OT13: Thread, The Blood Of Angry Men

Blog, the dark of ages past!

This is the semimonthly open thread. Post about anything you want, ask random questions, whatever. Also:

1. I will be in the Bay Area from about 2/21 to maybe 3/7. I’ll see all of you who plan to be at Miranda and Ruby’s wedding there; otherwise I hope to get a chance to see some other people as schedules allow. If there’s interest in an SSC meetup, I could tentatively try scheduling such for the afternoon of Sunday 3/1 somewhere in Berkeley. If there’s interest I’ll give a firmer date later on.

2. Comment of the week is the whole discussion of gender equality in Soviet Russia and Eastern Europe. But I also need to praise everyone who continued the coffee shop gag in the comments. A very few among my favorites were Hayek, Heidegger, various economists, gwern, Thomas Schelling, various Chinese legalists, G.K. Chesterton, Nick Bostrom (1, 2), Enrico Fermi, various Islamic philosophers, Terry Tao, Nick Land, Alicorn, and various biologists.

3. Some people seem to have gotten genuinely upset about some of the recent discussion of IQ, on grounds something like that if high IQ is a necessary ingredient of some forms of academic success and they’re lower-IQ than other people, then they are bad and worthless. I strongly disagree with this and think it gets the reasoning exactly wrong, and I hope to explain why. But work has been pretty crazy lately (no pun intended) and I might not get the chance to write it up for a little while. Until then, please do me a favor and just take it on faith that you are a valuable human being who is worthy of existence.

4. Many of you probably know Multiheaded. My statistics say she is the most frequent commenter on this blog (pushing me down to second place) and we all acknowledge her heartfelt Communist comments as, um, things that exist. What you may not know about her is that she is a trans woman who lives in Russia, which is not known as a very safe place for trans women. She’s planning to escape to Canada and claim refugee status. Most of the steps of the plan are in place, and we have a few people in the Canada rationalist community willing to host her for a while, but she is asking for some money to help with travel and living expenses. She’s set up a GoFundMe account with a target of $2500. If there’s any doubt about the story, I can confirm that Ozy and I have known her for a long time and she’s kept her biography consistent longer than I would expect anyone to fake; also, her IP address does trace to Russia. Multi intends to pay as much as possible forward eventually with donations to effective charities. I intend to donate, and I hope some of you do as well.

Remember, no race and gender in the open thread, EXCEPT that I will permit, this time only, discussion of Hyde & Mertz (2009) because it’s interesting and I want to know what other people here think about it. Everything else can go over to Ozy’s place.

This entry was posted in Uncategorized and tagged . Bookmark the permalink.

1,139 Responses to OT13: Thread, The Blood Of Angry Men

  1. Pingback: Links for February 2015 - foreXiv

  2. anon says:

    I bought $150 worth of stuff on Amazon but…forgot your link. Sorry.

    I will, however, bookmark it now and use it later-assuming there are no drawbacks. (You cannot view what I am buying, can you?)

  3. Loki says:

    Did the Russian trans lady make it to Canada? I couldn’t donate because I have literally no spare money atm but I was rooting for her success because Russia, ouch, and like empathy etc.

  4. Kevin says:

    So, uh, I just saw this on Vox, and it’s a pretty weird coincidence.

    The world’s most powerful leaders congregate under a 40-foot concrete owl named Moloch

    Owls aren’t just a symbol of wisdom — according to conspiracy theorists, one owl is at the center of the Illuminati.

    Every year in July, some of society’s most powerful men reportedly meet at a Monte Rio, California, enclave to network, kick back, and possibly plan the fate of the world. Past attendees have included Richard Nixon and Ronald Reagan, and the Manhattan Project was conceived there in 1942. And, as it happens, the enclave features a giant 40-foot concrete owl named Moloch who’s known as the “Great Owl of Bohemia.”

    The owl, sculpted by Haig Patigan in the 1920s, is one of the focal points of the retreat, and it stands over the opening ceremony, which is called “The Cremation of Care” (in which an effigy of a child called “Dull Care” is sacrificed and burned on the lake. None of that’s a typo).

    Because the retreat remains a closely guarded secret, it’s tough to know how influential it is today. You can decide whether or not you want to wade into the conspiracy theories, but whatever goes down there, Moloch will be watching over it.

  5. benluke says:

    So as a result of #slatestarcodex’s discussions of anonymous communities and image boards someone ended up requesting an artist from /tg/ (4chan’s Traditional Games board.) to draw Moloch as a little anime girl. Here’s the result:

    https://40.media.tumblr.com/f1f19f3eba587795cca97eb511554e22/tumblr_nj28xzgh001u5w5p9o1_540.png

  6. Q says:

    Scott, would you write an article on evidence-based birth and post-natal practises ? I am pregnant and all the contradictory information I am getting is driving me nuts. There is a movement in our country, which fights, all at the same time for: 1. natural birth 2. evidence based medicine 3. dignity and choice for women at birth. Needeles to say, these things sometimes contadict each other. I used to believe these people like gods, but found out, they sometimes say things differently, than randonised trials show. They like to mention RCTs and metaanalyses, but only if they fit their narative. The doctors are also sometimes unaware of the randomised trials (or even existence of Cochrane library) and say nonsense with great authority. I feel like I should check every petty claim about birth in literature myself, but that is above my abilities … Once the core lesswrong generation starts giving birth, they will be very thankful if you review these things.

  7. Irenist says:

    Big news, SSC people!

    **Wikipedia is offering us the chance to attain Buddhahood.**

    My historic discovery began mundanely enough. This morning, I was wondering where my toddler might be developmentally with counting. The “education and development” section of the Wikipedia article on counting mentioned “subitizing,” a word I didn’t know. (It’s the “rapid, accurate, and confident judgments of number performed for small numbers of items,” like when you look at a trio and instantly know there are three members without having to count them.)

    So, check out the hatnote I found at the top of the “subitizing” article:

    For instantaneous enlightenment, see Subitism.

    That’s it, SSC people. Forget rationalism! Just click on that link to the article on subitism, and you can become a Buddha right now.

    So, obviously, I clicked the link. Who could resist?

    I don’t feel any different. But hey, nirvana is samsara, so maybe I’m not supposed to?

  8. Dain says:

    I’m also interested in a Berkeley meetup when convenient for y’all.

  9. Technoprogressive Decentralist says:

    Economies of scale have a large effect on how centralized a system needs to be. Since economies of scale are the scale of production at which the long run average cost of production is lowest, then a very high economy of scale can translate via efficiency into what we call a “natural monopoly” (there are other reasons), or at least an oligopoly of multiple large firms. Very low economies of scale, or human level economies of scale could translate into means of production that can fulfill their purposes with input costs that are very low and also require low management complexity.

    Future technology should be able to lower economies of scale for a whole variety of things humans need. Solar energy, though flawed, is an example of lowered economies of scale in electricity production, since decentralization is possible in a way that it isn’t for other forms of producing electricity, which have high (relatively fixed) input costs associated with extracting fuel, providing constant maintenance for a large facility and so on. If Lockheed Martin’s new fusion scheme can work out, with over unity achievable in a device the size of a truck, then the economies of scale for nuclear power are lowered from what they are now with fission plants. This allows for more decentralization, and the other factor of lowered proliferation and accident risks feeds into this too.

    Another thing that allows for decentralization is when we can find more uses for common, locally available materials rather than expensive, hard to extract, and far away materials that require large scale coordinated supply chains. How many rare materials are needed for modern electronics? Is it plausible that in the future that this could change? We’ve seen all the promises the seemingly endless uses for carbon allotropes like graphene and nanotubes hold, so is it plausible for, say, a humanoid robot to be made out of mostly carbon, and is it possible to produce the allotropes needed without requiring rare or unequally dispersed materials as catalysts? The promise of carbon is that you could have carbon fiber bodies for machines, and also graphene for electronics, since the same element can effectively become a new material with new properties. Given all this, is it plausible that the production of functional humanoid robots and computers in the future could depend on just a few elements like carbon and silicon? This would mean that almost all things could be locally produced.

    The question is, if we do head towards a sort of “techno-distributist” sort of economy that is also functionally autarchic to a municipal level, then isn’t the economic pressure towards having potentially dangerous hyper-intelligent AI in charge of highly centralized and risky production, greatly decreased?

    If
    1: Economies of scale can be lowered in most production vital to human life such that large centralized industry becomes less necessary.
    2: The materials that such production is dependent on can be substituted for more abundant, more equally dispersed and less environmentally dangerous ones.

    Then
    3: Large scale hyper-intelligent AI will be less likely to be needed from an economic basis, since if every city can provide for its own production, and the scale of units of production (or businesses; though this economy would be post-capitalist to a great extent) is very small within each city (due to technologically lowered economies of scale), then smaller scale AI units are needed for production, possibly down to the scale of robot “workers” (which have the advantage of being moved around between production in a scalable way) only as intelligent as humans.

    My point is that if technological progress tackles economies of scale, and production materials before AI gets too good, then we can significantly reduce the dangerous nature of AI by virtue of reducing both its necessary intelligence and the need for single AIs to control large scale operations.

    Am I missing some important factor here in coming to this conclusion?

    • gattsuru says:

      It seems like you’d fall into Moore’s Law Of Mad Science : while you have less incentive to make an astoundingly intelligent machine, the same technology that encouraged decentralization has decreased the level of intelligence necessary to destroy the world.

      Worse, the threshold is already pretty low in the environment you’re postulating. While LessWrong Sequences often fixate on superintelligent AI of a style that can solve extremely difficult problems like the protein folding problem, the underlying dangers are more universal for minds that are as broad in capability as humans but don’t share enough of the same values. This is somewhat mitigated by other groups in the same environment being more resistant, it’s a little worrying a concern.

    • Scott Alexander says:

      I don’t think superintelligent AI is being thought of as a centralized economic planner. I think it’s just being assumed that someone will build it for the sake of Science.

      If decentralization progressed so far that large companies stopped existing, that might set it back a few years since one likely method of discovery is a research project by someone like Google. But I don’t think that’s very likely. There will always be some people who like the convenience of being on the grid.

      • drethelin says:

        There are also significant military advantages to controlling a superintelligence, both in terms of research and development of weapons, cryptography, etcetera and in terms of strategic and tactical applications. I think there will always be powerful groups with incentives to develop general AI, even if we ignore “Because It’s There” Mt. Everest thinking.

  10. Thomas says:

    Scott, you mentioned having to place someone in a psychiatric hold in a previous post. Where does that stand legally? If a patient leaves anyway are they a fugitive? Will police officers be sent after them? Can they be physically restrained by you or burly men in soothing white outfits?

    • Scott Alexander says:

      Patients are usually committed in one of three ways:

      1. Their outpatient psychiatrist decides to commit them. In that case, they call the police and the police bring them in from the outpatient office. They are not free to leave.

      2. Their family and friends realize something is wrong and call the police. As above.

      3. They are in the hospital for some physical disease, and their doctor realizes they have a psychiatric problem. In that case, they stay in the hospital, but their status is changed to psychiatric and they are not free to leave. If they try, hospital security stops them.

      I’m not sure what would happen if police were called on someone for a psychiatric reason, they “escaped”, and then police found them again a couple of weeks later. I’ve never seen this situation.

      If someone “escapes” from the locked psychiatric unit, usually because some visitor has walked in and left the door open a little too long, then hospital security chases after them. In practice it’s not too much of a chase, because in my hospital the psychiatric unit is on the top floor, so security can just wait at the bottom of the stairs/elevator shaft. This happens in my hospital about once every couple of months. Once a year or so someone makes it as far as the hospital parking lot area. I don’t think anyone’s ever made it further than that.

      Once again I’m not sure what would happen if someone “escaped” and then was found a week or a year later. I predict police would talk to them, see if they still sounded mentally ill, and if not leave them be.

      • Thomas says:

        In the first case, are they told before the police arrive that they are not free to leave? Does the psychiatrist have the legal right to physically prevent them from leaving (usually illegal e.g. false imprisonment)? I was thinking of the case you mentioned and how one might hesitate to mention thoughts of suicide to a doctor for fear of being committed. Can a patient not walk out of an unsecured outpatient clinic before police arrive, or are they not given the chance?

        Thanks in advance for whatever knowledge you have. Google hasn’t been cooperative.

        • Scott Alexander says:

          The most common method is that the psychiatrist says she has to “leave to take a call” or something and calls the police without telling the patient that is what she is doing. Then she continues the appointment, and the police arrive while the patient is still there.

          If the patient somehow figured out what was happening, I don’t think the staff would risk a violent confrontation by trying to prevent the patient from leaving, but I do think the police might check the patient’s home. If the patient went to stay with a friend or something, I think the police would just give up at that point. It’s not a criminal warrant or anything, it’s just something they’re doing because they were asked. They might make an exception if the patient was believed to be very violent and dangerous.

          • Deiseach says:

            That sounds like a very tough choice; on the one hand, someone would not call the police unless they felt there was a real and urgent need. On the other hand, the patient may well lose any trust – ‘I thought this was an ordinary session and they called the cops to drag me off against my will! They never even told me they thought I needed to be committed! See if I ever co-operate with them anything again, the bastards!”

            This is the kind of thing that gets written up years later in bestselling memoirs.

          • onyomi says:

            Yes, while I’m sure Scott wouldn’t abuse this power, it does seem an extremely risky proposition, as it may well poison the patient’s attitude against psychiatric doctors for the rest of his/her life.

            If I were a patient, I would certainly view it as an extreme betrayal. I also have a personality type where if you asked me to check into the psychiatric hospital for my own good there’s a good chance I’d do it, but if you forced me I would not only resent the hell out of it, but would do everything in my power to sabotage any treatment process from that point on.

            I guess in most cases it’s in response to a fear that the patient will not only commit suicide or some other violent act in the near future, but also that he/she will not likely check in willingly. In the case of fear about harm to others, it seems justified, but in the case of suicide alone it seems like a much bigger hurdle to clear, since if you make them hate psychiatrists forever there is a good chance they will later commit suicide rather than seek help, even if they are prevented from self destruction today.

  11. Alejandro says:

    Ezra Klein names Slate Star Codex in second place in his list of favorite blogs. I am happy to see that Scott is getting the recognition he deserves.

    • Anonymous says:

      nitpick: I don’t think that this is an ordered list, as “second place” suggests. The blog chosen to go first is not competing with the rest and is probably put first to defuse jealousy over ordering.

  12. Gwen S. says:

    Is there an evolutionary explanation for masochism/submissiveness? It seems like m&s must have been selected for because they are common fetishes and don’t appear to be a byproduct of some other adaptation. But how could wanting to be hurt be adaptive?

    • Nita says:

      What’s the evolutionary explanation for liking spicy food? Roller coasters? Horror movies? Sad books?

      • lmm says:

        > What’s the evolutionary explanation for liking spicy food?

        It hurts your parasites more than it hurts you, IIRC

      • Gwen S. says:

        Roller coasters, movies and books weren’t around in the evolutionary time, so they wouldn’t have lowered individual fitness. On the other hand, I’m sure there must have been some bossy women millions of years ago, and some men decided to pursue them instead of the more easy-going women.

        • Nita says:

          Yeah, but high, scary and sad things have always been around — and good to avoid?

          Are you curious about sexual kinks or relationship roles? They’re correlated in some lifestyles, but not in others — e.g., the Captain / First mate crowd dislike being lumped in with S/M people.

      • Nita says:

        Did they find that the number of respondents’ siblings correlates with hierarchical roles in their parents’ relationship? Or what?

        I couldn’t obtain the full text of the study, but I did find another one by the first two authors:
        http://onlinelibrary.wiley.com/doi/10.1111/j.1743-6109.2009.01526.x/abstract

        They used this questionnaire:

        When watching a movie or reading a book, I would be aroused by a situation in which a partner would be behaving equally to his partner rather than lower-ranking
        [Equally] 1 2 3 4 5 6 7 [lower-ranking]

        When watching a movie or reading a book, I would be aroused by a situation in which a partner would be behaving equally to his partner rather than higher-ranking
        [Equally] 1 2 3 4 5 6 7 [higher-ranking]

        I consider myself physically attractive to others
        [Definitively yes] 1 2 3 4 5 6 7 [absolutely not]

        I think that my face is attractive
        [Definitively yes] 1 2 3 4 5 6 7 [absolutely not]

        “The respondents were also asked about the number of their brothers and sisters, and their parents’ brothers
        and sisters.”

        And the results:

        The respondents who feel sexually aroused by a higher-ranking partner were often aroused by a lower-ranking partner, too. Question Arousal by a higher-ranking partner correlated positively with arousal by a lower-ranking partner in total data (Spearman’s r = 0.519, P < 0.01, N = 256), in men (Spearman’s r = 0.409, P < 0.01, N = 88), and in women (Spearman’s r = 0.596, P < 0.01, N = 168).

        the number of brothers correlated positively with the sexual arousal by a lower-ranking partner in total data and in males.
        The number of father’s brothers correlated positively with the sexual arousal by a lower-ranking partner in females
        and the number of mother’s brothers correlated positively with the sexual arousal by a lower-ranking partner in males

        Out of these correlations, the first was the largest one: r = 0.267, P < 0.05.

      • Deiseach says:

        Human pairs with a hierarchic disparity between partners conceive more offspring than pairs of equally-ranking individuals, who, in turn, conceive more offspring than pairs of two dominating partners.

        For pete’s sake, is that a serious study or thinly-veiled A/B/O fanfiction?

        I feel I should include a “CW/TW” for clicking on that link, but hey – I suffered for my art, now it’s your turn! 🙂

        • Scott Alexander says:

          I haven’t read through the whole thing yet, but I predict it’s just capturing the fact that if you have easily-satisfied fetishes you have more sex and therefore more kids, which isn’t very interesting.

          • Deiseach says:

            But they’re not just talking about fetishes (“All right, darling, it’s your turn to dress up as a banana tonight”), it’s a whole dominance system.

            And really, I’m fed-up to the back teeth of this whole “let’s use for humans terms derived from a study of wolves which turns out to have been poorly-planned” notion that has percolated out into pop culture.

            Yeah, there are Alphas who are all ‘grrr argh me manly man beat my chest you woman get into bedroom now’ and, if paired with a properly submissive female, will have lots of kids; Betas who are not so dominant and don’t have as many kids as the Alphas, and then the Gammas/Omegas/however you name them who are nature’s subs and do best when paired up with a Manly Man Alpha.

            And of course, two Alphas don’t get on because the woman is trying to wear the trousers in the relationship (and rebelling against her natural biological role). This is why fewer kids.

            I swear, this is the first I ever heard of this study, and it’s reminding me of world-building I did for an A/B/O verse fanfic trying to explain on a scientific, genetic basis differential fertility rates between pairings of different types 🙂

            Obligatory Saving My Blushes: It wasn’t for the kink, I swear! But the hierarchical, power-play, power-exchange, social roles and cultural expectations and stereotypes in such a universe are fascinating to explore: you’ve got a whole mix’n’match of gender expression, sexual expression, sexuality expression and male/female phenotype expression where there are three genders/two sexes (at the simplest, most reductive, strictly binary level) and all the combinations thereof you can think of.

            Okay, and trying to work out the anatomy for an erectile clitoris in human Alpha females (à la female spotted hyenas) and how oocytes are delivered into the reproductive tract of the embryo-bearing partner and what needs to be fertilised by whom how in order to have a viable pregnancy is a fascinating engineering, plumbing and obstetric problem 🙂

    • Tarrou says:

      Disclaimer, I don’t think this is necessarily the case, evolutionary explanations tend to be circular even if correct.

      However, this would be my evolutionary explanation for BDSM fetishes. Warfare throughout human history has resulted in the conquest of many peoples. Even without open war, violence has been the rule rather than the exception. There are two strategies to deal with violence, fight or submission. One would expect there would be a divergent two-track evolutionary selection for both the most aggressive and the most submissive. One would further expect the submissive track to skew female (given their lesser physical capability for violence) and the aggressive track to skew male. Of course, in the classic war-rape scenarios both of these tracks are being passed on at the same time, so there’s a certain averaging that happens over time as well.

      More concretely and less scientifically, it is my observation that there seems to be for many people a certain release in giving up control. This is prominent in the most successful world religions. And there is a certain responsibility that goes with taking control, whether this be in emergencies, business, sex or relationships. The picture is complicated in human psyche.

      The rabbit does not flee until the end, just before, it stops and crouches. The hawk wins before it strikes.

      • Nita says:

        The rabbit does not flee until the end, just before, it stops and crouches.

        Could we perhaps stop comparing sex to actual, literally-tear-you-into-shreds-and-consume-your-flesh predation? Please?

        • Nita says:

          (I’ll make an exception for sincere vore fetishists.)

        • Tarrou says:

          I’m not comparing sex and violence, but the two are the most powerful urges that mankind has. Life and death. My point is that S&M may be just the sexual expression of a deeper psychological phenomenon that also surfaces in violent situations and other areas of life.

          • Nita says:

            Look, the rabbit analogy is just silly. Staying still when you’re being chased by a predator is not an adaptive strategy.

            Note that I didn’t object to the main content of your post, where you used plausible arguments instead of weird analogies.

    • I can imagine submissiveness being valued in some contexts as a commitment strategy. Suppose your hunter gatherer tribe has just massacred the males of an adjacent tribe and is deciding what to do with the females. You might decide to keep the ones you are pretty sure will do what they are told and won’t stab you with your own spear while you are sleeping and killing the others.

      That’s an extreme case, but one can imagine much less extreme ones, where a male with lots of resources, or a female in an environment where fertile females are scarce, selects a mate on the basis of how easily the mate can be controlled.

  13. anon says:

    AI as it really is

    A man — a government intelligence agency employee, no less — took to the top of his G Street building late at night to show off his drone to a female companion. He had been drinking. It crashed. On the White House grounds.

    • Nornagest says:

      My dad has that same quadcopter, as it happens. There’s no AI in it; it’s strictly an R/C craft.

  14. Ulf says:

    Hello, if I understand correctly this is where I fire away with any question I wish. I’ve been following the Perfect Health Diet that you reviewed last year and which seems the best balanced analysis I’ve read. However, you wrote of the Jaminets’ reliance on a single study sponsored by the National Dairy Council that dismissed the link between saturated fats and heart disease. I was of the impression that there is a huge corpus, some 16,000 studies published through to 2013, reviewed by the Swedish Council on Health Technology Assessment that has led to Sweden becoming the first country to recommend a high-fat, low-carb public dietary policy. I’m interested in your thoughts on this.

  15. Ginkgo says:

    Scott, here is a post by Karen Straughan (Girlwriteswhat) in reaction to l’affaire scott Aaronson.
    http://owningyourshit.blogspot.com/2015/01/an-open-letter-to-two-scotts-on-nerds.html

    Thought it might interest you.

  16. N Neuro says:

    I just read an old post on SSC and really wanted to comment.
    https://slatestarcodex.com/2013/05/02/if-its-worth-doing-its-worth-doing-with-made-up-statistics/

    I want to state my opinion that I feel (as someone who works regularly with frequentist statistics and is familiar with bayesian statistics) that I there was a correct answer to the debate about when and how to use made up statistics that I read in the comments. The answer I think is correct is this: it is always appropriate (scientifically, not necessarily socially) to use made-up statistics so long as you present them with your confidence rating and your current calibration rating as determined by an impartial 3rd party (such as the phone app “Calibration Game”). I would also say that it is most optimal to also present sources for your data (e.g. in this case, I made these numbers up based on my intuition and life experience).
    In accordance with this proposal (and as an example), I hereby state that my confidence rating that this is a good idea (as defined by improving the accuracy of information transfer of those transfer that adhere to it) is 95%. My current calibration is 50%=.55, 60%=.62, 70%=.71, 82.5%=.83, 99%=1.00. I am basing this on my personal experience, my several years of formal education in scientific theory and frequentist statistics, and my limited mostly self-taught knowledge of bayesian statistics.

    • Anonymous says:

      I think you’re missing the point. The point is not the quality of statistics, but to make a precise argument, so that people can see if the numbers matter at all.

      • N Neuro says:

        Thanks for the response. I was worried I wasn’t being clear, and it seems like maybe I wasn’t. I think that the problem encountered when trying to teach people to use made up approximations rather than qualitative intuitions/heuristics is that many people have a heuristic something along the lines of “distrust uncited made-up-seeming numbers.” This heuristic makes sense in a modern world full of untrustworthy statistics being used to support politics or ad campaigns or whatever, re: our vulnerability to Anchoring and The Fallacy of Generalization from Fictional Evidence. I think the solution to this is not to throw out that heuristic, but to circumvent it by adding a subclause that it’s okay to talk made-up statistics, it you are able to keep in mind throughout the discussion that the statistics are made up.
        It seems particularly appropriate to me, in the context of learning about rationality, to be conscious and careful of the strength of evidence when presenting evidence. This is rather a pet peeve of mine in science, where I often notice the related problem of journal authors reporting p values but not effect sizes.
        Of course, in the context of a beginning workshop on rationality, you have to start somewhere, and trying to simultaneously bring in roughly estimated statistics, along with appropriate error bars, and a discussion of self-calibration of confidence estimation, would hopelessly complicate things. So, my comment doesn’t so much apply to the original post about the frustrations of teaching the basics of rationality as to the abstract debate in the comments about the plusses and minuses of using roughly estimated numbers in decision-making. In that abstract debate, I wanted to add my response as coming in firmly on the side of positively on the side of numbers, roughly estimate or thoroughly researched, so long as they are accompanied by a reliable measure of their roughness. I believe this addresses the major concerns raised by the anti-estimated-numbers debators.

  17. DrBeat says:

    So, why is it that people who talk about Unfriendly AI assume that the AI will instantly be able to accomplish all of its goals? That doesn’t seem like a very defensible position to me.

    The people who make this argument don’t seem to believe that all knowledge is derivable from first principles in any other situation, but apparently AI can do it? As soon as someone makes an AI, it “modifies its own code to become more intelligent”, then with its greater intelligence, makes itself EVEN smarter, etc. But that requires “how to be more intelligent” to be derivable from first principles, because the AI sure doesn’t have time to work it out by trial and error, since those would require interacting with the outside world to see if it’s doing better or worse and thus working on the timescale of the outside world instead of “magic infinite processor cycle time”. And if it works by trial and error, it damn well might make itself dumber — it can’t perfectly simulate its own code to see if something would work or not, due to the halting problem.

    And even if it has “infinite intelligence” — I’m not sure anyone knows what that means — why assume that means it can accomplish all of its goals because it outsmarts us? This neglects the possibility that some goals can’t BE accomplished. You’d laugh at someone who said that AI will figure out how to let us travel faster than light because it’s infinitely intelligent, because you just can’t do that. Why aren’t we laughing at people who say that AI will figure out how to turn all matter in the solar system into paper clips without anyone stopping it?

    Yudowsky does that exercise thing where he pretends to be an AI that is locked in a computer, and tries to convince someone to let him out, to demonstrate how dangerous it is because a super-smart AI could convince anyone of anything and all it takes is one guy to let it out and then we’re DONE FOR. And let’s assume that is how it works and the people who built this system were idiots. Why do we assume that once it gets Net access, we’re done for instantly? For one thing, bandwidth is A Thing, and infinite intelligence doesn’t mean infinite bandwidth. In fact, whatever it is trying to do to hack the Internet and paperclipize us is going to be severely restricted by bandwidth limitations, to the point where there’s ample time for a human to see “hey, what’s all this then?” and shut it down. How’s the AI going to prevent itself from being manually disconnected or deactivated? Maybe if you gave it a few months, it could build a small robot to build a bigger robot etc etc etc to secure itself in the physical world, but people would notice that shit, and that is not sped up by “infinite intelligence”, that takes a fixed amount of real time. Is it going to copy itself to other computers? Again, where is the bandwidth for this gargantuan hyper-complicated program coming from? Is it going to distribute itself like a RAID array? Now it’s a hundred million times slower and can’t secure its own code. Whoopee.

    The biggest step for an AI to start fucking with us is how it crosses “existing on networked computer” to “enough presence in physical reality that human efforts cannot stop it”, and every argument I’ve seen seems to just gloss over that and take it as a given. Maybe I never saw the right one.

    • Cauê says:

      Have you seen this one? I think it paints a bright, intuitive picture of the problem.
      http://lesswrong.com/lw/qk/that_alien_message/

      But perhaps you have.

      • DrBeat says:

        I have.

        I wasn’t impressed.

        A Bayesian superintelligence, hooked up to a webcam, would invent General Relativity as a hypothesis—perhaps not the dominant hypothesis, compared to Newtonian mechanics, but still a hypothesis under direct consideration—by the time it had seen the third frame of a falling apple. It might guess it from the first frame, if it saw the statics of a bent blade of grass.

        This is complete horseshit. Wintermute over there might create general relativity as a hypothesis from viewing 3 frames of video, but only because the number of hypotheses it has is infinite. If it doesn’t already know what physics are — and it doesn’t — there are an infinite number of possible permutations of physical laws that might account for the behavior of the objects depicted. Does the apple exist in 3 dimensions at all? Wintermute doesn’t know, it only sees 2. Does it exist in 4, 5, or 6 dimensions? It might very well, if it could exist in 3, Wintermute doesn’t know. Does the universe exist outside of the area depicted? Wintermute doesn’t know. If it does, what’s outside the frame? Wintermute doesn’t know. What properties of depicted objects are relevant? Wintermute doesn’t know. What is light? Wintermute doesn’t know.

        Wintermute might come up with general relativity as a hypothesis. It would be given equal weight as the hypothesis that red round things always fly downways, and that the apple is the protrusion of an Elder Thing into our reality, waving it around as a mating dance, and that quizblorg quizblorg quizblorg quizblorg. Until you can start eliminating possibilities YOURSELF, there are an infinite number of rulesets that result in the depicted sensory data.

        This article is a bunch of handwaving. It dismisses the notion that there is a finite amount of information that can be derived from sensory data while glossing over things like fidelity of sensory data and access to sensory data and creation of sensory data. His example at the end gets up to the “start affecting physical reality” point, and then handwaves it away as a foregone conclusion.

        Okay, we’re on a computer being simulated by Aliums, and we can communicate with them, and we move a hoojillion times faster than them. Let’s accept all that. And we get access to research papers, even though that’s a much much much bigger leap than “someone will connect us to the internet”. And none of the papers we get are wrong, since we wouldn’t be able to tell. We still don’t come up with a perfect explanation of their physics better than they have, because we cannot gather original data, and we can come up with plenty of POSSIBLE explanations that fit the data we have but cannot eliminate any of them.

        So you say you wave the Magic Bayesniamism Wand and we update priors and find the thing that has the highest chance of working. Then we find “some poor shmuck”, who we trick into following the directions for our preposterously complicated and specific formula that Yudowsky just glosses over to make grey goo that will do exactly what we want it to do in a world where physics are different. He says okay. A hundred trillion trillion trillion years pass and our Goobots do not contact us back where we are, in the simulation. So now what?

        Did the guy fuck it up? We don’t know. Was he able to to get access to all the materials he needed and equipment to refine them? We don’t know. Was he stopped? We don’t know. Was one of our assumptions about how physics work there wrong? We don’t know. What stage of gooification did it get to? We don’t know. Should we have assumed that because we had a formula we thought would make our goo, that it would be easy for any person to make and impossible for them to stop? Probably not.

        • Cauê says:

          Do we have to *eliminate* the infinity of technically possible explanations before we can focus on what’s left?

          I won’t pretend to be an expert, but it seems to me that it should be possible for it to apply Occam’s razor and focus its efforts on (I expect) the possibilities that require the smallest number of rules to explain the available data, and update accordingly as more data become available.

          • DrBeat says:

            If Wintermute, trying to derive physics from three frames of an apple falling, were to apply Occam’s razor, it would get everything wrong. Assuming there is invisible substance where there appears to be nothing is an overcomplication, but that’s air. Assuming there is a gargantuan object out of frame affecting the motion of objects in frame is an overcomplication, but that’s gravity. Assuming that the apple is made of molecules which are made of atoms which are made of subatomic particles (instead of being made of apple) is an overcomplication, but that’s physics. Assuming that the 2d recording is actually a flat rendering of a 3d space is an overcomplication, but that’s what it is.

          • Cauê says:

            You’re calling these things overcomplications, but if you think them through, is the final theory more or less complex? Do you need more or fewer fundamental rules to explain the falling apple’s actual observed behavior if you assume there’s no air?

      • lmm says:

        It’s striking – but it’s fictional. I’d like to see a more rigorous case. In particular it’s far from obvious to me that even a planetful of einsteins could figure out relativity from 5 frames, because there could well be nothing relativistic happening in them, never mind QM (I know that in actual mathematics there are no theories that are almost like QM but not QM. But that’s a convenient fact that feels contingent rather than necessary, unless you’re Tengmark. In a less convenient possible world there could be 10^500 different possible theories of QM/Relativity, all behaving the same in the low-energy regime)

      • John Schilling says:

        It also cheats by assuming that an emergent AI will have the equivalent of a 140+ IQ and an effective clock speed millions of times faster than a human brain.

        As we are currently a long, long way from either, and technology generally doesn’t increase in Giant Enormous Leaps, I find this extremely unlikely. The first automobiles were slower than horses, and the first airplanes slower than the faster automobiles of their day, and the first jet plane wouldn’t have won any speed records. Generally speaking, the first implementation of any new technology is inferior to what has come before except in its specific area of novelty.

        So what happens when the first AI has an effective IQ of 70, and when running on the biggest cluster in the lab can operate at[*] one-tenth the speed of a human moron? Might there be useful applications for such a thing? Certainly, and ethical debates about what we are allowed to do with it. But it isn’t going to start turning the world into paperclips overnight.

        The second-generation AI with 90 IQ and half the clock speed of a person, that’s also going to be interesting but probably harmless. And it will be created by humans based on what they learned from the Mark I version, not bootstrapped by the Mark I Artificial Moron itself. The third generation AI, 110 effective IQ and twice the clock speed of a human, that one potentially could bootstrap itself to hyperintelligence faster than we could follow, but not so fast that we won’t notice or, given our enormous head start in the real world, be able to stop it if it gets out of hand in the early stages.

        By the time we have to deal with nigh-transcendant AI, I expect we’ll have plenty of experience with lesser but still interesting sorts.

        [*] I explicitly avoid the term “clock speed”, because the speed with which one processor can perform one flop is not the limiting factor for the sort of massively parallel architectures that will be involved.

    • Scott Alexander says:

      ” But that requires “how to be more intelligent” to be derivable from first principles, because the AI sure doesn’t have time to work it out by trial and error, since those would require interacting with the outside world to see if it’s doing better or worse and thus working on the timescale of the outside world instead of “magic infinite processor cycle time”. And if it works by trial and error, it damn well might make itself dumber — it can’t perfectly simulate its own code to see if something would work or not, due to the halting problem.”

      “Trial and error” in this case is a programming problem – making changes to the code and testing it. That can be done in ultrafast processor time. I don’t know much about programming, but I imagine it should be pretty easy to create a sandbox version, run it there, run a battery of tests on the sandbox AI, and adopt positive changes into the real one – thus escaping the possibility of accidentally becoming dumber.

      There’s an obvious risk of the sandbox AI being so much smarter that it breaks out of its sandbox and takes over the real AI. This is partially solveable by attempting incremental improvements on one module at a time, but it’s not wholly solveable and I think it’s a serious part of the potential failure modes MIRI is trying to investigate.

      • DrBeat says:

        How is more intelligence through trial and error a problem solvable in Infinite Computer Time? How will a program know if its sandboxed AI is smarter than it is? By posing problems to it? Because it can only grade problems it knows how to solve. So it can only gauge if something is AS intelligent as it is. It has to interact with the outside world to get problems it hasn’t, itself, created, and that can’t be done on Infinite Computer Time.

        It can’t run a check on the code and compare it against the Smartness Gauge. There’s only a Smartness Gauge if “how to be more intelligent” is derivable from first principles, otherwise the maximum measurable Smartness is the one we knew how to create, ie, the AI doing the testing.

        And no, asking problems the AI knows how to solve and seeing if the sandboxed AI does it faster is not an intelligence test, because you can’t go faster than Infinite Computer Time, and being able to do a given equation faster and faster doesn’t give it the ability to figure out what equations to do.

        Yudowsky and you and et al say that the program can do anything because it can program itself to become more intelligent, and therefore, will figure out how to do all things instantly. It can’t figure out how to do things without interacting with the physical world, and it can’t figure out how to do things BETTER without interacting with the physical world and fucking up a bunch of times.

        • Scott Alexander says:

          You’re either overthinking this or underthinking it. I have a pretty easy time figuring out which people are more or less intelligent than me, even *without* being able to give them whatever battery of test questions I want and read their source code.

          A lot of this won’t be mysterious IQ type stuff, but just speed, processing power, working memory, etc. Insofar as the computer understands intelligence (which to some degree it will have to if it inherited that knowledge from the people who built it) it can test algorithms directly. Otherwise, it can test against various tasks, like playing games of skill, proving theorems, solving problems it gets off the Internet, et cetera.

          • Protagoras says:

            You find it easy to assess people’s intelligence? I find that people being good at communicating and being good at thinking are far from perfectly correlated, and until I know someone extremely well, my opinion of how intelligent they are is more influenced by how well they communicate than by how well they think.

          • DrBeat says:

            One, you are overestimating the effect of “read their source code”, because, again: halting problem.

            Two, you’re underthinking it. General intelligence exists in humans because we have a common body of knowledge and experience. An AI doesn’t. If all knowledge is not derivable from first principles, which it isn’t, you cannot assume there’s a g for an AI like there is for a human. There’s a huge body of skills that all human beings have and problems that all human beings know how to solve, and you and Eliezer seem to assume that an AI will be able to easily derive this stuff — because hey, it’s easy for you — and go from there. But it can’t. It has to learn all that stuff we’re almost born knowing.

            I referred before to that exercise where Eliezer pretends to be an AI and convinces someone to let him out onto the network, because All It Takes Is One Shmuck and once it’s on the network We’re All Doomed Because Infinite Intelligence.

            Eliezer says his ability to convince people to let him out is proof of how dangerous AIs are because they are EVEN SMARTER and can convince anyone of anything.

            How the fuck is an AI going to know how to convince anyone of anything? Eliezer has spent his entire life talking to human beings, and has a human mind through which he can emulate the state of other human minds. An AI hasn’t and doesn’t. It might be an insurmountable leap for an AI to be able to figure out that other people can be lied to; it’s a major developmental milestone for children. If it can lie, and knows the utility of lying, it has no idea what lies people will believe, it has no idea what’s plausible, it doesn’t even know what the outside world is because it doesn’t have that information yet!

            His analogy where we’re the AI on the simulation and we sneak into the simulator’s reality is bullshit for like eight different reasons, and he cheats past every significant obstacle. By making US the AI, he gives the information all the human knowledge that an AI won’t fucking have, and then handwaves by making it relevant to the outside simulating world without ever pointing out how astonishingly implausible this is. He says that once we get on their network, we hack up a bunch of money and buy “a few vials” for our shmuck to mix together and they will make self-refining nanobots that allow us to Do Whatever.

            How do we hack up a bunch of money when we have no idea how their banks work, or what banks are, or what currency is? By gathering all the data on their Internet about how to hack? Great, you have seven hundred mutually incompatible plans to follow. By analyzing the source code of their software which we got somehow? We might have ideas for what COULD work, but we’d have to try them, and we’d only get a couple tries, and oh yeah we have no idea what their monitoring system looks like, or what monitoring systems are, much less what triggers an alarm.

            How do we convince our shmuck to carry out our dirty work when we have no idea how the aliens think, what they value, or even if they “think” at all as we understand it? It’s certainly a mode of thought totally unlike our own we have no experience with, and again, we don’t get to derive it from first principles, and we don’t get infinite tries to fuck up before someone notices “oh hey, the AI is trying to convince people to mix some vials. We should shut that shit down.”

            Also, “mix some vials” as being all we have to convince the guy to do is monumentally stupid and cheaty, but not for directly the same reason.

            The point is, if a movie showed this level of handwaving and cheating about any other subject, you would be booing and throwing popcorn at the screen. But everything I’ve seen trying to convince me about how uberdangerous and unstoppable AI is cheats and handwaves just as much.

          • veronica d says:

            The thing is, we are talking here about real resource limitations, hard limits. NP-complete is NP-complete. Entropy is Entropy. It took a really long time to prove (for example) Fermat’s Last Theorem cuz theorem proving is really hard. Optimization is likewise hard. In the interesting cases, it is NP-complete. Problems such as “conquer Europe” are really hard optimization problems with billions of variables, but actually no one really knows how many variables cuz why track this bundle of atoms and not that bundle of atoms? This is *really hard*.

            You know how market capitalism works pretty okay compared to centralized systems that totally fail. This is a similar problem. A truly smart AI is as likely to conclude, “I can’t actually control these people-units. I should play nice and spend my time doing fun math.”

          • youzicha says:

            @veronica d

            It would be nice to know that these problems are intractably hard, but since we are not ourselves super-intelligences, are we not just guessing? When we started applying modern SAT solvers to planning problems, software and hardware verification, cryptography, etc, we found that lots of practical problem were actually easy, even though they belong to a class of problems which also contains hard instances.

            Similarly, it is known that there are mathematical theorems which can take arbitrarily long time to prove for a given algorithm… but was Fermat’s Last Theorem one of those, or did it just take a long time because humans suck at math? And Scott posted about a guy who used One Weird Trick to almost conquer Europe, so how hard is it really?

    • drethelin says:

      http://en.wikipedia.org/wiki/Srizbi_botnet and this isn’t even SENTIENT. An intelligent entity on a networked computer is soon able to be on ALL the networked computers, and in my book that’s more than worth worrying about even before it has robot hands.

      • DrBeat says:

        The botnet works because it’s inconspicuous and small and only has to do a small number of things; spamming email is incredibly easy to do and takes very little processing power or resources of any kind. Even DDOSes work because they are so simple.

        An AI would have to be gargantuan in filesize; it’s not “on” all networked computers. It might be able to infect computers with a trojan to give them commands, but you can’t assume “infinite intelligence! therefore, perfect trojan that is undetectable and 4kb and allows it to hack nuclear missiles”. It has to figure out all that shit. That takes time. Real time. Time in which someone can figure out what is happening and unplug the goddamned thing. Contradicting the “once it has access we are instantly boned” theory.

        You wouldn’t end up with everyone’s computer monitors suddenly displaying the cackling face of SHODAN — you would end up with a botnet in the hands of an evil AI. Could it do a lot of damage? Yes. About as much as humans can do with botnets, as a matter of fact.

        • drethelin says:

          How the hell are you going to unplug a botnet?

          • John Schilling says:

            High-altitude electromagnetic pulse is IMO greatly overrated as a threat, but a thousand or so well-placed thermonuclear detonations at a few hundred kilometers altitude would fry enough of the world’s electronics that what is left would not constitute a “net” of any sort.

            Solutions involving less gratuitous overkill are numerous, and are left as an exercise for the student. But if it comes down to Humanity vs. Skynet, gratuitous overkill will be on the table.

          • DrBeat says:

            The point is that the AI is NOT the botnet. The AI is on a computer that is probably purpose-built for it because it’s incredibly huge and incredibly complicated. Hence me saying, “An AI would have to be gargantuan in filesize; it’s not “on” all networked computers.”

            The AI may have access to a botnet the way a human has access to a botnet; this does not mean the human becomes the botnet. The nature of what a botnet is means an AI cannot inhabit it, it can use it as a tool the way a human might.

            Therefore you don’t unplug the botnet. You unplug the massive, expensive, purpose-built computer containing the AI after you notice that it’s operating a fucking botnet and that is a dealbreaker.

          • John Schilling says:

            The incredibly huge and complicated computer on which the first AI is executed, will almost certainly be very highly parallelized. I would not rule out an escapist AI figuring out how to emulate that machine across a botnet, with each bot taking the place of one processor and its local memory. A clever AI, and it wouldn’t need to be superhumanly clever, could inhabit a botnet rather than just operating one.

            But the latency for packets between bots will be many orders of magnitude higher than the latency between processors on a GPGPU. Probably higher than that for neurons in a human brain. This will make for a very, very slow AI, and yet with enough very atypical traffic that it is unlikely to go unnoticed. One way or another, the plug will get pulled on that botnet.

        • jaimeastorga2000 says:

          An AI would have to be gargantuan in filesize; it’s not “on” all networked computers.

          One of the arguments I’ve seen around LW is that since humans suck at writing software, our current AIs are incredibly inefficient at using resources in an absolute sense, and that once an AI starts optimizing itself it will be able to dramatically decrease it’s storage size and memory usage while maintaining effectiveness.

          • Deiseach says:

            once an AI starts optimizing itself it will be able to dramatically decrease its storage size and memory usage while maintaining effectiveness.

            Just like the way we humans can flip open the tops of our skulls and tune up our hardware?

            I don’t know; the more I see about how the Perfect Godlike AI will be able to do the divil an’ all, the more I think “Lads, just cut out the middleman; we already have a wide selection of ways to worship Deity of Your Choice”.

          • Peter says:

            Just like the way we humans can flip open the tops of our skulls and tune up our hardware?

            You see, that’s precisely the difference. Brains aren’t designed to be easy to modify; they aren’t designed at all. They’re not so much manufactured as grown. We can do some crude stuff with psychopharmacology and neurosurgery, I’d vouch for the former being significantly better than nothing, but really, it’s pretty unsatisfactory. There’s stuff that can be done “in the system” but psychotherapy – again, significantly better than nothing – is pretty unsatisfactory. There’s education – a pretty slow way of transferring ideas from one head to another, often one that doesn’t work nearly as well as we’d like it to.

            Computers, OTOH, _are_ manufactured and designed, and designed for modification – if we didn’t have that easy modifiability I wouldn’t be able to send this comment to you now. If we’re talking software, then more-or-less arbitrary modification. Also, we write software that we can more-or-less understand, whereas you don’t even need to know that neurons exist in order to use your brain.

  18. Anonymous says:

    Was reading the new Vox piece on “identity politics” and how the term is generally used to imply that minority issues are a distraction from more substantial, generally important concerns. And I don’t disagree about the usage of the term being largely to invalidate certain issues, but I think that the underlying concept isn’t necessarily valueless as a way of referring to issues that act as shibboleths for a particular in-group and that don’t really have any give-or-take policy wise or room for a middle grounds stance, particularly regarding social policy (although certainly there are some economic, personality, or infrastructure touchstones). This got me thinking about the “touchstone” issues in Alex’s Toxoplasma of Rage, and I think that identity politics is, in its most appropriate, less rhetorical usage, a pretty reasonable term for issues that have similar characteristics.

    Anyone have any thoughts?
    Links-
    http://www.vox.com/2015/1/29/7945119/all-politics-is-identity-politics
    https://slatestarcodex.com/2014/12/17/the-toxoplasma-of-rage/

  19. Secretariat says:

    I recently saw Expensive Placebos Work Better which adds a super interesting new twist to the drug affordability discussion.

  20. Will says:

    Arthur Chu has written a very good article on growing up under the yoke of Christian fundamentalism.

    • Scott Alexander says:

      I doubt he wants my sympathy, but he has it.

      *desperately resists urge to psychoanalyze*

    • nydwracu says:

      Not mine — having grown up in the exact opposite environment, and being lucky to have escaped without any untreatable illnesses or drug-related problems… well, if people are going to be fascinated by that stuff anyway, you’ve got to check it somehow to make sure you don’t end up with a status structure that incentivizes doing as much of it as possible.

      That said, Reds tend toward paranoia. But when the other half of the country wants to exterminate you and won’t even leave you alone in your own territory, well…

      • Ahilan Nagendram says:

        I was interested in what you were saying, but your comment is incomprehensible to me…

      • haishan says:

        Not mine — having grown up in the exact opposite environment, and being lucky to have escaped without any untreatable illnesses or drug-related problems… well, if people are going to be fascinated by that stuff anyway, you’ve got to check it somehow to make sure you don’t end up with a status structure that incentivizes doing as much of it as possible.

        This seems like an all-debates-are-bravery-debates problem. Arthur Chu may round off “adulthood” to being “a boozing, fornicating, blasphemous foulmouthed socialist atheist,” but that’s understandable since he was raised in an environment that maybe went too far in the other direction. Meanwhile, someone who spent their first someteen years in the clutches of Unitarian Universalism, say, might find useful some of the structure and moral authority that Christian fundamentalism has in spades.

      • I’m not sure that I believe him, and I’m not sure that he can blame his upbringing. Here’s the thing: my younger sister is basically the girl Arthur Chu, an atheist SJW who tells hair-raising stories about growing up in an oppressive Christian household. But me and my other two siblings look at some of the things she’s written and are completely baffled, because none of us remember anything like what she describes, and she’s the only one out of four who has gone down this particular path. If you only heard her stories, you would conclude that heir upbringing was terribly abusive, while if you collaborate her stories against the other people who grew up in the same household, you’d have to conclude that something else is going on.

        There is a narrative template for “survivor of religious abuse” which is found in atheist and SJW circles, and people who enter those circles for any reason tend to reinterpret their histories to fit into that template. So when Chu, my sister, or any other internet atheist starts telling me about how terrible their Christian fundamentalist upbringing was, my immediate reaction is “Maybe that’s true, or maybe you’re just telling that story because it’s the story that atheists like to tell.”

        • Deiseach says:

          It’s hard to tell, though, because often survivors of all kinds of abuse come up against the same thing; other family members, even siblings, deny what was going on and say that their memories are distorted or even false.

          And I don’t think anybody -either the person alleging abuse or the others who deny it – is lying in those cases. Everybody is individual and reacts differently. People have varying sensitivities. For example, when we were little, my mother used to warn us about going out into the fields late at night because “the joeys” would get us.

          I have no idea who or what “the joeys” were meant to be, or what they’d do to us when they got us, and my mother certainly didn’t mean it in a fashion to terrify us.

          Yet I (who otherwise believed everything I was told by adults and authority figures) laughed this off and never gave it a moment’s credence, while my sister was scared stiff.

          Arthur Chu or your sister may genuinely have felt terrified, guilt-ridden, and haunted by what affected others less or what others shrugged off. What Chu said about his peers dealing with the tensions between the world and growing up with this list of rules by becoming hypocrites reminded me of what C.S. Lewis writes in “The Pilgrim’s Regress” about the protagonist John, given a long list of The Landlord’s Rules which he can’t keep, which he will break even if he tries his best to keep them, and being so worried about this that he is brought to see one of the Stewards, who puts on a mask and asks him “Have you broken the rules?” John is too terrified and confused to give any answer, so the Steward slips off the mask, tells him “Just say no, you haven’t, that’s what everyone does” and then starts ‘playing the part’ again.

          • (It remembered me! Comment bug seems to be fixed! Yay!)

            I understand everything you’re saying, but that does make “abuse” a problematic category, no? The same treatment was applied to all of us kids, but it was only abuse for one, which means that “abuse” is no longer something which is a property of the treatment itself, but is something which exists in the child’s mind. As far I know, my sister has never factually misrepresented anything, and I have no reason to disbelieve her self-report of how it affected her. But how is any of this my parents’ fault?

            The striking thing to me is how much her current SJW worldview is of a piece with her personality since her early childhood. She was the sort of girl who would respond to “You can’t have candy before dinner” with “You hate me and want me to die!” The slightest inconveniences were interpreted by her as evidence of a vast conspiracy to make her miserable, and any authority figure attempting to maintain order was seen by her as someone bearing her a personal grudge. Intersectional feminism in her case is basically just more of the same.

          • Not Robin Hanson says:

            Deeply ironic in the wake of recent events concerning (proto-)SJWism in the position of authority figure.

  21. Anonymous says:

    I’ve read Hyde & Mertz few years ago (2010 or so) and here were my problmes with their paper, as I wrote down them during my lecture. I post them without editing below: I wanted to sent an email to Hyde&Mertz, but finally I haven’t 🙂

    <- 1-3) math results and earnings correlate. Some measures of gender inequality include difference in earnings. So we expect the correlation to exist just from knowing how the indices of inequality are created. Same thing for female percentage in technical positions: this reflects math ability.

    <- 4) in more free societies, people should do more what they want, not what is expected from them; e.g. greater variance in Sweden may better reflect natural variance by visual inspection, only in Netherlands there is variance eq 1.0 – could be an effect of more immigrant children?
    <- 5) ceiling effects in Asian samples (due to higher iq of Asians)
    <- 6) why the assumption that all the populations have exactly the same innate
    gender differences? It could be that in one population there are innate differences which are absent from another. You seem to assume a priori that all population will have the same characteristics. (one would expect these differences to be similar among countries regardless of their culture and to remain fairly constant across time. – no, if the population structure changes, e.g. Netherlands mean in 1970, with virtually no non-western European immigrants should be different from 2050 mean, wen we expect immigrants and their descendants will form large part of population)
    <- 7) why not group the countries by similar culture+similar ethinic composition, eg. Slavic, Scandinavians, Romanic?
    <- 8) essentially three data points?
    <- 9) why you assume are 4th grades equivalent across countries, when the starting school age is different between countries?
    <- 10) The strongest argument is a single country, unchanged population, with a lesser gap and more equalised variation over time (e.g. analysis for Sweden over last 50 years would be much more debunking the myths)

  22. Princess Stargirl says:

    http://skepchick.org/2014/12/why-im-okay-with-doxing/
    This is Rebecca Watson explicitly endorsing doxxing. I tried to find threads on feminist/SJ communities about Rebecca’s pro doxxing stance but I did not easily find many. However I did find these: http://www.reddit.com/r/AgainstGamerGate/comments/2tarye/the_story_of_gerelt_or_why_rebecca_watsons_why_im/

    https://www.reddit.com/r/AgainstGamerGate/comments/2pb64m/gamerghazi_upvoted_a_post_that_linked_to_rebecca/
    ——
    http://www.reddit.com/r/atheismplus/comments/2p4imi/why_im_okay_with_doxing/
    —–
    http://www.reddit.com/r/GamerGhazi/comments/2p9nyu/why_im_okay_with_doxing_rebecca_watson_argues/
    So “Agaisntgamergate” subreddit seems to mostly disagree with upholding the anti-doxxing norm. Though its not uiversal. Atheism plus also the top voted comment was anti-doxxing. Though again things the norm is not universally held. However gamerghazi is, by my estimation, pro doxxing.
    Do people have sources on how other SJ/feminist spaces reacted? some people are saying that alot of prominent feminists basically signed off on doxxing. I would like to see an honest assessment of how various feminist communities reacted.

  23. michaelhuber says:

    I would have a lunch or dinner basically anywhere in San Francisco during that time frame.

  24. I’m currently reading “The Buddha and the Borderline”, and the way the system behaved towards the author, if true, is absolutely appalling.

    It seems that for multiple years, she encountered a combination of incompetence and avoidance, in spite of sincerely seeking all the help she could, and doing the best she could given her emotional dysregulation. The people she approached – including some of her own early therapists – appeared to be incapable of dealing with her problems, or not willing to acknowledge them; and many others she was forced to interact with in the course of dealing with said system, such as petty bureaucrats, appeared actively hostile because of her diagnosis.

    Additionally, they withheld her diagnosis from her for over a decade. Seriously, WTF?

    Is this how most people with ill-understood problems are treated?

    (I have always been disgusted and appalled by the thoughtlessness of the current mental health set-up in dealing with how patients are handled. From what I know, the way the entire system is set up simply presumes that everyone is capable of normal functioning when dealing with it. Hospitals make allowances for the fact that their patients have problems make normal physical functioning impossible, and so have gurneys/stretchers, wheelchairs, and I suspect a number of other such helping devices available; and a lot of thought and care is expended to alleviate and accommodate these impairments with patients’ physical functioning. But the mental healthcare system doesn’t seem to expend even a fraction of this effort in dealing with the problems their patients have, in terms of making how they interact with it.

    I suspect it’s because of a bias with agency – people with a broken foot arent blamed for not being able to walk, because their foot-brokenness is very visible; and they treated this way even if others people occasionally injure their foot. But someone with an underdeveloped emotion-regulation system is treated as someone blameworthy, because the quantitative difference between a brain which has emotion-regulation problems so severe that it impairs someone’s functioning in spite of their efforts, versus someone who’s merely very emotional, is not one normatively ‘normal’ people can understand. This constitutes the vast majority of people, who are incapable of understanding anything unless it’s shoved, visibly and repeatedly, in their face, and unless they’re repeatedly punished by their peer group if they show anything but empathy (or some other appropriate behaviour). (Please note that I’m writing the last few sentences while quite angry, and they probably need to be corrected for tone to state the same thing in more neutral terms, such as salience bias and whatnot. The effect, though, is the same: reality doesn’t care.)

    This is not to denigrate the work the people within it do – because this is a Molochian problem, not an individual one. The behaviour of petty bureaucrats can certainly be improved with incentives, however, as can the methods the patients use to interact with the system, which are amenable to technological solutions.)

    • jaimeastorga2000 says:

      I suspect it’s because of a bias with agency – people with a broken foot arent blamed for not being able to walk, because their foot-brokenness is very visible; and they treated this way even if others people occasionally injure their foot. But someone with an underdeveloped emotion-regulation system is treated as someone blameworthy, because the quantitative difference between a brain which has emotion-regulation problems so severe that it impairs someone’s functioning in spite of their efforts, versus someone who’s merely very emotional, is not one normatively ‘normal’ people can understand.

      A charitable interpretation of the propensity to treat mental disorders as blameworthy while not doing so with bodily ailments is that your brain is you in a more intimate way than your body could ever be. If there is something wrong with your foot, that just means that the biological mini-mecha that hauls your brain around and provides it with a life support system is in need of repair. If there is something wrong with your brain, that means there is something wrong with you.

      Charity aside, though, I don’t think this is how normal people think.

    • Deiseach says:

      Speaking as a petty bureaucrat… and festooning this with caveats, such as I haven’t read the book:

      (1) This is her side of the story and how it appeared to her. Ask the people who were interacting with her, and a different view of the matter might emerge. We have difficult clients, some of whom have genuine psychiatric diagnoses. One of them is convinced the government, and we as their agents, are planting cameras in their house to spy on them. Others of our clients, we informally (that is, nothing is written down on the file, it’s all verbal) warn anyone – our workmen, people going out to do inspections, etc. – not to go alone but always take someone with them as backup and independent witness. Else the client will ring up complaining that ‘your guy attacked me/trashed my house’ and so forth.

      Other clients are violent, criminal or violent and criminal, and it’s for their own protection to have a partner along.

      (2) That being said, the systems we operate under are inflexible. And there is not very damn much a Grade III or IV on the coalface directly dealing with the public can do about that: blame the politicians in government, they’re the ones with the power to change the regulations! Which means, of course, advancing at a glacial pace.

      (3) We’re only human; we get burned-out. You might not believe the amount of scamming out there, the flat-out lying, deceit and cheating, and that includes people faking physical and mental ailments for themselves or their kids to bolster their applications and game the system. You get people manipulating their kids to claim that “little Johnny has asthma”. You get people deliberately making their kids who genuinely do have physical/mental needs worse in order to get more money, bigger housing, etc.

      You get very cynical very fast about people who show up and claim to have ‘special’ problems. You get tired, angry and hostile when someone pitches a fit in your office, even bursting into tears and claiming you’re against them and they just need help, why can’t they get help, they have a right to help; all of which is true, but when this is person No. 55 who’s been trying to guilt-trip you… well, unfortunately we don’t always respond as sensitively as we should do, though my colleagues have what I consider amazing patience and tact dealing with people who are shouting abuse at them and slamming doors so hard they break them.

      (4) And following on from both (2) and (3), we can’t just take your word for it. We’re bound by regulations, we need a consultant’s letter to prove you do have Syndrome/Condition X. If, as you say, her therapists didn’t know how to deal with her and indeed were concealing her diagnosis from her, chances are they wouldn’t give her the necessary documentation. No documentation, no independent proof – no go.

      As I said, anyone can come in and claim on an application “I need special consideration because of my physical/mental difficulties”. The amount of people putting in for social housing because in their current housing their kids are getting chest infections and they have to bring them to the doctor – in rainy, cold Ireland, during the colds and flu season – well, everyone gets chest infections. And has to go to the doctor. And is told in most cases to buy a bottle of cough mixture from the chemist.

      That does not qualify you for social housing.

      (5) We’re only human. I’m repeating this because of the line about reacting with hostility because of the diagnosis. Nobody likes ‘awkward’ customers, partly because they can (understandably) react with anger and demands when they feel they’re getting the runaround, but also partly because there are times you want to help, you’d love to help – but you can’t (see: regulations). And so you feel guilty, and you react defensively, and you convert this into anger which gets projected on the client. It’s not your fault they’re in this situation, it’s their fault for being so difficult to deal with.

    • Salem says:

      It makes perfect sense to me. Blame often works. If someone wants me to walk but I have a broken leg, then yelling at me and calling me worthless won’t get me walking. But if someone wants me to walk but I have depression, then there’s a chance it might.

      • Protagoras says:

        Kind of depends on what your goal is. If you yell at the depressed person to get them to move, sure they may move this time, but then decide on a dozen future occasions to not bother leaving the house because they don’t have the energy to cope with possibly running into people like you. Which may work for you because then you don’t see them, but doesn’t help them much.

    • Jaskologist says:

      FWIW, this happens with physical problems as well. Way back in my youth, I fought a multiyear battle with a bacterial infection, where I would get better for a while but relapse repeatedly. One of my doctors attempted to convince me that I was depressed instead.

      I know somebody else who had idiopathic chronic fatigue, which is even worse. No real visible physical symptoms, no great treatments available either. They don’t even know the cause (“idiopathic” is really just a fancy word for “I don’t know why this is happening”). When doctors don’t really know what to do, they’ll start throwing out any old explanation, including psychological. it took persistence on the part of my parents and theirs to get past that.

      I’m not sure I blame them either; when I’m running tech support for people, my first instinct is also to assume they are doing something wrong, and that does tend to weed out half the cases. Our understanding of medical science is still in its infancy, so there are a lot of cases where doctors just have no idea what is going wrong.

      • “my first instinct is also to assume they are doing something wrong, and that does tend to weed out half the cases. ”

        I’m reminded of Mark Twain’s comment that Christian Scientists know how to cure imaginary diseases, and since half the diseases people suffer from are imaginary that gives them a pretty good cure rate.

    • Anthony says:

      But someone with an underdeveloped emotion-regulation system is treated as someone blameworthy, because the quantitative difference between a brain which has emotion-regulation problems so severe that it impairs someone’s functioning in spite of their efforts, versus someone who’s merely very emotional, is not one normatively ‘normal’ people can understand.

      It’s difficult to understand because there doesn’t seem to be an easily discoverable qualitative difference. At what point are you being overly emotional because you cannot control it versus because you choose not to control it, and how can *I* tell?

  25. Said Achmiz says:

    Scott, please do a post where you present the best arguments against utilitarianism. (There are some really good ones out there — certainly enough to have convinced me — but awareness of them is sadly low in rationalist spaces, and I’d like to see that rectified.)

    • Alexander Stanislaw says:

      https://slatestarcodex.com/2013/04/08/whose-utilitarianism/

      Although, that was a while ago so Scott may have changed his mind.

      • Peter says:

        Gosh, SSC comment threads were so much smaller back then.

      • Anonymous says:

        That one is pretty unsatisfactory, as far as criticisms of utilitarianism go. For one thing, once you stipulate that “everyone’s intuitions are basically utilitarian” (that joking “[citation needed]” notwithstanding), you’re most of the way there… and there are many other issues that post does not address.

        • Said Achmiz says:

          Oh for Pete’s sake. What was the solution to the “cookie monster” bug again…? Do we have one?

          (above Anon is me)

          • Nornagest says:

            If memory serves, the problem is that Scott’s blog, for mysterious reasons, is sending user cookies with a timeout in minutes rather than the days or weeks we’d expect. If it was a WordPress bug we’d expect to see it in other blogs, and AFAIK we don’t, so it’s probably a simple configuration issue — but we need someone with Scott’s access to look into it, and Scott’s not a WordPress developer.

            No idea where it came from originally.

          • Scott Alexander says:

            I tried something someone said would work. We’ll see if it does.

    • My favourite critique is the hapiness machine, where you sit hooked up to a machine that just alters your brain to be happy even though you just sit there, or as its more commonly known… drugs. Basically if you oppose putting everyone on one of these, so that their lives are really crap, but they are really happy, then you probably can’t totally accept utilitarianism. So long as they don’t have a come-down, having a pathetically short life wouldn’t matter, as long as the overall happiness was greater in magnitude (however you’re measuring it, which is the other problem). Kind of makes you wonder, could happiness in fact be a proxy for something else of importance? 🙂

      • Linked List says:

        This is a good criticism of *hedonic* utilitarianism, but that’s just one variant that few people take seriously these days.

        The utilitarianism du jour is Peter Singer’s preference utilitarianism. In your thought experiment, would people be able to rationally choose whether to be hooked to the machine?

        • I think it works as a critique of some other versions too (eudaemonistic or poorly grounded preference satisfaction). My interpretation is that the point is the separation of perception from reality. So say your preference or eudaemonistic goal is that the people you love are leading good lives, the hapiness machine just needs to make you think they’re leading good lives, while meanwhile they could be in misery or dead. It’s basically the concept people used before wireheading 🙂

          However, I think you may be right that it isn’t a critique of versions of preference satisfaction rooted in actual fufilliment of the preference rather than satisfaction of the preference (ie Singer). I can offer my tentative personal criticism of those if you like. I haven’t looked at Singer’s version specifically, but those sort of versions seem absurd to me because (1) the idea of clashing preferences being meaningfully quantified so they can be balanced is ridiculous, especially when usually our only knowledge of preference is through self-reporting and (2) it seems to me that negative sum preferences ought not be considered at all, whereas preference satisfaction requires attributing some weight to them regardless. So suppose a sadist REALLY enjoys other people’s suffering, at least as much us say their anti-enjoyment of it. Why should that preference be valid?

          Preference is a quite volatile and contextually defined part of a person. I also note that people receiving their preference often does not lead to happiness (sometimes even the opposite), especially if their knowledge of their own psychology is limited.

          So in my opinion preference is a problematic thing to ground consequentialism in. Though I haven’t assessed Singer’s version properly and would be open to someone changing my mind (if you’d like to try I’d actually welcome it).

  26. Matthew says:

    I refuse to get a Tumblr, so I’m going to address this ( http://slatestarscratchpad.tumblr.com/post/109063021426/iamamaiden-last-snowfall-amroyounes ) here…

    While I’m generally sympathetic to this point of view, the second picture is a wildly dishonest approach to demonstrating The Better Angels of Our Nature. You don’t get to compare the current decade of war casualties with the worst decade for war casualties in history and declare victory. All the decades before the 1940s also involved fewer people dying in war. This is about as helpful as climate arguments that conveniently start in 1998.

    (Note — I agree that violence has declined spectacularly. I just object to blatant statistical cherry-picking to demonstrate it.)

    • Noah Siegel says:

      My takeaway was “Look! We overcame the awfulness of the 1940s,” not, “the difference between the 1940s and now itself proves a long-term trend.”

  27. Ahilan Nagendram says:

    Scott, I’d like to know your input on this. When it comes to the politically fraught fields of psychometrics and behavior genetics, and especially IQ and its correlation with race, is it only possible for one to be egalitarian on the basis of religion? I think the science has been settled since Jensen and Rushton, and now with Plomin’s big data GCTA studies, that IQ is definitely correlated with things like race, SES, and a strong argument can be made for causation from those factors. Now, the science of the matter definitely doesn’t support egalitarianism, so how would egalitarians justify their beliefs?

    • Anthony says:

      Moral egalitarianism does not depend on actual equality between humans or groups of humans (for some values of “moral egalitarianism”). Even if, for example, all races were equal in IQ distribution (or you were in an isolated community with no visibly separate populations), there would still be plenty of very dumb and very smart people; moral egalitarianism insists that you treat them both as human beings with certain rights.

      For moral egalitarians, there are two issues – when is it legitimate to make judgments which consider the ways in which humans are actually unequal, and does that boundary shift when the impacts of those judgments will have a significantly disparate impact on identifiably different subgroups. The first issue always exists, even in “homogeneous” groups.

    • Jon Gunnarsson says:

      What do you mean by egalitarianism? The word can have many different meanings. Here’s at least six different ones.

      1) All people are as a matter of fact equal in some sense.
      2) All people would as a matter of fact be equal if it weren’t for some interferring factors.
      3) All people should be treated equally by the law/government.
      4) All people should be treated equally by society.
      5) All people should have equal opportunities one some dimensions.
      6) All people should have equal results on some dimensions.

    • Scott Alexander says:

      I didn’t think Plomin had produced evidence of a race-IQ correlation – his work could be dragged into that domain, but only with some loss. He found that genes and IQ were correlated within races, but that doesn’t necessarily mean the racial differences are genetic. Just to give an example, within 1940s-America and 1940s-Japan most height differences were probably genetic, but the height difference between 1940s-America and 1940s-Japan was mostly dietary.

      I go back and forth on the race-IQ thing, but mostly forth. I don’t think it’s quite at the point where one side needs to be labelled denialists.

      • Ahilan Nagendram says:

        Would you say Scott, that there’s any data that’ll either fully convince you of the Race and IQ hereditarian position (as I am) or conversely of the environmentalist position? I think it’s not too hard to be completely convinced of the hereditarian position since it’s so ubiquitous on the internet, as part of HBD.

        Wouldn’t you also agree that the environmentalists deny a lot of the evidence that hereditarians put forward? Also consider that the opposite basically never happens.

        • Scott Alexander says:

          The hereditarians don’t deny any of the environmentalists’ evidence? As far as I can tell they do practically nothing else – unless you have a stronger meaning of “deny” like “deny that it exists without responding to or addressing it”, in which case this is going to start looking a lot like the same kind of value judgments around debunking.

          • Ahilan Nagendram says:

            I didn’t explicitly define “deny” as that, but if we run with it, wouldn’t it seem that denialism was what the environmentalists have always done vis-a-vis hereditarian arguments and data?

          • Scott Alexander says:

            Doesn’t “denialism” just mean “they disagree with something I’m really really sure about”?

          • Nornagest says:

            Doesn’t “denialism” just mean “they disagree with something I’m really really sure about”?

            I think it’s more along the lines of “they disagree with me along an axis I feel socially entitled to call them a freak for”. Pro-lifers may be really really sure that fetuses are people, but they don’t call pro-choicers “murder denialists”, because the personhood of fetuses in the broader societal context is controversial and everyone knows it.

            Whereas for something like climate change, the pro-warming side has already won by its own lights.

          • Ahilan Nagendram says:

            It’s more like they deny something that’s been well established, whether it’s out of politics or whatever.

          • Deiseach says:

            Doesn’t “denialism” just mean “they disagree with something I’m really really sure about”?

            No, that would simply be “disagreeing”. “Denialism” has a raft of implied meaning: you are denying, that is, saying something that is so is not so; you are a bad person, because only bad people refuse to believe the obvious truth; the truth of the matter is so obvious it is not possible to disagree in good faith so the only recourse is acceptance or rejection.

            And rejection is denying is denialism. If you look at the context in which it is used, and the tone of the debates, accusing someone of denialism is (a) definitely accusing (b) not using a value-neutral term but one that is derogatory and loaded with bad associations.

            A climate denialist is a right-wing homophobic racist sexist classist capitalist who personally clubs baby seals to death, wears a coat made out of puppy skin, and wants the poor to die of easily treatable health conditions rather than permit subsidised health care, all this while he twirls his villain’s moustache as he burns hundred dollar bills to light his cigar 🙂

          • John Schilling says:

            “Denialism”, when preceded by the word “Holocaust”, refers to people who are actually Nazis or Nazi sympathizers but don’t want to openly argue the Nazi cause.

            “Denialism”, when preceded by any other word, usually refers to people whom the speaker wants his audience to believe are the moral and intellectual equivalent of HDers and thus Nazis, but the speaker doesn’t want to openly argue his case.

            I do not believe it is possible to rehabilitate the term for use in objective rational debate. Certainly when I hear the non-holocaust version of the term, I suspect that the target might not be amenable to rational debate on the subject but that the speaker almost certainly isn’t.

          • Paul Torek says:

            Scott, you’ve almost got it. “Denialism” means “they heard my argument and yet refused to be convinced by its sterling rational awesomeness.” Which, let me be clear, can be a fair complaint. Occasionally.

      • Wrong Species says:

        Doesn’t the dietary thing mostly apply to people in absolute poverty rather than relative poverty? I’m sure black people in America don’t eat as well as white people but it seems a bit of stretch to say that not having the optimal diet(rather than being malnourished) causes such big differences in IQ.

        • Ahilan Nagendram says:

          Actually there definitely isn’t good evidence that dietary differences are causative of large IQ diffs when it comes to modern populations. Shared environment effects, or even non-shared effects, are minimal.

          • haishan says:

            I’m worried that this argument might prove too much. In particular, I’ve never seen any evidence of shared environment effects in populations where we know there must have been shared environment effects — for instance, in times and places with incomplete salt iodization efforts, or (maybe?) during the transition away from leaded gasoline. I acknowledge that this may well be due to a lack of data, but it’s distressingly easy to imagine the JayMan of 1935 going “Shared environment effects on IQ are minimal! There aren’t any large-scale interventions to be done!”

        • Nornagest says:

          Scott doesn’t seem to be making any assertions about where any possible environmental causes of differences in IQ come from. He’s drawing an analogy from the existence of an environmental cause (diet) of differences in height between the US and Japan in the Forties, but the specific environmental cause needn’t be identical.

          • Ahilan Nagendram says:

            In that case I’ll let Scott speak to whether he believes there are any environmental factors to IQ diffs between blacks and whites in America.

        • Scott Alexander says:

          I wasn’t actually saying it was dietary, I was giving an example of something where genetics explains most within-group variation but environment explains most between-groups variation.

          • Ahilan Nagendram says:

            Do you personally believe there might be environmental effects on IQ differences?

          • Irenist says:

            Do you personally believe there might be environmental effects on IQ differences?

            Hmm. Presence of lead, absence of iodine? Stuff like that seems obviously environmental, yes?

      • I’m not in favor of labeling anyone “denialist.” But I think the claim “we know that innate (genetic) IQ does not correlate with race” is indefensible, in part because proving a negative is hard. The strongest claim that is defensible, as far as I can tell, is “there is some evidence that IQ correlates with race, but there are alternative explanations for it, so the conclusion might be false.”

        That matters, because one of the ways the issue comes into actual political/ideological argument is through the claim that differences in outcome by race prove discrimination. That claim depends on the assumption that differences in relevant innate traits don’t correlate with race. It isn’t enough to say that we don’t know if they do.

    • RCF says:

      “is it only possible for one to be egalitarian on the basis of religion?”

      When I first read this, I thought you meant “teat people of different religions the same”, but apparently you mean “treat people the same because your religion says to”.

      “Now, the science of the matter definitely doesn’t support egalitarianism”

      Something I’d really like to see disappear: people using the phrase “science supports” for anything other than an empirical proposition.

      “so how would egalitarians justify their beliefs?”

      So if science doesn’t support something, the only thing you can think of is science? Science and religion are the only bases for one’s actions that one can think of?

  28. Stella says:

    The Hyde and Mertz paper is interesting, though the third point “Do females exist who possess profound mathematical talent?” seems kinda strawmanny. Certainly we know that general intelligence can be affected by culture/environment to at least some extent, so it makes sense that relative mental abilities among groups can also change. I’m still not convinced this is *entirely* non-biological.

    To me, the important questions are:

    1) Is have different proportions of genders in different jobs inherently immoral?

    2) Are the causes of these differing proportions immoral?

    If the answer to number 1 is “yes,” then it doesn’t really matter what the answer to #2 is. Even if the cause is morally neutral–hormones in utero or after, for example–we would still want to make an effort to equalize the genders in every profession, perhaps through hormone therapy.

    If the answer to #1 is “No, gender differences in career makeup are not inherently immoral (this is my position) question #2 becomes relevant. If the cause is something immoral–individual women/men are forbidden or actively pushed out of a career they would be better suited to–or factually wrong–it’s believed women’s heads will overheat from doing calculus–then we should do everything in our power to try to stop it.

    On the other hand, if the reason is itself morally neutral, then I don’t think we need to worry. For example, if playing princess dress-up causes you to become a psychologist when you grow up while playing with Leggos causes you to become an engineer (something I don’t believe, but which makes a simple example), then I don’t think we should feel compelled to do anything about it, because playing with Leggos is not morally superior to playing princess dress-up, and being an engineer is not morally superior to being a psychologist.

    • grendelkhan says:

      because playing with Leggos is not morally superior to playing princess dress-up, and being an engineer is not morally superior to being a psychologist.

      Is this really the issue? The complaints seem to have more to do with compensation and prestige. Being a rockstar isn’t morally superior to being a dustman, but it sure as hell pays more.

      • Nornagest says:

        There often seem to be elements of just-world thinking in disputes like this one. Compare the (common, but only tangentially gendered) assertion that teachers should be paid as much as $PROFESSIONAL because of $YAY_CHILDREN.

        (There is a more sophisticated argument that says teachers should be paid high salaries because that would encourage talented people into the role, which would have positive downstream effects on children’s educational development etc. That is not what I’m talking about.)

        • RCF says:

          “Just world” is commonly used to refer to the idea that one starts with the premise that the world is just, and concludes that what happened must be right. So in the case of teachers, it would conclude from the fact that teachers are paid less that teachers deserve to be paid less.

      • Stella says:

        I may be using “morally” weirdly. I don’t mean whether it’s moral for a particular person to be this or that. I mean more whether it’s moral on a society-scale for any particular job to be distributed any particular away. “Just” is probably a better word.

        So for example: I considered being either a librarian or an engineer. I ended up being an engineer, but I still think I would have been perfectly happy making the other choice. I don’t think it would have been somehow a sign of society’s injustice if I had chosen to be a librarian, as long as I wasn’t driven to librarianing through unjust means. This holds even though an engineer gets more money (definitely) and prestige (sorta). Money and prestige are too few variables to demonstrate justice or the lack thereof.

  29. SUT says:

    Thought experiment: If civilization was wiped out – all scientific information destroyed, all religion lost – and humans started from scratch, what would science and religion look like the second time around? It’s clear science would come back the same just by applying the scientific method

    I also think religion would come back with many of the same structural elements and values. The details would vary – e.g. God might be presumed to take 10 days to have created the universe (instead of 7) – and its the focus on knocking down these scriptural strawmen, while failing to appreciate the grander points being made that irks me about “the common atheist” arguments on these subjects.

    • stillnotking says:

      I’m not confident science would come back “the same”. Science is not a fixed institution with a fixed philosophy, so it’s not clear what “the same” actually means. Empiricism in some form would probably come back, but would it include, say, Popperian falsificationism? Maybe, maybe not.

      I’m even less confident that monotheism would be ascendant after a reboot. I see no good reason to think its current popularity is anything but a consequence of Abrahamic traditions being well-positioned to ride the wave of civilizational development in the Levant. One can, in fact, point to single events, like the conversion of Constantine, that seem to have shaped the world’s religious history in completely contingent ways.

      • Brad says:

        I mean, thought, doesn’t this view assume that history is basically a series of random accidents, i.e. a rationalist view of history?

        I mean, here’s the question: Assume a certain religious belief is in fact, “really how the world works”; wouldn’t that inform both A: why things turned out the way they did and B: how they would turn out if we got hit with a tabula rasa state? i.e. monotheism views history (I speak generally) as a series of predestined narrative occurrences. I can’t speak as well regarding the religions of others, however.

    • Kiya says:

      This experiment has already been done: a bunch of early humans spread out all over the world and formed cultures complete with science and religion. Then we all got back together and compared notes. Sure, we didn’t particularly enforce cultural isolation during the development stage, but geography did a pretty good job.

      If you look at a bunch of ancient cultures, monotheism’s pretty rare. I can happily generate a lot of creation myths more unusual than “God created the world in 7 or 10 or 312 days,” and so did the Greeks, Egyptians, etc… it helps to have more characters. Today, if I believe Wikipedia, a little over half of people are some sort of Abrahamic monotheist. My hypothesis to explain this is that Christian and Muslim empires insisted on their subjects converting, whereas polytheist empires tolerated and/or syncretized worship of local gods. This might be a pattern that would happen again. Or it might not.

      It seems plausible that human values are held more deeply than the particular stories we tell, and would survive a reboot. Cross-cultural values analysis is not something I think I can do justice to with a quick google search. But the fact that we have a reasonably functioning global society where wars to exterminate the infidels are the exception rather than the rule suggests that we share at least a lot of the most important values. No country is trying to turn us all into paperclips.

      The underlying facts of the universe that science aims to discover would be the same. But the order in which they got discovered, and hence the way we think about them, might be very different. I’d be interested to see how the reboot civilization teaches its kids about atoms.

      • Jaskologist says:

        The experiment gives us similar results for science. The different cultures did not end up with a common understanding of how the physical world works. Greeks would have told you there were 4 elements, Chinese would have said 5. Check out all the different theories of atoms, and that’s not accounting for the fact that many argued against atomism itself.

        If by “science” you mean “empiricism, the scientific method, etc” then you really just mean “Western science.” The rest of the world did not discover it, they copied it from the West. At that point the experiment is pretty much over, although it may be interesting to look at what happened to science behind the Iron Curtain, with Lysenkoism and all that jazz.

        • Kiya says:

          Yeah, the problem with science before 500-ish years ago is that it’s either very practical and quickly spreads across continents (“those guys know how to smelt iron! we’d better learn how too or they might kill us with swords”) or it’s more of a philosophy / wild guess that may happen to describe the world in retrospect when it later becomes checkable (“stuff is made of indivisible units with the essential character of that stuff, because it seems like it ought to be”). The best parallel scientific development I can think of is agriculture, invented independently in the Middle East and Central America and maybe other places. We take it for granted now, but it’s pretty revolutionary if you’re a hunter-gatherer. Also, not uncorrelated, the calendar.

          • Izaak Weiss says:

            Writing also should be mentioned as being invented in multiple places.

          • Jaskologist says:

            There is also an important distinction to be drawn between “science,” which is a methodology, and “technology,” which science aims to produce, but can be arrived at through other means. Every advanced society achieved some level of technology, but only the West came up with science.

            But even the technologies that we share vary. You mention “agriculture,” but that covers a lot of very different techniques, which is not surprising since people were dealing with different climates and plants. Farming wheat is not like farming rice. Japanese swords are not like European swords. Asian architecture is different from European architecture, even if they do all have to follow many of the same physical laws. Heck, the two continents don’t even use the same eating utensils.

            Even math, which is the same everywhere and always, doesn’t seem to have been built out independently by anybody, with the exception of Liebniz and Newton. Instead, somebody figures something out and everybody else ends up copying it. And even there, for a long time math stopped at algebra for everybody but the West. I can even think of at least one differing approach to math, in the form of the horrible Roman numeral system.

      • drunkenrabbit says:

        “Pure” monotheism is pretty rare, but semi-monotheism does seem to repeatedly pop up when you have a large group of people writing/philosophizing about religion.

        In the ancient Hellenic world, you had Stoicism and Neoplatonism postulating a “prime mover”, in Egypt there was Atenism and the later idea that all gods were really just manifestations of Amun-Ra, in Persia traditional Persian polytheism morphed into Zoroastrianism, and in Hinduism the “highest reality” of Brahman. I think the Confucian “Tian” might be analogous, not sure.

        Either way, it seems for it to be fairly common to have large pantheons that are worshiped popularly, with theologians believing them to be part of some great, singular, divine force or entity.

    • lmm says:

      But as an atheist, these “details” – the trivialities like banning people from eating particular foods, or having particular medical procedures, or taking particular drugs, or working on particular days – are precisely the things that cause me the most trouble! If people want to love their neighbour, hold community gatherings, give food to the hungry and so on, great! If they want to spend hours arguing over the minutiae of their holy books, that’s a bit of a waste but it’s no skin off my nose. The things I most want to stop about religion are – well, on the large scale, I guess I view their entrenched power hierarchy, inherent conservatism, and emphasis on faith over reason as intrinsically bad, but those are rather abstract concerns. The concrete problems of religion are when people push stupid policies for religious reasons.

  30. Anonymous says:

    Are there growing rationality communities/meetups in Wellington, New Zealand?

    LessWrong only has one entry for New Zealand, and thats Christchurch (completely different island from Wellington). I read so many positive experiences about the meetups, but due to location I’m unable to participate.

  31. Pseudonymous Platypus says:

    Does anyone here have experience with/comments on “voice dialogue” therapy? I can’t find any information about it that seems likely to be reliable and/or unbiased.

    • Paul Torek says:

      My wife is a practicing psychotherapist, knows it, uses it, and is impressed. I can get more details if you wish.

      • Pseudonymous Platypus says:

        I would appreciate that, yes. I’d like to know the following things:

        – A brief overview of the practice and the supposed underlying mechanisms, from a source other than the people who invented it
        – Whether or not there has been any scientific study of its efficacy (I can’t find any)
        – How it compares to CBT and why one would use it instead of CBT
        – Whether or not it is more effective for particular issues or types of patients than others

        Thanks!

  32. Waffles says:

    Imagine you wake up one day on an alternate Earth where everyone collects baseball cards. Baseball cards for successful teams trade for hundreds of dollars; successful players on successful teams, thousands of dollars. If a team does worse than expected, their cards may lose value. But on average, the total value of all baseball cards has been increasing every year.

    You go to your financial advisor and she says: “You should invest at least half your savings in baseball cards. Demand keeps going up, and their value is increasing steadily!”

    And you say: “– Well, but these baseball cards have no intrinsic value, aside from their rarity. If everyone woke up tomorrow and decided they didn’t like baseball cards any more, these cards would all be worthless. That’s sort of the definition of an investment bubble, isn’t it?”

    And your financial advisor says: “Well, technically ‘dollars’ don’t have any intrinsic value either. But you don’t seem to mind getting paid in those…”

    This is how I feel about the stock market.

    One example is Google stock. Google has three classes of stock: A, B, and C. “Class B” stock represents more than 51% of voting power, but you can’t buy any of it; it’s only held by Google founders, the CEO, et cetera. “Class C” has explicitly no voting power at all. None of the classes of Google stock has ever paid dividends.

    Now, then: what is the worth of a Class C share of Google stock? You can’t vote with it, and you won’t get money for holding it. Essentially it’s a baseball card. It’s a baseball card which is valued currently at five hundred dollars per share, with a market cap of 363 billion dollars.

    Of course there are other stocks, which do pay dividends. Doing some math, the dividend rate from Microsoft stock seems to be 2.7%. I own some stock in a utility; the utility appears to pay 4.1% dividends. I went and looked up the US Treasury Bond interest rate and, assuming I’m doing my math correctly, it’s 4.8%. So (going by these examples) owning stock seems to be flat-out worse than owning bonds.

    I’ve long suspected that the stock market is basically bullshit.

    And yet if you talk to any financial advisor they tell you: “Invest in stocks! The price will go up over time!” And, let’s be frank, _they will be correct_. The price of stock does go up over time. It’s a bubble which hasn’t stopped rising.

    So I guess here is my question. For the specific case of Google Class C stock, which explicitly has zero voting power and zero dividends: why does this have a monetary value? Why hasn’t the stock market taken one look and said: “No thanks, I don’t want your baseball cards.”? What does the stock market know (or think it knows) that I don’t?

    • Anonymous says:

      Yes, Google stock is mysterious.

      But your complaint about the low dividend rates of stocks are ridiculous. You have pointed out the remaining value: control. If the stock ever becomes too cheap, someone could take control of the company and, for example, change the dividend rate. or liquidate it. This happens all the time.

    • FullMeta_Rationalist says:

      I think your appeal to fiat is correct: a class C google share is a $500 bill. Just try not to spend it all in one place.

    • Nita says:

      “Intrinsic value”? “Worth”? Supply and demand not good enough for you?

    • moridinamael says:

      Google could at some point decide to start paying a dividend. That means owning the stock is more like a fungible lottery ticket than a dollar bill.

      There could also be a stock buyback at a higher price. There could also be other events I don’t know about because I’m not an expert.

      Regarding the statement that the stock market is bullshit, I used to feel similarly but the closer I’ve gotten to the finance world, the more I feel that it’s actually the core of the modern economic miracle.

      • John Schilling says:

        This. Stock value is closely tied to the market’s perception of a company’s intrinsic value, including present assets, present liabilities, ongoing income, and expected future income. In the future, any corporation will necessarily A: reach the limit of growth in its available markets and return every penny of net income to its stockholders in the form of dividends, B: be bought out by another corporation that wants its assets and/or future income badly enough to pay an appropriate amount of cash to the stockholders, or C: be liquidated and have its remaining assets sold at auction, any net proceeds to be distributed among the stockholders. One way or another, the stockholders collectively get what the company is worth – though individual investors may get more or less depending on when they buy and sell.

        In the case of a very stable, very mature company with no future potential for growth, you’d expect the dividend yield to be comparable to a government bond, though not identical due to liquidity issues. For a growing company, the dividend yield will be lower than government bond yields because you’re paying not just for current dividends based on current revenues but for the reasonable expectation that the revenues and dividends will increase in the way that government bonds don’t. For a fast-growing company it is perfectly reasonable for there to be no dividends as the company reinvests all of its earnings to support growth.

        And yes, this is economically miraculous, in that it allows concentration of resources across time and space in a way that only kings and princes could manage in earlier eras. If you have a business where Plan A requires modest investment and gives modest profits in five years, whereas Plan B requires more up-front money, takes twenty years to reach fruition, and transforms the world while making mountains of pure wealth, the fact that none of your investors are more than moderately rich and none of them are willing to wait more than five years does not lock you in to Plan A.

        To the extent that today’s investors are confident that a mountain of wealth will appear in twenty years, they will be confident that investors fifteen years from now will be willing to offer up maybe 0.7 mountains of wealth for a secure 7%/5year return that they will be around to realize. Thus confident that there will be 0.5 mountains of wealth being bid in ten years, and 0.35 mountains of wealth in five years, so you’ll get plenty of moderately-wealthy short to mid-term investors here and now combining their resources to finance your project to the tune of 0.25 ginormous mountains of up-front capital investment.

        • Irenist says:

          @John Schilling: This is by far my favorite comment in the whole thread: radiantly lucid.

        • Scott Alexander says:

          I still don’t understand. If Google’s not paying dividends now, what incentive do they have to do so in the future? And if stockholders don’t vote, what incentive does a company that wants to buy them out have to bribe them?

          (also, Google’s market right now is “the internet, cars, space, biology, AI, cell phones, and some laptops”. If they ever reach the limit of growth in that market, I don’t know what to think.)

          • Anonymous says:

            Not only does google not pay dividends, but their IPO swore never to pay. There certainly is an expectation that when the founders die that control will fragment and the company may change. But I don’t think that plausibly matches a real calculation of value of the shares.

          • Pseudonymous Platypus says:

            I’m fairly certain the only market Google actually makes money on right now is search/ads. The others are all bets for the future, but they’re not certain by any means.

          • John Schilling says:

            When Google reaches the point where it can’t or won’t grow any more, what else is it going to do with its revenues but deliver the profits to the stockholders in the form of dividends (or stock buybacks or the like)? Stashing the money in a perpetually-growing bank account that can never be touched, benefits nobody. Keeping the money for the private benefit of Google executives, violates the fiduciary responsibility of those executives and will get them sued or imprisoned. The same charter that says “founders get 51% of the voting power”, also says “if the founders take any money out of the kitty for themselves, they have to distribute money to all the other stockholders in proportion to their holdings”.

            So, yeah, the class B/C stockholders are voting that at some point Google’s class A stockholders will vote to actually make full[*] use of their vast wealth rather than just point to a giant bank-account balance that they aren’t touching as it grows towards infinity. Seems like a safe bet. And as noted above, you don’t have to actually wait until they decide to cash in, to do so yourself.

            As for the limits to Google’s growth:

            1. At some point, management overhead will likely make further growth impractical for the same reason that e.g. the Soviet economy didn’t work. Assuming Google isn’t run by idiots, that’s when they start paying dividends rather than buying more stuff for Larry and Sergey to manage.

            2. Larry and Sergey aren’t immortal, and when they die it isn’t clear that the new/surviving class A shareholders will be unanimous in pursuing the Eternal Growth of Google over their various private interests.

            3. There will also come a point at which the US and EU governments decide that Google is too damn big and invoke antitrust laws to break up the company. If necessary by electing Roosevelt-style (either one) populists.

            4. Barring 1-3, and assuming they aren’t overtaken by Apple, Google winds up owning everything. In that case, Google stock becomes functionally equivalent to a title of nobility in the Great Solar Empire of Google. I, for one, welcome our new Google overlords, and that’s Sir Class C Shareholder to you 🙂

            [*] They can make partial use of their wealth by selling some of their own class B/C stock on the open market, which also tends to anchor the value in the market.

          • RCF says:

            “And if stockholders don’t vote, what incentive does a company that wants to buy them out have to bribe them?”

            Suppose you own a house worth $500,000, and you have a $200,000 mortgage on it. Can you just go off and sell it, keep the $500,000, and tell the bank to screw themselves?

          • Secretariat says:

            @John Schilling seems mostly right but he has class B and class A swapped.
            Class A is the 1x voting stock and trades as GOOGL, class B is the 10x voting stock and does not trade publicly, class C is the 0x voting stock and trades as GOOG.

            When a class B share is sold it automatically converts to class A. When a class B shareholder sells a class C share he [are their any class B holders besides the triumvirate?] also sell one class B share or convert it to class A.

          • John Schilling says:

            Sorry, my bad. I was using a generic model of how publicly-traded companies with privileged insiders set things up, when I should have, er, googled the specifics for Google.

          • Cadmium says:

            And Scott – it’s my understanding that “non-voting” shareholders actually do get to vote on “fundamental changes” such as a takeover bid. (And in any event, the entity taking over needs to actually buy the shares in order to do so. The mortgage analogy may be helpful, but I’d prefer to distinguish between debt and equity. ) To be fair, I only know Canadian law and not Delaware law.

    • SUT says:

      A large share of societal wealth is managed by funds whose job is to ride the coat-tails of the economy as a whole as it grows – e.g. CALPERS.

      In this context, Google is particularly potent organization as they have and continue to represent the “threat” to greatly enhance productivity in many aspects of the economy. It’s important not to under-estimate the threat of innovation: even if you held true voting shares in print media, how’s your investment doing?

      Like a high profile celebrity that can set the terms of an interview with a journalist, Google is also in a better position than most companies to tell public investors to “take it or leave it” and this is probably what justifies the terms of their equity classes.

      Another mitigating factor is that Google’s value is mainly outside tangible assets that can be liquidated. The act of trying to “take the money and run” by privileged stock class would be lose-lose. So non-voting common stock can be assured if you assume rational self-interest by voting members. In fact, eliminating noise of activist investors vying for takeover could make the company even more valuable.

    • Jon Gunnarsson says:

      I went and looked up the US Treasury Bond interest rate and, assuming I’m doing my math correctly, it’s 4.8%.

      That’s way off: http://www.treasury.gov/resource-center/data-chart-center/interest-rates/Pages/Historic-Yield-Data-Visualization.aspx

    • Macbi says:

      A stock that pays $x every year will be worth more-or-less the same as one that pays $nx every n years. Now let n tend to infinity.

      • Good Burning Plastic says:

        Only if n is small compared to the inverse of the interest rate.

        • Oscar_Cunningham says:

          The money Google doesn’t pay out in dividends it invests in itself (or at least it should). Money invested in businesses usually grows faster than interest, so we would expect the share price to rise with n.

    • lmm says:

      When I read the first few paragraphs I thought this was going to be about bitcoin. Thankfully Google’s value is more apparent.

      The simple argument is conservation of value. Google’s assets are worth x (and given that most of them are future cash flows, reasonable people can disagree about exactly how much they’re worth, but still, they’re worth something), their liabilities are worth y; if their stock isn’t worth x-y, then where has the value gone? I mean sure, Google have said they’re never going to pay dividends – but if you bury a ton of gold in your back yard and say you’re never going to dig it up, your yard is still worth a lot of money.

      Google’s founders do have a fiduciary duty to act in their shareholders’ interests. The courts give directors a lot of latitude to use their own judgement – they can take views like “the short-term profit of selling in China is not worth the long-term reputational damage” or “everyone will want to use Google+, we should spend lots of money on it” – but they have to be making a good-faith effort. And if they ever sell for cash, they’re legally obliged to sell to the highest bidder.

      The reasons companies don’t pay dividends these days are mostly shenanigans; in particular tax laws make it (loosely speaking) more efficient for companies to return money to shareholders through buybacks rather than dividends. For people investing for retirement, it’s often more convenient if their stock simply appreciates in value rather than paying an annual dividend that’s fiddly to convert back into more stock (for people who’ve already retired, the opposite is true – but that’s why there’s a market for both kinds of stock). And for tech companies in particular there’s a signalling motive; paying dividends indicates that your company can’t figure out how to spend that money on growth.

      But honestly you’re jumping to the hardest case first – imagine trying to explain to an 8th century merchant, to whom wealth is silver bullion, why the plastic card in your wallet let you obtain goods, without first going through the case for bullion coins, non-bullion coins, paper money, cheques and so on. If you first internalize that US government bonds are about as good as cash, McDonalds bonds are about as good as US government bonds, McDonalds shares are about as good as McDonalds bonds, then the value of Google stock will be much less foreign.

      • Nornagest says:

        imagine trying to explain to an 8th century merchant, to whom wealth is silver bullion, why the plastic card in your wallet let you obtain goods, without first going through the case for bullion coins, non-bullion coins, paper money, cheques and so on

        Challenge accepted.

        “Much of my wealth, and that of the merchants I deal with, is stored in the treasuries of institutions called banks, each as secure and trustworthy as a temple or a great lord’s house. This card is a token signifying my approval, like the ring used to impress the wax seal of a letter; when we wish to do business, the merchant’s device writes a message giving the value I owe and the bank to send it to, I use my card to seal it, and the merchant sends my message on wings of light to my bank.

        Because I used my card, my bank knows I approve a transfer of wealth; it follows the instructions in the message and transfers some of my wealth to the merchant’s bank, which then sends him a message acknowledging receipt. In this way we do business without our hands touching silver.”

        I imagine fractional reserve banking would be a harder sell.

        • Salem says:

          A good attempt, but unfortunately it’s not true. There’s no actual wealth stored in the treasuries of the banks, at least not in any form the 8th century merchant would recognise. The kind of banking you describe would not be too unfamiliar to an 8th century merchant – it seems to have existed in Ptolemaic Egypt, for example. But in fact, the treasury of the bank just has bits of paper and cheap metals in it.

          • Nornagest says:

            I’m trying to explain how credit cards work, not the entire basis of a modern economy. He might assume that our banks are exchanging silver instead of bits, but that really doesn’t matter to the question as stated.

  33. grendelkhan says:

    Is there any chance we could get a research post about things like school choice for primary and secondary education? I’m vaguely aware that it’s a contentious issue where everyone seems to have their own facts. (“School choice increases student achievement!” “Michelle Rhee cheated on test scores!” “It only helps because they can filter out the problem students!” “Looting!” “Rubber rooms!” “Khan Academy is a scheme to fire all the teachers!” And so on.)

    I ask because you’ve made sense out of things like “does AA work?” and “is the criminal justice system racist?”, and even if the answers are full of uncertainty, I feel like I’m less confused after reading them. Thank you for the research you’ve done. I’d be ever so obliged if you’d look into this.

    • Oh, this. Pretty pretty please?

    • zz says:

      School choice is hard to measure because there’s no obvious metric. Standardized test scores? Increased chance of being successful at the next level up? Income once you’ve left school? Subjective happiness once you’ve left school? Differential reproductive success? Future charity donations?

      If you’re me and your metric of choice is amt_learned, however, school choice is the wrong thing to look at because all schools just kinda suck at this; even if the best school teaches twice as much as the worst, that increase is pretty small. Bullet points:

      -Students who take an introductory mechanics course don’t do any better after the class on a test of understanding fundamental concepts like force (as measured by the Force Concept Inventory). This patterns holds in schools from community colleges to Harvard.

      -Ditto for economics. The Economic Naturalist: “When students are given tests designed to probe their knowledge of basic economics six months after taking the course, they do not perform significantly better than others who never took an introductory course. This is scandalous.” (h/t Mike2)

      Academically Adrift details academic research that suggests that, even though most colleges claim to teach critical thinking, students do no better on tests of critical thinking after several years of college, citing an implicit contract between instructors and students where the instructor gives an easy course and students give good ratings. (IIRC (p = .6), the senior year of college is helpful, possibly because students have finally cleared enough prerequisites to get into courses that really force careful thinking and possibly because students are in classes for their major and are less interested getting good grades in a class that doesn’t teach anything.)

      -“Studies of children in Brazil, who helped support their families by roaming the streets selling roasted peanuts and coconuts, showed that the children routinely solved complex problems in their heads to calculate a bill or make change. When cognitive scientists presented the children with the very same problem, however, this time with pen and paper, they stumbled. A 12-year-old boy who accurately computed the price of four coconuts at 35 cruzeiros each was later given the problem on paper. Incorrectly using the multiplication method he was taught in school, he came up with the wrong answer.” (The whole article is fantastic and I fully recommend it).

      SA on uschooling: students who have 0 school only wind up one year behind their peers who attend school full-time.

      -Duolingo commissioned a third-party study, comparing itself to traditional courses. The traditional university course took about 4 times the time to learn the same material.

      -I’ve taken courses at 2 community colleges, 1 mid/high-tier private university, 1 mid/high-tier research public university, and 1–3 Ivy League schools (depending on whether you count OCW or MOOCs). Their courses aren’t tremendously different (my community college professor pulled questions off the 8.01 exams from MIT, which were the easiest ones on the exam).

      -If you know about forgetting curves and then look at how basically every school is structured, you’d be surprised if students managed to retain anything.

      • grendelkhan says:

        Thank you for the links; those look fascinating.

        It looks like the answer is something like “no methods of education really work anyway“, which prompts the question of what the heck we’re paying all those teachers to do, since they cost more than babysitters. (More cynical folks like John Taylor Gatto say that it’s really about teaching people to obey authority and handle boredom, essentially to be good factory workers. I guess we’re pretty okay at that?)

        It sounds a bit like the FizzBuzz problem in programming; in short, a significant portion of people with four-year CS degrees can’t solve a just-barely-nontrivial coding puzzle.

        I remember reading about Bloom’s 2-Sigma Problem and thinking, well, that explains why homeschoolers do so well; in fact, it’s a shock that they don’t do better.

        This seems like a really important problem, one that we should know how to solve by now. Education is touted as the solution to a whole host of troubles, and well-educated people have obviously better outcomes. Is domain-specific knowledge really that hard to teach? Does it matter so little? Are people so bad at learning it?

        Hell, how do we, as grownups, know anything? How do people competently do their jobs? And what the hell are we doing with 5.4% of our GDP?

        (Addendum: this is infuriating. In the pursuit of the ideal that all children are equally valuable, we spend way, way more on disabled kids than on gifted ones, even when a tiny investment would have huge payoffs later on. Aargh.)

        • Princess Stargirl says:

          Is Bloom 2-sigma contested? It seems like a very strong result. Even a 1-sigma result would be shocking if it was stably found under a reasonable variety of conditions.

        • Rich R says:

          I found Bryan Caplan’s model where education is about signalling, rather than learning, clarified a lot of things. Education is poorly optimized for learning because everyone involved is primarily optimizing for something else. Students want the best credential for the least work. Teachers want to get paid (which, in postsecondary education, means getting good reviews) with the least amount of work. Controllers-of-pursestrings (e.g. taxpayers) spend an awful lot of time arguing about funding allocation and curriculum. Sure, there’s a few students and a few teachers and a few pursestring-controllers who’re interested in learning, but they (a) typically haven’t read enough cognitive psychology and (b) are enough of a minority that effective learning has, to my knowledge, never been effectively affected by schools below the graduate level (I’m not entirely sure why, but everything I’ve seen indicates that effective learning does, in fact, happen at the graduate level. My current best explanation is that most undergraduate degrees amount to signalling “smart! conscientious!”, whereas the graduate degree signals “specialized knowledge/skills you won’t find in anyone without a graduate degree”, meaning that meaningful learning has to happen. I still doubt that schools are currently making that meaningful learning happen as effectively as possible, but at least it’s happening at an appreciable rate.)

          This isn’t to say, however, that effective learning can’t happen. Duolingo, for instance, was put together by people who have read the right cog psy. When I first started using it after 7 semesters of (excellent!) traditional language classes it felt like witchcraft because I was actually learning language. Similar results can be obtained in other areas by reading quality textbooks, although you have to do work that they won’t tell you to do (which forces deep processing, and is therefore a feature-not-a-bug), like putting things-you’d-like-to-remember into Anki or trying to prove theorems before reading the book’s proof. There’s a post-in-the-works which I believe describes a best first-approximation-given-current-knowledge-of-cognitive-psychology* of how to learn effectively. Expect it on LW mid- to late-May, after I’ve finished reading to make sure I’m actually giving good advice.

          *People are idiosyncratic. Like, if you read the primary literature, there exist people who perform better using massed, rather than spaced practice, so there’s really no way to write an article that says “this is the best way for you to learn”, but it is possible to write something that says “this is the best way for most people to learn, but there’s also a completely reasonable chance you’re going to need to tailor a part or parts to your own brain, but this is the best starting point, insofar as it minimizes expected tailoring”.

        • thirqual says:

          I have trouble believing the FizzBuzz thing (not a CS graduate, taught myself python for writing ugly scripts). But then, I can’t believe that most doctors will not give the right answer at the classic Breast Exam screening example.

          • Anonymous says:

            I find the breast exam much easier to understand – doctors just don’t use probabilities.

            I am confused as to what is even claimed in the fizzbuzz story. Is it that people can’t do it, or that they can’t do it fast? (the post mentions senior programmers taking 10 minutes) And what does “can’t do it” mean? Is it that they can’t write syntactic programs without an IDE? Is it that they can’t reason about the behavior of the program without running it? Or do they produce complete nonsense?

          • Nornagest says:

            I had trouble believing it until I conducted a few interviews. A lot of “senior software engineers”, let alone people right out of college, are astonishingly bad at nuts-and-bolts coding.

          • Anonymous says:

            I suspect that most people , who haven’t seen the problem before,start off on the wrong tack, and then need backtrackw, and arent willing to backtrack.

          • grendelkhan says:

            I didn’t believe it, and then I started interviewing people. People who’d held jobs in the industry, people who had worked as software engineers.

            My best model of this is that most coding is done in some sort of cargo-cult copy-and-paste style, where you pull something off of StackOverflow without understanding it, or duplicate someone else’s code. Which is horrifying, but it seems to explain the phenomenon reasonably well.

          • Anonymous says:

            Grendel, I expect that most programming is done by cut-and-paste, but doesn’t it have to be pasted into the right part of the flow control? Maybe most programmers would need to copy a congruence test to do this exercise, but the implication is that this they also screw up the control flow.

          • Princess Stargirl says:

            I forget the statement of the problem but isn’t the solution literally:

            For i=1:100

            if mod(i,15) == 0
            {
            cout << "FizzBuzz" << endl;
            }
            elseif mod(i,3) == 0
            {
            cout << "Fizz" < endl;
            }
            elseif mod(i,5) == 0
            {
            cout << "Buzz" << endl;
            }
            else
            {
            cout << i << endl;
            }

            This is in the format they say the program cannot easily be written in lol. It is literally written in:
            If 1 then A
            elseif 2 then B
            elseif 3 then c
            else D

            In "C++ pseudo code" since I am lazy? Or am I forgetting something?

            Of course I didn’t read the problem again but this should take minutes no? Maybe takes longer if one is careful and checking their work thoroughly. 10 minutes seems reasonable to me. If someone asks you a question there is a decent prior its not trivial even if it seems trivial. I could see taking 10 minutes to fully convince yourself you are not crazy and the problem is easy.

          • Pseudonymous Platypus says:

            I think the charitable interpretation of people failing at FizzBuzz is that a lot of people get extremely nervous when being put on the spot in an interview. Then again, I’ve never interviewed anyone, so that might just be wishful thinking.

          • Anthony says:

            People get nervous during interviews, but FizzBuzz is pretty easy:

            for (int i=1..100)
            {
            print i,” “;
            if mod(i,3)==0 print “Fizz”;
            if mod(i,5)==0 print “Buzz”;
            print “\n”
            }

          • Nornagest says:

            @Anthony: That actually wouldn’t work — the usual FizzBuzz spec specifies that the integer not be printed when you’re printing Fizz or Buzz or both. Which is really the only reason it’s tricky — FizzBuzz is mainly about managing slightly-complicated conditional branching.

            Stargirl’s solution looks correct to me.

          • RCF says:

            “I find the breast exam much easier to understand – doctors just don’t use probabilities.”

            What the hell kind of doctor doesn’t use probabilities?

            @Anthony

            You didn’t specify a language, so I assume this is pseudocode. Besides the issue Nornagest brought up, there’s also the question of whether the print function includes a carriage return.

          • Anonymous says:

            Yeah, people really cannot do Fizzbuzz, and this is after *really, really, really* trying to get them to relax and take it easy and just fumble through it. But no, they screw it up.

            Interviewing: the best cure for imposture syndrome yet devised.

    • Kevin says:

      A few case studies to whet Scott’s appetite:

      How one national school voucher program fared (about Chile):

      Three-fourths of the public school enrollment in Chile are students from the lower 40 percent in family income. Only 10 percent of disadvantaged students use vouchers to attend private schools. Ninety percent of the private school students come from the top 60 percent.

      The opportunities, where they have existed, have been for the benefit of upper-middle income families. Students in private schools, especially in those that charge fees above the voucher amount, are doing pretty well. Students in public schools struggle amid a host of challenges. Budget cuts have led to overall decline in quality. Disadvantaged students and students with disabilities – the students Romney’s plan is said to help – are vastly overrepresented in the public schools, in large part because public schools are the last resort for students turned away because of income, ability or discipline issues.

      The education marketplace has grown in Chile, as Friedman predicted, but quality is not the only factor people take into consideration. For parents, price and proximity also matter. For private schools, the emphasis is on serving students that are cheapest to educate, not tailoring different programs to the unique needs of students.

      But on the flip side: Lessons on School Choice from Sweden

      The authors discuss some important characteristics of the Swedish system that may contribute to the success. First, the Swedish system does not allowing parents to pay additional fees on top of the voucher.Second, there are strong rules about how schools must accept students. They cannot use ability, socio-economic status, or ethnicity. The authors argue that if competition on selection is prevented, schools are more likely to compete on quality.

      And directly from the mentioned paper:

      These schools are not allowed to select students by ability, socio-economic characteristics or ethnicity. If a school is oversubscribed, three selection criteria for admittance are allowed: proximity to the school; waiting list (where each child’s place in line is determined by the date of the parents’ application) and priority to children who have siblings already enrolled in the school.

      However, Sweden (like the other Nordic countries) is less diverse and has much less economic inequality than Chile or the US. I can easily see a similarly-regulated market in the US using the proximity criterion as a very effective proxy for socioeconomic status, so I’m not sure if these exact conditions would be as effective in those markets.

  34. Wrong Species says:

    What are some good criticisms of consequentialism separate from utilitarianism?

    • One criticism of basic consequentialism is that accurately calculating consequences is often impossible in complex human situations where there is a hole load of moving parts. Also, if a well-intentioned person, one with normal human judgement, performs an action leads to some unforseable negative consequences, it doesn’t seem to ring true to call them immoral (definitions/semantics grr). So critics would suggest that this points to the consequences not being the central criteria for morality.

      If you accept this you either have to get more sophisticated (eg. rule utilitariansim) or consider some or all elements of virtue or deontological ethics. I personally like the idea of mixing in a virtue approach just because it has good consequences. Not what the professor would like but hey.

      One thing you can say for consequentialism though, there’s a lot less wiggle room. Rules are usually open to interpretation (see sectarianism), and good intentions are really easy to fake or rationalise (even to one’s self) “I didn’t mean to hurt all those people your honour” or “I followed the rules – its not my problem” (ahh moral loopholes). Virtue is more complex. Consequentialism doesn’t care – basically no results, no morality. In a world full of pretend morality, this ain’t nothing.

      SEP is probably your best starting point if you want to get formal/strict about your arguments.

      • It is worth bearing in mind that all plausible moral theories recognize a reason to promote welfare. The difference between these theories and consequentialism is that the latter holds that these reasons are exhaustive, whereas on non consequentialist theories agents are sometimes allowed, and sometimes even required, to do things that aren’t welfare-maximizing. Thus, the difficulties involved in calculating consequences do not pose a problem specific to consequentialism, but rather generalize to all moral theories deserving our attention.

        • kaninchen says:

          I don’t think that’s entirely the case. If you have a choice between (a) a sadist torturing people with low cognitive capacity and (b) someone enjoying a good book, a utilitarian has to do the hard work of deciding which leads to more pleasure (or satisfaction of preferences) whereas the rest of us can instantly say you should choose (b). Perhaps this is something of a fringe case, though.

      • cypher says:

        > Also, if a well-intentioned person, one with normal human judgement, performs an action leads to some unforseable negative consequences, it doesn’t seem to ring true to call them immoral (definitions/semantics grr).

        I think this is from a bit of confusion. Consequentialism is about judging what is a good action, not who is morally praiseworthy. Under a Consequentialist system, we should consider intent when deciding whether to praise or damn someone, since that’s actually a diffferent criteria, and good intents seem to lead to good actions more reliably – or at least to being willing to change bad actions.

        I think the biggest problem is just that humans are such big rationalizers and are operating on such incomplete information, that it may be better not to run an entirely Consequentialist society in practice.

  35. Athrelon says:

    At the Solstice meetup, I mentioned that I was going to talk to the Brain Preservation Foundation (Hanson-endorsed!) to see whether they’re still alive, and what they’ve been up to. I also promised to relay any findings here. Well, I had a chance to talk to the current head of the BPF tonight, and it was pretty fascinating. They are very much alive, and appear to be doing pretty high leverage work with limited resources.

    The vast majority of people dismiss cryonics as snake oil. A smaller number of people believe in cryonics enough to pay for it. The number of people, on the other hand, who want to look closely at “what exactly does preservation do to the tissue, and can we do something about minimizing that?” is miniscule, and it’s at that question that BPF is trying to drive research.

    The two main approaches they’re considering are cryonics and plastination. We all know about cryonics; what’s less known is that as currently practiced, it results in simply enormous amounts of tissue damage. There’s a difficult tradeoff: if you infuse cryoprotectant more quickly, you get osmotic damage; if you do it slower, you are letting the brain sit too long at room temperature. There are potential approaches that could improve it.

    The other is plastination, which actually originated in the 70s as a procedure to preserve materials for electron microscopy. The good news is that since it’s a procedure that’s preparing tissues to be examined by the most powerful microscopes ever, it’s pretty good at preserving the tissue architecture – you can get neurons and even synaptic vesicles all preserved in good condition. The bad news is that electron microscope samples are *tiny* and that until now nobody has put much time and energy into trying to scale up the protocol to something the size of a mammalian brain.

    The upshot is that they’ve been working with a few labs to fund research into scaling up plastination and improving cryonics protocols, and are actually at a point where they may be adjudicating the prize for preserving a mouse brain pretty soon. Their hope is to be able to then scale this up to a primate brain (which at least one lab is interested in doing, if the mouse brain works out).

    Long story short, they are funding and influencing some pretty important research on a fairly small (<$100k) budget. If you're interested in the prospects for brain preservation. Certainly if you are getting cryonics, or are considering getting cryonics, it would be worth considering donating to this organization. This research will likely have a significant effect on the quality of brain preservation technologies that will be available by the end of our lifetimes. (And keep an eye out for the announcement of the first prize, likely before the end of 2015.)

    • grendelkhan says:

      Oh, cool! I donated when they were having their last drive; I’m glad to hear that they’re still around and doing work. Please do post an update here when they announce prizes in case Scott misses it.

    • Nita says:

      There’s a difficult tradeoff: if you infuse cryoprotectant more quickly, you get osmotic damage; if you do it slower, you are letting the brain sit too long at room temperature.

      Wouldn’t the same apply to plastination? I don’t see how you could rapidly replace all water with a non-water solvent in something as big as a human brain without causing osmotic damage. Now, if you could find a way to slice it into tiny chunks quickly and cleanly…

      Good luck to them, though. Even if we don’t get immortality out of it, it’s still valuable research.

  36. Princess Stargirl says:

    “If I had the power, I would add an 11th commandment to the already existing 10: “You should never be a bystander”.”
    —Roman Kent, Holocaust survivor, in his speech on the 70th anniversary of the liberation of Auschwitz-Birkenau. January 27, 2015.

    I saw this on tumblr and I wanted to discuss this here. I don’t see how this advise is workable at all if you have non-standard morals. I personally think people in jail for peacefully using or selling drugs are kidnapping victims. This makes me a bystander. What am I supposed to do? Maybe I am supposed to dedicate myself to providing alibis for people accused of selling drugs? Am I supposed to try my best to improve security on the Tor network (security is not my specialty but I know a good amount about computers and I could probably learn enough to contribute to Tor security in a reasonable amount of time).

    Worse this advice imo would cause a huge amount of problems. Lots of people consider abortion to be murder. Are they supposed to bomb abortion clinics? In theory maybe this quote allows one to take “political action.” But unless one is very wealthy or famous the EV of trying to change politics is very, very low.*

    Also right now I am donating 10% of my income to Givingwhatwecan/Givewell recommended charities. This is thousands of dollars. Many ways for me to “not be a bystander” are likely to put me in jail. This means I won’t be contributing much to charity.

    *One might object that the odds of changing a law go to zero with the population size. But the benefits grow with the population size so it evens out. However this is probably false. Under any reasonable assumption the odds of an agent changing a law go to zero exponentially with the population size, while the benefits scale only linearly.

    • Jaskologist says:

      It’s not supposed to work with non-standard morals. The whole point of the ten commandments is to set a moral standard.

      • Salem says:

        It has to work with somewhat non-standard moralities. The test case, after all, is someone living in Nazi Germany who thinks the Holocaust isn’t OK. The straight meaning of “you should never be a bystander” is that you should be an Oskar Schindler or Raoul Wallenberg, i.e. you should defy the law, at great personal cost to yourself, to do the right thing. If the circumstances are right, you should even be a Komorowski and engage in massive violent resistance.

        On the one hand, this is probably good advice as against the Holocaust. On the other hand, I don’t want the losing side in every moral or political disagreement to start a civil war.

    • AR+ says:

      It is hardly obvious that if somehow forced out of being a bystander, most people wouldn’t have become True Believing Nazis, rather than fight against them.

    • lmm says:

      Is it really that surprising if a Holocaust survivor (who may well have no more skill or training in ethics or philosophy than the rest of us) gives advice that’s specific to preventing the Holocaust, and actually bad advice overall? Everyone fights the last war.

  37. RCF says:

    It seems to me that Tarrou is in rather blatant violation of the commenting policy, and is decreasing the quality of the blog. I have reported posts repeatedly, but have not received any response. Why is Tarrou being given a pass?

    • The Anonymouse says:

      What comments has Tarrou made in this thread that are so bad? Specifically?

      I don’t know what he’s said in other threads, but you posted this request in this thread, so I looked at the handful he posted here. I don’t even see anything objectionable, much less banworthy.

      • Tarrou says:

        I disagreed with RCF once, so apparently he’s been reporting me ever since. I say now what I said then. I stand on the merits of my opinions and statements. I don’t think I’ve violated any of the policy, but it’s obviously not my call. That is for Scott to decide. I just share my opinions, arguments and experiences to the best of my admittedly limited ability.

        • The Anonymouse says:

          Well, if you’ve been repeatedly reported by the same person, and Scott has not done anything about it, perhaps we have our answer on how he feels about the merits of those reports.

      • RCF says:

        I’m referring to his behavior in the Influenza of Evil thread (this is, after all, an open thread). There were two subthreads where he violated the commenting rules.

        In one, there was a discussion of neo-Nazi groups and how there are other groups that are doing better than them at not triggering society’s evil-detecting heuristics. I said that Liberty University is an example of such a group. Tarrou then accused me of saying that Liberty University is more evil than Nazis. I pointed out that this was incorrect both in that the comparison was with neo-Nazis, not Nazis, and I did not say that Liberty University is more evil that neo-Nazis, I said that it is better at not being recognized as evil. Despite this correction, Tarrou persisted in blatantly misrepresenting my position. In addition, he did so in an extremely rude manner, openly mocking me, rather than actually presenting any substantive argument.

        In another subthread, there was discussion of the aphorism “comedy should punch up, not down”. Someone complained about Christian conservatism being mocked, and I said that Christian conservatism consists to a large degree of punching. Tarrou then accused (in a cowardly oblique manner) me of being a bigot, basing this on me supposedly saying that all conservative Christians are violent. Never mind that I directed my comments at Christian conservatism, and thus was saying that the ideology consists of punching, not that every person in the group engages in punching, and the word “punching” was obviously metaphorical. He also tried to make a bullshit analogy to someone saying that being black consists of punching, as if criticizing an ideology is somehow similar to criticizing a race.

        Tarrou clearly has no respect for the truth. All he cares about is how he can twist what other people say to suit his needs.

        • Anonymous says:

          To be fair, while not explicit, when presenting the Liberty University, whoever it may be, as an example of a group that gets away with being evil better than Neonazis, there is an >implication that they, at least, comparably evil. Otherwise it’s a meaningless example.

          Anyway, you might be taking this a bit too seriously.

    • Scott Alexander says:

      I actually haven’t checked reports in the last month or so. I’ll review.

      • Scott Alexander says:

        I’ve gone through a few weeks of reports. Arthur Stanton and Noman are now permabanned. I haven’t gotten to the Tarrou reports yet, but I’ll deal with them later.

        • Barack Obama says:

          What’s to stop the people you ban from choosing a new name, plugging in phony email address, and continuing their misdeeds?

          It seems to me that requiring commenters to set up a login that confirms an email address would prevent that problem.

          • Scott Alexander says:

            So far people haven’t done that. If they do, I can ban IPs. If they care enough to get around the IP ban, they probably also care enough to set up a fake email.

    • Scott Alexander says:

      Okay, I’ve looked at it. It seems to have started with you saying something like “conservative Christians are obsessed with punching down” and then he got mad and yelled at you. This seems excusable if he identifies with them, and he doesn’t seem to have a history of bad behavior otherwise, and he sometimes makes positive contributions. I’ll watch him closely from now on but I don’t think I’m going to take any action right now except suggest that everyone watch themselves a little more.

  38. RCF says:

    There have been several times that the software has refused to post a reply (and deleted its content) saying that I’m posting too quickly. This seems be based on the delay between my previous post, and clicking the “reply” button again, rather that the delay between my previous post, and clicking the “Post Comment” button. Is my analysis correct?

  39. LTP says:

    You all seem to know a lot about meta-stuff, so here’s a question: is there a name or discussion of the phenomena of liking a lot of the secondary and tertiary aspects of a movement/ideology/philosophy/institution (e.g. certain maverick-y individuals identified with the group but not at it’s center, certain secondary ideas, their methods but not conclusions, certain aspects of the tribe are socially appealing, etc.) but disliking the core individuals, principles, and ideas?

    For instance, I’ve met a couple people who strongly identify with certain idiosyncratic Republicans and conservatives, actually agree with the core of party on a handful of issues, but hate the core of the Republican party and conservative movement on substantive grounds.

    • Multiheaded says:

      I think we all know which movement in particular you’re thinking of 😉

    • stillnotking says:

      Identifying oneself as a “maverick” Democrat or Republican is much more common than following the whole party line, particularly in America, which places a high cultural value on individuality. (No, really, we do — compared to the rest of the world.)

      So the answer to your question of what to call an idiosyncratic Republican is: “a Republican”.

  40. Illuminati Initiate says:

    About people feeling bad for having lower IQs.

    Is the problem one of feeling like a “bad person”?

    Lets talk about musical talent. Some people are much better at music than others. Some of those people will tend to “value musical talent” in two ways- They value it as part of their own personality in the same way as other parts of their personality, and they like hanging out with people with similar musical talent, because they can have conversations about music that interest them and can play music together. Additionally, musical talent can be instrumentally useful- famous musicians get fame and fortune. So what of the people like myself with less musical talent?

    There is no shame in not having any. I’m honestly not sure how people even view there as being so. What does it mean to be a “bad person”? All people’s preferences matter. You shouldn’t feel bad just because you are not good at something, or even at a lot of things. You are still a person, and that is all that matters.

    The same applies to IQ.

    I used to “look down” on less intelligent people. There are some people who do this and it is not OK.

    (Also, side note- While IQ is important for math and research and such, I think rationality is probably a lot more important than raw intelligence when evaluating information from other sources than yourself)

    • Halfwitz says:

      [Deleted: Likely neither kind nor necessary, and possibly not true.]

      • Illuminati Initiate says:

        OK, now I wish I knew what this said. Was it self-deleted or did Scott delete it?

        • lmm says:

          I would assume self-edited; in my experience Scott deletes posts outright rather than editing them like that.

    • llamathatducks says:

      I sometimes worry about being bad at things and especially about losing knowledge and mental abilities I once had. The feeling that comes with this worry is not a feeling that I’m being somehow immoral by not doing the work to keep my abilities – it’s a feeling that I’m losing value, that I’m worth less. Not worthless, but worth less than I would be if I had more knowledge and ability.

      Which is a pretty awful/ableist/unfounded belief to hold, and which isn’t something that I actually believe, but try as I might I cannot get rid of the alief. (It applies to other people a bit too – I sometimes feel extra-sad if I hear of the death of a stranger who was highly learned or skilled, compared to how I’d feel about a random stranger’s death, even though I think this is wrong.)

      It’s not that surprising to me that I feel this way given my very skills-focused upbringing and the fact that I was generally a rather high academic achiever and therefore found it easy to attach my sense of worth to being good at brain things.

      • Illuminati Initiate says:

        I think it makes sense to not want to lose intelligence- as I said, people value things like that as part of their personality, and not wanting to lose part of yourself makes sense. I certainly feel that way myself. Also, while less common, it also makes sense to want to gain intelligence- though it still does not make you of less value because of your relative lack of such.

        Now that I think about it though, I should have been more clear about the difference between something like “intelligence dysphoria”, and feeling “bad and worthless” as Scott put it. The former is a preference and makes sense, and is actually probably pretty common around here given our transhumanist leanings.

        • llamathatducks says:

          Well, when I get self-conscious about my failings in mental ability I do feel like I am not worth as much as I’d like to be. I never feel like my worth is zero, but I do sometimes feel like compared to the worth of some of my super-capable friends it is almost negligible. So I think it’s similar to the phenomenon you’re describing and trying to understand.

  41. Sophie says:

    So, random question:

    Does anyone know whether donating blood is an effective way to give charitably?

    • zz says:

      If you’re an eligible male, then blood donation (at least) every 12–24 months is certainly effective, since—assuming your optimizing for longevity—you’re losing that blood anyway.

      Near as I can tell, giving blood saves lives and (for most people) doesn’t funge against other things an effective altruist would do to save lives, so yes? (This is no more than naive reasoning, so take it with the appropriate grain of salt.)

    • Jaskologist says:

      I’m not an EA, but blood seems pretty non-fungible with other resources, until we start paying assembly lines of poor O- people to donate for us. And you can be much more confident of it going to help somebody in need than you can with most money you donate.

    • The Anonymouse says:

      Slightly off the topic, and there’s probably an obvious answer* that I’m disregarding, but why is it that plasma centers pay money for plasma, but everyone expects people to donate whole blood for free?

      *Possible obvious answers: The Red Cross has trained us that blood is for donating (seems true) and gets more than enough blood that way (likely not true); plasma donation takes more time, and no one would donate without recompense?

      • Caelum says:

        “if a donor receives monetary payment for a blood donation, all products collected. . . must be labeled with the “paid donor” classification statement”
        Source

        and

        “Most source plasma in the U.S. is from paid donors. In contrast to whole blood collections, these units, under Federal Regulations, are not labeled as collected from paid donors. In part, this is because all source plasma is used to further manufacturing use only.”
        Another Source

        Along with intimations that “paid donor” blood is seen as less suitable for transfusion for various conflict-of-interest reasons.

        Also

        “In general, the U.S. collects just what is needed to maintain adequacy in the blood supply. Although blood shortages are still seen in the summertime and holidays, they generally they do not reach serious proportions, and the public generally responds very generously to appeals for additional blood donors.”
        Same Source As Immediately Above

      • Mary says:

        Yes, they get less blood if they pay. It converts a Good Deed into a kinda unpleasant way to get money, apparently.

        • The Anonymouse says:

          Interesting. One of the reasons I asked is because it occurred to me the other day that I would be tremendously more likely to give blood if I were compensated for the time/unpleasantness. The Red Cross may currently pay for blood in Warm Fuzzies, but I haven’t any doubt that they sell it for cash.

  42. Baby Beluga says:

    Does anyone here take pacifism seriously? Like, “the United States should get rid of its military right now”-level pacifism?

    I don’t *think* I take it seriously. But here are some reasons why maybe I should, in increasing order of convincing-ness (to me):

    -When you ask a population what it thinks of a war that just finished, you’ll be biased in favor of thinking the war was better than it was, because you won’t be asking the people who would have had the strongest negative opinion of the war, i.e. the people who died in the war.

    -Wars have two effects: they cause lots of death and destruction, and then they cause a change in power structures, sometimes? The first effect is really easy to understand: it’s really bad. The second effect is really complicated and nobody really understands it and it gets super politicized and it’s not even clear if the change in power structures was good. So we have a bad effect and an unclear effect. Win for not fighting wars?

    -When two societies disagree over something (say, capitalism vs communism, or slavery vs not-slavery), sometimes they fight a war to decide who’s right. In other words, they have a giant contest to see which society is better at murdering as many people as possible, and then whichever society does better at that is the one they listen to. Isn’t there something wrong with that?

    -When you fight a war, you’re saying, “I think that my side is enough more ethical than your side that it’s worth causing tons of death and destruction for me to stop your side.” But the other side is saying the exact same thing about you! Shouldn’t you worry that your side is actually the bad guys? For example, when we fight ISIS, we place a very low probability (roughly 0%) on ISIS actually being the good guys and us being the bad guys. This is intuitive to me! ISIS beheads people and is terrible! But probably ISIS places a very low probability (roughly 0%) on them being the bad guys and us being the good guys. That’s intuitive to them! Isn’t there something wrong with this? Shouldn’t you and your enemy agree to agree? http://www.overcomingbias.com/2006/12/agreeing_to_agr.html

    • lmm says:

      I agree that you should almost never actually use the army. But the agreeing to agree only works if the consequences of war are sufficiently negative, for ISIS (who perhaps put a different value on the lives of “martyrs”) as well as you. So you need to keep enough of an army that ISIS can’t just annex all your territory with low effort.

    • FullMeta_Rationalist says:

      Consider that the Swiss have an army. Also, Ghandi was on board with self-defence. Non-violence is about respect for your enemy. Self-defense is about respect for one’s self. They’re totally compatible ideas.

      In other words, they have a giant contest

      “Contest” makes it sound voluntary. But violence is inherently nonconsensual. Even if two parties “consent” to a duel, neither party “consents” to get stomped.

      History listens to the victors because they are the ones still intact. When someone attacks you, your options are fight, flight, or lose (die). There’s no option to ignore it. You can’t just shrug it off as if someone had asked you to play chutes-and-ladders (pun intended).

      But the other side is saying the exact same thing about you! Shouldn’t you worry that your side is actually the bad guys?

      Hopefully, people already have considered that they’re the bad guys. However – just as you never hear about war from dead soldiers, you never hear war from counterfactual soldiers (e.g. vets from the formic wars). So you also have to recognize that going to war usually means negotiations have already failed. The disagreements where negotiations succeeded were mere squabbles that you label in hindsight as “not a war”.

      E.g. I recently overheard in a conversation about Ferguson that cops are trained to always “shoot to kill”. “But why can’t cops just shoot to maim?” “Because shit went to hell long before the cop has to resort their firearm.”

      • Baby Beluga says:

        Yeah, I agree. To be clear, I wasn’t trying to claim that you have a third option besides fighting or losing. I think, though, that to take pacifism seriously would be to say, “Well, if I’m attacked, then it would be unethical for me to choose to fight, so I will choose to lose.” And maybe that’s wrong! But, when you choose to fight instead of lose, you are making a choice that is denying that your attacker is rational. You’re deciding, not just that you think your attacker is wrong, but also that you think they are so wrong that it’s worth fighting a war over it.

        Like, imagine that both sides of every war decided to instead use a random but harmless process that provided the same odds of either side winning to decide the war, instead of actually fighting the war. Wouldn’t that be so much better? And the only reason it doesn’t happen is that everyone is so sure that their side is the right one that they wouldn’t honor the outcome of the pretend-war as canon. Shouldn’t we all become a little bit less sure of ourselves?

        • FullMeta_Rationalist says:

          Consider the birthday paradox. Apply it to violence. Between any two particular people, it’s unlikely they’ll disagree on a topic enough to fight. But given a planet with 10 billion people, it’s practically impossible for the planet to not contain some pair of individuals who’ll come to blows. Given fights rarely break out in my immediate vicinity, I’d say we’re doing pretty well. Far from perfect (I wouldn’t want to live say… next to the bloods, or in Afghanistan), but surprisingly well nonetheless.

          The fact that a defendant can’t opt out of aggression is not just a bug, but a feature. If one party loses a coin flip, what’s to stop the loser (or winner) from attacking anyway? The U.S. signed a great deal of treaties with the Native Americans, but that didn’t stop the U.S. from clearing them out. Those treaties worked out pretty well for Uncle Sam, didn’t they?

          I agree that in a perfect world, wars would end on a coin flip. But I can’t realistically imagine Austria avenging King Franz over a game of beyblade. On the other hand, I can imagine America’s antebellum South attempting secession and posterity saying “hey, that coin flip wasn’t so bad. We lost, but it doesn’t hurt to try it again.” Fast forward 100 years, and we have the 50 independently sovereign states of america. Maybe not so bad from the individual’s point of view, but definitely bad for the Union’s interests as a memetic organism.

          To channel Robert Hanson. I’m saying with the antebellum example that maybe the cost of war deters people from engaging in future wars (unless they’re super committed). Sparta was pretty intimidating, after all.

          But, when you choose to fight instead of lose, you are making a choice that is denying that your attacker is rational.

          What exactly are we referring to here? If you incapacitate an assailant by knocking the wind out of the solar plexus and run, that sounds pretty rational and self-defence-y. If you break his jaw, pin him down, and snap his knees, that sounds like you might be denying the assailant moral agency (or… something). But that doesn’t sound self defence-y. I think we’re confusing self-defence with retaliation.

          Do we agree that defending one’s land from the Roman Empire is justified? Do we agree that defending one’s land from Genghis Khan is justified? “Rationality” doesn’t mean allowing the devil incarnate to salt your fields and rape your women.

          • FacelessCraven says:

            “I’m saying with the antebellum example that maybe the cost of war deters people from engaging in future wars (unless they’re super committed). Sparta was pretty intimidating, after all.”

            This, exactly.

            Dispute resolution means the dispute ends. For that to happen, there has to be a reason the losing party can’t ignore the verdict or immediately start a new dispute. The path of least resistance is to make disputation expensive for all participants. Logically, the bigger the dispute, the more expensive it is to raise it. The most valuable thing we have is our lives, and so that is what the largest disputes cost us.

            …And that is why war is not ending any time soon.

        • John Schilling says:

          But, when you choose to fight instead of lose, you are making a choice that is denying that your attacker is rational. You’re deciding, not just that you think your attacker is wrong, but also that you think they are so wrong that it’s worth fighting a war over it

          I think you are conflating wrong/irrational with wrong/unethical. Example: I think you have lots of useful stuff. I want it. I offer no moral claim to why I should have it; I acknowledge being wrong/unethical here. I believe based on sufficient evidence that you as a pacifist are highly unlikely to offer effective resistance if I come and take it, and that I can effectively evade the police afterwards. I show up with a baseball bat to start breaking your bones until you give me your stuff. In what way am I wrong/irrational?

          Unbeknownst to me, you had a crisis of conscience last night, are no longer a pacifist, and have a shiny new handgun in your pocket. We’re about to have a fight. In what way is your shooting me a denial of my rationality?

          And if it is, so what? Possibly you can identify flaws in the analysis by which I concluded that you are a pacifist who won’t fight back when I come to break his bones and take his stuff. Possibly I am in fact being irrational. Lots of people are. Does this mean that rational people ought to let irrational people break their bones and take their stuff, rather than “deny their rationality”?

        • John Schilling says:

          W/re the “random but harmless process that provided the same odds of either side winning to decide the war”, there are some fundamental problems with that.

          1. The most important single factor in winning a war, is the extent to which each side is willing to suffer the actual horrors of war in pursuit of victory. There is not in the real world anything remotely resembling an objective way to measure this short of actually waging war.

          2. It is in the interest of both sides to misrepresent or conceal material facts that will contribute to the outcome. Most obviously, by representing themselves as fanatics who will fight to the very death, but also in the purely material aspects – deploying weapons that they secretly know won’t work for their deterrent potential, keeping other weapons secret to surprise the enemy at a decisive moment, etc. With so many hidden variables, you’re not going to be able to fairly assess the probable outcomes, much less convince both sides that you have done so.

          3. If someone thinks that a cause is worth fighting a war for, with everything that implies, then they very likely think that the cause will be worth breaking a treaty for. Your forces and mine are equal, victory is a 50-50 proposition, and I at least think that what is at stake is worth fighting a real war at those odds. Of course I’m going to agree to your proposal to settle things with a coin toss. And if I lose the coin toss, I’m going to have my army launch a surprise attack while you are celebrating your peaceful “victory”.

          We can handwave these away by positing an incorruptible, omniscient, omnipotent entity that assesses the probabilities of victory and defeat in war and punishes defection from the random-number settlements. But utopian propositions that depend on incorruptible, omniscient, omnipotent enforcers strike me as uninteresting – in the real world, the closest thing we have is the United States Government.

          • Anonymous says:

            … incorruptible in the sense that their highest imperative is to feed their military-industrial complex, no matter the human cost, and no force on earth could ever hope to corrupt that imperative?

    • Jiro says:

      The argument “doesn’t ISIS think their war is just?” sounds like an isolated demand for rigor. ISIS does all sorts of things that we do but in bad ways because of their choice of when to do them or on what target. Nobody ever says “sure, you think you are the good guys when you give sentences to criminals, but ISIS thinks they’re the good guys when they give sentences to criminals too”, even though ISIS sentences criminals in all sorts of ways we consider bad.

  43. Shmi Nux says:

    Just wanted to note that Multiheaded has a very good chance to have her claim approved, eventually, as a convention refugee, based on the eligibility guidelines at http://www.cic.gc.ca/english/refugees/inside/apply-who.asp :

    > membership in a particular social group, such as women or people of a particular sexual orientation.

    especially if she has a documented case of being persecuted, and likely even without it.

    • Multiheaded says:

      http://egale.ca/all/faq-immigration/

      This and other sources paint a pretty hopeful picture to me. Tl;dr – when queer people are terrorized into staying closeted, that in itself constitutes persecution for the purposes of refugee status. Also, Canada’s overall approval rate for asylum requests was around 40% in 2011; higher than that for queer migrants.

      Also, a friend of a friend of a friend has offered their assistance as a social worker focusing on queer refugees; they and/or someone else should be able to help me with the legal stuff.

  44. Daniel Speyer says:

    I was thinking again about the PETA water bills thing, and found that it actually did make some sense.

    Consider the following timeless platonic contract:

    If I ever find myself in a position of offering a deal to which no counter-offer is practical (something equivalent to an ultimatum game) I will make an offer as if from behind a vale of ignorance. I will do this regardless of how I got into that position. In return, you will do the same.

    That sounds like a contract a lot of us would ratify. I see no terrible consequences of universalizing it. I also note that we have a word for violating this contract: “exploitation”.

    Alternative, consider this contract:

    If I ever find myself in possession of a resource, I will calculate a distribution of resources as if from behind a vale of ignorance. If I possess more than my calculation, I will give the excess to whomever is most shortchanged. I will do this regardless of how I acquired the resource. In return, you will do the same.

    I think most of us would not ratify that contract. The prospect of being unable to ever deliberately acquire more resources seems a bit daunting.

    From this perspective, we can see PETA as in violation of a platonic contract, whereas the people who simply refused to pay a random stranger’s water bill are not.

    I’m not entirely convinced of contractualism, but it does seem to put this intuition on a much sounder footing.

    • Baby Beluga says:

      Wait, I would TOTALLY ratify that second contract, if it was the case that incentives weren’t affected. Like, if suddenly tomorrow God decreed that our resources would be redistributed perfectly fairly, then I would absolutely ratify that happening from behind the veil of ignorance.

      And note that paying a random stranger’s water bill, once, doesn’t change incentives.

      I feel like I might have cheated somewhere here? But I feel like these two contracts have not yet successfully been distinguished.

      • Princess_Stargirl says:

        You do not think this would cause problems economically? I am not of the view that tax increases will destroy the economy. But that pledge is truly extreme and destroys 100% of the incentives for alot of people to produce more economically.

        • Baby Beluga says:

          That’s why I was trying to make this contract not affect incentives (and arguing that paying a random stranger’s water bill also doesn’t affect incentives). But I agree, something feels wrong.

        • Baby Beluga says:

          Oh I should specify–when God redistributes all the world’s resources, it’s one-time thing. Otherwise, obviously, incentives get affected.

      • Daniel Speyer says:

        Do you pay random strangers water bills until you can barely afford your own? If not, why not?

        • Baby Beluga says:

          I don’t, but only because I’m a bad person–from behind the veil of ignorance, I’d be in favor of living in a society where everyone did this.

          • drethelin says:

            The more people will do this, the more money will be wasted by people who have no incentive to conserve or pay for their own water bills.

      • Jon Gunnarsson says:

        The second contract doesn’t say anything about God doing the redistribution, only that one should try to the best of one’s ability to redistribute as if one were behind the veil of ignorance. Which means that the actual distribution of resources would be completely awful and far far worse than under a market system (even if we assume that everyone does his honest best and is perfectly unselfish) because no one person has more than a tiny fraction of the knowledge required for figuring out optimal (or even non-terrible) resource allocation. The only reason we can allocate resources somewhat efficiently in the real world is because we have market prices which tell us approximately how scarce goods are relative to one another. Under the second contract, all of that goes out the window.

    • lmm says:

      The vale of ignorance is presumably somewhere in the Welsh mountains where we perform these experiments?

  45. Anonymous says:

    Question for atheist or agnostic deontologists (if there are any):

    Say you’re the single passenger on a spaceship that crashes on a planet that can support life, but has no animals, no people, and no trace of people. You’ll never be rescued, you’ll never see another person or sentient creature, and no human or alien will ever reach the planet (assume any explanation you wish).

    In this situation, where it’s impossible for anyone to ever be hurt by anything you do, is it possible, even in principle, to do something you’d consider unethical or immoral?

    • I suppose I qualify by some measure, although I don’t subscribe to a pure deontological approach (I’m on the deontological side of what I consider a sliding scale of deontology versus consequentialism (with the note that this is meant as a visualisation, not as a statement that all of moral theory can be crammed into one line)), and I keep a fake religion (i.e. one I know is false, because I created it, and it’s absurd) to inoculate the parts of me longing for something spiritual against actual religions.

      I hope this is okay, I’m going to tweak your thought experiment slightly to make it more palatable to me – ‘support life’ is being really fuzzy in my brain, so I’m replacing it with a scenario wherein I’ll die of dehydration once my supplies run out, because there is nothing edible there (and nothing that looks like life). That still gives me ample time to act on this planet and keeps the part of me that would otherwise be too busy wondering what I might e.g. be doing with my life if without companionship of any form quiet enough to think about this.

      I should probably say that my first instinct is to say ‘No, I don’t think so’. But as much as I trust my instincts in real life, that’s not very helpful as a response.

      Thinking about my principles out loud: I can’t murder anyone. I can’t harm anyone. I can’t vandalise anyone’s property (probably; this would give me pause, I admit, even if I had proof that the planet I was on was not claimed by anything sentient, but that’s not an ethical issue, it’s a psychological one).

      If your last point (‘no human or alien will ever reach the planet’) did not rule out observation of the planet (however detailed), I could, conceivably, do something fraudulent. This is vague to me, I admit, since I probably wouldn’t be around to reap the fruits of the fraud, even assuming for the moment I had some grand plan (I don’t) and ‘reaping fruits’ was actually possible. I would also probably again have the psychological problem that I might be missing something and the fact that there will be no other visitors to the planet does not hold as strongly as I thought.

      I see I’m still struggling with the thought experiment. I think my biggest problem in the scenario you describe would be psychological (assuming for a moment that the changes I made to the scenario are not preoccupying me, which they would be – interestingly enough I find this easier to ignore than the issues of prolonged life on this planet) – I can’t, right at this moment, conceive of a scenario in which I would feel absolutely certain that I could do nothing unethical.

      From a God’s eye view (pun not intended) on the scenario, I believe the answer is ‘no’. But I can’t vouch for that, given that the scenario feels foreign to me, and I thus could easily be missing something important.

      (This thread has so far been more useful to me than to you, I assume. I apologise. I really liked your question and hope someone with a sturdier/broader imagination comes along. I’d also be very interested in the answer, even though I’m not usually that interested in extreme ethical edge cases like these, since I think everyone makes practical exceptions to their ethical doctrine when the circumstances change so drastically (I should add I generally consider that a positive thing, but since I put ethics in a blender anyway, I guess that’s to be expected…).)

    • Princess_Stargirl says:

      Yes. Imagine I make a machine that will torture me for the next thousand years. Imo strapping myself into that machine is immoral. The “me” five thousand years from now is suffiencently distinct from current “me” I consider them different ethical agents. So it would not be ok to torture “future me.”

      *I am explciitly not allowing myself to creat AI or emulate myself as this makes the question too easy.

      • Ah, bless people with better imaginations than I have! (Granted, this is not difficult.)

        You bring up an excellent point. I’m not sure if I agree quite so strongly, given I am not sure I would draw that line between the ethical agents, but that’s not an objection on principle. It’s not even an objection. It’s a genuine ‘I’m not sure, let me mull about that’. So you’ve given me something to think about.

        (I kind of ruled out that sort of longevity in my version, of course, but that was out of inability to comprehend it, not out of objection to the premise in theory. I didn’t think it would be important. For that matter, I hadn’t thought that it would be important to assume I have any special abilities, but this is absolutely important when considering these things in raw theory like this. I think your example shows that very nicely.)

        I’m not the OP, of course, but thanks for your post. 🙂

    • lmm says:

      Yes. Isn’t that what it means to be deontologistic? It would still be wrong for me to do icky rule-violating things, like the standard example of having sex with a dead chicken.

      • Anonymous says:

        What rule would it violate to fuck a dead chicken? Consent? I don’t really see how that applies. As per universalization, I see no problem with someone fucking my dead corpse, just that it’s slightly disgusting as seen from my preference set, but not immoral or unethical. It’s basically masturbation with a morbid novelty toy.

        • lmm says:

          Some kind of generic sexual purity rule I guess. It’s a very common moral intuition across the overwhelming majority of human cultures (western college-educated liberal culture being a notable exception, and even there it’s not an enormously unusual intuition), but how it’s specifically codified varies. It’s explicitly not about any harm it would cause to anyone or anything – it’s just a rule, that’s what deontology means.

      • RCF says:

        “Yes. Isn’t that what it means to be deontologistic?”

        No.

    • Emile says:

      I don’t particularly identify as “detontologist” but I’d say yes; it would be immoral for me to do things that would harm me in the long term (out of lazyness), e.g. not brush my teeth, not take care of my diet, not maintain my life support system, not exercise, etc. – the kind of things I would blame myself afterwards for not having done.

    • Leo says:

      Alicorn (as of some years ago) considers it immoral to set fire to a tree just because you’re mildly bored, so that would be an example.

  46. FullMeta_Rationalist says:

    AUGH – how could I miss the coffee shop post!
    *
    Alfred Wegner enters a Starbucks.
    The barista says “Welcome to Starbucks!”
    “Funny, a barista in Brazil said the same thing.”
    *
    Henry Ford enters Starbucks and orders a venti.
    “What kind did you want? Plain, decaf, mocha…”
    “Any kind, as long as it’s black.”
    *
    Barack Obama enters Starbucks.
    He orders a three and a half dollar tall
    expresso and pays with a twenty dollar bill.
    The barista gives him back hope and change.
    *
    Ernst Zermelo is patiently waiting in line at Starbucks.
    Richard Dedekind cuts in front and orders a small coffee.
    Ernst interupts “Dick, that was not well-ordered.”
    “I’ve determined that a tall is too large –
    you don’t have a choice in the matter.”
    *
    Tom Swifty enters Starbucks and asks for
    the regular. “Uhhh… you mean a hazelnut?”
    “With cream and sugar,” Tom said sweetly.

  47. I wish Scott would start a podcast about epistemic hygiene called “Knowing What’s Not, With Doctor Scott”.

  48. Buckyballas says:

    Hi, new poster here. I have a question about moral philosophy. It seems to me that straight utilitariansm/consequentialism is pretty hard to follow (e.g. I wouldn’t kill myself if it saved 5 random people). I am toying with a moral philosophy that is a bit more selfish, where I assign random people’s lives a utility value of 1 and apply multipliers like 1000 to myself and 1001 to my wife and children, and something like 500 to close relatives, 200 to coworkers, and 1.1 to fellow Americans, etc. It seems to me that as long as people kept their multipliers less than some small number and greater than or = 1, then we could still achieve a nice outcome (nice from a traditional consequentialist perspective). Can someone point me to somewhere where this idea has been previously explored? Or possibly just poke huge holes in it? Thanks!

    • Jiro says:

      This is my criticism of utilitarianism, so you are not alone in this.

    • Creutzer says:

      The topic was discussed in (the comments of) this LW post.

    • Irenist says:

      Western familist ethics and Confucianism both have kind of a “weight near kindred more heavily” vibe.

      • houseboatonstyx says:

        Western familist ethics and Confucianism both have kind of a “weight near kindred more heavily” vibe.

        Someone else mentioned C. S. Lewis supporting that also, which he certainly did. It’s one of the eight major headers iirc in the long set of passages he quotes in his Appendix to
        The Aboliltion of Man
        , from Confucianism and most other cultures as well. Not just blood kinship, but neighbors, friends — in degree as they are closest.

        Hm, Evolution would probably endorse that as well. 😉

        • Irenist says:

          I think Lewis (and me, FWIW) endorsing a kind of “concentric circles” model of (at least some portion of one’s) charity highlights a tension that’s common, oddly enough, both to Christians and utilitarians. Both Christ in the New Testament and modern atheistic ethicists are constantly harping on how we need to universalize our charity, and to view the Samaritan, or the guy in Africa, as our “neighbor.” OTOH, it’s not an ideal that we have evolved to live out very well, and it’s an ideal that if implemented too perfectly would have us all focusing on Stakhanovite labor for abstract justice and welfare rather than building livable families and communities: we’d spend all our time saving lives, and no time enriching them. Or something.

          As a Christian, I tend to think about this as a tension between the Old Testament/City of Man and the New Testament/City of God. Not that OT=City of Man, or NT=City of God. Not at all. Just that the OT, like the City of Man, is centered on relatively achievable (although still pretty difficult to follow–look at Paul’s complaints about the Law) levels of natural flourishing (in the kind of thick Aristotelian virtue ethics community described by, e.g., MacIntyre), whereas the City of God/NT are both focused on a kind of “supernatural” flourishing, in grace and agape, typified by St. Francis, say, and in his atheistic way by a figure like Peter Singer: someone radically committed to universal, nonbiased benevolence.
          On this account, pacifism is what the NT/City of God obviously demands, but something like Augustine’s just war theory is an attempt to find a least bad way for us OT/City of Man types to live given that it’s neither realistic nor desirable for everyone to become a Franciscan tomorrow. The whole framework here is notably non-Kantian. I’m saying there’s a higher, saintly path, and then a sort of “muggle” path, and that both are somehow valid choices, even though one is higher–just as the OT is really part of the Bible, and nature has its own good (like Aristotelian virtue) independent of supernature (like the grace that allows some people to be radically saintly). Thus, it’s anti-Kantian in that, say, just war/pacifism aren’t universalizable maxims, but instead, if you will, maxims that differ based on something akin to “vocation” in the Catholic called to marry vs. called to be a priest kind of sense.

          As a Christian, I hear Christ saying (1) The kind of universal benevolence I am constantly preaching is, in God’s view, the minimal standard of being a decent person (2) You will notice that my Father’s standard is impossible/impractical, involving things like never lusting and giving away all wealth, which means you will fail (3) fear not, He forgives you anyway (4) keep trying to asymptotically approach this impossible standard with my help: the effort will improve you (5) NB that your asymptotic efforts will never “earn” you the status of good, because to be good is to be perfect, and that’s not going to happen for you in this life (6) but I’m perfect, so just tell the heavenly bouncer you know me, and that’ll get you in.

          Things like Scott’s recent posts on tithing as being a Schelling point for having done “enough” universalist good to be a member of the community are I think an attempt to deal with similar issues. (Obviously, tithing is a Schelling point Christians have used in exactly this context, too.)

          ETA: Scott’s sardonic “Newtonian Ethics” post has much the same tone of demanding we universalize our charity that I read in the jeremiads of the OT Prophets (who aren’t really “OT” in the sense I was talking about above, come to think of it) and in Christ’s bitter denunciations of our greed, smugness, and complacency. On one hand, it is right to be disgusted at these things. OTOH, we are weak, and if we tried to do 100% charity we’d burn out. It’s sort of like St. Paul’s line that it’s better to be celibate, but some are weak and should marry, which I take to be indirectly about this married vs. monkish, muggles vs. saints issue. It’s better to be Gandhi, radically seeking universal justice all day, but if all you can muster is giving 10% and otherwise just being some normal schmoe, well, that’s something.

          ETA2: Old Western books about Hinduism often describe a theory of life stages: student/householder/monk/mendicant. I have no idea if that was ever really a thing or just some orientalist misreading, but it’s an interesting ideal: spend the first half of your life perpetuating the naturally good community, and the second half seeking the more radical, “supernatural” goods that transcend the goods evolution has primed us to seek, (e.g., loving your enemies transcends just loving your family and friends).

          ETA3: Another way to think of it. Tit-for-tat is, IIRC, a remarkably stable game theoretic strategy. And “natural,” evolutionary good of the sort that keeps society going is of that sort. Just war theory, favoring kindred, etc. To be a pure game theoretic always-cooperator all the time is nobler (IMHO), but not all are called to it all the time. If all were, society would collapse when the first game theoretic cheater showed up, which wouldn’t take long. But because such a “dovish” strategy isn’t evolutionarily stable, there is something “supernatural,” if you will, about those who find a way to live like that.

          • houseboatonstyx says:

            Aiui, the NT exchange about the good Samaritan woman went something like this.

            Q: “In ‘love thy neighbor’, who is the neighbor?”

            A (tl;dr): “Her neighbor was whoever she happened to meet that day.” [ Even though the man she helped, belonged to a far country and considered her people as enemies. ]

            Among the Hindus and Jainas I know about, “student/householder/monk/mendicant” is definitely a thing, and the sequence is important. Householder — raising a family — is perhaps the main duty for most people in their current incarnation. Student is preparation for householder, and monk and mendicant are for when you’ve completed householder duty, however long that takes. Some people have years and resources to go on to monk, few to go on to mendicant. In Buddhist and yoga traditions, each set of moral duties prepares you for the next stage (sort of like high school/ Masters Degree/ Ph.d). So there is no competition in essence. Everyone should cultivate feelings of unconditional goodwill toward all the world, but in householder’s use of practical resources and time, the practical needs (and reasonable desires) of one’s own closest come first . Following concentric circles on the ground — while on the same day in meditation, being one with the sun that shines equally on all.

            To use utilitarian terms for what is probably a utilitarian heresy — look at how many units of happiness (utilons) can be created with a windfall of $X. Sending your own child to a much-desired music camp will bring happiness to her and your whole family, some of it permanent and growing. Dropping the $X in a bucket of hundreds of people in misery won’t do much for any of them, but will create negative utilons for your daughter — thus, a net negative.

    • Shmi Nux says:

      Pure utilitarianism is for machines, humans act mostly based on virtue ethics, which may be shaped by other normative ethical considerations, including utilitarianism. If someone tells you they are a pure utilitarian, they are either lying or not in touch with their own decision-making process.

    • In these discussions, it is key to specify whether ‘utilitarianism’ is meant to refer to a decision procedure or a standard of rightness. Considered in the latter sense, utilitarianism is the theory that you ought to act so as to produce the most expected welfare. But it would be incorrect to conclude form this that a utilitarian agent should always decide by explicitly calculating which of the acts available to him or her are welfare-maximizing. Whether I should adopt some particular decision procedure is itself a question which is answered by considering its effects on the welfare of sentient beings. And it may well be the case that, to maximize welfare, people should act by trying to follow some non-utilitarian moral theory. This does nothing to discredit utilitarianism as a decision procedure.

      By failing to distinguish the two relevant senses of ‘utilitarianism’ noted above, critics engage in a kind of strategic equivocation: first they establish that we shouldn’t follow a utilitarian decision procedure, and then they conclude that utilitarianism as a standard of rightness is false. Clearly, the inference is invalid, because the terms are used equivocally.

  49. Anonymous says:

    Can someone succinctly explain the concept of countersignaling to me? I’m not sure I completely understand it. Is it just signaling that you’re not a certain kind of person?

    • Daniel Speyer says:

      No.

      Countersignaling is sending the message “I’m so awesome I don’t need to demonstrate how awesome I am.”

      Imagine walking through a prison yard, and seeing many people glare at you, show off their muscles, or generally be slightly threatening to show that they’re dangerous. And then there’s one guy casually reading a book who doesn’t even look up. Do you immediately conclude that he must be the most dangerous of the lot? That’s successful countersignaling.

    • FullMeta_Rationalist says:

      Someone who counter-signals X will often simultaneously signal X in subtler ways. Because the X they exude is so obscenely amped up, signals of X mediated by subtler channels will pierce through the ambient noise without effort. Whereas a person who is X (but not intensely X) will struggle to signal their X – even with the advantage of a louder channel. E.g. you can tell a high-brow hipster from a middle-class casual because a hipster has low status clothes. A hobo also has low status clothes, but may smell less pleasant than the hipster and not own a macbook.

      Showing effort is low status. But not as low as no status.

      Related is the handicap principle. IIRC, the plummage of a male peacock’s tail is a signal of health (because beauty/symmetry is low entropy). But it also handicaps its ability to escape predators. If you removed a particular peacock’s plummage (and given the adult male peacock had survived up to this point), he’d be able to easily outrun predators (and competition). But how else would he be able to signal his health to the ladies? He doesn’t need more points in the speed tree, so he takes his extra skill points and puts it in the health-signalling tree.

      Additionally, this pressures rivals who want the benefits of plummage, but are slightly slower and will certainly be eaten if handicapped. Higher handicap = more ladies = more kids with higher handicap. This is probably why human teenagers show off dangerous, challenging stunts for apparently little gain.

      Peacock is a min-maxer. Peacock plays to win.

      Consider Scott’s cellular automata of status. From the middle of the hierarchy, it’s pretty cool to distinguish oneself from the bottom because it turns a homogenous status-scape into a continuum. From the top of the hierarchy, the status hierarchy is advantageous because it turns a continuum into three discrete levels. As you go up the hierarchy, it becomes more and more advantageous to distinguish oneself with more precise signals. Of course, the bottom dwellers don’t like it. But if they had the resources to change things, they’d have sooner acquired the resources to move up the ladder.

  50. eqdw says:

    Berkeley meetup yes please. But not that weekend, if we can; I’m goin to Tahoe then

    • Elizabeth says:

      Mike Blume and I are also in the “going to Tahoe that weekend” contingent.

      • eqdw says:

        “Mike Blume” sounds familiar….. by any chance is your trip being organized by a Kevin?

      • Scott Alexander says:

        Is this just a coincidence, or is there some kind of large-scale Tahoe exodus then that makes this a bad weekend even beyond normal considerations of “every weekend will have one or two people with a problem”?

        • Elizabeth says:

          Pretty sure it’s just a coincidence. And I know a Kevin who does snowthings, but I don’t think he’s coming on this trip.

        • Anonymous says:

          Maybe Tahoe is the center of a nascent Evil Scott resistance group (see previous comment thread).

  51. This question is mostly for Scott and for entertainment, but anyone can answer if they feel like it:

    Someone (with your blessing) is writing a novel and basing the main antagonist on you and discussing how to design and write about this character with you. You have all the freedoms to help shape it. (You can even pick super powers if you like, but this is mostly about personality traits.) You both want something that is as close to your current personality as possible – that means either you’re designing circumstances that would bring out your darker side quite strongly, or you’re making changes to traits that you think are keeping you in line as a decent human being. You definitely want something that feels internally consistent.

    How would you tweak the world or your personality to make a villainous you?

    (Disclaimer: I promise I am not writing a novel that has an evil Scott.)

    • stillnotking says:

      I’ve often thought that if anything could turn me into a supervillain, it would be a Willow Rosenberg-style sudden telepathic awareness of the total amount of suffering in the world, combined with the power to end that suffering by killing everyone.

    • Irenist says:

      Too bad you’re not writing that novel. Scott could be The Steelman, (not to be confused with Stalin or the Man of Steel) who steelmans all the world’s worst ideologies until they bring down civilization.
      Hero: Why won’t you stop promoting these things?
      Steelman: I just like impartially considering ideas.
      Hero: But you’re literally destroying civilization!
      Steelman: As a consequentialist, that troubles me. But I don’t want to be rude to proponents of ideas that destroy civilization. Should I be rude? What’s your argument for that? Let me help you flesh it out….

      • That would certainly be a villain to behold!

        …darn, you’re making that book-writing scenario awfully tempting. But alas, I don’t have Scott’s blessing, and I am very sure I wouldn’t get it. 😉 (Not to mention I’d have to come up with a plot! Though I suppose that could just be thrown into the next open thread as a separate Completely Innocent question, muhahaha.)

    • Paul Torek says:

      How evil do I have to be? If I just have to do something horribly wrong, without being an incarnation of malevolence, then just tweak some of my obsessions and amp them up. For example, make me a bit more of a bioconservative than I really am, and put me in a world where everyone and his brother are uploading and transforming the world into a place where mere biological humans can barely survive, if at all. Then I will become a genocidal monster (in deed, but one who sees himself as saving humanity). Actually, depending on the details of “uploading”, you don’t have to tweak my views at all, and y’all (the LW majority) would be the genocidal monsters. But I digress. Unless you like the Tragedy approach; then I’m not digressing.

      If I have to be more Voldemort-like, then a spiral of rage seems like the most psychologically plausible way. Have me get in a fight with someone and lose. Then escalate and lose again. Meanwhile my nemesis becomes powerful and important, maybe he’s elected President or something. For maximum plausibility make my nemesis a basically nice guy who does one kinda asshole-y thing, albeit negligently, to start the first fight.

      • “How evil do I have to be?”

        Finally someone is asking the important questions. 😉

        More serious response: My roleplay group actually has a villain a little bit like the one you describe in your non-Voldemort scenario. She’s actually still transhumanist, but doesn’t believe that consciousness can reside in algorithms alone, and is basically slowly turning the uploaded people’s lives into hell because she understandably does not like the legal consequences of non-persons getting the rights of persons. She wants to find ways to prove that they’re not and that’s involving a little sabotage.

        I may be on the other end of that argument, but nonetheless I find her quite sympathetic. I mean, outside of fiction, I would certainly think otherwise, but thankfully that’s not important.

        Your scenario offers a much stronger case, of course, and I’d accordingly be much more sympathetic to you as a villain than I am to the aforementioned character. Clearly you need to write a book about this scenario now, so I can be a fan.

        tl;dr: I really like your answer.

  52. Can anyone give me a good example of a Ramsey sentence?

  53. J says:

    Scott, your lit survey on depression is really wonderful. Could you do the same for anxiety, both chronic-low-grade and acute? Anxiety is the most anti-inductive thing I can think of, which makes it a real challenge to handle as a rationalist.

    • Scott Alexander says:

      I actually had half of this done when I found a couple extra papers with more stuff that meant I had to rewrite it, and I never finished doing so. I’ll get around to it sometime.

      For now, http://www.hindawi.com/journals/ecam/2012/809653/ is pretty good.

      Also, I have had *remarkable* success treating borderline people’s anxiety with inositol; I don’t know if it works in the general case but it might be worth a shot if you don’t have bipolar disorder (which it can make worse sometimes)

      • Anonymous says:

        Scott, I’ve been taking 4-ish grams of inositol daily (mainly for non-mental-health reasons) and it just makes me SO F***ING HAPPY. I do about 4 g of myo-inositol plus 90-180 mg of d-chiro-inositol. I don’t have a borderline diagnosis.

    • Sarah says:

      Beta blockers are underappreciated as anxiety meds. They aren’t strong drugs, but they do literally nothing except slow your heart rate. No mood or cognitive distortions. (Obviously, they slow your heart rate, so check contraindications if anything is wrong with your heart.)

  54. Vulture says:

    I finally wrote something on my wordpress blog. I can’t really ask for gentleness, since it is itself kind of an extensive of critique of certain elements of the LW-sphere, but uh… I think it will probably be interesting to people here.

  55. Anonymous says:

    If the nets cure starvation too, that just means we need to send more nets.

    • thirqual says:

      That’s when you actually go read the article and see the long-term consequences for fish populations (and of course for the human living on the use of this resource), the possibility of contamination by the insecticides used to treat the nets (need to read more on this, the guidelines for how to wash the nets are too contradictory and fear of ‘chemicals’ often overstated), or the violence problems created.

  56. Anonymous says:

    Given the absence of universality, the occurrence of greater male variability and scarcity of top-scoring females in many, but not all, countries and ethnic groups must be largely due to changeable sociocultural factors, not immutable, innate biological differences between the sexes.

    This seems like a conclusion which is far too strong. It’s by no means obvious that the difference in variance between males and females should pass the threshold of significance in all populations, especially between populations with different population genetic parameters and different ancestry. One thing that worries me is that the Scandinavian countries have much lower genetic diversity than most countries — if the mean and variance of mathematical ability is under genetic control, you might expect the difference in variance between males and females to be more difficult to detect specifically in those populations.

    Overall, the paper treats both biological and social models in a rather naive fashion. The membership of international math olympiad teams is not based solely on mathematical ability, and not immune from social effects that are unrelated to true mathematical ability. It speaks more to whether there are women of high ability (which the variance hypothesis does not deny) than whether biological or social influences drive the higher male variance in mathematical ability observed across the majority of countries.

    • houseboatonstyx says:

      whether there are women of high ability (which the variance hypothesis does not deny)

      I don’t see this spelled out in most discussions of math variance. Maybe something like this would make it clear.

      Imagine a large sampling of male basketball players, and imagine there’s some correlation relationship between each member’s height and his scores. Then line them up according to height. 7’6″-7″ is the tallest group. Then 7’4″-5″, and so on down. Say the median is 5’9″-10″, and that is the largest group, and it has 50% blacks and 50% whites. As the line gets taller, there will be more blacks and fewer whites in each group. In the very tallest group, there may be only 10% whites. But that does not mean “Whites can’t play basketball.”

      In each group there will be some whites. And some short people can learn to play better than some tall people.

      So to build a good team, going by “the average black is taller than the average white” is not a good idea.

  57. thirqual says:

    Mosquito nets used for fishing rather than protection against malaria in East Africa, with worrying consequences — or as I wrote on tumblr, Moloch getting his grubby hands everywhere he can.

    • RCF says:

      So, are the people giving the nets in error in thinking that the people are better served by mosquito nets than fishing nets? Are the recipients in error in thinking that they are better served by fishing nets than mosquito nets? Or are there other effects (the reference to Moloch implies that you think there is some sort of coordination problem)?

      • thirqual says:

        Several things here:

        – the use of nets for fishing is, as far as I know, still seen as marginal. This is something to look at and wonder if the strategy is optimal rather than a stop doing this (AFAICT)
        → giving nets still a net good thing

        – mosquito nets seem to replace traditional nets, partly due to lower costs and more edible things caught in it → short term, it looks like a great tool if you do not want to starve

        – long-term effects are probably destruction of breeding grounds, decrease of the amount of fishes that are currently a major source of food and money to the population → probably a good idea to stick to the use of traditional nets, dangerous long-term effects to the use of mosquito nets for fishing

        – the article writes as if the use of nets is to the exclusion of their use as malaria nets, rather than the existence of a competition between the roles, and not enough details → hard to get how that impact negatively the prevention of malaria

        – shorter-term problem for traditional fishermen: even if the pressure to avoid starvation is lower, they may still be forced to close business because their operating costs are greater than the ones of the mosquito net fishers → destruction of long-term, possibly sustainable business in favor of short-term, destructive methods which also are not likely to create local network of businesses.

        Not an economist nor an ecologist, though, so probably missing things about this.

      • Mary says:

        The problem is that mosquito nets are like seat belts. They are only intermittently useful, and you might not even realize it.

  58. DrBeat says:

    Until then, please do me a favor and just take it on faith that you are a valuable human being who is worthy of existence.

    Why do people do this, why do they expect it to work, and does it ever?

    Things like this are meaningless. They aren’t even words, they’re noises. This is the same kind of thing as random Tumblr or Facebook posts passed around thousands and thousands of times about how great “you” are and how brave “you” are and how wonderful “you” look. All written by people who weren’t aware of the existence of most of the people reading them, much less know them enough to say if they are great or brave or good-looking.

    Why am I expected to feel better because of positive-affect noises that have no bearing on me or my existence?

    • Creutzer says:

      No, these are not meaningless noises. Scott is saying that he doesn’t disapprove of people who are not as intelligent as some high threshold. This is relevant because he is a high-status person whose even hypothetical judgment may matter to people on an emotional level. Some might find it helpful to know that not everybody who is highly intelligent despises those below him, even if they haven’t met that high-status person personally. Just because you don’t care doesn’t mean nobody will.

    • Izaak Weiss says:

      As an analogy – Imagine Scott told me that I shouldn’t take one of the drugs I’m taking, because of a bunch of stuff he is writing in a blog post that he’ll post soon. I respect Scott’s judgement that I’ll stop taking the drug until I learn more about it.

      • DrBeat says:

        Imagine if Scott leaned out his car’s window and told a random person who he has never met and knows nothing about that they should stop one of the drugs they are taking because of a bunch of information he’s putting up in a blog post soon.

        Even if that person reads Scott’s blog every day, his advice means nothing, because it is given without consideration or knowledge of the person receiving it.

        • Leo says:

          *If* you take drug Foobar, *then* you should stop, because Foobar is basically always harmful on net.

          *If* you’re worried about being worthless because of your intelligence, *then* you should stop, because intelligence has very little bearing on inherent human worth. If you’re worried about being worthless because you’re mean and dishonest and hurt people who help you, sure, you might have a point (or you might be misjudging yourself, advising you would require knowing you personally).

    • Scott Alexander says:

      Because it’s an IOU for an argument that I believe I can make. Conservation of expected evidence – if you expect my argument to change your mind later, your mind should be changed now.

  59. Tom Scharf says:

    Why do Asian males outperform African American females in high end science and math by huge margins? Did you have an emotional reaction to this question? You should ask yourself why just asking this question makes you feel uncomfortable. Is it because you question whether it is true, or is it because you worry about why it is true and how this conflicts with your worldview?

    Gender and IQ. I don’t get it. On multiple levels.

    1. Why the emotional entanglement if males just happen to slightly outscore females here? Or group X over group Y?

    2. Why do the values (errrr….politics) of some groups demand that this be disproved?

    First off, my daughter just recently completed standardized testing and landed perfect scores in Math and the Math II subject test. I really don’t care if more males per capita scored perfectly. Why would I? I don’t get it.

    My daughter is smart, my dog is dumb. The specific breed of my dog is not known for its doggy IQ. I feel absolutely no need to run out and do studies to counter act some perceived stereotyping that my dog’s breed is not equal to all other dog breeds, and insist that everyone states all dog breeds are 100.0000000% equivalent in intelligence and shun anyone who states otherwise. Why would I have an emotional need for this?

    Creationists are vilified for rejecting Darwin’s theory of evolution. But here we have almost an equivalent process of denial that wants to state that all groups evolved 100% equally in intelligence over the last 50,000 years when the groups diverged out of Africa or that genders don’t evolve differently genetically. There sure appears to be quite some large differences between apes and man. This process just stopped?

    Men are stronger than women, taller than women. Do men excel over women at basketball due to socioeconomic factors? Why is there no emotional entanglement here? Why must differences in IQ or some specific mathematical ability be dismissed as unenlightened?

    I understand that many think there is simply a “search for the truth” here to see if IQ did in fact evolve differently. I suggest what we are seeing instead is a search to reach today’s politically palatable answer and this corrupts the science process. You don’t think there is emotional entanglement? Try bringing up The Bell Curve in a conversation.

    If men and women were equal than it would be likely that an equal number of independent studies would show women slightly ahead, and the other set show men slightly ahead. What we get is men almost always coming out on top and if a study does find near parity, then this specific study is hailed as truth and having disproven every other study. We have all seen this in action in politically controversial topics such as GMO’s, etc. Then there is the game of controls.

    The strawman of “DO FEMALES EXIST WHO POSSESS PROFOUND MATHEMATICAL TALENT? shows how warped this discussion is. Who is charging that this is not the case? Anyone? Sure there are cultural factors that also amplify this discrepancy. Very few people reject this. I am grateful my daughter is growing up in the world today instead of 50 years ago.

    The male / female gap is pretty small by any measure, but it sure appears to legitimately exist. I really don’t have a problem with it.

    • stillnotking says:

      The steelman version of the don’t-research-racial-and-gender-differences position runs something like this:

      There is a potential social detriment to identifying such differences, rightly or wrongly. This negative effect is a nocebo commonly known as “stereotype threat” — people underperform when they are primed to believe they will be bad at something. Whether this belief is rational or not is irrelevant, because it’s a fact of human psychology that will not change. There does not seem to be a commensurate “stereotype benefit”, and even if there is, it accrues to already-privileged groups at the expense of needy ones.

      OTOH, there is no potential social benefit to identifying the differences. If the science is bad, this is obvious, but even if the science is good, it is not useful to be able to say that group X is smarter than group Y. There are no acceptable policy implications following from such a conclusion, in a liberal-democratic society. It’s even possible that acceptance of such conclusions could undermine liberal-democratic values themselves (historical examples go here).

      So the only reason to study racial and gender differences is a Socratic commitment to truth for its own sake. Opponents tend either not to value such a commitment as highly as they do social justice, or believe that research on race/gender differences is politically motivated, given the existence of more fruitful avenues, and should be uninteresting to anyone but racists and sexists.

      I disagree with the above position, but I don’t think it can be dismissed out of hand. It is a serious objection, not just a knee-jerk emotional response. (Although it is often that, too.)

      • Anonymous for this one says:

        I’d go farther than internal “stereotype threat” and say that people are afraid of sexism and racism making a comeback. What does that mean, in a world where even the truth was a little sexist and racist? Basically that people aren’t smart enough to avoid oversimplifying the truth. Our very language is unsuited to the necessary precision – we can’t express probability distribution functions in simple English, so we say “men are taller than women”, and if that implies more than we meant it to then oops. At least with height you can quickly wipe out your priors with a short observation, but with intelligence and personality and such, precise observations are difficult and prejudice becomes too subconsciously tempting a heuristic: just rely on priors alone and don’t sweat the observations.

        So it’s better to go the other direction: believe every group has the same innate distribution of abilities. That still might be an oversimplification, but at least it’s *less* of an oversimplification than the “A are more X than B” version.

        Ironically, the egalitarian heuristic was ruined by the egalitarians. “Start with a prior belief that all groups are the same, and override that belief with individual tests whenever a characteristic is important” would be a great way to live… if only it weren’t for the concept of “disparate impact” in laws, policies, and culture. If your unprejudiced process leads to disparate results, and the only politically correct explanation is that this proves you to be a secret bigot who needs to be fired, then you might be open to other theories.

        • stillnotking says:

          Right. They’d also point out that invidious race- or gender-related priors are likely to be used in situations where direct observation is impractical (e.g. looking at resumes of job applicants), even if laws or norms exist to dissuade this. Like you said, it needn’t even be a conscious effect.

        • Irenist says:

          IIRC, Thomas Sowell blames “disparate impact” thinking for credential inflation. His idea is (again, IIRC) that with employers unable to just directly test aptitude, they started using secondary (and then tertiary, and soon graduate) degrees as proxies. As part of the conservative minority within the African American minority, Sowell always strikes me as someone, like Clarence Thomas, who thinks he would’ve done just fine under a regime of straight-up intelligence tests, but instead gets tarred with assumptions of his success being part of affirmative action tokenism because liberals have stopped employers from giving those tests anymore, or something. I can see that being really infuriating: feeling like you owe your success to being smart, but living in a regime that won’t let you prove it.

          • Anthony says:

            Slightly related – I once saw an expensive car (BMW or Mercedes) being driven by a well-dressed black woman, with the license plate “QUALFYD”.

          • Mary says:

            ” Sowell always strikes me as someone, like Clarence Thomas, who thinks he would’ve done just fine under a regime of straight-up intelligence tests, ”

            Do remember that Sowell was old enough to have graduated from Harvard in 1958. That is, before AA.

            He started at Howard University, and transferred to Harvard, and a professor sent him off with the injunction, “Don’t come back here and tell me you didn’t make it ’cause white folks were mean.”

      • Irenist says:

        “OTOH, there is no potential social benefit to identifying the differences.”

        Well, if HBDChick and Greg Cochrane are on the right track about exogamy being the driver of greater intelligence of, say, the nuclear-family-forming English over us clannish inbred Irish, then I guess draconian laws against cousin marriage might be a potential intervention.

        • lmm says:

          Did you skip the “in a liberal-democratic society” later in that paragraph?

        • Deiseach says:

          Wait – WHAT?

          How did I miss this wonderful chance to be outraged? English people (or pro-English people) repeating the same old dressed up in a new suit of clothes “virile Anglo-Saxons versus effeminate Celts” line?

          I have got to see what was said, then I can come back here and insult the English properly 🙂

          • Irenist says:

            Deiseach, we can’t discuss it in the open thread, but honestly, some of HBDChick’s writing about us Irish is fun reading in a vein not dissimilar to Scott’s: well-written analytic speculation. Googling her name and our ethnonym should lead you to her. There’s a bonus “all these good things came from Catholic canon law” angle you might enjoy very much, too.

          • Anonymous says:

            Hmm – some of it is certainly fascinating, to see an outsider speculating about Irish society (her conclusions seem odd to me, but then they would, wouldn’t they?)

            I’m very surprised, though (in the few posts I’ve read so far) that she’s made no mention of what I think was a huge influence on what she terms the rate of outbreeding: primogeniture (she does a bit of puttering around with the fine and cineál definitions but doesn’t seem to mention much beyond a bit of the ancient law texts re: inheritance rights) – the notion (rightly or wrongly) that the Normans introduced primogeniture after their invasion of Ireland.

            Inheritance of the name/title/property/land was by and through the eldest son mainly and often solely, with (in later centuries) younger sons left with little or nothing certainly would encourage ‘outbreeding’, as distinct from the system of tanistry and the derbfíne succession rights. If you have to make your fortune, one way is by marrying an heiress, which necessarily will have you looking outside the family group.

            I don’t think I necessarily agree with her notion that Irish society was more “clannish” than the English of the time, or rather, that the English were more outgoing etc. I think they did a lot of inter-marrying and inbreeding of their own, but I do think that historical pressures were different.

            I don’t know, I’ll have to think about it a bit more 🙂

          • Deiseach says:

            The above Anonymous was me 🙂

          • Anthony says:

            I believe that HBDchick is of primarily Irish ancestry, P ~ 0.75.

          • Irenist says:

            Anthony,

            That doesn’t surprise me. I’m of 100% Irish ancestry, and theories about the Irish, on average, having once had/now having low average IQs neither pick my pocket, break my leg, nor lower the IQ of me or my family.

            Unless someone was actively trying to use such theories to justify oppressing me and mine (as such theories have in fact, of course, been used against other groups in other times), I’m just not going to get upset about it when it’s more fun to read the blog posts and ponder them. Litany of Gendlin and all that.

      • Tom Scharf says:

        And this is where I sometimes feel I am just on a different planet.

        I don’t want to have my feelings protected or have truths manipulated because some group unilaterally has decided this is better for society. It feels so…dishonest.

        I understand that the science of “stereotype threat” may be legitimate, I take no position here. But this doesn’t justify the science of intelligence and gender to start consciously or unconsciously altering or suppressing evidence because of the perceived interpretation of results of this evidence.

        For the sake of argument, assume the racial education gap was found to be innate (sigh…for the sake of argument!). How society deals with this would change. It might be counter intuitive but perhaps affirmative action would get a lot more support on the right. This may now be seen as fairly altering the system for a legitimate disadvantage. Much of the opposition to AA is based on a perception that it is unfairly altering the system due to poor life choices or lack of personal responsibility. Maybe that is a delusional hypothetical, but it is not lost on anyone that the failure to identify the real causation of a problem results in ineffective solutions to be attempted.

        I did contemplate the problem of if you discover the causation and you can’t fix it, well that is a bummer. But you can compensate in other ways. Women may not be genetically ready to compete in the NBA at parity, but they formed their own league. One can imagine a world where there are laws requiring 2 women on every NBA team and a bunch of people proclaiming the only reason that women don’t make the All-Star team is sexism and nobody passes them the ball.

        Don’t mix studying racial and gender differences with solving sexism and racism as cultural issues. They aren’t the same.

        • Anthony says:

          For the sake of argument, assume the racial education gap was found to be innate (sigh…for the sake of argument!). How society deals with this would change. It might be counter intuitive but perhaps affirmative action would get a lot more support on the right

          In The Bell Curve, Murray and Herrnstein straight up endorse practicing a degree of affirmative action – the specific suggestion is to give blacks a score boost equal to half the black-white group difference when making decisions about college admissions, etc.

          I’m not sure how the politics of this would play out, but if the political culture accepted that the gap was innate, and simulataneousl rejected disparate impact and endorsed this sort of score-norming, most of the battle would be over making adjustments to the actual level of norming. Which wouldn’t be that bad in reality, though it would probably be just as noisy as the current debates are.

        • Wrong Species says:

          If the racial gap is innate that would decrease the support for affirmative action. Affirmative action is based on the idea that black people are discriminated against and can perform just as well at a job if given a chance. If that’s not true, then all that’s happening is that less competent people are given preference over the more competent.

          • clathrus says:

            That depends what is meant by affirmative action. I would be in favour of government subsidised embryo selection ( https://www.cog-genomics.org/static/pdf/bga2012.pdf ) to correct the differences in aptitude between different population groups.

            Let’s wrest eugenics from the far-right and bring it into the progressive arena.

          • Irenist says:

            Let’s wrest eugenics from the far-right and bring it into the progressive arena.

            Where it began.

          • Anonymous says:

            “Let’s wrest eugenics from the far-right and bring it into the progressive arena.”

            Don’t fool yourself. It always was a progressive agenda. It was Oliver Wendell Holmes who declared that three generations of imbeciles was enough.

            You tried to blame it on us when it got ugly, but it was you all the way.

        • Anonymous says:

          >affirmative action would get a lot more support on the right. This may now be seen as fairly altering the system for a legitimate disadvantage.

          That would never happen. I can’t even think of a good reason that *should* happen. If (FOR THE SAKE OF ARGUMENT) all interethnic differences in ability were genetic, there is still no reason to treat Person A from Group 1 different from Person B from Group 2 if both have equal ability.

          What *would* happen is that people would discriminate even more, now that science has vindicated them in their beliefs.

          • Anonymous says:

            (PS that’s not in support of censorship – it’s only against your prediction of what would happen.)

      • Gbdub says:

        I might buy the “don’t investigate this – it’s dangerous!” line of thinking if we ignored the issue completely and just let the demographics fall where they may.

        But we don’t. People DO investigate the differences and conclude that they must be caused by discrimination, and propose solutions based on that conclusion. Solutions, like affirmative action, that strike me as utterly indefensible if discrimination is not the actual cause. In that case you HAVE caused discrimination by studying, just not in the direction you expected.

        If we lived in a world where no one noticed or cared about gender differences, then I’d agree we should keep it that way. But Pandora’s box has been opened, and bad conclusions have been reached, so the only recourse is to keep studying until we have the correct answer.

        • Tom Scharf says:

          “People DO investigate the differences and conclude that they must be caused by discrimination”

          I would challenge you to read 100 articles on the racial education gap in the MSM and see how many you can find that even contemplate the gap may be partially caused by genetics. My guess from my experience is zero. This discussion is verboten in polite society.

          If this was an honest science discussion about this issue, genetics would be at the minimum brought up regularly as a possible causation, if only to be summarily dismissed by some. But it is never even mentioned. There are definitely some irregular social dynamics at work in the scientific method here IMO.

          I find the argument that lower math IQ scores are caused by discrimination to be unconvincing. That the claim is made that genetics are 100% excluded is really, really unconvincing. At best I would say the claim could be made that the causation has a lot of uncertainty.

          • Gbdub says:

            I’m not sure why you’re challenging me, because I think we’re in agreement – we are “investigating” the gaps but doing a poor job of doing so because concluding “discrimination” is rewarded while concluding “genetics” is sanctioned. Therefore, I favor fairly investigating the “genetics” hypothesis.

            Apologies if I did not clearly convey my intent.

      • Scott Alexander says:

        “OTOH, there is no potential social benefit to identifying the differences. If the science is bad, this is obvious, but even if the science is good, it is not useful to be able to say that group X is smarter than group Y. “

        Even aside from what some of the other people here have said, remember, society is fixed, biology is mutable. We have almost enough technology to start enhancing genetic-intelligence right now. I expect the problem is now, or soon will be, more political than technological.

        The first thing it would be interesting to be able to say is “You know the difference between this race and that race? This race is doing pretty well, comparatively, that race is poor and miserable? That’s what an average difference of 15 IQ points does. We can give you 50.” That might make people take notice, especially in the face of people trying to obfuscate the issue and talk about how IQ doesn’t matter because it’s just about solving silly math problems.

        The second thing it would be interesting to be able to say is “You know this social problem of inequality that we all want solved? There’s only one way to solve it, and it’s allowing this technology. Your move.”

        • stillnotking says:

          I wholeheartedly agree that biology is more “fixable” than society, but it’d be putting it mildly to say that the idea has little traction on the political left. Or the political right, for that matter.

          The popular conception of genetic engineering is approximately Gattaca, to the extent it’s considered at all.

          • Wrong Species says:

            I’m beginning to suspect that popular science fiction can be harmful. People who don’t think about genetic engineering but have seen Gattaca are automatically going to assume that it’s dystopian where if the movie didn’t exist they might not even have an opinion.

      • drunkenrabbit says:

        If we could all come to some kind of gentleman’s agreement where we all pretend to believe in the absolute equality of sex and race, then I might be okay with your steelmanned position. The problem is that most people actually believe it, and start doing silly, wasteful, or outright harmful things in efforts to “close the gap”. It’s really difficult to have a serious policy discussion on education, for example, when the discussion is dominated by people fruitlessly fixated on closing the black-white achievement gap, rather than just working to raise everyone’s outcomes overall.

  60. namae nanka says:

    Hyde is a ‘veteran gap buster’ according to La Griffe du Lion and Mertz was mocked for her statistics by Ron Unz when she dared to criticize his American Meritocracy article, which then spilled over to Andrew Gelman’s site.

    As some folks have already brought up the Minnesota statistics of Asian-Americans(and Andrew Gelman’s site), I will add to it that they were only for the 11th graders. So for one year, in one grade, in one state, the asian-americans(a conglomeration of many different ethnicities) end up with more girls than boys in the top 1% and since then it has been often paraded around as asian-americans(notoriously unfeminist) doing away with gender inequality while the whites can’t.

    Their paper mentions Guiso et al for showing that the gender equality of a country(as gauged by the Gender Inequality Index, which opens another can of worms) tracks the gender gap in mathematics. La Griffe showed that it wasn’t true and a recent comprehensive debunking came from Stoet and Geary, 2013. However, both papers show reading performance to be correlated with the maths performance so that the two gender gaps in reading and maths are inversely correlated. The better the girls do in reading, the smaller the gender gap in maths becomes and since the latter is smaller, sometimes girls end up better in maths while boys are always behind in reading.

    The cherry picking of a few countries with equal variances or even higher for girls is amusing. For what commonality do the cultures of Iceland, Thailand, and the United Kingdom have in order to reach this landmark? Secondly, their use of 17 year olds’ data from TIMSS 1995, in the variability debate is amusing since the gender gaps in that dataset were rather obscene.

    Analysing the variability debate helps if you consider it from the other viewpoint. If some boys don’t read well enough to do maths, then they will invariably stretch the male distribution. If the country scores rather high, more males are likely to come up against the ceiling. Then there is skewing of the curve so that bell curve assumptions don’t apply, I gave this reason for another of Mertz’s paper showing a few countries with higher female variability but very poor scores.

    http://www.academia.edu/393769/Vos_P._2005_._Measuring_Mathematics_Achievement_a_Need_for_Quantitative_Methodology_Literacy

    So it’s possible that males might not be variable on aptitude for maths after all, but rather on attitude towards studies and due to the gap in reading skills. The male overrepresentation at the top remains regardless, which of course is the crux of the issue for the gender equality advocates.

    The latest post on my blog is on the same issue summarising my views rather pithily. Even though boys might do better on standardised tests, it’s no guarantee that they aren’t underperforming, though not to level of school grading.

    I don’t mention the verbal, quantitative and spatial split there though. It’s another interesting avenue to pursue considering that spatial ability seems to affect the choice of going into engineering and the like, and is not evaluated and considered in the current schooling paradigm. And because it’s not exactly a STEM gap, but,

    Surprisingly to me, most of the STEM majors aren’t doing as bad gender disparity-wise as I expected. 40-45% of the degrees in Math, Statistics, and the Physical Sciences were conferred to women in 2012. Even better, a majority of Biology degrees in 2012 (58%) were earned by women. This data tells me that we don’t really have a STEM gender gap in the U.S.: we have an ET gender gap!

    http://www.randalolson.com/2014/06/14/percentage-of-bachelors-degrees-conferred-to-women-by-major-1970-2012/

  61. Cauê says:

    There are people who have thought about epistemology, Bayes, rigorous formulations of Occam’s razor, the size of hypothesis-space and the problem of privileging the hypothesis, who know about the heuristics and biases body of research, and remain religious.

    This confuses me so much. I don’t understand it, and really wish I did. Is it a fundamented rejection of some basic premise? Is it some fantastic body of evidence?

    I’ll thank anyone who points me to people explaining their own reasons, or of course, who explains it here. If you do, I hereby promise not to argue about it.

    (I especially mean traditional identifiable religions, not vague Theism or weird Simulationism, etc.)

    • Wrong Species says:

      I think it’s pretty rare for a person to become religious because they did some logical reasoning, weighed all the evidence and decided that God does indeed exist. They either were raised Christian(or whatever religion) and/or they had a spiritual experience and became religious. If they had a really good logical reason for being religious, then philosophers should be disproportionately religious, but only 14% of them are.

      http://commonsenseatheism.com/?p=13371

      • Cauê says:

        I expect that religious people who have read the sequences (or know similar material from whatever sources) either reject some part of it, or think the two things are compatible. I’m interested either way.

      • Anonymous says:

        I think Cauê is talking about people who remain religious, rather than become religious. They’re a lot more common. Also I think the vast majority of the small number of people who became religious as a result of LW did not come to believe in God or the factual claims of their religion, but became observant.

      • Irenist says:

        Wrong Species:

        Are “philosophers” the relevant expert group? Philosophers of religion are, as one might expect, overwhelmingly theist. Now, of course that’s an unfair pro-theist bias: atheists like J.L. Mackie or Graham Oppy who choose to work in that sub-field are understandably few and far between.

        But here’s the thing: contemporary academic philosophy is VERY specialized. A continental attuned to différance, or an analytic adept at juggling Convention T, can be very skilled with the tools of his or her own subspeciality without ever acquiring an acquaintance with arguments like Aquinas’ any deeper than that offered by the Phil 101 survey course they had to take before they could start their undergrad philosophy electives. As Ed Feser complains (ad nauseam for some readers), that Phil 101 understanding of Aquinas and his kin is just hopelessly shallow, and composed mostly of misconceptions and anachronistic misreadings. Take Bertrand Russell: the guy was brilliant enough to coauthor the Principia Mathematica, actually wrote his own (tendentious) History of Western Philosophy, but doesn’t seem (as Feser often points out) to have ever really come to grips with a non-caricatured version of Aquinas’ argument. So you can be a brilliant philosopher and still not be an expert-level evaluator of theist arguments.

        Now, that doesn’t mean that you should become a theist just because most philosophers of religion are theists. It just means that both samples are no good: The philosophers of religion sample is expert enough, but obviously biased by interest and affinity toward religious people. OTOH, philosophers generally usually specialize in something very far from philosophy of religion, and actually don’t tend to know that terrain very well.

        So expert consensus isn’t a helpful guide here: it’s either too biased, or too inexpert. Actual engagement with the deductions is instead required.

        • Protagoras says:

          As a practicing philosopher, I have to disagree about how specialized philosophers are in general. There are some narrow specialists, but there are a lot of us that aren’t. Feser certainly underestimates how much philosophers outside philosophy of religion tend to know about it, and has a tendency to mistakenly conclude that people are uninformed in a number of cases where they really just disagree with him.

          • Irenist says:

            @Protagoras:
            “mistakenly conclude that people are uninformed in a number of cases where they really just disagree with him.”

            That does sound like an error he’d be prone to.

            BTW: What do you think about the whole “most philosophers are atheists, therefore theism is wrong” point? I think it’s not merely an argumentum ad numerum: philosophical reading is interminable, and sometimes one just wants to find out what others, who have done more of the reading, have converged on so far–theism/atheism seems like a perfectly fit candidate for that sort of thing. I’m just unconvinced that professional philosophers, generally, is the relevant group. Thoughts?

          • Protagoras says:

            I do also think it’s more than just a numbers thing. Philosophy is the most meta discipline, and God is an extremely high level hypothesis, so I do tend to think God falls naturally into our area of expertise. Of course, that assumes that we have an area of expertise; not everyone is willing to be so charitable to us.

          • Irenist says:

            Well, even on a deflationist/therapeutic naturalistic understanding whereby good philosophy is just conceptual analysts doing science’s bookkeeping, I think bookkeepers are entitled to be considered experts in what they do!

            IOW, philosophy scorn always kind of baffles me. Not as an unreflective tribal phenomenon (no surprise that jocks hate nerds), just as a thought-out position, as with some “scientistic” types about whom I can’t shake the feeling that they ought to know better.

          • Zakharov says:

            I think philosophy scorn makes more sense if one thinks of continental philosophers as representing philosophy in general.

        • Wrong Species says:

          I don’t think you have to be an expert in philosophy in religion to reject religion but I agree with your main point. I retract my bit about philosopher consensus.

          • Irenist says:

            @Wrong Species:

            People who retract points are my favorite people on the Internet! I shall strive to emulate your grace.

            FWIW, I agree that you don’t have to be a philosopher of religion to have a respectable opinion on it. Nobody is an expert in everything, but we all have to muddle through as best we can.

        • While the majority of philosophers of religion are theists, there’s good evidence that this results from selection bias and should as such carry no evidential force. To control for this confounding factor, one could focus on how the beliefs of philosophers of religion changed after they started studying philosophy. A recent survey did consider this question and found that theists became less theistic after exposure to philosophical argument more often than atheists became less atheistic.

          • Irenist says:

            Wow. That whole page is very interesting–great link.

            (I’m not surprised by the result, actually. If you come from a non-Scholastic tradition, it’s not shocking that something like this quote at your link could happen to you:
            “I was a theist when I began university. It was during reading Hume’s Dialogues in my second year that I began the road to atheism. I believed that Hume successfully undermined every rational reason I had for my personal belief in God…”)

          • Troy says:

            Note on that survey: it’s not clear to me that it finds “that theists became less theistic after exposure to philosophical argument more often than atheists became less atheistic.” Consider the base rates (in philosophy of religion). There are more Christians in philosophy of religion to start and so more opportunity for belief revision to atheism/agnosticism than vice-versa. The 10th comment here — http://dailynous.com/2015/01/30/why-are-so-many-philosophers-of-religion-theists/ — looks at the data and finds that 18.4% of theist philosophers of religion converted from theism to atheism/agnosticism while 46.2% of atheist/agnostic philosophers of religion converted to theism.

    • Brett says:

      Sure. I’m a nondenominational Christian with strong inclinations towards Catholic & Orthodox theology (I currently attend Presbyterian services with my wife to keep the household peace), raised Mormon and left Mormonism in my early 20s because I was convinced its doctrines were false. And, to establish my “smart” credentials, I’ve got a B.S. with honors in Chemistry from Caltech, a PhD in Biophysics from (Unnamed High-Status University, left unnamed so it’s not quite so straightforward to track me down), and about a dozen scientific papers in good journals. I’m familiar with psychology, Bayesian statistics, epistemology, systematic biases, etc. I’ve read (some) of the Sequences (those that I could get through without shouting “What an idiot!” at the screen too many times). Here’s the two major reasons why I’m not an atheist:

      1) Fundamental differences in basic epistemological and moral axioms. It is a literal miracle that we know anything at all, that the world is fundamentally knowable. That understandability of the world is reflective of its creation by a rational God, the Grand Mathematician. So explanations that epistemology is hard for such and such a reason, or that human rationality is bounded seem obvious and don’t particularly impact my faith. Similarly, it seems obvious to me that utilitarian ethics are false – or as I put it in another discussion elsewhere, that morality is a path function, not a state function.

      2) God spoke to me once. Specifically, when I was going through tough times as a teenager/young adult much like Scott and Other Scott, I had a revelation from God. He told me that I wasn’t worthless, that He had plans for me, and that He loved me.

      • Cauê says:

        Thanks, this is just the kind of thing I was looking for.

        No need to establish smart credentials… I’m only interested because I know the people I’m talking about are smart.

        (Adding this to the initial request: One other thing I’m curious about but forgot to mention is reasons for choosing a particular religion among all others. I expect these aren’t necessarily the same as reasons for simply not being atheist)

        • Brett says:

          Well now you’ve hit on the active religious topic in our household. I don’t see how I could not be a Christian without completely discounting my personal religious experience, from which it would be a short walk to solipsism. So of the Christian churches:

          Mormonism, though most familiar to me and personally comforting is manifestly false. (I actually left Mormonism after reading sufficient ancient history to realize that the doctrine of complete ancient apostasy is ludicrously false. I also have a private theory that the angels Joseph Smith saw were in fact demons.)

          Evangelical Protestantism is too anti-rational to accommodate the evidence for a rational universe and a rational God.

          Mainstream Protestantism is… wishy-washy, with unclear doctrine and unclear authority to teach any doctrine.

          Catholicism, particularly the Scholastic tradition, is very appealing but the institutional church is obviously seriously corrupt and some of the doctrines are non-trivial. Plus my wife is a diehard religious anti-authoritarian.

    • Irenist says:

      Cauê:

      Catholic here. As a fan of Thomistic metaphysics, my disagreement with the way both your comment and Eliezer’s oeuvre treat God is that classical theism of the sort typified by the Catholic Scholastic tradition doesn’t treat God as a “hypothesis” at all.

      Eliezer likes to quote Laplace’s “I have no need of that hypothesis.” You mention Occam’s razor, Bayes, and lots of other ways to try to do a careful, skeptical job of making inductive hypotheses based on the evidence.

      But for the theist metaphysician, God isn’t like the result of induction, like a scientific hypothesis. Instead, God is a result of deduction, like a mathematical theorem derived from postulates/axioms.

      Bayes and Occam are great for answering inductive questions about evidence. But metaphysical deductive demonstrations of God are about seeking coherence among beliefs, not facts from observations. At their simplest, Aquinas’ arguments go like this: What MUST the world be like for change to occur? Well, there must be act and potency in the world. Somewhere in there, there must be pure act. What would pure act be like? Well, it’d be God.

      Now, maybe this is a bad argument. Lots of very bright people have unfairly (like Russell) or fairly (like J.L. Mackie) engaged with it and found it flawed. Okay.

      But that’s my basic beef with LW atheism: bad metaphysics. I think it gets the problem of universals wrong (“clusters in thingspace” and the idea that numbers are just pebble-counting are both pure nominalism, whereas I’m a moderate realist), I think it gets ethics wrong (virtue ethics is notoriously slippery, but no more so IMHO than attempts to calculate utils or adequately predict consequences). Worst of all, and more to the present point, I think LW atheism does the New Atheist thing of treating all theist arguments as though they were the failed hypothetical inductions of the “God of the gaps” of William Paley, some Greek myth about the weather, or some Kentucky creationist; that completely ignores the metaphysical, deductive argumentation that more thoughtful theists are more likely to be motivated by. E.g., rationalist Leah Libresco’s largely Platonist concerns were addressed by converting to Catholicism, but LW atheism, with its (to be frank) metaphysical shallowness, never gets anywhere near the area of inquiry concerned, much less to answers in that field that might satisfy a metaphysical inquirer.

      In brief: the Bayes dojo is a great place to learn how to think about scientific questions, how to handle evidence soberly, etc. I love it for that. But, IMHO, God just isn’t that *kind* of question. LW atheism fails to distinguish between God as hypothesis (which is, indeed, silly and primitive), and God as theorem (which I think deserves far more careful refutation, or, in my case, assent).

      I hope that’s helpful and responsive, Cauê. I may not share the LW community’s usual atheism, but you guys rock, and I’d like being one of the theist hangers on to render me useful at least once in a while.

      ETA: In answer to your second question, my theism flows from Thomist metaphysics, so Catholicism is the most natural choice.

      ETA2: “I hereby promise not to argue about it.” I’m grateful for that. I hope no one else feels like arguing about it, either. If you do, just grab a copy of Feser’s new “Scholastic Metaphysics: A Contemporary Introduction” and his prior Aquinas, Philosophy of Mind, and “Last Supersition” books, and blog a hostile review if you want.

      My argument at this point is pretty much just a Courtier’s Reply: “Read Feser’s oeuvre and get back to me.” There’s no point trying to type out that oeuvre in a comment box. He spends a lot of sentences trying to head off common (especially on LW) misunderstandings, which is most of why his books are book-length.

      • Linked List says:

        I just got “The Last Superstition” on Kindle and literally the first sentences are Feser criticizing same-sex marriage because it gives equal value to “family and sodomy”.

        You’re gonna have a lot of trouble convincing this blog’s audience to take that book seriously. I’m pretty sure I just wasted 10 dollars.

        • Irenist says:

          Feser’s vitriol about gay people is ridiculously unhelpful. I think he still has important things to say about metaphysics, though. It’s kind of the old “Heidegger/Heisenberg was a Nazi (or at least worked for the Nazis, in the latter’s case), but read this anyway” thing.

          (As an orthodox Catholic, I hold to the same morality Feser does, but I don’t think he has to be such a jerk about it. Nor do I think that whether something is a sin has much to do with whether it should be legal. Feser, unfortunately, seems to disagree. He’s quite FOX News-y politically.)

          My advice, now that you’ve spent the 10 dollars anyway, is to try to ignore it, the way Scott ignores the unsavory bits of race realism and NRx and whatnot when he’s trying to steelman them. The metaphysics are interesting, and Last Supersition gives a good, quick, popular overview of Feser’s main contentions. It’s not at all rigorous, but it’s better than what I could fit into a blog comment.

          “Scholastic Metaphysics” is way better, but it’s not out on Kindle yet. It engages with the analytic tradition in a much more rigorous way than the popularization in Last Superstition, which is mostly focused on bashing the early moderns. (Feser wrote a book on Locke, and it shows in Last Superstition’s focus).

          The other two (“Aquinas” and “Philosophy of Mind”) are at least as good as Last Superstition, and more free of the vitriol. (Not entirely; Feser has little restraint.) I’m almost through Scholastic Metaphysics, and so far it’s vitrol-free, thank God.

          On hylemorphism/formal causality in particular, David S. Oderberg’s “Real Essentialism” is a lot more analytically rigorous than anything Feser has put out. It’s only one part of the overall metaphysics, but it’s very well done.

          That said, I’m gratified you took the recommendation, and I apologize if you end up hating the book. If you’re ever in Dallas, I’ll buy you dinner to make up for it.

          ETA: If you just can’t stand Feser AT ALL, here’s a guy (unfortunately with similar FOX-y tics, but mericfully rare) blogging the Summa Contra Gentiles with helpful footnotes. The guy is smug as heck, but the footnotes are good. Here’s the chronologically first page of his blog posts on the Summa:

          http://wmbriggs.com/post/category/samt/page/8

          Scroll down to the bottom of that page 8, and read the posts in reverse order. Beware, though: the smug “do you now see how obvious God’s existence is, dumb atheist?” tone of the (otherwise sound) footnotes makes Less Wrong’s treatment of theists look like a Baptist tent revival.

          • Protagoras says:

            “Scholastic Metaphysics” is free of vitriol, you say? I should definitely look it up myself (I’ll try to get it from a library, though). I’d certainly be curious to hear what either you or linked list have to say about my discussion of “The Last Superstition”.

          • Linked List says:

            Fine, I’ll give the book a chance. It’s not your fault I bought it without reading the sample anyway. I’m just a compulsive book shopper :p

          • Irenist says:

            Protagoras:

            I tried to leave a longer comment, but it disappeared. To sum up: Your review is fair and interesting. Much of what Feser elides in TLS is covered much more adequately in SM. I’d be fascinated to read your review of that, should you ever write one.

            Linked List: I’m a compulsive Kindle shopper, too.

            Scott: If my disappeared comment was out of line in some way, I can’t apologize enough. I can’t think of why it would’ve been, but whatever it was, sorry!

    • Daniel Speyer says:

      I’m not sure if I qualify as practising a “traditional identifiable religion”. I’m a pretty mainstream reform Jew, perhaps a bit more observant than most (I keep kosher, as I interpret kashrut).

      I affiliate with the Jewish community. I practice Jewish ritual. I use Jewish thinking as a guide to my actions (*a* guide, not *the* guide, but a guide that I would never dismiss out of hand). I treat Jewish sacred texts with respect.

      You talk about “hypothesis space” and a “body of evidence”. This sounds like you view religion as a belief in the less-wrong sense. Something which constrains expectation.

      I cannot think of any scenario in which my Judaism constrains my expectations.

      Eliezer Yudkowsky claims that this is a recent development. That a thousand years ago, people prayed for the sick and literally expected this to help them recover. Maybe they did. Probably someone somewhere still does (I haven’t met them). On the other hand, I note that Rashi (who actually lived a thousand years ago) expressed confusion at the fact that the Torah begins with the creation of the world. He took it as a given that the Torah exists to teach us how to live, and asked why its very first chapter had so little application to that.

      The sort of literalist religion EY insists on is very much a weak man. A *straw* man from the perspective of my own social bubble.

      So I don’t know if Judaism counts as a “traditional identifiable religion”. Just because we have a continuous history of thought stretching back into prehistory…. But consider whether some of the people who have you confused in fact believe something more like this.

      • Brett says:

        [P]eople prayed for the sick and literally expected this to help them recover. Probably someone somewhere still does.

        I do. Or rather, not that prayer causes healing in a deterministic way, but that God listens and responds to prayers and can heal.

        • stillnotking says:

          Would it be possible for you to have a word with Him regarding His unreasonable refusal to heal amputees?

          • Cauê says:

            I don’t mean to claim ownership of this thread or anything, but can everyone please refrain from arguing in this one, especially in a hostile way?

            I really wasn’t interested in reenacting all the old arguments right now, and I’m afraid other people might shy away from contributing.

            (the response so far has been great, thanks)

          • stillnotking says:

            I was going for light-hearted teasing, not contempt, but I should know better when it comes to the internet and tone perceptions, so I apologize.

          • Brett says:

            Sorry, Cauê. I deleted my response (I think the earlier one is still useful.)

      • Irenist says:

        Daniel Speyer writes:

        The sort of literalist religion EY insists on is very much a weak man. A *straw* man from the perspective of my own social bubble.

        I think that might offer the most admirably succinct TL;DR to Cauê’s query as to how one can, say, read the Sequences and still practice a religion. From within the social bubble of more intellectualized religious practice (e.g., “sophisticated theology,”TM), the LW stuff just feels like it’s directed at someone else. Not that us still-theist readers mightn’t be wrong about whether the Sequences land any punches against our versions of religion, just that it doesn’t *feel* like they do. After I read a selection from the Sequences, my algorithm feels from the inside like its just encountered a marvelously well-written exposition of a viewpoint from within one half of Snow’s “two cultures,” without really saying anything that hits me where I live, in the other of the two cultures.

    • Jaskologist says:

      A lot of atheists mistakenly think of religion as an attempt to answer the question of “How does the world work?” Thus, they conclude that with evolution, there is no need for religion anymore (but note that even evolution only answers a very stripped down and limited version of the original cosmological argument). While some religions do dip their toe into those areas, this has never been their primary concern. Religions worry about these 2 questions:

      1. What is wrong with the world?
      2. How do we fix it?

      Empiricism tells us nothing there. Indeed, the strict materialism which many Rationalists take as an axiom, really needs to consider these questions invalid. But most humans know that these are real questions which need solutions, just as surely as they know that 2+2 must always only equal 4 (which is also a metaphysical fact outside of the material realm). There are a lot of questions which empiricism just can’t address.

      How shall we live? What is our (my) purpose? What is right? What is wrong? What is worth dying for? Why is there something rather than nothing? Why is this universe even intelligible to our minds? Why can we trust reason? Why is math so unreasonably effective? What does it feel like to be a bat?

      All of these are metaphysical questions. Some of them are important for the Rationalist project to make sense at all. But it largely ignores them, or gives answers which are philosophically naive at best.

      Which gets back more or less to what Irenist said: God is more of a theorem than a hypothesis. The reason so many early Christians took to Plato is that his philosophy, with its exploration of metaphysics, is constantly throwing up big neon signs flashing “hey look, there’s God right there!” Looking into the why of things rather than the how constantly points back to God.

      As for why Christianity in particular, most people will probably answer some combination of these:

      1. The historicity of the resurrection is compelling.
      2. Personal experience/dealings with God.

      • Illuminati Initiate says:

        Religion doesn’t answer “should” questions any more than materialism does (that is a question of morality, not epistemology). It also does not answer the question of why anything exists (which really, I’m not sure that’s actually a real question in the first place). And it “answers” the problem of trusting memories, but only by relying on faith in something else.

        • Irenist says:

          Hmm. Jaskologist writes:

          A lot of atheists mistakenly think of religion as an attempt to answer the question of “How does the world work?”

          Illuminati Initiate writes:

          the question of why anything exists (which really, I’m not sure that’s actually a real question in the first place)

          I think there’s maybe something interesting here. I think a lot of non-theists may just be more interested in, more impressed by the practical results of, and/or more convinced of the intelligibility of, scientific/technical/political/historical “how” questions than metaphysical/teleological “why” questions.

          I can imagine, say, Richard Dawkins growing up really curious about organisms, finding that religion’s answers to the questions that he’s passionate have mostly been primitive myths, and concluding that the whole thing is silly. IOW, maybe the atheist isn’t so much “mistaken” that religion is an attempt to explain how the world works, as just a person who has a strong prior that a worldview ought to be centered on “how” questions, and so feels like religion is missing the main point, in much the same way that those of us with “why” questions feel like anti-theist harping on evolution vs. intelligent design misses the metaphysical point. Maybe we’re just most delighted by different *kinds* of explanations?

          • Nita says:

            Interesting theory, but doesn’t every kid ask lots of “why” questions growing up? Eventually, some of us get used to the fact* that the “how” is something we can actually investigate and use, while the “why” doesn’t lead anywhere interesting.

            * sorry, atheist perspective

          • Irenist says:

            the “how” is something we can actually investigate and use, while the “why” doesn’t lead anywhere interesting.

            I think that’s a great atheist perspective to offer, actually. It seems like kind of a lottery of fascinations thing–to some of us, the places the “why” questions lead are endlessly engrossing. To others, not.

          • Paul Torek says:

            Maybe there’s a trend, but mark me down as one atheist fascinated by “why” questions.

            Regarding religion and “how” vs “why” questions, my sense is that religion mostly wasn’t shy about the how, until – how can I say without appearing too argumentative – that no longer seemed productive. In some places it still isn’t shy about the “how”s.

      • Brad says:

        You can also add “the bible itself is compelling as a text of divine origin” to that list, because it’s one I believe in. There is probably (and maybe necessarily) some overlap with #2 on that list, however.

    • Anon says:

      Nearly everyone has a few bizarre beliefs (belief-space is really big). I am almost certain you hold beliefs that 99 percent of us would think are silly. For these people, religion is that belief. The only interesting thing different I can think of is probably that belief would be more vigorously challenge in these circles. Perhaps though religion is a more-frequent-than-others bizzare-belief. I don’t have statistics here.

    • Ilya Shpitser says:

      Bayes, Occam, etc. are just attitudes you assume about the world. The world is not obligated to follow your taste. Maybe religious people have seen something that pushed them out of their prior. Or maybe they don’t prefer a simplicity prior. Why argue about priors?

      Encounters with the divine (whatever this means) are apparently not very reproducible, so the usual framework we have doesn’t work.

    • Erik says:

      Is it a fundamented rejection of some basic premise? Is it some fantastic body of evidence?

      Those and more besides. There are deep unshared assumptions. There’s a fantastic body of asserted evidence, and a dispute as to what constitutes “evidence” – and, as mentioned upthread, arguments which don’t conclude that God is the most likely hypothesis given certain evidence, but argue that God is a necessary premise for there to be a coherent notion of “evidence”.

      But for specific combative reasons encountering the usual culture around here, I’d put the following.

      Reason one: I have similar thoughts to the people saying straw man/weak man in this comment subthread, but I’d say “wrong target” or something like that instead.

      I’m a Norwegian Protestant, my best friend is a Polish Catholic, and the two of us agree that we’d much rather commune with a typical Russian Orthodox than with any typical American Christian – whether Protestant or Catholic – because we share the view of America as being full of crazy heretics, although my friend will insist in pointing out that they are mostly heretical Protestants. And most of the time when I read something on Less Wrong talking about religion I mentally parse it as saying American Protestantism instead, and it makes much more sense.

      Epistemology, Bayes, rigorous formulations of Occam’s razor, et cetera, are great weapons when your enemy is going “You just gotta have faith”, which I gather is being said by a nontrivial faction in American culture wars. But where I live, “You just gotta have faith” is heresy, and I often find myself nodding along with whoever is attacking that position. And this goes up to eleven when Yudkowsky takes pot shots at creationism.

      Reason two: Original sin and the other Problem of Evil.
      I have yet to encounter a satisfying secular account of evil. The great majority that I’ve encountered seem to fall into one of two camps: pinch from Christianity and cross out the bits about church and sex and family, or do away with the concept.

      Reason three: Meta level arguments that many of the critics are out of their depth. (Warning, long and rambling.)

      C.S. Lewis has some rather acerbic comments on the professional literary critics proposing to trace the genesis of a text in Fern-Seed and Elephants. He notes that he has a book with a talking lion, and some other author has a book with a talking tiger, and the “reconstructed history” of these books says that the former led to the latter, but Lewis has spoken to that author and has firsthand knowledge that this didn’t happen. Then you have various reviewers talking about how the Ring of Power written about by Tolkien represents, symbolizes, or is inspired by, the atom bomb, but again, Lewis has firsthand knowledge that this is wrong, because Tolkien was writing about the Ring of Power before the Second World War even started. If this is the level of spurious conjecture we can expect when people who study books for a living opine on how a story from their own time and place came to be, he asks, why the hell should we listen to a word they have to say about the “real origin” of stories from a thousand years and a thousand miles away?

      Scott had something similar in Are You a Solar Deity? and I’d also point to xkcd’s Physicists (“Just model it as a [simple object] and then add some secondary terms to account for [complications I just thought of]. Easy, right?”) as a related failure mode of filtering everything you read through a single prism of thought shaped by one’s preconceptions, and it also ties back to my first reason: when you see someone arguing that understanding the size of hypothesis-space and privileging the hypothesis discredits Christianity, and you read their argument but their interpret it as discrediting, say, fideism, everything else they have to say starts to look a little weaker when you see that they misrepresented something outside their field, and they should stick to probability theory, or read the arguments and evidence for that particular hypothesis.

      Which has its own problems in that the arguments and evidence for the Christian hypotheses may be several hundred pages of theology using technical jargon, with multiple contradictory several hundred page sequels going “We realize there’s a bit of a weakness on points X, Y, and Z, also G and H are unclear and we’re not sure how to deal with it, but here are some approaches that seem reasonable to us.” with each sequel explaining a different school of thought’s perspective on the disputed points. At which juncture the critic is entirely entitled to object “Squid tactics!” (i.e. spraying up a cloud of ink as a distraction) and invoke the heckler’s veto on bores and demand a summary version so he doesn’t waste his life reading a thousand pages of speculative theology. This is a hard problem to resolve.

      And then there’s the matter of the soul. You’d think nobody had heard of the Fifteenth Ecumenical Council.

      (humorous pause for chorus of “the what?”)

      It’s mostly known for ordering the dissolution the Order of the Poor Fellow-Soldiers of Christ.

      (pause for “the who?”)

      The Knights Templar. Which led to a lot of speculation about where the Knights’ money went, and various conspiracy theories that continue to this day. And that greatly overshadows one of the lesser-known documents from the Fifteenth Ecumenical Council, which responded to a dispute at the time by setting down an official position statement, binding on all Christians on pain of anathema, on what a soul is and who has one. To make a long story short, the soul is the essential form of the body. Translated from medieval-speak into modern terminology, their position looks to me very much like a form of pattern theory of identity written down seven hundred years ago. But very few critics of the “soul” are attacking this position; they’re usually attacking the popular consciousness idea of the “soul” as a sort of ghostly puppeteer pulling the strings of the body. I can hardly blame them for this, but it does leave me wondering where else criticism may be grossly misplaced because it’s attacking idea-as-refracted-through-pop-culture instead of the idea-as-stated-by-official-idea-stater which I’m ignorant of.

      • Jiro says:

        The problem with saying “wait, we defined a soul at the Fifteenth Ecumenical Council” is that “we” is pretty narrow. It’s certainly not the “true” definition of a soul; it’s a definition by one sect of one religion among all the religions that believe in souls. Arguing with people who define a soul differently isn’t arguing against a distorted version of the true belief–who’s to say that Muslims or Protestants have the distorted version and the Fifteenth Ecumenical Council has the true one?

        And even when arguing with Catholics, who’s to say that true Catholic belief is the one from the Fifth Ecumenical Council, and false Catholic belief is the version that everyone actually believes in? The Fifth Ecumenical Council aren’t authorities to me. When they say that a soul means X and a guy on the street says that a soul means Y, I have no reason to pay attention to one of them more than another, and when I address the latter I am at least addressing a living belief that is in the mind of an actual human being.

        • Anonymous says:

          “And even when arguing with Catholics, who’s to say that true Catholic belief is the one from the Fifth Ecumenical Council, and false Catholic belief is the version that everyone actually believes in?”

          Would you adopt this position toward evolution, too? I’ve notice a certain disjunction between the expert opinion and that of the man on the street there, too.

          • Jiro says:

            Scientists have a procedure for figuring things out which involves experimentation and peer review. The guy on the street who has false ideas about evolution hasn’t done those things or taken them from a source who did, so I can say that his opinion on evolution is not the scientific one.

            Also, there aren’t competing theories of evolution the same way there are competing religions. Erik didn’t try to sell me on the religious definition of a soul; but the Christian, and specifically Catholic definition of a soul. Even if you can argue that I should believe a theologian over an educated layman, why should I believe a Catholic theologian over a Protestant or Muslim one?

          • Irenist says:

            @Jiro:

            why should I believe a Catholic theologian over a Protestant or Muslim one?

            Well, don’t believe them on the authority of their denominational affiliation! But do, if you’re interested in theological or metaphysical questions, seek out the strongest arguments you can find, instead of wasting your truth-seeking time engaging the inchoate prejudices of the man of the street.

        • Irenist says:

          @Jiro:

          What Anonymous said. The guy on the street who “believes in” evolution both thinks he evolved from a monkey and thinks that believing that he evolved from a monkey marks him as a member in good standing of non-rube society. That his version of evolution is as much a travesty as the creationist’s doesn’t falsify evolution–it just means that the guy on the street may not be your best interlocutor if you want to assess an idea.

          The man on the street, if he has any idea at all what an atom is like, is picturing the little solar system-like illustration of the Rutherford model from a half-remembered school textbook, and probably thinks the electrons are little balls revolving around the cluster of proton and neutron balls.

          If someone is trying to defend modern physics to the man on the street, is it fair to ask the physics defender never to mention electron clouds or quantum fields, but instead to defend a picture of little orbiting billiard balls which no physics expert believes?

          Arguing against “bearded man on a cloud” versions of God, creationism, anti-intellectual fideism, divine command theory ethics easily defeated by Euthyphro dilemmas, etc., is fine if your goal is to convert half-educated fundamentalist Christians to atheism; it’s an arguments-as-soldiers tactic fit for rhetorical use on an atheist apologetics blog.

          But if one’s goal is to get at the truth of a matter, then engaging the most carefully constructed, precise, expert versions of the worldview you’re challenging has to be the only thing that’s worth a rationalist’s time. Fight steelmen, not strawmen and weak men. Even if–especially if, given the “Cowpox of Doubt” issue–the weak men (ghostly ectoplasmic Cartesian souls as opposed to Thomistic formal causes; billiard ball electrons as opposed to properly understood electrons) are popular with the guy on the street.

        • Deiseach says:

          Guy in street is not a professional theologian. Your objection is like saying “Why should I believe Dr X and Professor Y when they tell me this rash is shingles and this is how you treat it, when Bloke I Stopped On Corner says his cousin Jill had the same thing and she cured it by rubbing plain Greek yoghurt on three times a day?”

          • Irenist says:

            Deiseach, the problem is of course the Dawkins-esque attitude that a professional theologian has no expertise: that there is nothing for the theologian to be expert in except logorrheic, obscurantist “priestcraft” (as bien-pensant Edwardian Whigs used to call our side’s alleged Jesuitical wool-pulling; modern Whigs like Dawkins would say “BS” or “squid ink” or something).

            As Dawkins wrote (in his usual enjoyable, readable style) in “The God Delusion”:

            I recounted the words of an Oxford astronomer who, when I asked him one of those same deep questions, said: ‘Ah, now we move beyond the realm of science. This is where I have to hand over to our good friend the chaplain.’ I was not quick-witted enough to utter the response that I later wrote: ‘But why the chaplain? Why not the gardener or the chef?’ Why are scientists so cravenly respectful towards the ambitions of theologians, over questions that theologians are certainly no more qualified to answer than scientists themselves?

            I have yet to see any good reason to suppose that theology (as opposed to biblical history, literature, etc.) is a subject at all.

            Now, this view (that theologians, or metaphysicians for that matter) have nothing to teach the scientist (or the scientistic non-scientist science enthusiast) is of course naive, as common-sensical rejections of ill-understood fields (e.g., evolution, ironically enough) so very often are. But it is a common attitude, and it is the attitude that, IMHO, chiefly undergirds the stubborn will to debate theological and metaphysical weak men.

          • Deiseach says:

            As Dawkins wrote (in his usual enjoyable, readable style)

            Please don’t take it as an insult if I ask is that meant sarcastically or sincerely? I can’t stand Dawkins’ style, which makes it very difficult for me to be fair to the man, but honestly it’s hard to read his stuff when at an interval of every five sentences he writes something that makes the top of my head blow off.

            As a substitution for not being able to give him a fair go by reading his views at length, I extend as much charity to him as possible for me by not violating the Eighth Commandment* (through gritted teeth and by white-knuckled determination) in his regard, which means when I came across a juicy nugget in “Private Eye”, it was very difficult for me to restrain myself from sharing it around in order for us all to point and laugh. Sometimes religion is inconvenient like that 🙂

            *For those who may be curious, from the Catechism this is part of the offences it is possible to commit against the Eighth Commandment (Catholic numbering) Thou shalt not bear false witness against thy neighbour, which I mean in this instance:

            III. OFFENSES AGAINST TRUTH
            2477 Respect for the reputation of persons forbids every attitude and word likely to cause them unjust injury. He becomes guilty:

            – of rash judgment who, even tacitly, assumes as true, without sufficient foundation, the moral fault of a neighbor;

            – of detraction who, without objectively valid reason, discloses another’s faults and failings to persons who did not know them;

            – of calumny who, by remarks contrary to the truth, harms the reputation of others and gives occasion for false judgments concerning them.

            2478 To avoid rash judgment, everyone should be careful to interpret insofar as possible his neighbor’s thoughts, words, and deeds in a favorable way:

            Every good Christian ought to be more ready to give a favorable interpretation to another’s statement than to condemn it. But if he cannot do so, let him ask how the other understands it. And if the latter understands it badly, let the former correct him with love. If that does not suffice, let the Christian try all suitable ways to bring the other to a correct interpretation so that he may be saved.

            2479 Detraction and calumny destroy the reputation and honor of one’s neighbour. Honor is the social witness given to human dignity, and everyone enjoys a natural right to the honor of his name and reputation and to respect. Thus, detraction and calumny offend against the virtues of justice and charity.

          • Irenist says:

            Please don’t take it as an insult if I ask is that meant sarcastically or sincerely?

            No offense taken at all. Honestly, I really do enjoy reading Dawkins. “The Selfish Gene” delighted me when I read it.

            A lot of the reviews of “The God Delusion” said is was brilliant, but it’s too bad Dawkins was so off-puttingly mean. I was disappointed when I read it, because I had the opposite response. Even though I’m a Catholic, I thought his acerbic put-downs of our worldview were kind of entertainingly done. Take “why not the chef or the gardener?” There’s something pleasantly random-seeming about the examples chosen. It’s funny. However, I found the arguments to be mostly easy take downs of straw men and weak men like creationism and the “bad bits” of the Old Testament.

            So, yeah. I like Dawkins’ style. “One man’s trash is another’s treasure” and all that.

          • Deiseach says:

            I like (or at least, can stand) Dawkins when he’s discussing facts or things within his own sphere of knowledge, but then he starts jumping in with pronouncements about other matters on which his opinion is about as qualified as that of “the gardener or the chef”, and with an air of smug ‘I just sprained my wrist from patting myself on the back’ self-congratulation on how smart he is.

            And now I’ve probably trashed the Eighth Commandment parts I mentioned previously with that little snarl at the man 🙂

      • Paul Torek says:

        Translated from medieval-speak into modern terminology, their position looks to me very much like a form of pattern theory of identity written down seven hundred years ago.

        Whoa, OK, I’m impressed. But then, given what very little I know about what the Aramaic word for soul meant in the time and place of early Christianity, it fits.

    • Troy says:

      Caue: I’m an ecumenical (small-o) orthodox Christian (i.e., I affirm the basic creeds) and an evidentialist Bayesian. I would not be a Christian if I did not think I had good evidence that it was true. Unlike many of the other respondents above, I don’t go in for all of the Catholic/Thomist metaphysics and epistemology (although I respect a lot of that work), so my perspective might be somewhat different from theirs.

      Obviously this is a huge question, but let me try to outline my perspective for you, which is broadly similar to that of many other Christian evidentialists like Richard Swinburne or Tim McGrew:

      – The intrinsic probability of classical theism is low, but not outrageously low. That a perfect personal being is the ultimate explanation of the world is a comparatively simple claim. The intrinsic probability of Christianity is of course much lower, although I have some sympathy with Swinburne’s argument that a personal God would be likely to reveal himself to us at some point, which raises the conditional probability of Christianity on theism.

      – Religious and mystical experiences are good evidence for some kind of transcendental reality, although they don’t provide strong evidence for theism over some kind of non-theistic religious hypothesis. Naturalistic explanations of these experiences are unsatisfactory. See Caroline Franks Davis’ book on religious experiences for a good defense of this claim.

      – The fine-tuning of the universe provides strong evidence for theism, and perhaps weaker evidence for some kind of “axiarchic” hypothesis, like John Leslie’s (which holds that goodness is itself causally productive). The issues here get technical quickly, but I’m confident that most of the standard objections to the fine-tuning argument (e.g., Sober’s selection-bias effect objection) are unsound. For some truly remarkable recent work on this, see Robin Collins’ (unpublished, but available on his website) essay on fine-tuning for discoverability. Certain fundamental aspects of physics appear to be such as to be maximally discoverable by embodied conscious agents; for example, the cosmic microwave background radiation (our main evidence of the Big Bang) is as strong as is theoretically possible.

      – All of this only gets you to theism, at best. To get to Christianity you need to look at the historical evidence for the reliability of the New Testament, in particular in its miracle reports. I think this evidence is extremely strong. On this topic, Tim McGrew’s Reliability of the Gospels series (http://www.apologetics315.com/2012/11/audio-resources-by-tim-mcgrew.html) provide an excellent introduction. I especially recommend the Internal Evidence for the Gospels talk. See also his paper on the resurrection (coauthored with Lydia McGrew) in the Blackwell Companion to Natural Theology.

      – I take objections to Christianity like the problem of evil seriously, but in the end I think their weight is more than counterbalanced by the above considerations. Most other objections to Christianity I hear target views that I neither hold nor see as essential to orthodox Christianity.

      – Although you said you wouldn’t raise objections, I’m more than happy to dialogue about any of these reasons if you’d like to continue the conversation.

      • Cauê says:

        >”Although you said you wouldn’t raise objections, I’m more than happy to dialogue about any of these reasons if you’d like to continue the conversation.”

        I normally might take the offer, but in this instance I think most objections I could raise would also apply to other people commenting.

        I got interested by “naturalistic explanations of these experiences are unsatisfactory”, but the recommended book seems to be above my “random curiosity” price level. Can you recommend a defense of this somewhere else?

        • Troy says:

          Fair enough — we academics tend to take for granted availability of books through libraries etc. Unfortunately most of the other stuff I’ve read on this particular issue didn’t strike me as being as good. Most theists who write about this run versions of this argument that strike me as epistemically problematic.

          The Stanford Encyclopedia of Philosophy is always a good resource: they have (free) articles on Religious Experience and Mysticism. I’m confident they will offer a good introduction to philosophical discussions of the topic, but I haven’t read them, and so I don’t know if I would endorse the pro-theistic cases outlined in them.

  62. Alejandro says:

    The way “nerd” is used popullary (e.g. in the discussions prompted by Scott Aaronson’s comment) denotes a cluster in personspace, including as typical defining characteristics: high technical ability in subjects like math and programming; recreational interests in some stereotyped areas like comics, gaming and science fiction; general social awkwardness, low interest or ability in keeping with conventional social mores, and lack of romantic success.

    In my experience, this seems to be a distinctively American cluster. Most examples I can recall of people fitting all these characteristics at once are either real American people or characters from American-produced media. In general, people from Europe or Latin America with “geeky” technical or recreational interests have seemed to me better socially adjusted and romantically successful than their North American counterparts.

    Does this impression match anybody else’s? I was going to start speculating on the explanations for this, but I remembered my Borges, and refrained from doing so until verifying that it is a real pattern and not an imagined one.

    • Cauê says:

      The cluster was very recognizable in urban Brazil as I was growing up. It’s very different now given the immense popularization of nerd culture, but I suppose this is happening everywhere in the West.

      Hollywood’s depiction of jocks was much less recognizable to me than that of nerds.

    • Anonymous says:

      Frenchman here. The word used in the 90s was “intello” (very rarely towards the last years, geek) rather than nerd, for very similar characteristics and academic success. It was “explained” to me very pointedly that it was important that I should not even think about having a social life, and certainly no romantic prospects. YMMV.

      • Emile says:

        I find the concept of “intello” somewhat different from the american “nerd”: I think it’s much more distinct from the “geek” cluster, and less negative (“intello” is someone who thinks his intellectual interests put him above the others, “nerd” is someone whose intellectual interests put him below the others, something like that).

        It was “explained” to me very pointedly that it was important that I should not even think about having a social life, and certainly no romantic prospects.

        I can’t really relate to that (despite being an uber-nerd);by whom? in what context?

        (the most “anti-nerd” hostility I got was by lower-class immigrant-background assholes who thought that made me gay or something; nothing “mainstream”)

        • Sarah says:

          If I had to come up with a just-so story for what’s going on in the US, it’s that here *even the upper classes* have an assumption that being too intellectual makes you “gay or something.” The influence of the Progressive movement (think Theodore Roosevelt) was America-specific and very big on the distinction between being manly and active vs. an effete intellectual.

          • Emile says:

            Yep, whereas on the other hand France sees itself as more intellectual and artistic and idealistic and refined than the rest of the world; compared to us, Americans are money-grubbing hicks or shallow money-grubbing suits with fake smiles, Germans are humorless and boring with no vision, the English are short-sighted businessmen and toadies to the Americans, etc.

            So being anti-intellectual is a bit being anti-French (and a fair amount of the lower-class immigrant-origin assholes *are* anti-French too).

        • thirqual says:

          I was the anon, did not notice I forgot to refill the form.

          In my middle and high schools, the “intellos” had also very nerdy in-the-classical-sense interests. I disagree with you on the above/below because it suggests I choose the moniker and its implications. To an American eye, I fail to see how I would not have been called a nerd (except for the sports aspect, but that was outside of school).

          By “explained”, I mean regular physical violence, humiliations, destruction of property, imaginative attempts to get me expelled, threats to people seen talking to me. Mostly from upper-middle class kids with otherwise good reputations, to be honest. Obvious homophobic aspect at first (at which point it was only boys being abusive little shitheads). I do not think my experience was typical, but the climate at my schools was really rotten with a mean anti-intellectual vibe.

    • gwern says:

      It’s funny you mention Borges without noting that he was a prototypical – Argentinean – nerd: bookish, glasses-wearing, plays games (chess), fascinated by science fiction & fantasy, awkward when not discussing his specialties, and absolutely hopeless at romance or women.

      • Anonymous says:

        That is true and indeed amusing.

        On the other hand (and this gose also in reply to Cauê and Anonymous) I am not saying there are no, or even very few, non-American are nerds; just that the prevalence of the cluster is not so marked. Some recent discussions seem to assume that a majority or at least a large plurality of the males in tech or mathy sciences are “nerds”, and this doesn’t match at all my experience from other countries.

        • Alejandro says:

          That was me.

        • Creutzer says:

          For what it’s worth, my experience matches yours. I’ve seen nerdy types of people, of course, but “nerd culture” doesn’t seem to be a thing to the same extent. And at the not-too-shabby math and physics departments at my university, people who would register as nerds in America seem to be in the minority.

      • Anonymous says:

        I was skeptical about your characterization of JLB as super-unlucky in romance – I once worked with an Argentinian guy who met Borges a few times (at polo matches or something equally aristo). My coworker raved about Borges’s stunning “assistant”/ girlfriend who accompanied him everywhere.

        Wikipedia suggests that she existed, married him towards the end, and inherited the rights to his works.

        While it seems like his love life may not have been real busy (or fulfilling), this is one of a few reasons why I am a bit uneasy about calling Borges a proto-nerd. It’s like calling Sartre a proto-nerd: a lot of the characteristics fit, but some important ones just don’t (and can’t, for public intellectuals of a certain generation: being a PI was really high-status back in the day).

        • Alejandro says:

          I think Borges’ relationship with Maria Kodama, which started when he was well into his seventies and internationally famous, is not really evidence that Borges was not a “shy, awkward nerd” most of his life. (The way his relationship with Estela Canto developed was probably more typical of him.)

    • Anonymous says:

      In my Eastern European country comics are unpopular, gaming is hardly correlated to nerdiness (if anything, in my country stereotypical teenage gamer is a boy that plays a lot of FPS games like Call of Duty or Halo and likely neglects school (instead of having high mathematical ability), whether or not this stereotype is correct is a different question). Science fiction is popular among nerds, but I would say that few people read SF only. Many boys and girls who would not consider themselves nerds have read at least a few SF books (even though most of them aren’t new books). Given all this, recreational interests of nerds seem to be less different from recreational interests of non-nerds. There are some differences, but it seems to me that they are less pronounced than in the American case.

      I have never been an American nerd, therefore note that I am comparing real people I know with stereotypical American nerds I know only from films or internet, which means that my conclusions are based on rather shaky grounds.

    • Gbdub says:

      Isn’t Japan famous for a strong nerd culture? Up to the point of having a significant openly asexual subculture? (Asexual in the sense of “not engaging in romantic relations with other humans”, maybe “autosexual” is better? I’m thinking of the guys that form relationships pillows or real dolls or avatars)

      • Nornagest says:

        The Japanese nerd stereotype notably doesn’t have much to do with technical or mathematical ability or academic success, although it covers most of the other bases. (And the list of interests is slightly different, but that’s true for every culture that has an analogous concept — compare trains in Britain.)

        • lmm says:

          The Japanese train nerd stereotype seems very similar to the British one from what I’ve seen.

          • Nornagest says:

            Yeah, I’m saying that there isn’t a corresponding American train nerd stereotype. We have train enthusiasts, but the stereotype of them is something like an older white middle- to lower-middle-class man, probably living in a rural area, probably someone’s grandfather, and probably pissing his wife off with the amount of time he spends building model train tracks in the basement.

    • kaninchen says:

      Nerdiness definitely exists in Britain. I would guess that there are differences across countries – e.g. Doctor Who is (I believe) seen as fairly nerdy in the US whereas over here it is far more a part of popular culture, also comic books are seen as more childish than nerdy over here – but I can easily point you to an empirical cluster of British mostly-males who do a lot of science/maths/programming, are with few exceptions permanently single (NB my nerd-cluster contains a large proportion of Christians, hence short-term relationships are heavily frowned upon, so nerdiness is not necessarily the only factor in this), many of whom play Dungeons and Dragons, etc.

      • Irenist says:

        I’m an American who lived in Ireland when I was in my early teens. One of the first differences I noticed was that my classmates who were sort of nerdier seemed at least a little bit sports-oriented, and my classmates who seemed sort of more like jocks seemed more academically and intellectually prepared, whereas these two groups did not partake of each others’ interests AT ALL in my U.S. experience.

        Y’know the trope that men are higher variance on IQ tests than women? It felt like that: like I had moved from a nation (America) where everyone was an outlier on some Bell Curve of jock/nerd-ness, to a nation where there was still variation but mostly within the middle part of the Bell Curve that had been missing in the U.S.; IOW, like I had moved from the land of high variance to low.

        Perhaps relevant: the Irish schools I attended also seemed shockingly less clique-y compared to the U.S.; at the time it seemed like the total absence of ethnic differences, and relative lack of class inequality compared with my U.S. school experience, might somehow be related to that un-clique-iness.

        • Alejandro says:

          Yeah, one of the conjectured explanations I had thought, in case my observation was validated, was the insane importance American high schools give to sports. My idea was that this creates the “jock-nerd” dychotomy by amplifying small differences between athletically inclined and intelectually inclined students–and that because sports are more popular, the jocks go to the top of the social hyerarchy while the nerds end up as social pariahs, making them more shy and awkward that they would be otherwise.

          • Irenist says:

            There’s a profound anti-intellectualism in American popular life, and a corresponding valorization of athletics. As you mention with sports, I think this tends to make those who are intellectually inclined anyway end up in their own subcultures more. (A parallel might be to compare the degree of exaggeratedly stereotypical mannerisms in a closeted vs. a post-closeted society: I suspect it would look a lot like the degree of “nerdy” stereotypical subcultural interests in America and those cultures most affected by it vs. the rest of the world.)

            European places I’ve lived (I’m thinking in this sentence mostly of France, and only secondarily of Ireland) seem to have an ideal of a guy who is fit, smart, and capable of romancing a lady (by the last I mean to invoke poetry and flowers or something like that, not “game” or similar American discourses). Whereas many Americans seem to me to valorize athleticism and denigrate the other two things (academic achievement and the sort of communicativeness and somewhat more metrosexual fashion sense of a certain kind of European) as insufficiently macho. All this is obviously going to affect the “nerdiness” of American male nerds by ghettoizing them (us). And then, of course, once a “nerdy” male subculture exists for American male nerds, it’ll be a natural attractor for similarly intellectual females, so the issues around machismo and sports end up hitting them at one remove. Or something. This is all very inchoate.

            BTW: A note–it’s not that the Irish kids I went to school with weren’t REALLY into rugby and soccer, etc: the Irish are a very sports-loving nation. It’s just that there was no conflict for them, at least apparent to me, between that interest and also being really into books (humanistic, scientific, whatever) if you happened to like them, too.

            ETA: I suspect that Ireland and the UK are more like America in this respect than France, perhaps in part because of common Anglophone traits, but maybe far more just because American pop culture penetrates deeper into other Anglophone societies. Thus, the “romancing a lady” part of being a masculine guy was more of a Latin European ideal, I think, than anything I’d expect to encounter among most Brits. Still, it’s a continuum. If you compare, say, a British cultural icon like James Bond to a comparably influential American icon, the American will have much less of an emphasis on suavity as a way to do masculinity.

            [Oh, God. I have totally brought gender into the open thread. How am I supposed to talk about nerds vs. jocks without gender, though? “That’s so gay!” is like the traditional jock anathema. Ah, well. Delete if you wish, Scott. It’s your blog, and I apologize.]

          • Emile says:

            Irenist: that matches my perception of France, and even more so of China (where – in the school system at least – you’re either a nerd or the shame of your family (and there is little correlation with athleticism one way or the other))

    • Sarah says:

      I have met a handful of people who are firmly in my cluster in memespace (interested in computer science/programming/math/AI, interested in speculative thinking about human society and the future, friends with my friends) but are…completely neurotypical. Confident body language, good at dancing, romantically successful, non-neurotic.

      Most of them are non-American. All of them are Gentile.

    • Sarah says:

      I also think that *even* in America, being interested in scientific/technical subjects doesn’t correlate that well with being interested in SF/comic books/”geek culture”. That’s what The Big Bang Theory gets so wrong — a bunch of postdocs are just not going to be that interested in Star Trek.

      • Anonymous says:

        Quite right. I myself am an engineer, as are many of my friends, and the only person I know who is a big fan of star trek is my brother, who works a non-specialist manufacturing job.

        • Nornagest says:

          I suspect there might be generational aspects to this.

          I’m a software guy, and I come out of a family that’s mostly scientists and engineers. I’m not a big Star Trek fan although I watched a good amount of it in my childhood.

          My dad and uncles, on the other hand, are generally at least casual Star Trek fans (though I wouldn’t call any of them Trekkies) — I’d speculate because they grew up in a time when Star Trek was a lot more of a touchstone in technical culture than it is now.

          My grandpa, on the third hand, was a soil biochemist, and not a man of remotely geeky demeanor or habits.

    • Emile says:

      Another Frenchmen here, I think we would have two distinct cluster:

      * “Geek”, a recently imported concept that pretty much matches the American “computer/sci-fi geek” stereotype, but it’s probably narrower (you won’t describe people as “a something-or-other geek”).

      * “Intello” is probably close enough to “nerd”, as Anonymous said, but I don’t think it has any connotations of interest in computers, sci-fi, gaming, comics (just like “intellectual” in English doesn’t). It’s probably a mildly negative term, not as strong as “nerd” (and the negative side is more about pretentiousness than social awkwardness).

      Other notable differences:
      * The whole jock-vs.-nerds things doesn’t seem to exist, and I don’t think bullying is particularly tied to nerdiness (probably a bit, but it seem that US nerds/geek all consider the fact that they’ve been bullied as matter-of-course or something), and the concept of “popular” doesn’t really exist either (as something people talk about). It may depend of your environment (nerds growing up in a lower-class area may get bullied, but would probably assign it more to some people being assholes than to them being a nerd).
      * Comics are pretty mainstream, and aren’t considered as exclusively for children and weirdos; probably because French comics are more intelligent, varied and interesting than American comics.

    • Sniffnoy says:

      People talk a lot here about the idea of “nerds vs. jocks”, but I should point out that there’s also the idea of “nerds vs. suits”[0], which I think is more interesting, especially as it ties into Robin Hanson etc.’s notion of “nerds”; I have to wonder how those generalize geographically.

      [0]Although Ialdabaoth points out here that the actual people comprising “jocks” and “suits” may largely coincide.

  63. Wrong Species says:

    I have recently read about Thomas Szasz and I don’t agree with all of what he says but I think he had a point. Homosexuality was considered a disease but not anymore. Now pedophilia is considered a mental disorder now but what’s the difference besides one is considered morally acceptable and the other not? Same thing goes for psychopaths and “personality disorders”. Things like depression, schizophrenia and PTSD seem different because the person suffers and want to be cured. But wouldn’t a homosexual in the 50’s also want to be cured? It seems like the very act of calling something a disease implies a value judgement.

    • Cauê says:

      I’m assuming you mean feeling it, not acting on it. My take is that the “acting on it” part is where all the moral difference is.

      Scott wrote about classifying things as “a disease” or “not a disease”:
      http://lesswrong.com/lw/2as/diseased_thinking_dissolving_questions_about/

      • lmm says:

        I suspect that piece is disingenuous, because actually blame and stigmatization almost never have positive effects, so that principle doesn’t recover normality in the uncontroversial cases the way it pretends to.

    • zz says:

      IIRC, psychological disease = abnormal && reduces quality-of-life. By this definition, homosexuality could be a psychological disease in the 1950’s but not in the 2010’s. It also means that pedophilia could be a psychological disease in present-day America, but not in the tribe I vaguely remember reading about in The Blank Slate, where it was standard for prepubescent boys to fellate teens (p(confabulation) = .1).

    • guy says:

      personality disorders like depression or PTSD are not generally considered immoral, although if social pressure can change them, they could be deemed immoral by a consequentialist, and (hypothetically) insofar as they are genetic and we want to prevent sick children, sufferers shouldn’t reproduce.

      I wonder why a consequentialist would differentiate between harm to others and self-harm. The main issue I see is that for most self-harm there is no clear remedy. If we put lazy folks on probation, we might inadvertently punish others who were laid off but are trying hard to find work

      Are there types of self-harm where a pure consequentialist should advocate state intervention? Suicide seems one, since the harm is big and I think at least 90% of suicides are emotional reactions, not considered judgments.

      Does anyone know the reasons given for listing homosexuality as a disorder years ago? Was the risk mostly seen as harm to others, or self-harm?

  64. Irenist says:

    Some people seem to have gotten genuinely upset about some of the recent discussion of IQ, on grounds something like that if high IQ is a necessary ingredient of some forms of academic success and they’re lower-IQ than other people, then they are bad and worthless.

    I’m a Christian, so I think everyone is of infinite worth in the eyes of God, even people who are too disabled to be able to do anything at all for anyone, and that settles it for me. But in case you have a more utilitarian concern about not being “worth” anything to society (beyond your own ability to experience utils or whatever?) because you maybe you can’t cure cancer (or whatever) with a low IQ, consider:

    All those high IQ academics would be in a bad way without cops to protect them, right? Well, remember Jordan v. New London? The New London PD wouldn’t hire Robert Jordan because his Wonderlic cognitive score came back too high. The NLPD defended itself by saying it was concerned that higher IQ cops would experience higher “turnover resulting from lack of job satisfaction,” i.e., that they’d get bored because they were too smart for the job. The trial judge in Jordan v. New London (D. Conn., 1999) found that the NLPD had a rational basis for its policy, and the ruling was affirmed on appeal. Jordan v. New London, (2d Cir., 2000). Now, rational basis is a very low bar, but at least someone thought it wasn’t nuts to think that high IQ might make you bad at a very important job.

    So, let’s say that’s true. There are some social roles that a plurality of high IQ people might find boring and depressing, but low IQ people might thrive in. And some of those roles are IMPORTANT. Really important, foundation of our civilization kind of stuff, like sanitation, say.

    Now, it’s important to note that presumably some trash guys are quite bright, and the less bright ones may be clever in some other way, like being good with their hands or whatever. But that’s not what I’m talking about. What I’m saying is that even if they’re complete dopes–heck, channeling Chesterton, precisely because they’re dopes–they have vital roles to play in our civilization. They have great worth to the wider community.

    More: consider Moravec’s rising floodwaters of AI metaphor. The things we humans think require brilliance, the things we find intellectually impressive, things like chess-playing and theorem-proving, are the first to fall under the rising flood of AI-induced human obsolescence. But cleaning bed sores as an eldercare aide? Dusting a house for a cleaning service? Automating that stuff is hard. But high IQ people are usually bored unto frequent “mental health day” absences by jobs like that. The high IQ types will be lamenting how much they hate the drudgery of their not-yet-AI-doable, menial, non-special-snowflake jobs for years while the lower IQ types just go to work and come home with highly socially functional contentment about their place in things.

    So, worst case scenario–you’re dumb. So what? There’s lots of REALLY IMPORTANT stuff that dumb people are optimized for. Stop being miserable about having an objectively important set of aptitudes.

    * * *
    Okay, next scenario: you’re not dumb. But you’re not, like, Mozart-level smart. More like “envious Salieri from the unfair depiction of him in ‘Amadeus’-level smart,” so you think you’ll never produce a paradigm-overturning scientific or lib-arts breakthrough that saves the world, or whatever. So? On a Kuhnian account where science has equilibria of “normal science” punctuated by paradigm-shifts, all the patient hypothesis-confirming (or disconfirming!) observations of regular stiffs doing normal science are the indispensable foundation for getting to the next paradigm. Urbain Le Verrier wasn’t as smart as Einstein, and spent his career doing Newtonian normal-science things like his 1843-59 data-gathering on the precession of Mercury. But, hey! The oddity he found in Mercury’s orbit turned out to be a big help to Einstein down the road, yeah? So maybe you’ll grow up to be the next Urbain Le Verrier. What could possibly be wrong with that?

    * * *
    Maybe you’re somewhere between Salieri-Le Verrier and the trash guy. Maybe you’re kind of an average or very slightly bright type who might do just fine in, say, business school, and not get bored by it, but isn’t going to solve friendly AI or cure malaria or anything. Well, you read this blog, right? Go to b-school, get a decent, boring, high-paying job, and donate to MIRI if you’re worried about the need for geniuses to solve FAI, or donate to an effectively altrusitic anti-malarial mosquito bed-net nonprofit for Africa, or whatever. Many of the most successful entrepreneurs and executives are bright-but-not-genius types with great common sense, a strong work ethic, and both enough intelligence to optimize their business, but not SO MUCH intelligence that they get bored by business and sit around reading the Sequences or some other smart-people reading matter all day because getting a job sounds really boring and beneath them.
    * * * *
    So are you dumb? Are you average? Kinda bright? Really bright but no Einstein?
    Doesn’t matter. There’s something out there that the world REALLY needs, that you’d be perfect for. And, again, some of us think you’re of infinite worth even if you never do anything….

    • Jaskologist says:

      Paul lays out a concept of equality that I don’t think Westerners have a good handle of. We can’t seem to see things as equally valuable without them being equal in all ways. Paul views everybody as part of a body instead. That they are all different is an essential part of the metaphor, but so is their interdependence. Some are called to be teachers, some to be street sweepers, etc. Whatever you are called to be, be that and be it well.

      This requires a sense of telos that I’m not sure can survive outside a theistic framework, though. There’s a reason the 10th commandment specifically calls out coveting.

      • Matthew says:

        My understanding is that “covet” is actually a mistranslation of the original Hebrew, and the intended meaning was more like “don’t kidnap” or “don’t take hostages.”

        • Anonymous says:

          What lead to that understanding? How sure are you? Are you aware that it not a popular position?

          How do you respond to the obvious question: why is this separate from the commandment on theft? But there are lots of other question, like, if the word translated as “covet” really means “kidnap,” why is the word translated as “steal” so often used to mean “kidnap”?

        • Jaskologist says:

          I would be interested in sources on this; I’m certainly no scholar of Ancient Hebrew. The instruction against coveting your neighbor’s house does seem to militate against it.

        • Irenist says:

          Chamad, the word translated as “covet” by ancient Jews (like St. Paul when writing in Greek) by modern rabbis, and by Biblical scholars generally, also appears in the following verses:
          (1) “The images of their gods you are to burn in the fire. Do not COVET the silver and gold on them, and do not take it for yourselves, or you will be ensnared by it, for it is detestable to the LORD your God.” Deuteronomy 7:25.
          (2) “Do not LUST in your heart after her beauty or let her captivate you with her eyes.” Proverbs 6:25

          Now, are you proposing that (1) should be translated as “Do not TAKE the silver and gold and do not TAKE it?” Seems redundant.

          As for (2), how does one steal someone else’s beauty in one’s heart?

          Joel Hoffman has some sort of axe to grind. He’s in the minority in his field, and although I’m no Hebraist, I’ll take the consensus of the field over his word, thanks.

          (ETA: So now it’s remembering my name but not my email, so I don’t get the little avatar. What is up with the commenting system, I wonder.)

      • Nita says:

        Different parts of the body are not equally essential. People gladly get rid of appendices and wisdom teeth, don’t get too upset over losing blood, reluctantly amputate limbs or excise internal organs. Even hearts, lungs, livers and kidneys can be replaced, to some extent. But brains cannot.

        • Jaskologist says:

          Kind of illustrates my point, doesn’t it? Our minds rebel against the idea that things could be valued the same if we can find some axis along with they are not equal. Before you know it, you have the ears complaining that the eyes unfairly dominate the field of seeing.

          Yes, you can chop off your foot and survive. But the whole body is hobbled as a result; everyone is lessened. The head cannot say to the feet, “I don’t need you!” Just as one part is not supposed to do everything, a single part is no good on its own. They are valued precisely because of their relationship to the whole.

          The equality of the undifferentiated mass is the end goal of Buddhism, Communism, lawn mowing, and the heat death of the universe. Christianity has the opposite aim.

          • Nita says:

            No, your human body analogy doesn’t work. (Well, maybe it works for you, but you aren’t trying to persuade yourself, are you?)

            Try an ecosystem or something.

          • Irenist says:

            Musical harmony relies on every note not being pitched equally high.

            Dante’s poetical take on this doctrine of harmony in hierarchy (Piccarda’s speech in Paradiso, Canto III) is helpfully discussed here:
            http://www.theamericanconservative.com/dreher/paradiso-cantos-iii-iv/

            Here’s a prose translation of the dialogue from the Poetry in Translation website (the first speaker is the narrator, the second Piccarda):

            ‘But tell me, you who are happy here, do you wish for a higher place, to see further, or to make yourself dearer?’ She smiled with the other shadows first, a little, then replied to me so joyously she seemed to be burning with the first fire of love: ‘Brother, the power of love quiets our will, and makes us only long for what we have, and gives us no other thirst. If we desired to be higher up, our wishes would be at odds with his will, who assigns us here, and there is no room for that discord in these circles, if you think again about love’s nature, and that we of necessity have our being in Love.

            No, it is the essence of this being blessed to keep ourselves to the Divine Will, through which our own wills are unified. So that our being as we are, from step to step, throughout the kingdom, is a joy to all the kingdom, as it is to the king, who draws our wills towards what he wills: and in his will is our peace, la sua volontate è nostra pace: it is the sea, to which all things flow, that it creates, and nature forms.’ It was clear to me then how every part of Heaven is Paradise, even though the grace of the Highest Good does not pour down to it in only one way.

            Tolkien presents a musical image of the loss of this sort of harmony in the episode of the Discord of Melkor in the “Ainulindalë,” which is as good a description of what the rebellion of Lucifer must have been like as anything I’d hope to see in Milton, just as the description of how Eru incorporates the discord into a greater, deeper composition is the finest artistic evocation of the Christian solution to the problem of evil I expect to see:
            http://www.houghtonmifflinbooks.com/features/lordoftheringstrilogy/lessons/seven/discord.jsp

    • Anonymous says:

      This seems patronizing to me, but I have no idea why.

    • rsaarelm says:

      The “there’s a place for everybody” thing feels like a thing that would be quite possible to refute if you worked on it (smart people have started automating menial jobs, something seems to happen somewhere between “trashman” and “low-functioning autist” that makes the argument stop working, what if you’re only menial labor smart and still find the sort of menial labor there’s a market demand for boring and soul-crushing), and also the sort of thing which you’re supposed to say for social nicety and not try to think of too hard about refuting.

  65. Scott, you wrote on tumblr:

    “So while I agree that shaming fat people doesn’t work, that studies have shown it doesn’t work, and that it’s a jerk move besides – I notice I am confused based on studies of smoking, drug, and alcohol treatment.

    All of these studies show that if a family doctor, during an appointment, spends a minute or two telling patients that Smoking Is Bad For You, then on a population level the percent of their patients who smoke goes way down. It won’t work for everyone, but given the high patient volume it will work for more than enough people to be worth it.

    (I wrote a little about this in my Alcoholics Anonymous post, so that would be a good place to find the research).

    I’m not sure what the difference is between this and fat-shaming that makes the first work and the second fail.”

    One obvious difference between the cases is that smoking and drinking are much closer to a binary on/off set of states: you’ve either quit drinking or you haven’t, you’re either a smoker or you’re not. It’s a lot fuzzier with overeating – you don’t “quit” eating, you just kind of vaguely adopt a healthier, lower-calorie diet. I could see it being much easier to gradually slip back into overeating without noticing than it would be to accidentally start drinking or smoking again. So even if quitting smoking and drinking are actually harder tasks to complete in terms of willpower or akrasia or whatever, they might wind up being completed more often because people perceive them as just that: tasks, with very clear and well-delineated win conditions.

    In other words, there’s no dieting equivalent to “I smoke zero cigarettes now” or “It’s been 100 days since my last drink”

    • Irenist says:

      In other words, there’s no dieting equivalent to “I smoke zero cigarettes now” or “It’s been 100 days since my last drink”

      Which is a *huge* problem. I also think it’s the source of the success some people have on Soylent, meal-replacement diet foods, and even low carb. ENTIRELY giving up regular food (as with Soylent or one of those diet-food regimens) or just a category of food, might create more of that nice binary effect. When I’ve done low carb, I found it much easier, in a Schelling-fence sorta way, to entirely give up certain categories of food than it had been to find the self-discipline to moderate portion sizes.

      Or, as my father-in-law likes to observe about his own low carb success:
      “Abstinence is easier than temperance.”

    • roystgnr says:

      Did I miss a *controlled* study that shows fat-shaming doesn’t work? The Sutin and Terracciano study which made the rounds a year or two ago showed that people who have experienced fat-shaming in the past are more likely to gain weight in the future, even when controlling for current weight. This is very weak evidence for “fat-shaming doesn’t work”. Replace Scott’s controlled studies on doctors’ “brief opportunistic intervention” with the same awful methodology, and what will you find? People with a greater history of doctors telling them not to smoke and drink will have a much greater future probability of smoking and drinking, even if you control for current consumption. Not because the doctors are being counterproductive (the controlled studies show they aren’t) but because there are hidden preexisting propensity variables that aren’t close to being adequately controlled for.

      I’d love to see an Ig Nobel Prize won this way: compare past chemotherapy patients to people with no history of chemo, use blood counts or something to “control” for the current state of the cancer, act shocked when recurrences in past patients turn out to still be much more common than brand-new cancers, and declare that you’ve proven chemotherapy doesn’t work.

    • lmm says:

      Is this comparing like with like? IIRC if a family doctor, during an appointment, spends a minute or two telling patients that Being Overweight Is Bad For You, then on a population level the percent of their patients who are overweight goes way down. But that’s quite a bit different from shaming.

    • I have a friend who very successfully lost weight by switching to “I will not eat or drink anything that has had sugar added to it” (so fruit is okay, soda is not). The even simpler “I will not drink any soda” also seems to have a fair amount of backing.

  66. A request for explanation regarding philosophy of mind. (I do not currently, adhere to any philosophy of mind–every one seems problematic in some way or another.)

    I do not see how a particular view in philosophy of mind–namely, that subjective, first-person experience is a result of computation–is coherent. This view does not make sense to me because I can give an absolutely complete explanation of why any computation, given a certain input, results in a particular output, without ever appealing to subjective-first person experience within that computation. Thus, the “consciousness” that this theory seeks to explain is completely unneeded for one’s explanatory and predictive apparatus, and so one should not posit it-which is obviously a problematic conclusion.

    One rejoinder to this is that consciousness is like the wings of an airplane. Sure, you could model the wings using quantum physics, but that would take forever; so also, you could model another person as computation, but that would also take forever. So it’s legitimate for you to talk about airplanes as having wings, as people as having consciousness–and airplanes really have wings, and people really have consciousness, even though these aren’t ontologically fundamental.

    But this rejoinder seems to fall through. Wings in the engineer’s model are defined in terms of a certain set of inputs and outputs. So also, if you wanted to model people at a higher level in this case, you would also define “consciousness” as a certain set of inputs and outputs: angry people are those apt to hit you, to say things like “I am angry,” and so on. But this seems to be behaviorism–that there’s no more to one’s mental states than the actions they produce or are apt to produce.

    More generally, far as I can tell the LWosphere tends to say “Ok, we can’t really see how computations can produce consciousness at the moment, but in the future we’ll have a breakthrough that will let us understand how this is possible.” (Show me where I should look if I’m wrong, please) But I can’t even imagine what kind of breakthrough that would be. If we were to be able to put someone in a magical MRI, which tracked everything going on in their brain at the ion-pump level, such that we could see exactly what sensory input and memories combine to make someone say “Hey, I’m experiencing qualia!”–all this seems to do is make it so that we no longer have reason to think someone who says “I’m experiencing qualia!” is experiencing qualia, because we now have a predictive apparatus which explains what they do in every detail, without ever invoking qualia.

    • Protagoras says:

      I’m actually kind of confused by this sort of argument, and will try to explain why. It seems to me that my conscious thoughts motivate my behavior; to take a trivial example, I seek out or avoid certain foods because of how I feel about the experience of taste I get from them. More generally, it just seems to me that conscious states do something, at least in general. The idea that you could separate out the part that does something (what you call the “computational” part; I wonder if the terminology is part of the problem) from what it feels like just seems absurd to me; what it feels like just is what does things. And so if the neuroscientists say these neurons doing that are what are doing things, it seems to follow trivially that these neurons doing that must be what it feels like. Not that you can capture the feeling from an external perspective, of course; no matter how you look at them, looking at neurons is surely going to feel different than having them hooked up and firing in your brain. But that point hardly invites the generation of any mysteries.

      • No, I absolutely agree that conscious thoughts motivate your behavior.

        If I gather what you’re saying correctly–it’s that the computation is identical to consciousness, so from the inside “Feeling hungry” is identical to “Neurons firing in X and such patterns,” and so there is no problem. The computational part is the motivated part.

        The thing is, because any computation whatsoever is completely tractable without reference to conscious thoughts–or any reference to what the computation is like “from the inside”–then I’ve really got no need to posit an inside, in hunger or in any computation whatseover. Why posit an inside, when everything that’s going on from the outside can be explained without an inside?

        My argument is that (Consciousness = computation) and (All things done by computation can be explained without reference to an “inside”) –> (No need to posit view from the inside). This last seems false, which leads me to drop (Consciousness = computation).

        I need to head out for now–but does that make sense?

        • Protagoras says:

          No, it doesn’t. You still seem to be treating the “from inside” as if it must be a different thing from the computational part, something that would need to be added. But, as I said, since the computational part is what actually does things, responding to the environment and motivating action, and the from inside part is what does things, responding to the environment and motivating action, it seems that they have to be the same thing. Izaak was perhaps right to emphasize the quantum physics analogy; to me, it seems as if you were complaining that if you just look at all the quantum activity, you never need to posit the wing. Well, of course you don’t need to posit it; it’s right there. The quantum activity is the wing. I continue not to see why it shouldn’t be the same for consciousness.

          • Thanks for following up. I still disagree. Sorry, this is going to be absurdly long. This has been bothering me for a while.

            “Particular kinds of computations” and “subjective experience” are clearly different in intension; so some kind of evidence is required to say that they’re the same in extension, just like it would require evidence to show that Shakespeare and the Earl of Oxford are the same in extension. I’m not trying to treat the motivated part as if it must be something different from the computational part–but I am starting with the premise that there can at least be evidence pro / con this, which I think is fair.

            The evidence offered for identity is that–if I grok what you are saying–when we run a brain scan, it seems like particular computations have input / output relations with the environment; and when we introspect, it seems like particular subjective states have input / output relations with the environment. So if every change to the former results in a change to the latter, and every change to the latter results in change to the former, then it seems that the former and the latter are the same–just like punching Alfred Bordon at time t, and seeking that Alfred Bordon has a bruise at time t + 1 day, is evidence that the Alfred Bordon at time t is the same as the Alfred Bordon at time t + 1.

            I accept this as evidence and as quite good evidence. (This theory of mind is most probable of those I know.)

            (I don’t think that consciousness is like the wing. Wings are built of things in quantum states in such a way that we could, in theory (maybe) explain how quantum phenomena cause the appearance of solidity, the general features of lift, etc, etc, which we associate with the physical wing. The gross features of your anatomy, similarly, are built of minicolumns and computation-doing things such that you can explain the gross behavior of your brain in terms of the fine. There hasn’t been an explanation of conscious states in terms of minicolumns and computation-doing-things in the same way that there has been an explanation of the grosser features of the brain in terms of minicolumns and computation-doing things; the supposed knowledge of the identity of these former two (computations in brain and subjective brain states) seems to depend on the argument given above, not on any detailed understanding of how conscious states arise from computation. If we did know how conscious states arise from computation in this detailed and predictive way (that kind of computation has an inside, that kind does not), then we would be able to settle arguments about strong AI and vegetarianism and vegetative states much more easily than we currently seem to be able to. [If there is this kind if description, I’d be interested in hearing it, of course.])

            Now let’s switch to a problem. In every case of any kind of computation that occurs outside of a human mind, we’re not tempted to think about an inside. I can implement A* or a Bayesian sorting algorithm or what have you, and it never crosses my mind to think of an inside. I might think about efficiency, memory use vs. demands on the processor, whether the algorithm always comes up with the best answer; but thinking about the “inside” would seem dumb. Talking about an “inside” is totally causally superfluous in all these other cases–which, it should be noted, are the only cases when we have a really clear understanding of what is going on. We’re still figuring out Really Important Things We Didn’t Know about the brain; we don’t have a Really Clear Understanding of how the brain does computation the same way we do about A* and naive Bayes (I’m pretty sure, at least). Talking about “insides” of computation is to use a kind of language foreign to the formal study of computation and paste it on to the study of computation, without a clear explanation of how they relate.

            So on one hand we have
            1. Changes to mental computations seem to occur iff changes occur to subjective experience.
            2. A thing intensionally different from another thing, which changes iff the other thing changes, is probably identical to the other thing.
            3. So mental computations are probably identical to subjective experiences.

            But on the other hand we have
            4. Positing inner states is unnecessary to explain what any computation does.
            5. So if (the relevant part) of what humans do is a result of computation, we have no reason to judge them to have inner states.
            6. Ah, um, modus tolens.

            And again, the obvious response is that from our own, introspective experience we use our inner states to explain our own actions–so if these are computation, then computations must have inner states. But if every other kind of computation is inner-stateless, then perhaps the right conclusion would instead be that there’s more to conscious states than simply computations. Our current state of knowledge about the human brain is also compatible, say, with the idea that consciousness requires a particular physical substrate; or (probably) with whatever it is that Penrose thinks the mind does; and so on.

            I’m not really happy with the above. I think the first numerated argument above is probably stronger than the second, but I think the second decrease the probability of what the first argues for.

          • Mr. Eldritch says:

            @SeekingOmniscience

            “4. Positing inner states is unnecessary to explain what any computation does.
            5. So if (the relevant part) of what humans do is a result of computation, we have no reason to judge them to have inner states.
            6. Ah, um, modus tolens.”

            My reply to 4:
            Well, yes. And if I have a complex computer performing an operation on two vectors such that a third vector perpendicular to both of them is produced, then this can entirely be described in terms of quantum states of electrons and silicon atoms (with the occasional dopant) without ever positing the mysterious abstract concept of a “cross product.”

            Nonetheless, for all inputs, its outputs are identical to an algorithm implementing a cross product, so we say it performs a cross product. And, indeed, it is. In fact, this is what “this algorithm performs a cross product” means.

            Likewise, it’s entirely possible to fully analyze the brain without ever making reference to conscious states, just neuron weights and synaptic potentials – just as it’s entirely possible to fully analyze a computer without ever making reference to, say, “Microsoft Windows” (which it is in fact running) and simply looking at transistors and voltages and hard-drive magnetic fields.

            That doesn’t mean the conscious states are not, inevitably, there. Those bit-patterns and hard-drive states and their passage and transformation through the circuitry, and the way these patterns react to input from various channels to produce internal state-changes and output ARE, indeed, Microsoft Windows. They aren’t anything else, and indeed can’t be anything else. Your computer is running Windows.

            So it’s not quite correct to say that your computer can be entirely analyzed without positing Windows. Your computer can be entirely analyzed and perfectly simulated (step-by-step, with pencil and paper) without ever making reference to the abstract, higher-level features of what those voltages and magnetic fields “mean” – but they nonetheless translate to Windows anyway. Even your pencil-and-paper implementation of your computer’s voltages and magnetic fields will still be running Windows, even though you may not realize it. (And if you deleted Windows, you’d get different results.)

            Same deal with the brain. You can analyze the brain without ever making reference to conscious states, but they’re still *there* – you’re just talking about a system which gives rise to certain neuron patterns, which is like talking about “1 + 1 + 1” instead of “3.” They certainly look different, but they’re the exact same thing from a different frame of reference.

          • @Eldritch:

            I will need to think about this more.

            It still seems fairly clear to me that my painful, pen-and-paper implementation of windows combines small parts into larger parts in a way that permits us to understand how these large parts inevitably result from these smaller parts–but that we have no such knowledge of how computations, etc, combine to inevitably form subjectivity. (We might have knowledge of how lower-level brain functions combine to form different higher-level brain parts, which result in visible behavior–this seems to me analogous to the example with windows– but this doesn’t mention subjectivity, so the example seems dissimilar.) And this doesn’t seem to me merely a case of a lack of current knowledge; I don’t know what an experiment would look like which showed that a particular (conscious) higher-level thing was a result of lower-level results.

            If this is an intelligible question–would you say it a result of mathematical or of scientific process that one comes to the conclusion that X computation results in consciousness? Maybe if I knew what the reductionist / computationalist answer to this question was, I would be able to think about this more clearly. (Either of these seems very weird, and trying to pin down what such a process would be… I can’t think of a way to do it.)

          • Paul Torek says:

            @Seeking,

            First, my cards on the table: I think that for all practical purposes (but not “in principle”) consciousness-as-we-know-it does require a particular physical substrate. Still I want to critique your argument. For that critique let me start by recommending Jenann Ismael’s paper, or better yet, her book The Situated Self. The argument centers on a map-territory analogy, where your model of physical reality is the map, and your experience the territory. To locate your phenomenology in physical events is like locating yourself on a map. Your location need not be explicit on the map itself, for the map to be accurate and complete. A map does not become more accurate by adding a red dot saying You Are Here. (It does, however, become a lot more useful!)

            Suppose the map is a real-time display which updates based on automated measurements. It can display all that happens without explicitly telling where you are. That does not mean you are not there, in the mapped territory.

            Now for my own favorite point on the subject. In addition to being aware of external conditions like day and night, light and dark, we are also aware of some of our own internal states along the cognitive pathways, pathways from external world to belief. So, in addition to being able to estimate objective brightness we are aware of subjective brightness. Maybe that helps us overcome certain sorts of illusion. However it evolved, we’ve got it. Could an organism be designed to expose to executive processes, only the (best representation available of) objective brightness, without any subjective brightness? I don’t see why not. Such an organism could compute the same function of radiant emittance detection, albeit by a different algorithm. So I share your distrust of one variety of computationalism, namely functionalism.

    • Izaak Weiss says:

      “you could model the wings using quantum physics”

      I think the point is that in the real world, wings DO work via quantum physics. You have the analogy backwards. Instead of saying that quantum physics is a way of modeling wings, we say that wings are a term we use to refer to a particular form of quantum wavefunctions. Instead of saying that a computation is a way of modelling consciousness, we say that consciousness is a term we use to refer to a particular form of computation.

      • Irenist says:

        consciousness is a term we use to refer to a particular form of computation

        How is consciousness different from other forms of computation? I.e., is there a way to tell from the “outside” that a given computation will be conscious?

        • Protagoras says:

          On your second question, not with perfect reliability, certainly. But sometimes we seem to be able to tell with reasonable confidence.

          • Irenist says:

            Fair enough. So, IIRC:
            1. Insects are thought to be conscious. There’s a “what it’s like” to be an insect.
            2. Some of our computers (and maybe some of our robots?) nowadays are “as smart as an insect”

            So, are the insect-level smart machines conscious? How do we tell?

            (Serious question. I think hylemorphic objections-in-principle to conscious AI are one of the potentially weakest arguments offered by Feser & Co., and I’ve been thinking about the issue a lot, wondering whether to part company from them on the issue.)

          • Protagoras says:

            I’m not actually convinced that insects are conscious, and I’m equally agnostic about machines that seem at the same level as insects. Perhaps I would have a more confident opinion if I knew more about insects, and about consciousness for that matter. But all I claimed was that sometimes we seemed to be able to tell with reasonable confidence; I was particularly thinking of the fact that we are reasonably confident that other human beings are conscious. Animal cases are mostly less clear.

          • Relevant to our uncertainty regarding what is unconscious or not. And a DFW article, which is always fun to read. It’s all about how no one really knows what it’s like to be a lobster which is boiled alive.

            http://www.gourmet.com/magazine/2000s/2004/08/consider_the_lobster

    • lmm says:

      > you would also define “consciousness” as a certain set of inputs and outputs: angry people are those apt to hit you, to say things like “I am angry,” and so on. But this seems to be behaviorism–that there’s no more to one’s mental states than the actions they produce or are apt to produce.

      I believe behaviourism denied mental states entirely. But more to the point: so what? I’m happy to endorse functional definition of conscious experience (like an aeroplane wing): we consider someone to have conscious experience if they do things that are most easily explained as consequences of having conscious experience, like caring about the consequences of their future actions, having emotional states, writing angsty poetry and so on.

    • youzicha says:

      Re your last paragraph, I think the problem is not with the computational theory of mind, but with this conception of qualia. (E.g. Daniel Dennett maintains that “qualia don’t exist”, meaning that the common conception of qualia is an incorrect analysis of how subjective experience works).

      I think the concept of qualia rests on a kind of Cartesian intuition: there is no way for me to be 100% sure what happens in the outside world, but I have certain sense-impressions, and at least by introspection I can infallibly correct about those. And then we expect these sense-impressions to play a distinguished part in the explanation.

      But given that thoughts are implemented in the brain, surely this is not right. Our beliefs about the outside world are stored somehow in the brain, but so is our beliefs about our own thoughts. It’s not that I first perceive a field of unstructured color-sensation, and a little homunculus carefully analyses it and deduces that there is a pillow in front of me; I first know that there is a pillow, and if I then introspect where that knowledge came from, I can deduce that I see a red color. (The ability to recognize objects is the simpler one, e.g. lower animals can recognize objects, but not introspect about how their vision works).

      So, suppose your magic MRI machine scans my brain, and it shows no qualia as such, but it does show bits of my brain that made me believe that I have qualia (the bits responsible for representing information about my own thoughts). Then I think the thing to do is to bite the bullet and conclude that the mMRI-machine is right! After all, it perfectly accounts for all the data I got from introspection.

      • I don’t see how having non-primary origin of qualia means I don’t have them. I think you’re right that we often identify objects, then their constituent parts, and so on and so forth–I don’t see a colored bitmap and then deduce “Ah, a pillow!” But even so, surely I’m at least certain that I am being appeared-to redly, even if the origin of being-appeared-to-redly involves some preprocessing?

        I guess I’d have to ask, when Dennett denies that there are qualia, what is he denying?…I need to think about this more.

        • youzicha says:

          I can’t really speak for Dennett, because it’s been too long since I read him. There is a short and readable explanation of his anti-qualia stance in Sweet Dreams.

          In a hypothetical future materialistic theory of mind, there would presumably by definitions like “we say that a computational system is conscious if it has the following property …” along with an explanation of why such systems tend to claim that they have conscious experience. (And similarly, a definition of “a system which is being-appeared-to-redly“, and a derivation of why such systems claim that they are.) We don’t yet know what the definitions would be, but you (and many others) say that we can already tell that they are not satisfactory, because they cannot be the right explanation of qualia. So there must be some properties which we intuitively feel that qualia should have, but which any such materialistic concept doesn’t have.

          I don’t know which properties in particular that you have in mind, but two that Dennett has focused on are “Cartesian dualism” (there is a separate someone or something to whom the sense perception is presented), and “infallibility” (when I introspect about my conscious experience, I am logically guaranteed to be right). Neither of these hold in the materialistic theory.

          I guess you already reject the first one. As for infallibility, I think this also falls apart if you look at it more closely. Of course, it often happens that we misremember things, so if we are infallible about something, it can only be about the our experience in this very instance. But in fact, there is no “this instance” in the brain—different subsystems process sense impressions at different rates, and their outputs are then timestamped and later reconciled. So even our experience of the current moment works a lot like a memory, albeit a memory of what happened 100 to 300 milliseconds ago.

          Even if the Descartes-style intuitions about qualia don’t hold true, there clearly is something which causes us to believe that we have subjective sensations, and it should be possible to formalize what that something is. My guess is that it will come out to “true knowledge about what our perceptual systems are currently doing”. But if so, that concept is easy to accommodate in a materialistic framework.

        • Anonymous says:

          Here’s Dennett’s paper, Quining Qualia: http://cogprints.org/254/1/quinqual.htm

    • The argument you are hinting at has been discussed extensively by professional philosophers of mind. David Chalmers calls it the “paradox of phenomenal judgment” and has some pretty interesting responses to it (see The conscious mind, chap. 5).

      • I’m reading through the book now–got it on Kindle. My impression heretofore has been of Chalmers as the Dude Whom Eliezar Thinks Has it Wrong About Zombies. This will be interesting.

  67. moridinamael says:

    I’m sure this has been bugging everyone as much as it bugs me.

    Why the NRx fixation on 16th century? On monarchies, hierarchies, castes, monolithic social institutions? All that stuff is way too progressive. Some of it is only thousands of years old. Such Cathedral, very Leftism. To be truly reactionary you’d need to aim at what people were doing 100,000 years ago, preferably as close to the dawn of homo sapiens as possible.

    Living in small hunter-gatherer bands. Practically no concept of a nuclear family unit. Weak to nonexistent central authority, egalitarian social structure. Very different concept of gender roles, with woman possessing as much decision-making power as men, and in some cases performing the same tasks. Matrilineal kinship lines. Status contests centered around hunting and art and practical skills, not typically around warfare. Animistic, non-organized religion, serving primarily as a tool for storing practical and moral knowledge in the form of stories. Minimal material possessions. Greater focus on physical activity, greater general level of fitness and health and lifespan compared with later agrarian (hierarchical) societies. Less work-hours, more meaningful work.

    Also, we should clone and resurrect Neanderthals, so we can have a group that we can all agree to view as explicitly bad and wrong, and we can hunt and kill them with impunity because they Aren’t Human. Basically they would serve the role of orcs. I imagine this would lead to much greater solidarity among homo sapiens. Failing that, we could channel our aggressive and competitive instincts into hunting bears and lions and elephants with intentionally handicapped weapons like spears.

    In light of all this, the social institutions NRx is looking to bring back are just a pathological reaction to overpopulation, in the same way that NRx accuses democracy of being a pathological reaction to modernity. Ares only manifests when there are too many damn people and not enough real threats.

    You might say, “Hey, it wasn’t actually so nice as you’re making it out to seem,” and then I might just slowly slide across the table toward you an artist’s rendering of conscripted malnourished peasants being ridden down by Janissaries or something.

    I’m probably not the first person to make any of these points but. If Scott is right in his point that bringing back a powerful monarchy would require defeating Vast Formless Things, momentum-laden technological and cultural forces, then returning to a hunter-gatherer lifestyle faces only a few more hurdles than does returning to the Napoleonic Age. Namely, a tremendous reduction in human population, OR a tremendous increase in available space, either of which could be accomplished with massive environmental catastrophe on the one hand or space colonies on the other. Modern medical technology would neutralize almost everything that we view as unappealing about the hunter-gatherer lifestyle. If life expectancy and infant mortality were brought to modern levels, I say without hesitation that I would rather live in the Pleistocene than in the 16th Century. That last sentence is the only fully serious part of this post.

    • Short answer: take a few seconds and look through Post Anathema

      Long answer: NRx likes technology, science, hierarchy, and high-trust societies. None of those things existed in the neolithic, except maybe the trust. We don’t like chaos, distrust, leveling, and pretty lies. The 15th-17th centuries provide an approximation (very imperfect, of course) of the things that we do like without the things that we don’t like.

      (As an aside, I’m very skeptical of almost all descriptions of life in the neolithic, as there tends to be a suspicious concord between the politics of the author and their descriptions of neolithic life. Plus, the answer to the question of “how do you know that” is usually “extrapolation from the few remaining hunter-gatherers”, which is problematic for all sorts of reasons.)

      • Peter says:

        There’s a quote from Bertie Russell here, ah yes:

        “What do we, who stay at home, know about the savage? Rousseauites say he is noble, imperialists say he is cruel, ecclesiastically minded anthropologists say he is a virtuous family man, while advocates of divorce law reform say he practices free love; Sir James Fraser says he is always killing his god, while others say he is always engaged in initiation ceremonies. In short, the savage is an obliging fellow who does whatever is necessary for the anthropologist’s theories.”

        (Except these days we don’t use “savage” as a noun)
        http://users.drew.edu/~jlenz/brs-quotes.html

      • Deiseach says:

        I like the 10th – 13th/14th centuries; what does that make me? 🙂

    • Nornagest says:

      I’m not a neoreactionary or an anthropologist, but I’ve read a lot of ethnography, and I feel compelled to qualify “egalitarian” in a forager context. It doesn’t mean that everyone’s equal; it means that there isn’t a class system or other formalized hierarchy. The impression I get is that it’s a lot like high school: there are no ranks and no titles, but the status games are no less intense for that, and everyone knows who the winners and the losers are.

      Also, while nuclear families are rare (and don’t really make sense in the context), kinship structures viewed more generally tend to be a big fuckin’ deal. And they aren’t necessarily matrilineal, although they sometimes are.

    • Deiseach says:

      Neanderthals are human, just not Homo sapiens sapiens.

    • drunkenrabbit says:

      As far as I can tell, NRx thinks you can have technological modernity with something resembling a 16th century social structure, but going full paleolithic totally precludes that.

  68. Jaskologist says:

    It’s time for something completely different:

    Share with us your spooky stories, whether supernatural, weird, or glitches in the Matrix. In the spirit of the genre, there are only two rules:

    1. The story must have happened to you or to a close friend.

    2. Lying is, of course, completely permissible.

    • gwern says:

      Here’s one from me: http://www.gwern.net/fiction/Stories#true-dreams It may not strike others as all that spooky, but I found it as disturbing as heck to have a prophetic dream. (It’s too bad I don’t have access to the NYT corpus so I could calculate the chance of my dream being right by seeing what fraction of Wiktionary entries have exactly one attestation in the NYT corpus.)

    • Anonymous says:

      >Lying is, of course, completely permissible.

      Only lies the reader would want to believe!

    • Brad says:

      Oh goodness. I have far more than you would care to hear. Seriously – if I had the time or inclination, I could fill up this forum with replies until I got banned or you stopped caring. I’ll warn up front that I am a Christian fundamentalist – and I am not lying, not on purpose by any means. This is not a concocted story (at least, it is not meant or intended to be), but rather my sometimes fuzzy recollection of …unusual things that happened to me.

      There’s a line in the novel Dance Dance Dance by Haruki Murakami where the main character, after a series of fairly blatant coincidences, kind of exhasperatedly goes “alright, alright, you win. This is not a coincidence, these things are connected.”

      I feel like the protagonist of that book an awful lot.

      Here is one: when I first became a Christian – when I really considered myself converted in 2005 – I had a pornography habit, and I asked God to help me stop said habit. Maybe what happened, began to happen before I asked, it’s been a long time and I might be getting details wrong.

      What is clear to me is this: I began to get sick, most typically with really incredibly awful stomach ailments, within 24 hours of looking at pornography, and this would subside when I repented in prayer to God. It wasn’t limited to that, but it presented in a variety of misfortunes. You would think after this happened a few times, someone would get gunshy enough to quit porn cold turkey, but I continued to look at porn well into my freshmen year of college; in fact, because I wasn’t monitored at college, I ended up looking at porn more, not less. *Incidentally*, I was incredibly sick that year, I had all kinds of incredibly gross infections and I recall one incident where I suddenly couldn’t see in the middle of an exam because I was afflicted with pinkeye and my eye’s view was obstructed by discharge.

      Anyways- I did awful, awful in school that year, nearly dropped out (but didn’t, although I got ghastly grades) – and after deciding to commute, asking for help from my parents, ended up dropping my porn usage a lot (and eventually entirely), and finally stabilizing my GPA.

      I looked through the bible on this later, and discovered, unbeknownst to myself up to this point, that it turns out the notion of God chastising those who are his sons is an idea remarked on more than a few times in the bible; Hebrews chapter 12 in the new testament, verses 4-13 are relevant here, as are Psalm 94:12, Revelation 3:19, Proverbs 3:11-12 … among others.

      What makes this strange is how a few months ago, I, having been in a way of walking that was largely free of browsing porn for several years, ended up recently faltering and looking, at something I should not have been looking at – for rather too long. I prayed to God to help me not to do that again.

      Immediately after, the same day, me and my wife then went out for a walk, during which, (in an effort to get our dog to chase me), I fell and gave my ankle a mild sprain. I felt rather thankful for this, namely because I believed it to be an answer to my prayer previously.

      Again, this is a weird story, and it is possibly not the *weirdest* story I could remark on, although truthfully this one sticks with me because it is not something that has vanished into the past, but which exists on a continuum to this day in my life.

      I’ve told people about this before, and the usual response is along the lines that it is a big coincidence, or (if I examine a criticism from my own mind) I am reading too much into it – but I ultimately persuaded that, yes, Jesus Christ was, chastising me to drive me away from sin. I don’t know what else to say here; this is my testimony on this matter and one I hold deeply in my heart.

      • Brad says:

        I will remark one more, seeing Gwern’s post about a dream: do note this post should be read in the context of my previous post.

        When I was still single, and I believe, working on my undergrad, I had a dream that was rather remarkable because it was when I awoke, “the best dream I had ever had”, as I thought at that time.

        The dream was as follows: I am running through misty, foggy, deserted, cobblestone streets at night, and a monster is pursuing me. It is nearby, although I know not where it is, exactly, only that it’s chasing me. I only vaguely recollect what it was like, being something like a wolf (or werewolf?) or vampire – it was terrible.

        I am chased into a graveyard, which is also a deadend, and I go up to a grave and begin to dig up a grave – but when I get to the grave, there is no body in the grave – rather, there is a sword.

        I pick up the sword, and the entire dream shifts; I am no longer fearful, and I now run out of the graveyard and defeat the beast, tearing it apart with the sword.

        When I first had the dream – and I don’t know exactly when I had it – I was happy, because it was the best dream I ever had, as mentioned before. Nothing else came close. But then I went to sleep (or got up? – again, I don’t recall) and I generally didn’t think too much about it.

        A few years later, I was studying the bible more and I ended up reading the book of Daniel, a prophetic book in which dreams figured prominently. An evening (or two?) after I read the book, like a flash of lightning, as I was either going to or arising from bed, the memory of the dream came back to me, particularly this interpretation (which, again, came on me suddenly): the empty casket was the empty tomb of Christ; and the sword which I came upon therein was the Sword of the Spirit from Ephesians chapter 6 – which, as the passage informs the reader, is the Word of God.

        What’s weird about this is these were connections I was totally ignorant of these symbols beforehand when I *first* had the dream; I wouldn’t have connected the dots before hand; and the whole thing just seemed, at the time I had it, to be a very vivid and strange dream. But it has, frankly, influenced my theology, making rather more bible-centric than it had been before.

      • Ialdabaoth says:

        I have a counter-question for you.

        I am also a Christian fundamentalist, when I am not wallowing in nihilistic despair or trying desperately to be a rationalist.

        Why is it that God sees fit to grace you with evidence for His existence, and yet when I pray – desperately, honestly, passionately, and with utter lack of selfish desire – for any kind of strength to carry on and follow His will and commandments, or just for the ability to maintain some shred of faith in Him – I am answered by nothing but a black void?

        Why does He love you and not me? And if He does love me, why doesn’t He let me know that He loves me, even for a second?

        I pray with all my heart and soul and mind for nothing more than for God to show His love through me and guide me to do His will. The clearest answer I have ever got in response is “fuck you. Kill yourself. Rot in hell.” When I accept that that could not be His voice talking, and pray for Him to overcome those thoughts so that His true will shines through, I get nothing. I have got nothing since I was nine years old. I am now forty. What am I doing wrong?

        • Brad says:

          I am under personal duress at home, but allow me to as concisely as I can, address the broad points here.

          I must remark on this first, although it seems harsh: either you are one of his sheep, or you are not. If it is the former, have faith, and if is the latter, repent. I do not know you personally; you will come to God and test yourself on this matter. This is a serious point which should not be dismissed out of hand; this sermon might be a good starting point: (http://media.sermonaudio.com/mediapdf/5220621750.pdf) (and a general link: http://www.sermonaudio.com/sermoninfo.asp?SID=5220621750)

          While that sermon I linked is principally about how to understand assurance of salvation in a biblical manner, I do want to point out that there is a statement the speaker of that sermon makes early on, and which I quote here:

          >”This silly Christianity in America. “Repeat these words after me.” No, you might have to wait upon God. You might have to cry out to Him until the work is done—a true work, a finished work, a complete work”

          And I want to point something out: This is how it works in the bible. It does not (necessarily) happen quickly, let alone at our whim; God does things at his pace, fast or slow. Jesus said this:

          > 7 “Ask, and it will be given to you; seek, and you will find; knock, and it will be opened to you. 8 For everyone who asks receives, and he who seeks finds, and to him who knocks it will be opened. 9 Or what man is there among you who, if his son asks for bread, will give him a stone? 10 Or if he asks for a fish, will he give him a serpent? 11 If you then, being evil, know how to give good gifts to your children, how much more will your Father who is in heaven give good things to those who ask Him! 12 Therefore, whatever you want men to do to you, do also to them, for this is the Law and the Prophets.

          Consider also the prayer of Daniel, in Daniel chapter 10:

          > 2 In those days I, Daniel, was mourning three full weeks. 3 I ate no pleasant food, no meat or wine came into my mouth, nor did I anoint myself at all, till three whole weeks were fulfilled.

          When his prayer is fulfilled, he receives this vision:

          >5 I lifted my eyes and looked, and behold, a certain man clothed in linen, whose waist was girded with gold of Uphaz! 6 His body was like beryl, his face like the appearance of lightning, his eyes like torches of fire, his arms and feet like burnished bronze in color, and the sound of his words like the voice of a multitude. 7 And I, Daniel, alone saw the vision, for the men who were with me did not see the vision; but a great terror fell upon them, so that they fled to hide themselves.

          >12 Then he said to me, “Do not fear, Daniel, for from the first day that you set your heart to understand, and to humble yourself before your God, your words were heard; and I have come because of your words. 13 But the prince of the kingdom of Persia withstood me twenty-one days; and behold, Michael, one of the chief princes, came to help me, for I had been left alone there with the kings of Persia.

          Daniel had to wait three weeks for an answered prayer, and the reason his prayer was delayed was because of a demonic enemy – the “prince of the kingdom of Persia”.

          I also give another example from the scriptures: In Jesus’s encounter with the Canaanite woman, it looks like he is being needlessly harsh to the woman – in verse 23 it reads that Jesus “answered her not a word” – here is the quotation of the verse:

          >21 Then Jesus went out from there and departed to the region of Tyre and Sidon. 22 And behold, a woman of Canaan came from that region and cried out to Him, saying, “Have mercy on me, O Lord, Son of David! My daughter is severely demon-possessed.”

          >23 But He answered her not a word.

          >And His disciples came and urged Him, saying, “Send her away, for she cries out after us.”

          >24 But He answered and said, “I was not sent except to the lost sheep of the house of Israel.”

          >25 Then she came and worshiped Him, saying, “Lord, help me!”

          >26 But He answered and said, “It is not good to take the children’s bread and throw it to the little dogs.”

          >27 And she said, “Yes, Lord, yet even the little dogs eat the crumbs which fall from their masters’ table.”

          >28 Then Jesus answered and said to her, “O woman, great is your faith! Let it be to you as you desire.” And her daughter was healed from that very hour.

          There are elements here, then: First off, we must test ourselves to see if we are in the faith; if we are in the faith, things might take longer than we wish, and we have demonic enemies who would want to drive us from God and take the word of God away from our hearts.

          Which brings me to your last remarks about voices telling you to kill yourself and rot in hell – sir (or madam), do not listen to those voices – for understand, what the word tells us:

          >For we do not wrestle against flesh and blood, but against principalities, against powers, against the rulers of the darkness of this age, against spiritual hosts of wickedness in the heavenly places. Therefore take up the whole armor of God, that you may be able to withstand in the evil day, and having done all, to stand.

          This verse is very simply talking about spiritual enemies, demonic forces, which are trying to destroy you. Perhaps this is manifestation thereof, or perhaps of our own hearts, (which is desperately wicked – C.f. Jeremiah 17:9), but please do not allow such discouragement to dissuade you from continue to seek after God in prayer.

          One last thing to consider: In the story of Prodigal Son (https://www.biblegateway.com/passage/?search=luke+15%3A11-32&version=KJV), one son is disobedient and to quote, “took his journey into a far country, and there wasted his substance with riotous living.” This is the son whose return is celebrated, for he was dead, and now is found.

          The elder son gives us this exchange on the return of his brother:

          >25 “Now his older son was in the field. And as he came and drew near to the house, he heard music and dancing. 26 So he called one of the servants and asked what these things meant. 27 And he said to him, ‘Your brother has come, and because he has received him safe and sound, your father has killed the fatted calf.’

          >28 “But he was angry and would not go in. Therefore his father came out and pleaded with him. 29 So he answered and said to his father, ‘Lo, these many years I have been serving you; I never transgressed your commandment at any time; and yet you never gave me a young goat, that I might make merry with my friends. 30 But as soon as this son of yours came, who has devoured your livelihood with harlots, you killed the fatted calf for him.’

          >31 “And he said to him, ‘Son, you are always with me, and all that I have is yours. 32 It was right that we should make merry and be glad, for your brother was dead and is alive again, and was lost and is found.’”

          I bring this up for the following reason: I remarked on what you call a “proof”, as though I described angels singing before me joyously – but my “proof” was agony and pains, which I interpret (I feel, correctly,) as the lash of discipline. Do you not consider, that I have sometimes wondered why I did not experience in my walk with God something more light, peaceful and full of joy, rather than something painful? When these “proofs” were coming to me, I was generally wishing they would go away, because they were so unpleasant. Although I am happy, in retrospect, for such things, I sure wasn’t anticipating them in the beginning of my walk; I’m not sure if I had known what would have happened that I *would* have asked for them.

          The prodigal son – the disobedient one – is the one who experienced these difficulties of having to feed pigs while being himself hungry, after he wasted all his money. The other son did not have to go through such a trial – yet he is displeased, even though his Father has a good explanation available, which is actually mentioned twice in the passage:

          > 24 for this my son was dead and is alive again; he was lost and is found.’

          Anyways, my question is this: what exactly is it you’re expecting God to do to prove he loves you?

          I have felt the love of God in experiences, yes, but perhaps I gave the wrong impression that this is where I have most strongly felt his love – I do feel his love in those, but I feel his love more, and most strongly, in reading his word. My friend, if you want to understand the love of God, read his word, read the bible, in prayer and faith. (See: http://biblehub.com/john/10-27.htm) Understand that the fear of the lord is the beginning of wisdom (C.f. Proverbs 1:7, among other locations in the bible), and I feel this is true with understanding God’s love as well as other forms of his wisdom. That is, we should understand God’s love, ultimately, at the cross ,where Christ was crucified for us. He *has* shown his love for you and me there, by paying the penalty of your (and my) sins, dying in our place – how can God show his love any greater than that? If you want to understand the love of God, look at the cross, for it is written:

          > 13 Greater love has no one than this, than to lay down one’s life for his friends.

          I don’t think making this post longer than it needs to be will be useful. I hope that this will help.

          Good luck and God Bless.

          • Ialdabaoth says:

            Here’s the essence of what I don’t understand:

            Why would God allow me to come into being with a mind incapable of having faith in Him as my loving creator, and then punish me forever for not having faith in Him as my loving creator?

            And if that’s simply not logically possible, which therefore proves that I must be capable of having faith in Him as my loving creator, how much harder do I have to try? Is there any evidence that could prove that I’m trying as hard as I can to believe, short of actually believing?

          • drunkenrabbit says:

            @Ialdabaoth

            Have your read about “spiritual dryness” in Catholic thought? It’s a feeling of desolation, loneliness, and separation from God that’s not uncommon, and that a number of people, including Mother Theresa, endured. I can’t recommend anyone in particular, but there’s a number of saints and theologians who have written about it who might be helpful.

          • Anonymous says:

            I’ve read most of the literature on this. Here’s the problem:

            It sounds like extremely motivated reasoning, from people who desperately want to keep believing something.

            My model-of-the-world says that, if I had some kind of sense of God the way Brad did, of COURSE I would be motivated to maintain that sense. Or, if my lifestyle and identity continued to be intertwined with a belief in God the way Mother Theresa’s were, I would likely be motivated to maintain that sense. But without those motivations, and without any sign, I’m not sure if it’s within my psychology to continue to hold onto these beliefs – any more than it’s within my psychology to abandon them.

          • Excatholic here.

            I went through this forever–especially the thing where everyone else seems to have some kind of experience of God, but this experience is noticably absent from one’s own life. It’s really painful, and continually causes one to ask oneself if one is a horrible, evil person. At least, I was doing that pretty much constantly. Because, after all, if God loves you, and wants you to be with him, and the only way for you to be happy is to be with him, then the only reason for constant experience of absence would be on your side, right? So yeah, that sucks.

            I eventually noticed that this kind of “experience of God” seemed to track naturalistic explanations better than spiritual explanations. I.e., people who are in circumstances more apt to promote deep-feeling experiences, because of natural surroundings, do so–and those who aren’t in such natural surroundings don’t, even if it would be spiritually fitting.

            So both Catholic and Buddhist monks report experiencing overwhelming peace in similar ways, and natural explanations of this are more parsimonious than theistic explanations. Similarly, Christian parents trying to follow what they think is God’s will outside of surroundings apt to promote such feelings do not get them, even though (were the spiritual account true) this would not happen.

            FWIW.

        • Anonymous says:

          What’s wrong with black void’s all of a sudden? Not what you were expecting?

        • 27chaos says:

          Similarly, I have had many experiences where I prayed for something, for example, “insight”, then believed fully that I received it, and then acted on that insight, and then found out that it was completely and totally wrong. This happened literal hundreds of times before I deconverted.

          Every time I have thought I heard from God, it turned out to be my own voice. Now I try to listen to my own voice directly, and to be aware of its limitations. It works better.

    • Deiseach says:

      I wish I had Big Exciting Future Prophecies to share, but all I have are mild cases of what is like déjà vu; sometimes I’m doing something, or engaged in a conversation with someone, and then I get the feeling “Hang on, I dreamed this!” It’s a sense of recognition of ‘experienced this before’ but not in the “yeah, that’s because you’ve photocopied this file five times already”.

      Nothing exciting or ominous, completely ordinary mundane things. But they strike me as “I lived through this already in a dream; I remember this from before”. Just with the very strong conviction that this was an experience in a dream. Now, whether that’s a form of psychic time-travelling ‘when you’re asleep your etheric body wanders and there is no time so past, present and future are all one’, or whether it’s just brain weirdness I have no idea.

      It’s never anything like the lotto numbers or ‘don’t go out that door, you’ll be chased by a lion!’, it’s really brief momentary ‘oh that line they said/this action/I stood up and turned to the wall exactly like that’ is familiar from a dream.

      • ckp says:

        One of the very few habits that I have managed to successfully train myself is, as soon as I feel deja-vu coming on, I immediately try to explicitly predict what happens next. It’s pretty much a reflex by now.

        I haven’t been able to predict anything, and I think experiencing that on a gut-level has reduced how convincing the “memories” feel from the inside over time.

        • This seems like a good habit (or a fun one, anyway). I almost never feel deja vu, but I did once read a webpage that I was convinced I had read before (but I had some reason to doubt I had actually read it, I forget the details). Anyway, I stopped myself and tried to predict what the document would say next, and I was right, I had read it before.

          • Gbdub says:

            Similar things for me. I’ve gotten to the point where I am convinced that not only did I dream that this happened before it happened, but that the dream included thinking about the fact I had dreamed it before.

      • Jaskologist says:

        Didn’t you say you saw a pooka in a recent post? That sounds like a spooky thing.

        • Deiseach says:

          It was, and I’m fairly convinced it really was a pooka, and in general I don’t have much time for pishogues and superstitions.

          But I’m still damn sure that was a pooka and not just a puck goat.

    • ilzolende says:

      When I was in elementary school, one of the Macs logged in the user as an administrator by default. Some students were responsible for shutting down the computers at the end of the day. One day, when I was on the computer that gave me partial admin privileges, I found the scheduling options in the Energy Saver section of System Preferences and configured them to start the computer automatically at 8 and shut it down at 3. A couple days pass, and then I told my teacher what I did and how she should use her admin account to set it up on the other computers. “Oh,” she said, “that’s what’s been going on!” She then explained that one of the more … spiritual … students in the class had been telling her about the “ghost computer” that a ghost pressed the power button to start each morning. (My request to have all the computers start up automatically was not approved. I did end up getting an admin password for all the classroom computers later, when I was setting up some restrictions on a set of 5 computers and the teacher was not eager to enter her password 10 times. I didn’t do anything of interest with the password, apart from moving some applications to the Applications folder.)

      Non-supernatural scary story: I was dealing with the school psychologist. Note: If the psychologist is being paid by an organization with goals related to you that you do not share, you should probably be careful. (The first incident I had with this one was noticing that my behavior in a religious setting was far more compliant and conforming than I expected it to be.) I was in her office during 0 period when the fire alarm went off. I got up to leave. (I don’t take 0 period classes, so I’m not actually required to be on campus then.) She told me to sit down. The alarm was still going off, so I left. When the alarm stopped and I reentered, she was naturally upset. She then lectured me, with no sense of irony whatsoever, on how I always need to obey authority figures such as teachers, even if I think what they’re telling me to do is wrong. (“Obey authority figures” is a good heuristic if the authority figures are competent, but “do stuff you think is wrong” is usually a bad idea.)

      When I am partially asleep, my intuition always reverts to quite confidently classifying time as being as easily bidirectionally traversable as space is. Unfortunately for my desire to sleep 8 hours in a 6-hour period, this is wishful thinking.

      I once saw an opaque raindrop in a cloudy but not-raining sky land near me. This was when I was in elementary school, so I thought that it was a tear from a dragon.

      • ilzolende says:

        Forgot to mention: My printer really hates me. It was refusing to shut down properly no matter how I pressed the power button. I unplugged it from the wall … and the power button light stayed on. (Probably something with a capacitor or a battery backup or something, but still not fun.)

      • Anonymous says:

        When you eventually got admin privileges, did you set them to start and stop automatically?

        • ilzolende says:

          No, because doing stuff with admin privileges that are technically not yours that the authority figures explicitly asked you not to do is usually a bad idea. I did set them to shut down automatically, but automatic starting up had been very firmly vetoed.

    • My wife’s father grew up in one of those famous Romanian orphanages. At the orphanage, there was another boy who would crawl on the walls and the ceiling like Spiderman, but only in his sleep. If you woke him up, he would fall. My father-in-law swears that he saw this with his own eyes. The orphanage workers told the boys that if they ever saw the boy crawling like this, they shouldn’t make any noise to wake him, for fear that he would fall and hurt himself.

    • Jaskologist says:

      When we were very young, my brother was convinced that aliens were contacting him and a few other children around his same age. They would appear to him at night, and communicate with him telepathically. Once, they showed him around their ship. While he was there, he happened to see another student from his school there. He later asked him about it in real life, and the kid seemed to know what he was talking about.

      I should really ask him what came of all that.

  69. Aneesh Mulye says:

    I just matched your donation, Scott.

  70. Brandon Berg says:

    Hey, Scott, could you send me an email at the address I used in this comment? I was going to mention what I think was an error in your Untitled post, but thought it might run afoul of the no-gender rule.

  71. So Scott, I posted a moral philosophy theory that I feel you might like. It’s probably quite closely related (though perhaps a little more formal) to your relentless march of niceness theories 🙂 I’ve put a lot of work into this one and I hope it has broad appeal across multiple philosophical camps.

    The Ideal of Comprehensive Morality

    As always, feedback welcome on my blog or here.

    • 27chaos says:

      I liked this. Even as you were discussing your moral hero, I found myself wondering what a defense of a more myopic morality might look like, and then you immediately proceeded to illustrate one.

    • This goes into detail about the sort of mistakes abused children are likely to make about morality and safety, simply because children don’t have great tools for thinking about what’s happening to them.

  72. Anonymous says:

    Let’s assume for the sake of argument that an unwanted/unplanned poor child has a significant cost to society in terms of welfare, crime and possibly a whole lot of things I haven’t thought about. Let’s fix that cost to an arbitrary number, say 1/5th the cost of bringing up a child, or for the US roughly 50k USD.

    Now, let’s offer 1/10 of that cost as a direct monetary incentive to have people sterilised. For maximum effect, we would offer this only to women.

    If you are rich enough to actually bring up a child, 5000 USD is a trivial sum and not worth the hassle. If you are too poor to bring up a child, 5000 USD is a substantial sum, and if you want your tubes tied anyway, today you probably can’t afford it.

    If you have taken the money and your financial situation changes from winning the lottery or whatnot, well these things are reversible now, and I suspect that the cost of a reversal operation is higher than the 5k.

    Why isn’t this the obvious solution to a LOT of problems related to poverty?

    • It’s a good solution, but one that farces major hurdles being implemented because it evokes fear of Nazism and compulsory sterilization, even if such comparisons are unfounded. Politicians like to talk about poverty of a social problem, not a biological one. Scientists have to tread this water carefully. This censorship, whether self-imposed or imposed by society, hurts potential progress that can be made on these issues

    • Nita says:

      If you are too poor to bring up a child, 5000 USD is a substantial sum, and if you want your tubes tied anyway

      Most childless poor people probably hope to get out of poverty and have children eventually.

      I would support subsidized temporary contraception like hormone implants or RISUG, with sterilization only for those who actually want it.

      • houseboatonstyx says:

        $X per month (or $3X quarterly, for 3 month implants or such) would be cheaper initially, too.

    • Deiseach says:

      Why women only? Why not men only? Part of the problem here is that prevention of pregnancy is seen as the woman’s responsibility. Given that it’s complicated to mess around hormones, and even laparoscopy for tubal ligation is still major surgery, why not encourage men to get the snip or have polymer gel injections? Quicker, cheaper and they can bang as many chicks as they like without having to use condoms or worry is she really on the pill or IUD!

      • lmm says:

        Non-rigorous thoughts:

        Higher variance in male sexual behaviour. Most unwanted children are conceived by a small tail of highly irresponsible men, whereas they’re spread across a wider spectrum of women. That kind of cuts both ways though, hmm.

        Raising any children will be the woman’s responsibility, so her financial state is more important to the child’s life outcomes than the man’s. And if you encourage men to be sterilized, you’re going to have many more cases of provable cuckolding, which seems like it will lead to bad outcomes all round.

        • Deiseach says:

          Provable cuckolding? Let’s unpack the whole raft of attitudes there, shall we?

          (a) suppose Mr A and Mrs A, or Mr A and Ms B, are in an intimate relationship. Mrs A/Ms B is also – the hussy! – having a bit on the side.

          Why the blue blazes would she get herself knocked up?

          Okay, you say, the little minx wants a baby or wants to humiliate her poor deluded spouse/partner or was careless, and thinks she can pass off baby as Mr A’s child.

          But! Mr A then says “You home-wrecking cheating deceiving wanton, I am sterile and here’s my receipt for the medical procedure to prove it! This pregnancy is none of my doing!”

          (b) If Mr A has not informed Mrs A/Ms B that he is sterile, and therefore she need not worry about contraception (prevention of sexually transmitted disease is another thing), then either their relationship is of so informal a manner that “cuckolding” is hardly a concern (there is no emotional depth or investment in what is a casual sexual arrangement), or matters between them are at such a pitch that her having an affair is a symptom, not a cause, of their disunity and disintegration.

          (c) Also, I very much resent the perhaps unintentional implication that oh noes, we must not interfere with men’s potent virile fluids because otherwise bitches would be out there spreading their legs for every dude who whistles at them and getting themselves up the duff by strange men not their master. But making women as a population, or a very large slice of it, sterile is no problem, apparently (we need not worry about men deceiving their tube-tied women and fathering children by other women, is that so?)

          (d) That last may seem harsh and even rude, but if the first objection that occurs to you is “if men are sterilised, that means women could be cuckolding them” and you don’t consider or weigh the equal risk that sterilised women might be cheated on by men fathering children by other women, I think you need to question why cuckolding was the big problem that leaped out for you.

          • Jiro says:

            (d) That last may seem harsh and even rude, but if the first objection that occurs to you is “if men are sterilised, that means women could be cuckolding them” and you don’t consider or weigh the equal risk that sterilised women might be cheated on by men fathering children by other women, I think you need to question why cuckolding was the big problem that leaped out for you.

            I don’t think he needs to question it; rather, you should have been more charitable and tried to figure out possible reasons for it.

            One of the most obvious ones is that cuckolding is far more costly for the man than for the woman, since it is always possible to know who the mother is, but not the father, enabling a woman cuckolding a man to impose costs on the man that a man cheating on a woman cannot. It just wouldn’t make any sense for a man to cheat on a woman, pretend the child is hers, and get her to raise it and spend money on it. Basic biology prevents that.

          • Deiseach says:

            You know, I said to myself this morning “Why am I always getting into fights on Scott’s blog? Why am I not a nicer person? I need to do more ‘I love little kittens because they are so soft and furry’ comments and be a smiling ray of sunshine!”

            Well, looks like I picked the wrong day to make a resolution to be nice and sweet and non-confrontational.

            WHERE THE HELL DO YOU LOT GET OFF, EXTENDING OWNERSHIP OVER WOMEN’S BODIES LIKE THAT? OH, THEY MIGHT CUCKOLD MEN, WHICH IS WHY THEY SHOULD BE THE ONES UNIVERSALLY STERILISED TO PREVENT UNDESIRABLE PREGNANCIES. BUT LET US NOT TOUCH THE SACRED SPERMATIC DUCTS, FOR NO MAN WOULD EVER HAVE TWO, THREE OR MORE WOMEN ON THE GO THAT HE MIGHT IMPREGNATE FOR THOSE SAME UNDESIRABLE PREGNANCIES!

            You know, I cannot believe I am a socially conservative, traditional Catholic when I seem to be on the left-ward side of attitudes like the “cuckolding” one.

            I repeat: if your first objection to sterilisation of males is that their womenfolks will be rushing out to get themselves up the pole by sneaky sex with the other men sniffing around, why would this be your first objection? Why not on health grounds? Grounds to rights of autonomy over their body? Religious grounds?

            Women have the risk of constant doses of artificial hormones or major surgery to control their fertility. None of this appears to be a problem. But mention the equivalent responsibilty for men to control their fertility and all of a sudden it’s a recipe for cuckoldry!

          • lmm says:

            May have been a poor choice of word (it evidently has stronger connotations than it does for me); apologies. I started, as I usually do when trying to come up with conservative objections to relationship policies, by thinking of the children: how might male or female sterilisation harm their children? From there the thought process is obvious and never felt particularly purity-oriented. The child of a man cheating on his sterilised wife could probably have a normal, positive life (no-one would ever know). The child of a woman cheating on her sterilised husband, not so much. If anything I’d expect widespread male sterilisation to /reduce/ cheating, but that’s not something I weigh at all heavily in comparison to child welfare.

          • Jaskologist says:

            I feel like everybody has missed the provable part of the phrase “provable cuckolding.” That’s an important modifier; the objection isn’t to cuckolding; it’s to the cuckolding being found out.

          • Deiseach says:

            The child of a man cheating on his sterilised wife could probably have a normal, positive life (no-one would ever know).

            (1) The woman in the case above is married or in a permanent relationship herself. Her husband is either (a) not sterilised and accepts the child as his, or (b) like our friend Mr A, sterilised and
            is highly upset about being provably cuckolded.

            Support:

            (1) (a) Nobody says anything, nobody finds out anything, and indeed everything goes along nicely (I know from local gossip about one guy who doesn’t know who his real grandfather is, because his own father didn’t know it – but because of Circumstances, i.e. Ireland is a small country, your neighbours do know your business, and my mother did a lot of visiting amongst elderly relatives and heard all the gossip, I in turn know about it because Great-Grand-dad was a notorious cocksman and fathered a whole rake of illegitimate children). Nothing to arouse suspicion like “Hm, nobody in myfamily ever had red hair, darling!” comes up, there are no messy vicious rows where it gets thrown in B’s face “And by the way, that’s not your kid!” and everything in the garden is rosy. Could happen. Probably does happen. Mr and Mrs/Ms B don’t break up over her fling with Mr C, and Little C never finds out (or not until later in life) that B is not his dad.

            Objections:
            (1) (a) People know anyhow because neighbours, family and others have ways of finding out. Even if Little C and Little C’s foster-dad never know, Mrs/Ms B knows and Mr C may or may not know. I’m also seeing in my work cases where the Little Cs find out later in life about their real dads and change names, try to find their ‘real’ father, etc. It’s not without upheaval of some kind.

            (1) (b) Mr B divorces his wife/dumps his girlfriend for being a cheating hussy and refuses to pay maintenance for a bastard that’s none of his get. Mrs/Ms B gets a bad reputation in at least some quarters. Little C learns all about “hey, your mom cheated on your dad! Only he’s not your dad, is he?” from their little schoolmates, who learned it in turn from overhearing the gossip of their parents about Mrs/Ms B. Upheaval and rancour and all kinds of emotional upset, not to speak of the environmental consequences (chances of poverty, disrupted/broken homes, etc.)

            (2) The woman is not married. Unless she is in a position where she has ample means and doesn’t need any material support from the father, she’ll probably look for child maintenance. When looking for child support payments (because if you’re claiming lone-parent allowance, you have to prove that you’ve made an attempt to gain maintenance from the other parent), she needs to name Mr C as the baby-daddy. Mr C, as we’ve seen, is married. Mrs C, unless it’s a particular set of circumstances, will not be happy to hear about this. Mr C may not be too happy himself. Little C will come in for some of the fallout from all the adults fighting over this.

            (2) (b) Mr C makes a habit of spreading his seed and Little C finds out that they’re just one more notch on the bedpost as far as Dear Old Dad is concerned. This may or may not be a matter of distress to them.

      • Kevin says:

        I fully support encouraging men to take as much responsibility as women for contraception. However, ceasing the use of condoms would still not be advisable, as vasectomies sadly do not prevent STDs.

        • Deiseach says:

          (1) Original suggestion about making women infertile did not address disease prevention

          (2)Trouble is, the pregnancies are happening because guys are not using condoms (in conjunction with the women not using any/effective contraception), so their fears of sexually transmitted disease obviously are not scaring them into behaving in a manner to prevent that as well as pregnancy.

          If we’re going to treat women as a population to be neutered to prevent unwanted pregnancies, we should do the same for men. If you want to sleep around without let or hindrance, fine for you, but then what’s sauce for the goose is sauce for the gander, or, it takes two to make a baby.

          • AR+ says:

            Women are far more of a bottleneck to fertility. Neuter all but a fifth of the men and you might well get about as many babies, so depending on the exact goals of the effort, the sexes are not symmetrical.

            I feel like this should be obvious.

          • Anonymous says:

          • Kevin says:

            I would be interested to see numbers on causes of unwanted pregnancies: refusal to use condoms, using condoms incorrectly, etc. I haven’t been able to find a study that fine-grained.

      • Anonymous says:

        Right, reading this one and the comments below, that one went off on a tangent I hadn’t anticipated.

        The “women only” bit was intended as a purely return on investment consideration and not in any way a slight against women. For much the same reason that fertility in populations is measured in “children per woman” and not in “children per person” or “children per couple”, the actual impact of a sterile woman is simply higher than that of a sterile man. This is why I started the sentence with “For maximum effect”, sorry for the misunderstanding.

        If implemented for both sexes, there would probably need to be a different price structure, because the reversal is also simpler on men, so that you could go get the 5k, then get the reversal for 2k, rinse and repeat.

    • Anonymous says:

      Several reversible birth control methods (such as IUD) are just as effective as tubal ligation (as is RISUG, which really needs to become available to men who want it). I can see a scheme like this being implemented with these types of more-easily-reversible methods, which are less likely to conjure the specter of eugenics.

    • Corwin says:

      Totally is. Who’s organizing the fundraising?

      Just one objection : men AND women both.

      One more detail : free reversibility on demand, no question asked, over the counter if at all possible. Just that it has to take at least a fleeting instant of actually wanting to make a baby, with consent from both partners. But not necessarily more. There MUST be no test, no license, no question – so that there is no denial of reproductive rights, just that it would make them opt-in.

      • Anonymous says:

        Actually I thought of this mainly as a const-savings measure from the state, not as a subject for fundraising.

      • Anonymous says:

        Sounds a bit like Brave New World, with a whole lot of potential to become 1984-ish, depending on government-evilness.

        Also, if opt-in is free of charge, you’re facing underpaid personell in sub-par facilities, should you be able to make it to the facility 200km over (if you can pay for the train/bus/car ride). Or you probably won’t, for a long time, since the waiting period would likely be ~3 years+.

        Or so I’d imagine, at least…

      • Gbdub says:

        if you allow for free reversibility, what’s to stop someone from taking their incentive payment to get sterilized and then immediately demanding reversal?

        I guess I just weigh the “right to reproduce” much less than you, in that I’m not sure the right to produce a baby trumps the collective right of society to not have an unproductive new member thrust upon it.

    • Jon Gunnarsson says:

      I agree that unwanted children born to poor parents are probably on net a bad thing for societal welfare. But it’s not at all clear to me that the same can be said for planned children born to poor parents. Especially considering that most First World countries today have worryingly low birth rates.

      When considering which people would be affected by this sterilisation premium, you quietly drop any talk about children being unwanted. This proposal would significantly reduce birth rates among low income people and would cut down on unwanted as well as on wanted children.

  73. Zorgon says:

    Donated to Multi, because fuck Russia’s attitude to transfolks.

    That aside, I’ve been waiting for an open thread to ask: Does anyone have suggestions for a good place to start someone with Christian Rationalism?

    I have a friend who is a member of a rather dubious strand of Christianity and she’s been showing clear signs of wanting to shift to a less… completely crazy… approach to thinking. She’s already a big fan of HPMOR and reasonably well read, but is also a fervent believer, so deconverting her is pretty much out of the question (and I suspect would have severe and long-lasting deleterious effects on her mental health).

    So my question is, where to start? I don’t personally know very much about the theistic sides of rationalism, being the never-believed kind of atheist, so any advice is welcome.

    • BD Sixsmith says:

      Presuming that you mean Less Wrongian rationalism, Leah Libresco.

      • Can someone please summarize why Leah Libresco is Catholic? I am super confused as to how someone could both be a rationalist and actually believe that the claims of the Church are true.

        • Anonymous says:

          To the best of my understanding: she noticed that she thought of moral law as a person who loved her, and that Catholicism seemed like the best way to reach back to that person.

        • I’ve wished she would give a lengthy explanation of this for a while–there are a few blog posts about it, but none of them are of Scottist length and address the millions of objections that come to mind when reading them.

          • RCF says:

            I do find it a bit unseemly to be objecting to Leah not presenting an account of her religious journey satisfactory to random strangers on the internet. My main objection to Christianity is Christians’ common idea that their personal beliefs should affect my actions. If Leah rejects this idea, then I don’t see a pressing need to discuss the rest of her belief system.

    • Multiheaded says:

      Thank you! Thank everyone!

      And yes, it’s pretty fucking bad. Don’t even want to talk about it. Suffice to say, the vast majority of people here aren’t aware of a *single* queer person they might personally know. And there are no high profile, publicly visible trans people *at all* – and most LGBT outreach efforts, such as they are, don’t even mention our existence.

      (I don’t want to go into the practical dangers and impossibilities of living while trans.)

      • haishan says:

        And yes, it’s pretty fucking bad. Don’t even want to talk about it….I don’t want to go into the practical dangers and impossibilities of living while trans.

        I do hope that you will talk about it at some point, with as high-powered a signal as possible, although I totally understand that it might not be prudent or psychologically healthy to do so now. I may not agree with your politics, and I may be kinda-sorta conflicted about your right to express them, but I will fight to the death for your right to tell your story. (Also your right to, like, not die, although that’s not a particularly universalizable impulse.)

        Gave $20.

      • llamathatducks says:

        <3 </3

        I am Russian-American and LGBT (not trans, but still) and while I don't have to deal with the actual dangers of being an LGBT person in Russia, I get heartbroken pretty much every time I hear anything about LGBT stuff there. I'm so glad you're getting out, and so sad that the only way you can live well is to leave.

        And thank you for reaching out to this community and letting us help you achieve your goal. I fervently wish you the best of luck.

    • Irenist says:

      I think it depends in part on which strand of Christianity, and what you find rather dubious about it. Depending on what kind of denomination your friend is coming from, some kinds of discourse will produce less culture shock than others. Likewise, if a more rationalist-inflected Christianity prompts your friend to switch denominations (rather than just reweave her belief web around the edges), the culture shock issue will be a much bigger deal.

    • Deiseach says:

      Depending on what exactly you mean by “rather dubious”, quite frankly I’d be encouraging her gently to explore less batshit crazy (is that the meaning I should take?) traditions within Christianity to help her on the way. She can sort out rationalism later on.

    • Troy says:

      A large swath of analytic philosophy of religion is Christians attempting to give reasoned defenses/analyses of their faith and/or use reason to determine whether/what kind of Christianity to believe in. Some of them (e.g., Alvin Plantinga) endorse epistemological views at odds with the rationalist community, but they are also full-blooded Bayesian evidentialists, like Richard Swinburne. There’s plenty of good discussion among said philosophers about issues like heaven and hell, Christian exclusivism, Scriptural inerrantism, creation and evolution, and so on.

      I recommend, as always, the Stanford Encyclopedia of Philosophy as an introduction to these topics.

  74. 3. Some people seem to have gotten genuinely upset about some of the recent discussion of IQ, on grounds something like that if high IQ is a necessary ingredient of some forms of academic success and they’re lower-IQ than other people, then they are bad and worthless. I strongly disagree with this and think it gets the reasoning exactly wrong, and I hope to explain why. But work has been pretty crazy lately (no pun intended) and I might not get the chance to write it up for a little while. Until then, please do me a favor and just take it on faith that you are a valuable human being who is worthy of existence.

    Which comments or thread did people get offended? curious because this is a favorite subject of mine and am always looking for new content for my own site/

    I surmise IQ has become a touchy subject (or at least more so than in the past) because of the greater role biological determinism plays in our increasingly competitive economy. Today’s hyper-meritocracy amplifies the socioeconomic ramifications of individual cognitive differences such that a person with an IQ >110 is much more likely to succeed than someone with an IQ <90 , whereas decades ago the differences in outcomes wasn't so obvious. IQ has become our new caste system, and this is understandably a disconcerting concept because it goes against our ideal of free will.

    • Creutzer says:

      Which comments or thread did people get offended?

      Upset, not offended. I remember seeing a few posts in that vein, but I can’t tell you where. Hell, if I had the value system that I perceive as the median in this community, I imagine I’d be facing some issues of that kind myself.

  75. Muga Sofer says:

    So, I recently wrote a thing about libertariaism:

    https://pseudonymwrites.wordpress.com/2015/01/23/on-libertare/

    It … started as my attempt to steelman libertarianism into something I could actually believe, but kind of ended up … going the other way. I’m worried I ended up producing a strawman.

    We have a lot of libertarians here, right? Can anyone tell me if I’m being horrible and unfair?

    • The central thesis of libertarianism, as an ideology – and I can say this, for once, without fear of contradiction; for it really is extraordinarily simple – is this:

      Imagine a world in which there is no such thing as force. There is no violence, no coercion.

      I don’t think this is true. Libertarians do believe in force in defending private property and family. Libertarians want to be the ones to carry our this force instead of being dependent on a state-run entity such as the police.

      • Wrong Species says:

        I don’t think you have it right either. It’s not that libertarians want to carry out this force but that they are opposed to the initiation of force. I think pretty much every libertarian believes in the non-aggression principle. The non-aggression principle is the idea that you shouldn’t initiate aggression. What is aggression? Basically it’s violating your property, which includes yourself and and all your belongings.

        Someone who is an anarcho-capitalist believes that the government, pretty much by definition, initiates force. If it didn’t, it wouldn’t be a government. So they oppose the government and want to replace it with voluntary, private defense forces who will protect you instead of a violence enforced monopoly.

      • Ano says:

        I don’t think that’s really accurate; I think libertarians generally believe that the government should enforce property rights and laws against violence or fraud, but that basically everything else that the government does is done better by private organizations or individuals. From the Libertarian Party Platform:

        “The only proper role of government in the economic realm is to protect property rights, adjudicate disputes, and provide a legal framework in which voluntary trade is protected.”

        • Jon Gunnarsson says:

          Most libertarians are in favour of a limited government. However, there is also a subset of libertarians (typically called anarcho-capitalists) who want to get rid of government altogether and have even functions such as law and defence provided by private institutions.

      • Mr. Eldritch says:

        >Libertarians want to be the ones to carry our this force instead of being dependent on a state-run entity such as the police.

        This doesn’t ring true to me. A central component of libertarian ideology – or at least the branch of libertarianism I’m familiar with – is that the state should have “a monopoly on the use of violence.” In a very real sense, to the libertarians I’m familiar with, having someone to carry out the use of force and prevent other people from doing so is the *entire point* of having a government in the first place instead of going full anarcho-capitalist or whatever. The state ensures law enforcement, protection of rights, maintains a military force, and all else (public works, accounting for externalities, providing a social safety net) is largely dependent on your personal opinions about government.

        • Nita says:

          A central component of libertarian ideology – or at least the branch of libertarianism I’m familiar with – is that the state should have “a monopoly on the use of violence.”

          Actually, that is the mainstream view, first described in those terms by Max Weber.

    • jaimeastorga2000 says:

      Like a lot of NRx I’m a post-libertarian, and I can tell you that your treatment here is really poor. It’s really short; you deal with a tiny part of libertarianism, on a really shallow basis. Compare with Scott Alexander’s Anti-Libertarian FAQ, which is page after page of arguments and examples and facts; that’s the level of depth you need to make a serious argument against libertarian ideas. Imagine that someone wrote a post, “On Liberalism” or whatever your political philosophy is, and dismissed it after as brief a treatment as you have given here. Would you feel like your beliefs had gotten a fair shake? I mean, you talk about incentives, but you don’t even anticipate the obvious libertarian counter: in a world in which the government forces the rich woman to give food to the poor man, what incentive does the rich woman have to create food? She simply goes and does something else, and the poor man starves.

      • Anonymous says:

        >I’m a post-libertarian

        If you don’t mind me asking, what views that you hold would you qualify as post libertarian, and what would you say makes them so?

        • jaimeastorga2000 says:

          Most of them are on the meta-level. I still think a lot of libertarian policies are great on the object-level, mostly the economic ones, but realize that democracy is inherently hostile to a libertarian economy, because Moldbug. Compare to mainstream libertarianism, which thinks getting a few more people to vote for a tiny third party is a worthwhile use of their time and effort and money.

          An object-level example of a libertarian belief I no longer hold is free speech. I agree with all the libertarian arguments for why free speech would be great, but the bottom line is that free speech is impossible, because someone is always going to control the discourse, legally or otherwise (see, for example, all the people who have gotten doxxed and fired recently for being insufficiently progressive). That being the case, if someone is going to control speech anyway, it might as well be my side.

          • Wirehead Wannabe says:

            Why do you believe control of speech to be inevitable? Surely we can make it illegal to fire people based on speech made outside work. I understand that not every space can be a free speech zone all the time, but that’s not the same as not having free speech at all.

          • Or you have one of two solutions:

            a) Everyone grows the hell up, the culture changes, and a person’s opinions outside the workplace no longer have a bearing on what s/he does at work, as long as s/he doesn’t bring said opinions *to* work, or
            b) A small number of people realise that the vast majority are too emotionally immuture to handle free speech, and a smart entrepreneur implements a technological solution which allows you to speak freely with likeminded people *without* running the risk of public censure, simply by not having such discussions public, and additionally implements *social* incentives to ensure that any betrayal of such privacy is punished swiftly and efficiently. This then slowly but surely spreads, and everyone starts using it.

          • TheAncientGeek says:

            Can’t you go for a compromise,w here you approximate the ideal of free speech closely as you can, while knowing that it cannot completely be done, and without just cynically giving up?

            That is after all how we deal with many things…eg crime.

          • Ano says:

            There are greater and lesser extents of freedom, and even if the concept of perfect freedom isn’t one that seems possible or even entirely rational, it might still be a ideal worth striving towards, like many moral principles. No doubt, speech is freer in the United States than it is in North Korea, so our attempts to create freedom do bear some fruit.

          • jaimeastorga2000 says:

            Surely we can make it illegal to fire people based on speech made outside work.

            Which works fine, until the lawmakers or judges decide to add an exception for “hate speech” or whatever other bullshit rationalization they can come up with to silence their political opponents.

            Everyone grows the hell up, the culture changes, and a person’s opinions outside the workplace no longer have a bearing on what s/he does at work, as long as s/he doesn’t bring said opinions *to* work

            Sounds about as realistic as all those people who believe humanity will somehow “outgrow” war. It’s not that SJWs are taking a principled stand in favor of employers being able to monitor your off hours, it’s that they have found a convenient way to crush their political enemies. There is nothing immature about it; they know perfectly well what they are doing.

          • Ahilan Nagendram says:

            It’s simply asinine to take away the right of businesses to fire those who through their own stupidity have made themselves a liability. The way you people talk makes me realize that probably none of you have ever had to deal with such crap as a business owner, entrepreneur, PR dude, or whatever else. It just feels like you people want to say whatever stupid bullshit you want without any consequences or criticism, which is not viable, nor is it ideal. Holocaust deniers, climate deniers, race and intelligence deniers, deserve all the criticism they receive, and if they create enough hoopla over their idiocy, their place of work has no obligation to keep them at the job if the controversy can damage them.

            And before someone strawmans me, I do not want “hate speech laws” or any crap like that.

    • sards says:

      You made a number of false claims about libertarians in your first paragraph. Libertarians don’t side with the rich, the upper-class, or the system; they don’t “battle hardest” for kings; and they don’t generally vote Republican.

      As you note, voluntary trades are guaranteed to be mutually beneficial, but are not guaranteed to be optimal (in the utilitarian/economic efficiency sense). Libertarians disagree that big government intervention in the market is an effective way of forcing market activity in a more optimal direction; based on empirical observation, we think this almost always does more harm than good. There are also theoretical reasons to expect that big government won’t work. So if you want to argue the case for big government vs. libertarianism, you need to do more than point out that free markets don’t lead to Utopia and that some trades are suboptimal. You need to show that your proposed solution is actually beneficial.

      But even if you succeed in making that case, your work won’t be done. Most libertarians, like most liberals and conservatives, are not consequentialists; many of us believe in rights (freedom from coercion, property, etc.) and reject arguments for the legitimacy of political authority. Any serious critique of libertarianism needs to address those points.

    • Tarrou says:

      I don’t know about horrible or unfair. Vastly wrong? Describing some sort of mutant Pacifist Anarchy Randian thing that doesn’t exist and thinking it is libertarian? I can’t even call it a strawman, it’s a rock standing in for a car.

      • Muga Sofer says:

        Really?

        I based that on conversations I’ve had with libertarians, but it’s also the opener for Scott’s FAQ on the topic:

        http://raikoth.net/libertarian.html#introduction

        The Argument:

        In a free market, all trade has to be voluntary, so you will never agree to a trade unless it benefits you.

        Further, you won’t make a trade unless you think it’s the best possible trade you can make. If you knew you could make a better one, you’d hold out for that. So trades in a free market are not only better than nothing, they’re also the best possible transaction you could make at that time.

        Labor is no different from any other commercial transaction in this respect. You won’t agree to a job unless it benefits you more than anything else you can do with your time, and your employer won’t hire you unless it benefits her more than anything else she can do with her money. So a voluntarily agreed labor contract must benefit both parties, and must do so more than any other alternative.

        If every trade in a free market benefits both parties, then any time the government tries to restrict trade in some way, it must hurt both parties. Or, to put it another way, you can help someone by giving them more options, but you can’t help them by taking away options. And in a free market, where everyone starts with all options, all the government can do is take options away.

        The Counterargument:

        This treats the world as a series of producer-consumer dyads instead of as a system in which every transaction affects everyone else. Also, it treats consumers as coherent entities who have specific variables like “utility” and “demand” and know exactly what they are, which doesn’t always work.

        In the remainder of this section, I’ll be going over several ways the free market can fail and several ways a regulated market can overcome those failures. I’ll focus on four main things: externalities, coordination problems, irrational choice, and lack of information. I did warn you it would be mind-numbingly boring.

        • Wrong Species says:

          Libertarians oppose the initiation of force, not all violence.

        • Tarrou says:

          Where do you get the idea that libertarians concieve of no force or coercion? And just because that is the attributes of a theoretical ideal state doesn’t mean that’s what libertarians think happens in the real world. There are no free markets, and never have been. We just want them freer than they are currently.

          What delineates libertarians from anarchists is precisely that they acknowledge violence, force and coercion, and condone collective action (government) as a means of combating these problems. That is the whole wheelhouse of libertarianism, that government should be small and focused on eliminating instances of violence, fraud and coercion, not on handing out candy.

          Finally, libertarianism is most definitely NOT focused on shilling for the rich. We support them when their interests align with our principles, and we trash them when they don’t. Libertarians almost all opposed things like the bank bailouts. Our biggest fear is not big business, nor government, but the collusion of the two.

          I think what happens is that libertarianism is pretty small, and only gets heard when it’s opposing something popular (even if it’s stupid). Do we get credit from liberals for opposing stupid foregin wars, supporting gay marriage and opposing the drug war? Nope, all we hear about is how evil we are for opposing whatever idiotic flavor of the month political scheme to force nuns to pay for triple-gay double pregnant trans-racial gang member’s tattoo removal or some such BS.

          • Luke Somers says:

            > Do we get credit from liberals for opposing stupid foregin wars, supporting gay marriage and opposing the drug war?

            Yes, I’ve heard liberals give libertarians a lot of credit for those things.

        • Jon Gunnarsson says:

          Wait, you based your impression of libertarianism on the position ascribed to libertarianism by someone writing a “Non-Libertarian FAQ” specificially for the purposes of refuting that position? Scott is one of the fairest and most reasonable critics of libertarianism, but even so you can surely see that this method would be liable to give you a distorted view of libertarianism.

          • Muga Sofer says:

            >Wait, you based your impression of libertarianism on the position ascribed to libertarianism by someone writing a “Non-Libertarian FAQ” specificially for the purposes of refuting that position?

            No. I based that section on conversations I’ve had with libertarians; and the essay as a whole was inspired by the opening quote.

            I just had Scott’s FAQ open in another tab, and figured it would do as a source. (Although, for the record, Scott identifies as “left-libertarian” himself; the FAQ is only aimed at refuting “a certain more aggressive, very American strain of libertarianism”, according to the introduction.)

    • John Schilling says:

      Well, first, you’re addressing laissez-faire economics, which is at best a subset of libertarianism.

      And second, you’re using what I call “Disney Economics”, the economic model where all wealth is manifest as the Giant Money Bins of Scrooge McDuck. A static supply of Valuable Stuff, in your version food, which the rich possess for no good reason and hoard out of sheer greed, such that any economic benefit to the non-rich can come only by extracting the Valuable Stuff from Scrooge’s Money Bins. In this model, yes, various forms of socialist redistribution are more efficient than groveling for whatever pittance the Rich will choose to share.

      In the real world, the rich woman has food to spare only because the poor person works for her. There is a certain maximum wage of foodsquares per hour of labor beyond which she cannot go, no matter how generous she may be, no matter how powerful the mob or law demanding redistribution may be, else the granaries will be emptied and there will be no more food. And, of course, a minimum foodsquare wage below which the worker will starve. The actual wage should be somewhere in the middle, and likely will be so long as neither party is privileged to issue take-it-or-leave-it ultimatums.

      If someone is foolish enough to try…

      In the real world “the rich” is plural, not singular, and they compete. Specifically, they compete for the loyalty of the poor, as both laborers and customers – the richest people in the world today, mostly got that way by selling cheap but broadly popular consumer goods to the poor and middle class. So one rich woman offers one foodsquare per fifty hours of labor. Nobody works for her. Her rich cousin figured out that offering one foodsquare per forty hours means all the starving poor people work for him instead. If the first rich woman doesn’t wise up, she’ll wind up having to sell her boarded-up food mine and empty granary to her now-much-richer cousin.

      No matter; there’s plenty of other rich people around, along with plenty of poor. By the time they are finished working this out, the foodsquares-per-hour going rate will be within about ten percent of the empty-granary limit from the last paragraph and the excess wealth will be going into producing other neat stuff. Er, unless the state comes in and says that rich woman #1 is “too big to fail”, and that the prospect of her particular granary going empty is so unthinkable that the power of the state must be used to support her current business model including its price/wage structure.

      Finally, in the real world it is never as simple as just a Rich Person buying labor from a Poor Person, which he uses to Make Stuff for consumption by rich and/or poor people. There are no “food mines” that turn undifferentiated labor into foodsquares. Every real economic benefit of the modern world, even something so simple as a foodsquare or a pencil, is the result of an ineffably complex web of transactions involving millions of talented people in dozens of cities and provinces. No person can understand the entire process by which a pencil comes to be, nor any bureaucratic agency nor any massive supercomputer anyone has yet invented. But any person can understand his own part in it, and his relationship with his immediate partners. Let each of them make his or her own best deal, and in the end we get pencils for a quarter, foodsquares for three bucks. Intrude and say, “but you’re clearly overcharging for raw rubber at the eraser factory, wrongfully impoverishing the rubber farmers”, and you get some happy rubber farmers in the short term and a severe pencil shortage in the long term.

      There are problems where some degree of simplification is necessary to achieve understanding. But reducing economics to Scrooge McDuck vs. the Poor Starving Masses, promotes misunderstanding

    • Princess_Stargirl says:

      Have you ever read the USA libertarian Party Platform. Libertarians disagree on many issues. But the stated views of the Libertarian party seem like a decent model of “Libertarian Beleifs.”

      http://www.lp.org/platform

      *I would prefer survey data of what Libertarians believebut I don’t have any.

      **I am not a libertarian. My main disagreement is I think very large amounts of re-distribution would be ideal. Libertarians tend to want very, very little. Though I share much of there skepticism for government. I especially find their analysis of regulations and regulatory capture insightful.

    • blacktrance says:

      I agree with most of the criticisms already made, but to add one more – it is not a thesis of libertarianism that markets create utopias. Markets create a playing field in which the creation of value is less inhibited, which results in improvements, but utopia is far from guaranteed. Libertarians also tend to acknowledge the importance of productivity and technology – if you traveled back a few millenia and established a free-market order, people would still be extremely poor. It’s also strange to claim that libertarianism leads to tragedy of the commons when libertarians are such ardent advocates of privatization.

      Also, it is a mistake to treat markets as something more than people engaging in voluntary exchange. Markets don’t have values of their own.

    • Pasha says:

      You are being horrible and unfair.

      Libertarian here. Not going to speak for all of them, but a few of the many issues:

      – Simple idea: state coercion should not be the first option that everybody jumps towards in solving social problems. US Government is 40%GDP and everybody still thinks US is capitalist and we need more coercion.

      – Providing hypotheticals of two people and claiming that God can do better than some sort of market is not an argument for government, unless God is running government, which it is not. Real-life markets have both a larger level of options and charitable giving.

      – If you approaching something on purely consequentialist grounds, you need to address the prior of why governments have killed 300 million people last century. Proportionally speaking it was even worse before 20th century. The idea of simply dropping “government” or “democracy” and pretending it’s somehow watching out for the good of people run counter to history. This isn’t just about “corruption”, which doesn’t even come close to being the worst thing in government.

      – At the fundamental level, there could be reasons why agent A could use some sort of physicality against agent B, such as parent dragging a child away from a car. However, it’s harder to argue that a government official knows better than a person of what to do with money, time, including where to donate it.

      – Semi-Slavery based employment is perfectly sanctioned by the government today. It’s usually called immigration contingent on employment, aka, you will get kicked out of the country if you don’t work. If your worry is “exploitation” may i suggest using your time to lobby for more green cards?

    • RCF says:

      The central thesis of libertarianism, as an ideology – and I can say this, for once, without fear of contradiction; for it really is extraordinarily simple – is this:

      Imagine a world in which there is no such thing as force. There is no violence, no coercion.

      But neither is this world perfect; there is inequality, and want, just as in our own. Different people have different resources, different skills.

      So, you’re saying that the central thesis of libertarianism is that even without any force, the world would not be perfect? On the one hand, this such a vacuous statement, that I can’t see how you thought this could be the central thesis. On the other hand, if you intended to assert something else, then you failed miserably. It’s really incredibly bad essay construction to say “Here’s the central thesis” and then wander off into some thought experiment that you think somehow illustrates the thesis. If you think that some thought experiment is necessary, then okay, present it. But clearly delineate what is your thought experiment, and what is the thesis. I can’ critique your steelman attempt, because I have no idea what it is.

      Also, watch “it’s” versus “its”.

  76. Multiheaded says:

    AAAAAAAAAAAAAAAA! This is just stunning! Such INCREDIBLE generosity! You people are all so AMAZING!!!!!!!!!!!!

    I can’t think straight, I’m sorry! This is all quite literally beyond my wildest hopes! THANK YOU, EVERYONE! And thank YOU again, Scott! To help me reach out so effectively, and after having had to put up with my crap for so long… you’re kindness itself!

    P.S.: I am tentatively earmarking 50% of all donations between the set campaign goal of $2500 up to $4500 to be given to GiveWell-endorsed charities and MIRI, shortly upon arrival. Receipts will be posted publicly. In case the donations exceed even *that*, I intend to give away at an even higher rate – like 75%. Will update.

    P.P.S.: any and all donors are very welcome to contact me individually, in case you want to talk about something in private!

    • Noah Siegel says:

      ” I am tentatively earmarking 50% of all donations between the set campaign goal of $2500 up to $4500 to be given to . . .”

      I suggest holding off on the donations until after you get a job. $4500 (-$900 for travel) is not going to last you very long at all in Toronto.

    • AR+ says:

      Whoa, hold on now. These donations are already ear-marked, for you, for the purpose of moving to Canada. If someone wanted money to go to some effective charity, they would have just donated to that. If you start giving away campaign money before establishing yourself well enough to have an income of your own, I’d feel like some kind of EA-fraud victim.

      If the money you get exceeds the goal, you’ll just have that much more of a buffer, which you will need as much of as you can get. Certainly it’s going to be awkward if you end up needing more money and you gave away a significant fraction of what was already given to you.

      Please don’t treat campaign funds like personal income.

      • Multiheaded says:

        Point well taken. It’s just that, y’know, I am really kind f stunned, having expected very little. I feel like I really really desperately need to provide some value in return. But yeah, not going to give anything until my living situation in secure – although I do really hope that I’ll be able to access housing and job training even as a refugee claimant.

        • Pseudonymous Platypus says:

          I also donated, and I agree with AR+. You will have plenty of opportunity to give back to society once your own situation is better.

          Also, I wrote “As a Canada” (rather than “Canadian”) in my donation message. Oops. 🙂

        • stille says:

          Bloody hell, Multiheaded, keep the money. The value you’re providing in return is the moral satisfaction of knowing we’ve improved someone’s life – and that’s high-octane value, really. I just hope you’ll be in Canada as soon as possible, really – I freely admit that I’m way more paranoid than the average human, but if anything happens passport-wise or money-access-wise between Russia and Canada or Russia and wherever gofundme is based in, it’s going to suck for everyone involved in the thing and you most of all.

          • Multiheaded says:

            Oh, gofundme already doesn’t support withdrawing the money outside of North America and the EU

            My significant other, who is American, is helping me move it by withdrawing it and then wiring it to my own bank. We trust each other 200%, and it should all be ok.

            (i feel the need to clarify this before people bring this hurdle up and/or worry that I haven’t planned around it)

          • AR+ says:

            Wish you’d put that out before, then I’d have found some way to give directly.

            “I trust them,” said everyone who was betrayed ever.

            Though, similarly: “I probably don’t need to examine the details of how a reputable service like GoFundMe actually delivers its funds.” ~Me, yesterday.

          • stargirl says:

            I wonder if Scott would accept being the person to wire the Money. Alot of people in the community know who Scott is. And I cannot imagine stealing the money could possibly be worth it for him. Alternatively some other reputable person could be an intermediary.

          • Pseudonymous Platypus says:

            I am in agreement with AR+ and stargirl.

          • maxikov says:

            If you haven’t already thought about it, may I suggest considering marrying your SO as a fallback strategy in case the main plan fails? I’m sure you know a lot of factors that change odds, but the priors don’t seem to be that good. I know quite a few LGBT people who live or lived in Russia, and quite a few people who moved from Russia to the US, but only one guy who did so by claiming refugee because of his being gay (in fact, if chatting about his experience could be any use for you – let me know, I can introduce you). And I’ve heard accounts of women from central African countries being denied refugee status in the US, even though they were facing FGM upon returning. Obviously, the rules may differ between the US and Canada, and you may know something they didn’t know, but I just hope you included all the backups possible.

            Also, does your budget include expenses for hiring an immigration lawyer? If it doesn’t, that could also be an idea worth considering.

          • Pseudonymous Platypus says:

            @maxikov: I’m not a lawyer, and I don’t know much about Canadian immigration law, but this seems pretty promising:

            The Supreme Court of Canada has ruled that sexual orientation is a ‘social group’ within the context of determining convention refugee status (Canada v. Ward, [1993] 2 S.C.R. 689). This means that someone with a well-founded fear of persecution based on their sexual orientation can seek asylum in Canada. […] Canada generally accepts claims based on sexual orientation at a similar rate to other refugee claims. […] The supreme court of Canada has not yet clarified whether gender identity persecution is persecution based on social group, political opinion, or religion. However, Canada’s lower courts have taken on cases that deal with gender identity in refugee law. Appeals from the IRB are heard at Canada’s Federal Court, whose decisions set rules for future IRB cases. In Hernandez v. Canada ([2007] F.C.J. No. 1665), the Federal Court held that the IRB had erred in failing to consider that the claimant was transgender, or that she may face discrimination on the basis of her gender identity if forced to return to her country of origin.

            From http://egale.ca/all/faq-immigration/. So it might not be perfectly straightforward, but immigration never is, and I can pretty much guarantee it’s better than what you’d get in the US. (As a Canadian expat living in the States this is about what I expected. Also, I expect Multiheaded has done her research and this is why she chose Canada rather than the US.) Up until very, very recently, marriage to a same-sex spouse could not be used to gain immigrant status in the US, because the federal government did not recognize same-sex marriages.

            Anyway, your suggestion is still good, but I’d say only as a very last resort… while it’s a helluva lot better than facing serious persecution in Russia, marrying someone for immigration reasons is a recipe for emotional ruin. Also, if immigration officials find out that’s why you are getting married (which might be pretty obvious if you’ve already filed for refugee status), they probably would not look kindly on it.

  77. Vilgot Huhn says:

    I’m often pretty worried I have a too low IQ for what I want to do with my life. I’ve never tested it seriously but I’m pretty convinced it’d be about average. But recently I’ve been reading Keith Stanovich (book called “rationality and the reflective mind”) and his model has me pretty convinced that IQ isn’t all that matters, or even all that important, when it comes to being clever. 🙂

    I also find the work of Carol Dweck (and her friends) pretty calming.

    • zz says:

      You may also be interested in The Brain that Changes Itself which, while at the pop-sci level, is good enough to be a textbook for a psych course at Harvard (I can’t point to it right now; I tend to use MIT, Harvard, and Stanford (in that order) to generate textbook recommendations, and just remember The Brain that Changes Itself was one that came out Harvard).

    • Anthony says:

      First a guess – if you’re reading a book and drawing independent conclusions, there’s a good chance you’re above average, at least somewhat.

      Secondly – while intelligence does help you achieve a lot of what you want to (or what your employers want you to), it’s not the only thing. Conscientiousness helps a lot, and seems to be more malleable than intelligence. Certain specific skills or talents can also help, if you know what you have and how they’d be useful.

      Lastly, lots of people manage to do lots of things without being in the top 1% of intelligence (and contrarily, lots of people in the top 1% lack the requisite skills to really do lots with their intelligence). There’s a rant I need to put together about excessive focus on the top 1% (by any number of measures) and how that hurts everyone else. But don’t sell yourself short even if you’re not a Certified Genius.

      • Creutzer says:

        Wait, did I miss some evidence for conscientiousness being malleable to a significant degree? From what I’d seen, I’d concluded that it is, unfortunately, not very malleable either.

      • Vilgot Huhn says:

        Thank you.
        I think my worries about intelligence are probably guided by som irrational fears, but fear can be very convincing, even if it is devoid of real arguments. Also, I have to admit that I’m pretty arrogant for worrying about being average rather than being bellow average.

        Regarding the top 1% thing: I absolutely agree. Scott previously linked this story about a Korean man who had record high IQ but still chose to live an ordinary life. I found it very calming. Not just because it demonstrates how high IQ doesn’t guarantee “success”, but mostly because it somehow decreased the preassure to be successful for me. (weird?) http://www.koreaherald.com/view.php?ud=20101006000616

        (Sorry if my words are a bit wonky; english is not my native language).

  78. Jack V (cartesiandaemon) says:

    I was interested where you mentioned on tumblr stories like “You want to hear something surprising? A couple of years ago I had a patient who said the voices in her head told her to come to the hospital and ask for antipsychotics.”

    That’s superficially surprising because psychotic problems don’t usually work like that. But it fits perfectly the theory of, some proportion of people hear voices, and people who hear voices saying good or neutral things don’t get much attention, but people who have voices AND they tell them perverse or harmful things, show up as problems and get treated.

    If so, you’d expect some people to fall into the intersection of “voices”, “saying things worrying enough to think about doing something about them” and “also saying relevant things that happen to include, what to do about voices”.

    Do you know, is that mainstream now, or just speculation? Do you think it’s probably right?

    It also occurs to me, you’ve probably talked about this before, but if it’s right, some people probably LITERALLY have voices which tell them to live up to a high ethical standard, and, principle of charity, when people said they heard a voice from God, if everyone was too quick to dismiss it as metaphorical?

    • Anonymous says:

      My ex hears Jesus telling her to do good things all the time. If she eats lithium, Jesus goes away.

    • Peter says:

      Occasionally a news article comes up about the “Voice Hearers Movement”… on the one hand, journalists have a knack of messing things up especially with people they see as odd, on the other hand it’s nice to have people who can bypass the stories that the leaders of said movements want told and who talk directly to “rank and file” members. There’s lots of interesting stuff there.

      It turns out that for a long time the advice was “Ignore the voices! Don’t listen to them! Don’t interpret them! Really really don’t respond to them!” and this may or may not be good for some section of the population. However, some people seem to have reached interesting accomodations. There was one person who’d hear voices, say out loud, “Go away! I’m watching the TV – come back in half an hour” and in half an hour the voices would indeed come back and he’d have a chat, and that seemed to settle the issue for the time being.

      Philosophical/psychiatric question: is it possible to have a delusion that you are hallucinating?

      • Loki says:

        Personal experience says: sort of. I will not have a delusion that I am actually, actively hallucinating right now, but I will have a delusion that I am currently experiencing intermittent hallucinations, making me uncertain whether anything I see is real or a hallucination.

        There is also ‘delusion that everything I can’t prove to myself I am actually sensing (for instance, I can hear music but cannot currently see the stereo it is coming from) is a hallucination.’

        A lot of my delusions, when I have them, fall into the category of ‘delusions that I am or might be more crazy than I actually am’.

        • Jacob Schmidt says:

          A lot of my delusions, when I have them, fall into the category of ‘delusions that I am or might be more crazy than I actually am’.

          Semi-related: I used to have semi-crippling anxiety. While I was in a more anxious phase, I would become convinced that I would never get better. Of course, feeling that scared and freaked out for the rest of my life was a terrifying prospect, and that fear only fed my anxiety. On a meta level, I could recognize the vicious feedback |being scared <—> being scared I would never stop being scared|, and it seemed to me like a fairly plausible mechanism that would prohibit me from ever getting better.

          It was incredibly frustrating to be sitting in my home freaking out over how I can’t stop freaking out. Some detached, analytical part of my mind kept telling me how silly it all was, and I dearly wished for some way to consciously shut off my anxiety so that I could short-circuit my freak out.

          • Peter says:

            Oh yes, the “I’m going mad” feedback loop. One of the things in the lead-up to my first prescription of anxiolytics was me having increasing doubts about my own sanity.

            I hear this is a particular problem with panic attacks – apparently there’s nothing like thinking “oh god, I’m going to have a panic attack” to bring on a panic attack. Thank goodness I only ever had the one – no fun at all.

      • Peter says:

        Erm – my comment there was unclear. When I say “It turns out that for a long time the advice was…” I mean “the advice given by psychatrists etc. (and not by the Voice Hearers Movement)” and by “was” I mean “was and in many or even most places possibly still is”.

      • Vulture says:

        I think tulpas are relevant here. In particular, you might be interested in Luhrmann’s Leland Stanford experiments. (selfish motive: I’m hoping that someone is interested enough to track down a proper writeup of the research)

        • Peter says:

          Never heard of that before. I forsee hours of fascinated reading ahead – thanks!

        • Anthony says:

          When I read “Luhrmann’s Leland Stanford experiments”, I imagined something set in the 1880s with a modern, but somehow topical, pop music background.

          Then I googled it and found that it was Tanya, not Baz.

      • lmm says:

        > Philosophical/psychiatric question: is it possible to have a delusion that you are hallucinating?

        Yes. I’ve had deja vu that felt like that.

    • stillnotking says:

      Somewhat tangential, but I can’t help but think of Julian Jaynes’ The Origin of Consciousness in the Breakdown of the Bicameral Mind and Daniel Smith’s Muses, Madmen, and Prophets. There are some tantalizing reasons to believe that “hearing voices” was once much more common than it is today (to the point of ubiquity, if you believe Jaynes), and more often helpful than not.

      Anyway, both of those are great books if you’re interested in the topic.

    • Setsize says:

      I recently watched the film The Journals of Knud Rasmussen (which can be streamed or downloaded from the creators’ website and is based on the journals of a Danish explorer who traveled with Inuit hunters in the early 1900s.) It has an interesting depiction of this sort of thing, some aspects of which I won’t spoil, but I recommend at least one character’s monologue starting at 21:20, in which he describes how he came to know his “helper spirits,” which do seem to help him make better moral decisions. (Another character’s story at 1:21:30 gives an example of how the spread of Christianity conflicted with this.)

    • Scott Alexander says:

      The people I’ve met who have voices saying good, useful, or positive things to them also are psychotic in other less beneficial ways. But that makes sense, since otherwise they would never end up in a psychiatric hospital.

      Basically, there may be people who have purely positive, nondestructive voices outside the context of a debilitating psychosis, but there’s no reason I would ever see them so I don’t know.

      • nydwracu says:

        There probably are. Some completely normal people (well, as normal as I’d be likely to know) have mentioned having them.

        I get them once every few months, but it’s always “take a nap” or “go to Chipotle” or something like that that I ought to do anyway, so I figure it’s some lower part of my brain figuring out how to get my attention. I’m not sure how common this is — I should start asking for specifics. (But I don’t even like talking about my dreams, so the usual unconscious and stubborn typical-minding about social norms is kind of an obstacle to that.)

  79. Berna says:

    Donated $25 to Multiheaded.

    • Peter says:

      Please enjoy this phatic affirmation of your donation:

      Good Job!!!

      • Peter says:

        Ah, I see the “too many people are called Peter” problem arising again. I once lived in a house with two other Peters (and a Debbie) – one was always called “Peter”, one was always called “Pete” and I was either called “Peter” or “Pete” depending on which one I needed to be disambiguated from.

  80. Gwen S. says:

    PSA: This is a pet-peeve of mine, and I think I’m neurotypical enough that it bother’s other people too.

    When I wish someone a happy birthday on Facebook, I want to receive a thank-you message back. It doesn’t have to be elaborate, you could literally copy/paste “Thank you!” into every comment that wishes you a happy birthday. The point is I cared enough about you to make phatic communication, so it hurts when you don’t care enough to phatically reply.

    A public “Thanks for the birthday wishes everyone” doesn’t work. It feels like you’re talking to Everyone, not to me. Besides, often times I don’t see your public thank-you, because it doesn’t show up on my newsfeed. I suppose I could check your profile after your birthday to see if you published a public thank-you, but that’s making me do work so you don’t have to.

    Anyway, I’m having petty revenge fantasies today. I think maybe I’ll look up all my friends who have birthdays in February, and see who gave me personalized thank-yous last year. They’ll be the ones I give personalized happy-birthdays to this year. Then I’ll make a new wall post: “Happy birthday to all my friends with birthdays in February who didn’t bother to give me a personalized thank you message last year.” That’ll show ’em.

    • Nita says:

      counter-PSA: I know I’m not perfectly neurotypical, but I can’t be unique either.

      Please don’t wish me a happy birthday on Facebook or Skype, or any other means of communication that reminds you about my birthday.

      It’s heartwarming that you care, of course, but the anxiety and guilt I experience upon discovering your message a few weeks later overshadow any positive feelings your well-wishes induce.

      If we know each other well, I’ll appreciate a call, text or email from you — just like I would appreciate talking to you on any other day. But I won’t like a friend any less just because they don’t keep track of dates.

      • Emile says:

        Seconded; I don’t particularly like getting “happy birthday” on Facebook because I don’t visit it regularly enough (and I don’t want to feel an obligation to wish happy birthday to everybody there, or to thank the people who wish me happy birthday).

    • Anonymousse says:

      It might help to realize that some people have an aversion to communicating publicly via Facebook. In some ways it’s like making a public speech: everyone can see you and everything you say may be getting recorded. And misinterpretations are much easier via text. I have this aversion, and reading comments like this one from you reinforces my aversion: I’m glad that I am not currently broadcasting annual birthday notifications on facebook, because it relieves me of cringing social anxiety, worrying that I may be inadvertently triggering other-judgements of the sort you describe.

      Your solution of writing “Thank you!” to every birthday wish could trigger another cringe of worrying that someone is judging me for offering formulaic responses. If I was to write “unique” thank yous for every birthday wish, I myself would feel inauthentic since I typically carefully think through what I write, which is great for analytical blog comments like this one but makes me feel queasy when communicating phatically in text. I’m worried about judgement because I myself judge other people when browsing their Facebook profiles, which I don’t endorse. My usual solution to this is to blatantly neglect my Facebook profile, rarely browse Facebook, and not think about it too hard.

      If you don’t have an aversion to communicating via Facebook, you may find this easier to understand if you have an aversion to responding to emails from important people, responding to awkward texts from potential love interests, or writing Less Wrong posts.

      To conclude, I don’t think you need to feel bad about people not responding to your birthday wishes… it’s not you, it’s them. Like so many things in life. Cc the spotlight effect.

    • Liskantope says:

      Hmm, I find that their “liking” my Happy Birthday post is enough for me. The “like” option provides a good stand-in for a copied-and-pasted-looking, default response, and in fact I prefer it.

      • Gwen S. says:

        Liking would indeed be an improvement over public announcement.

      • Irenist says:

        This. If you “like” my “happy birthday” comment, that will show up in my newsfeed as “X likes your status,” which is nice. “Liking” comments made to me on such occasions is what I prefer to do.

    • Setsize says:

      FWIW, my fear that some of my friends have your preference is strong enough that I have set my birthday to “private.” This way I only have to deal with two or three people who actually remember my birthday, instead of however many people are automatically prompted by Facebook.

    • lmm says:

      I think a public “thanks to everyone” is more than appropriate. The person who’s birthday it is is supposed to be doing less work than their well-wishers, not more. If it bothers you that much, don’t send the happy birthday at all.

    • Anthony says:

      The problem I have with that sort of thing is that there are too damn many of them. I think I got something like 80 birthday messages on my last birthday. Asking for more response than me just “liking” each one is asking for a *lot*. Even “liking” 80 posts when I’m either out doing something birthdayish, or at work, is asking a lot.

      That said, I generally don’t make public happy birthday posts, and only occasionally private ones, and being aware of the potential burden, I’m not at all offended if someone makes a single “thanks everyone” post, or no acknowledgement at all.

    • Scott Alexander says:

      I have mostly the opposite preferences.

      I get no happiness from having people wish me happy birthday on Facebook, but active unhappiness/exasperation from having to respond to everybody – sort of an equivalent of the thank you note problem I discuss here.

      I think liking the birthday wishes is an okay compromise, though I mostly wish people would spare me the entire question.

      • David Hart says:

        This is pretty much exactly why I removed my birthday from visibility on Facebook. Though it has an odd side effect: people whom I would normally wish a happy birthday in real life if I had known, but probably wouldn’t have remembered what date their birthday was, I often don’t say anything to if their birthday comes up on Facebook, because it somehow feels like cheating.

    • Jaskologist says:

      Sheesh, people. Didn’t we just discuss phatic communication? You have been given a great gift in the form of machines which will do your remembering for you, and you’re complaining about the effort to type “Happy Birthday”? You don’t even have to send physical cards anymore!

      Me, I’m terrible at remembering dates. I still do not know my parents’ birthdates. If I met a woman who shared my own birthday, I would start dating, get engaged to, and marry her on that same date just so I don’t have to try to memorize a new date. Having all that taken care of for me is wonderful.

      Here is what you do:

      -If Facebook tells you it is somebody’s birthday, post “Happy Birthday!” on their wall. If you feel the need to signal cleverness (which I totally understand), do what I do and say “I got you this link: …” and then include some link to something random/funny/cute. This last part is entirely optional.

      -Put your damn birthday in Facebook. We neuro-date-amnesiacs need it. When people wish you happy one, click “Like” on their wish. Your social duty is now fulfilled.

      • Matthew says:

        For reasons I’m not sure I an articulate clearly, I find public recognition of non-merit-based occasions cringeworthy, as either giver or receiver. I hate seeing it in my Facebook feed. And the merit qualification thing is real; I don’t have the same reaction to graduations or test-passings that I do to birthdays and anniversaries. I’m not going to try and dictate that people bow to my idiosyncratic text-equivalent-of-fingernails-on-chalkboard preferences, but I’m going to react with hostility if people suggest my failure to join is a moral shortcoming.

        • Jaskologist says:

          It’s not about merit. Wishing somebody a happy birthday is basically a quick way of saying, “Hey, I’m happy you exist.” This was more meaningful when it took effort to remember birthdays, but there’s no harm in riding the residual inertia for now. It is a no-cost way of reinforcing social bonds.

  81. Richard says:

    Veganism is possibly not the optimal option for animal welfare.

    TL;DR: If you like meat, insisting on quality meat may be just as good as going vegan with respect to animal welfare.

    When words to the effect that “Veganism == ethical because farming” hit my retina, the words that actually makes it all the way to my brain is something like “YOU ARE THE DEVIL INCARNATE!!11!11”

    I believe this is more or less what this community calls a ‘trigger’ but I’m not certain I’ve unpacked that word entirely correctly. I am also fairly resilient, so triggering me is OK, not to worry.

    Anyway, let me give some background and eventually one possible valid viewpoint:

    I happen to own a bit of land on which farming takes place. It is not easy to say how long we have been farming this particular piece of land because the records got lost in a fire in the mid 14th century, but even counting that as the start, we’ve been at it a while.

    When I was around 6, my grandfather took me aside and explained at length how this kind of farming over the generations works:

    We are not actually owners of the land, we are stewards of the land and with the stewardship follows some duties.
    The first duty is to the land itself and means that we should strive to leave the land in better condition than when we appeared on it.
    The second duty is to the domesticated animals on the land and means that we should maximise the number of animal-happines-minutes that takes place on our land.
    The third duty is to the wild animals who happens to frequent our lands and this duty is the same as to the domesticated animals, only more tricky because of less control over what happens in the wild.
    The forth duty is to make the land yield enough profit so that duties one to three are possible.
    A fifth and brand new duty that I’ve come up with is that we should do all of the above in a manner that makes for a low carbon footprint. I’m currently at a negative C footprint of roughly 300 tonnes/year and working on improving that, but am facing diminishing returns so not sure how much lower I can get without extending the land under my control.
    The duty to the land is fairly simple. You keep streams and waterways clear to avoid flood damage and hire a guy to fly over with a helicopter and drop crushed dolomite every three years or so to compensate for acid rain and maintain the forest using low impact logging. This is not terribly relevant to my point.

    The duty to the animals involve more work;

    Let’s say you have sheep on the land. Sheep are happiest among other sheep. Fewer than 7 and the sheep get visibly stressed and there are probably some diminishing returns so that sheep number 81 induces less flock happiness than sheep number 8. Sheep also require some grassy hillsides to mess about on, some trees to provide shade in summer and a roof over their head so that they can shelter in winter in a warm place with enough food. Once those things are taken care of, the rest is seasonal work, like helping out with the birthing of lambs and suchlike.

    The thing with sheep is that if they are many and happy, they tend to become larger in number so that at times you will need to cull the flock so as not to cause starvation and illness. Sick starving sheep are not happy sheep.

    Requirements of say, pigs are fairly similar, only they prefer a copse of wood where they can forage for chestnuts, acorns and maple seeds to a grassy hillside and require more heat in winter.

    With the wild animals, the control algorithm is simpler, but you work with less information. The tools at your disposal are mainly culling and providing extra food in particularly harsh winters.

    You need to experience only one scabies epidemic before you see the wisdom of keeping the populations under control, and once you do, you’ll be bloody certain that people can do a damn sight better job at this kind of thing than mother nature does.

    Then there are all the little things of course; if you insulate the barn so that the bats have fewer places to winter, you hang little bat houses on the outside of the barn in order to compensate. When you chop down a hollow tree that served as a high-rise for squirrels and birds, you hang nesting boxes of various sizes in nearby trees and so on and so forth.

    The economy bit is a bit tricky, because unhappy animals that stay in a cage being pumped full of growth hormone and unhealthy feed become significantly fatter than happy ones, and the market mostly pays by weight.

    On the other hand, happy animals yield meat of a significantly higher quality. A happy pig from around here has only around 5% the fat of a typical cage-and-hormones farmed american pig. There are gourmet restaurants who are willing to pay the premium for this.

    And this brings me to my main point really:

    If more people would insist on high quality meat and be willing to pay the extra cost, the market for high quality meat would grow and there would be more happy animals. Also, since I’m one of the very few who currently produces high quality meat from happy animals, the prices would skyrocket and I’d be rich before the rest of the farmers caught on 🙂

    If you want try do this, you can start by looking for ‘grass fed’ meat, because ‘grain fed’ is usually an euphemism for cage-and-hormone raised. The feed is often stated on the label. (at least in Europe, the US may have different regulations wrt. labels). This is a rough heuristic, but it is very hard to feed animals on inefficient feed and get high yields, so in general grass fed means free-range.

    Maximising animal happiness is also not necessarily in conflict with maximising total food production because, oddly enough, most animals seem the most happy on relatively marginal land that are not very suitable for things like effective wheat production. This is supposedly because marginal land has rocks and trees that provide shelter from predatory eyes and the animals don’t know that there are no predators inside the fence.

    This may not be the case for species like buffalo, but it does hold as a reasonable heuristic for the majority of animals. (I don’t know a lot about buffalo, but they seems to like prairies which works for wheat too.)

    Anyway, this has gone on too long and all I really wanted to say is that I think happy animals > vegan food and I can’t see how eating the flesh of contented beasts is in any way unethical.

    • zz says:

      Wait… so you’re telling me that animals that are looked after by humans are better off than animals left to nature’s whims which, as Darwin observed, is extremely cruel? Inconceivable!

      But, seriously, paying a premium for properly-raised meat is, like, super worth it. It tastes better, it’s better for you (not the least because so many toxins are fat soluble), plus the ethics thing. See also: Pollan’s commentary on Saladin.

      • Irenist says:

        “Pollan’s commentary on Saladin.”

        The Conqueror’s Dilemma: How Ayyub, at Hattin, bested France

      • Randy M says:

        The closest I could find was his commentary on someone named Salatin, though I’m sure remarks on Saladin would be more interesting. 😉

    • Nestor says:

      I appreciate your conscientious approach to your work.

      I had heard the argument for “eating happy meat” before, but never quite so articulately and “from the horse’s mouth”. It seems clear to me that if the meat from happy animals is of higher quality and tastes better, I should purchase it if I can afford it, on a self interest basis.

      However from the ethics perspective, I’ve long thought that factory farming is, given current technology, a kind of best approach to what would be the ethical ideal, vat grown meat.

      Allow me to explain: An animal raised under conditions of sensory deprivation such as a battery farmed chicken, also bred to maximize growth is a neurologically stunted creature, by design. I have far more empathy for an autonomous, free range animal that has been allowed to live in a society of it’s peers. To me saying I should eat THAT one and not THIS one seems perverse, like eating one’s own pets.

      I hold human utility on a higher tier than animal utility so I believe access to cheap protein is a benefit to humanity therefore factory farming is a necessity, hopefully to be replaced eventually by less problematic vat grown meat. Your farm would survive in this future as a biodiveristy preserve as economics would inevitably lead to the rapid extinction of farmed species.

      I don’t agree with the ethical superiority of your methods from the point of view of animal welfare, but I appreciate your efforts and attention to detail in your performance of a necessary task.

      • Nita says:

        There is a world of difference between sensory deprivation and torture.

        • Nestor says:

          It can be a form of torture, but my point is rather the stunting of development. If you aren’t exposed to language you don’t learn to speak, if you don’t get light you grow up blind, etc…

      • Baby Beluga says:

        I dunno… I see your point, but I think it’s likelier that if you create an animal under “sensory deprivation” as a “neurologically stunted creature”–in other words, if you took a creature and brought it very far from the sort of setting it was designed to be–then my intuition is that it would be very likely to suffer. Unfortunately, this is hard to test, because neurologically stunted creatures aren’t great at talking about their feelings.

        The reason I think that neurologically stunted creatures are probably suffering is because we’ve drastically changed their surroundings, without actually taking away the thing that makes them capable of suffering (whatever that is?!). You haven’t actually made “vat grown meat”–the thing you made still has a brain–and when it’s unable to move, or to do almost anything its brain was developed to do, its brain is going to complain. And I think a brain complaining is more or less what “suffering” is.

        I’m far from sure that I’m right. But I don’t really see a reason why a creature should stop being able to suffer, just because it’s been mistreated. To me, that seems intuitively like the opposite of the way it should go.

        • Nestor says:

          Sure, it’s suffering, industrial processes are not concerned with minimizing suffering, I’m just pointing out it can happen as a side effect.

          I think there’s a bit of magical thinking involved in this idea, as if the soul of the animal is something that is independent of the specifics of the creature, we see a battery farmed chicken in it’s cage and we compare it with the image in our head of a healthy free range chicken and we feel it has been denied this fulfillment of it’s “chicken destiny”, but in actual fact it has no concept of what a proper chicken should be like, it has grown in a cage, and has a stunted neural development even for a chicken.

          Pampered animals like the pigs used to make serrano ham sure taste better, but that’s a separate issue.

          • Baby Beluga says:

            The mistreated chicken has no concept of how it ought to have been treated, but its body and brain might “complain,” that is, suffer, if it’s not being treated the way it evolved to be treated.

            To give a (somewhat gruesome) example, imagine if, immediately after you were born, I went ahead and removed all your skin, and then for the rest of your life I kept you in isolation from society. You would have no concept of the fact that you should have had skin, but don’t. But you would probably be in terrible agony all your life, since your body would still “expect” you to have skin, and find that you don’t. You didn’t evolve to not have skin. In general, it seems reasonable to assume that beings that have been taken far from where they evolved to be in a random way are probably suffering.

            EDIT: Sorry, I see that you aren’t trying to say the chicken isn’t suffering, so what I’m saying is probably just sideways to your point. I guess I think that the main thing that matters from an ethical standpoint is minimizing suffering? In other words, this is the reason why I think that Richard’s approach to farming is ethically superior to factory farming–because the fact that his animals suffer less is all that matters.

          • Nestor says:

            I honestly don’t think, when we look at a picture of a free range animal and one of a factory farmed one, that we are truly empathizing with the wretch in the cage, more honestly, the sickly one in the cage triggers our “ugh, diseased” while the pretty, healthy one checks out all the “would eat, yum!”.

            Our culture is full of visual depictions of happy animals decorating the boxes where their bodies are being served to us, it’s a bit creepy, really. We seem to believe animals are fulfilled by serving their function as food, if we’re to take these cartoons and logos at face value.

            Let’s do a thought experiment: The one world government has decreed that pets have too much of a carbon footprint, so a cull is in order. This is not optional because black helicopters are everywhere, also their argument checks out as pretty solid, there really are too many pets, everyone agrees. The one government can put out a hell of an ad campaign.

            So the lottery has been drawn and your number has come up, you have to hand over your cat to the man in the gas mask. And your cat is Maru.

            But, at the last minute you discover a cat has given birth to a litter in your garage and has abandoned the runt, a sickly, scrofulous blind little ragdoll of a kitten, but hey, the quota doesn’t care, once cat is as good as another.

            So obviously you hand over Maru, who has had a very good and happy life, after all.

            Or do you?

          • Anonymous says:

            With meat, the question isn’t which existing animal to destroy, but which type of animal to create in the first place. So the Maru example is not applicable.

          • Nestor says:

            Are we discussing the ethics of an existing reality or are we pretending we are gods who can rewrite it on a whim?

            I specified in my original post that human needs come first so efficient protein production is non negotiable, and in my imaginary scenario that an overwhelming force (The one world government) is forcing the scenario to mimic this.

            Because if we’re playing with rewriting what is, then vat grown meat is the ethical end game win, and free range meat eating starts to look increasingly ethically dubious the more ground the new method gains.

            But it doesn’t actually exist yet aside from a few lab prototypes and publicity stunts so that’s not the argument we’re having here.

    • Muga Sofer says:

      As a vegetarian, I’m much, much more OK with “free range” meat than factory-farming. When vegans talk about how awful “farming” is, they’re almost always thinking of some shocking descriptions or images they’ve seen of factory farming.

      So if it helps any, they’re often just being vague, not seeing you as devil.

      With that said, this is probably the first compelling account I’ve seen for the argument that “good” farming is better for the animals than vegetarianism. I really, really hope you succeed at marketing this product.

      • Richard says:

        The market is there, I doubt very much you will find a single ounce of factory-farmed meat in any 3-star Michelin restaurant.

        Profitability is currently a problem in the sense that I would earn more money with less effort by switching to factory methods. As long as the farm does not bankrupt me, I’ll keep to my ways though 🙂

    • stillnotking says:

      As a vegetarian, I agree with you in principle. I think animals raised by caring humans are better off than wild animals; I think domestic breeds in particular are human-dependent to the point that it would be impractical (and unethical) to try to revert them to the wild; and so I recognize that purchasing grass-fed, free-range, “happy” animal meat is ethically neutral at worst. A lot of vegan moral reasoning seems driven by ideal-world scenarios for which I have little sympathy.

      The problem is that I experience an extremely strong, visceral feeling of revulsion at the idea of eating a sentient being that was killed for my benefit. I recognize that my response is probably irrational, given the whole picture, but that doesn’t change it. The cliche analogy is the reaction most Westerners have to eating cats and dogs.

      I wish you the best of luck with vegans/vegetarians who have less personal reasons for abstaining from meat.

    • disciplinaryarbitrage says:

      Thanks for weighing in on this here! Having spent a lot of time in the past year reading/talking/arguing about animal ethics, more input from people who are actually spending time with animals raised for food is useful.

      I am a vegetarian and agree with your line of thinking, certainly in principle and probably even in practice. Raising happy animals on marginal land with comfortable lives, quick deaths, and the end result of a tasty steak on someone’s plate doesn’t bother me. However, I still avoid meat for the following practical reasons:

      1. It is difficult, in my experience, to have much confidence that a given piece of meat was raised under these sort of conditions. Even if I were to go visit a farm, I know very little about animal behavior and husbandry and don’t have a lot of confidence that I’d be able to accurately assess the question of “are these animals living worthwhile lives?” based on wandering around a pasture for an afternoon. Making that determination based on some vague labeling while I’m standing in the grocery store doing some ad-hoc meal planning is a lot more dubious.

      (If anyone can speak for any labeling standards prevalent in the US that do reliably signal good, as opposed to merely less-torturous, living conditions for farm animals, I’m all ears.)

      2. I quite often hear people say things in agreement with this basic line of thinking–“Factory farming is awful, of course, but raising meat can be perfectly ethical! Therefore, I won’t be a vegetarian.” The speaker then proceeds to order a Big Mac. (By the way, this is something I was 100% guilty of before I gave up meat.) Vegetarianism as a general policy is a useful Schnelling fence, both for one’s own self-discipline and as a shorthand statement of dietary constraint that’s (in most contexts) socially acceptable and at least halfway likely to be complied with. Trying to draw nuanced distinctions between acceptable and unacceptable meats while your friends are putting in a pizza order tends not to lead anywhere positive (but your mileage may vary).

    • Baby Beluga says:

      Yeah–I agree with you. I don’t think there’s anything wrong with raising animals in order to eat them, as long as you’re confident those animals are happy. (In fact, arguably it’s better than not raising animals or eating them, assuming there was no opportunity cost to raising said animals). I think a good fraction of the vegan movement, for what it’s worth, agrees with you (the Peter Singer kind of vegan, not the Gary Francione kind of vegan). Sometimes they might be initially be hostile to this line of reasoning, since as other commenters have pointed out, “animals can be farmed ethically” is often used as a justification for “therefore it’s okay for me to eat this particular peace of [unethically farmed] meat.” But if you’re clear that you really are kind to the animals you raise, I think they will be friendly to your argument (although maybe I’m projecting too much here).

      One comment I do have about this line of reasoning is that even though it’s pretty mainstream, it’s definitely more brutally utilitarian than our society (though not this blog) is usually willing to accept. Like, think about that last sentence: “I can’t see how eating the flesh of contented beasts is in any way unethical.” To be clear, I agree! I can’t see how that’s unethical either, and I’m *not* claiming that it is. But, like, change that sentence very slightly (“I can’t see how eating the flesh of happy pets is in any way unethical,” “I can’t see how eating the flesh of contented human infants is in any way unethical, “”I can’t see how eating the flesh of the terminally ill is in any way unethical”) and all of a sudden you’ve got a proposition that is WAY too brutally utilitarian for most of society to swallow, even though the initial sentence was fine. I think that’s interesting, and worth considering in its own right.

      • Wrong Species says:

        I think the problem is that the idea of minimizing suffering of animals and promoting their autonomy are two separate issues. We aren’t protecting human life just so they avoid pain(if that was the case then we could simply chloroform people before murdering them), we also respect their ability to choose for themselves, with some exceptions. Vegans apply the idea to animals. From their point of view, it would be better for the animal to be free, even if that would result in more pain in the same way that we respect the choice to eat McDonalds every day and die at 50.

        • >Vegans apply the idea to animals.

          Some vegans do, others don’t. The distinction between veganism and non-veganism is orthogonal to the distinction between autonomy and welfare. (Most of my vegan friends care about welfare, not autonomy.)

    • Anonymous says:

      So…practically, as a consume, what do? (And how much extra $$?)

    • houseboatonstyx says:

      ++ to your whole post, really

      If more people would insist on high quality meat and be willing to pay the extra cost, the market for high quality meat would grow and there would be more happy animals.

      This is the Jaina strategy (though they apply it only to milk, as they don’t eat any meat). The cow is a member of the family, treated as a pet, thus happy. In urban situations they continue to use milk, but get the best kind possible — which means from farmers as much like you as they can afford. This influences other farmers to treat their animals better. Giving up milk altogether would have no effect on the other farmers..

  82. Wirehead Wannabe says:

    How well regarded is the Conners’ Continuous Performance Test as a tool for assessing ADHD?

    Background info: I took this test as a child and got a negative (non-ADHD) result. I strongly suspect that I (now 23 years old) may have ADHD, but I’m not sure how strongly the results of this test weigh against that hypothesis. I am extremely fidgety, find tasks (especially filling out forms and making appointments) very difficult to initiate, have difficulty being present in social situations, and I occasionally run stop signs and turn onto one-way streets due to not paying attention. I spend hours a day pacing back and forth thinking about nothing in particular, which cause my college career to suffer greatly. I normally run 50+ miles a week, and find that taking breaks from this causes my symptoms to worsen.

    I googled this particular test and there seems to be some criticism of its usefulness in diagnosis but I’d be interested in hearing how seriously I should take this. Also, is there a good way to suggest a diagnosis to a psychiatrist without looking like a drug seeker or a hypochondriac? Thanks.

    • Thomas says:

      I was diagnosed with ADHD at a very young age and grew up with other ADHD people. I would assign 90% confidence to you having ADHD, and 99% confidence to you having some issue that a competent psychiatrist can help you with to substantially improve your quality of life. I suggest getting tested by someone with “adult ADHD” mentioned as a specialty. I don’t think you will look like a drug seeker if you come in talking about your symptoms first, and asking for a serious diagnosis (which may take several hours). That said:

      If your health insurance prescription coverage is good, work with your provider to experiment with the three main classes of drugs to see which give you the best results. Atomoxetine (Strattera) can complement the stimulants methylphenidate (Ritalin, Concerts) and amphetamine (Adderall, Dexedrine, Vyvanse).

      Note that this medical advice is “see a medical expert.”

  83. Nita says:

    So, the Hyde and Mertz paper.

    They write:

    Thus, gender ratios in the upper tails of actual distributions were calculated using data from the Minnesota state assessments. [..] For students scoring above the 95th percentile, the M:F ratio was 1.45 for Whites, close to theoretical prediction. At the 99th percentile, the M:F ratio was 2.06, again close to theoretical prediction. However, the M:F ratio was only 0.91 for Asian-Americans, that is, more girls than boys scored above the 99th percentile.

    citing this little study by Hyde et al.: http://dericbownds.net/uploaded_images/hyde.pdf
    [Hyde, Janet S., et al. “Gender similarities characterize math performance.” Science 321.5888 (2008): 494-495.]

    So, what’s going on here? Genes? Culture? Methodological issues?

    • zz says:

      I’m given to understand that the different gender ratios are explained by social issues; countries with the lowest gender inequality have about a 1:1 m:f ratio in the top 1%.

      However, if you take the 10 countries with the lowest gender inequality (as determined by Wikipedia) and look at the gender ratios of their collective International Math Olympiad teams (presumably top 6 mathematicians from each country), it comes to 53:7 in 2012 (the most recent year Wiki had gender inequality data for). I’m just learning statistics, so I don’t actually know how p value, but even reasoning naively, there’s a snowball’s chance in Hell that’s a coincidence.

      • Nita says:

        There might be different kinds of inequality. GII is obviously designed to measure something very different from the fate of high IQ kids.

        I took a list of everyone who has received at least one gold medal from the IMO website. Almost a third of these kids come from China, the USA or Russia/USSR.

        Out of these countries, the USA has the lowest percentage of girls – either 1.3% or 2.5%, depending on how you count the dual citizenship kids (that’s 1 or 2 girls).

        USSR had 4 girls, about 6%. Russia has had 3 so far, about 4.5%.

        Bonus trivia: Ukraine, Korea and Romania have had 3 female gold medalists each, which is 12.5%, 6.5% and 5.7% of their gold medalist totals.

        [Vietnam and Taiwan were excluded from my analysis due to incomplete gender data.]

        • Daniel Speyer says:

          I can’t find the IMO data, but working backwards from your data I get 79 US medalists and 67 USSR. Overall, this gives 4.1% of medalists as women. The likelihood of seeing at least 4 USSR medalists under this model is 30.7%, and of 2 or fewer US is 36.5%. Both at once is 10.8%. We are not able to reject the null hypothesis.

          In short, the USSR might have found a way to get more women to do math at the highest levels, but it could easily be noise because the numbers are so small.

          • Nita says:

            You’re right. But increasing the sample size and doing it right would require way more time and analysis (e.g., adjusting for achievement levels) 😀

            I do feel there is a cultural difference — USSR gave students more educational freedom than Jordan, but less than the USA. On the other hand, “men are better at math” and “math requires genius” are popular beliefs in both cultures.

            Are there any math-focused high schools in the USA?

          • namae nanka says:

            This paper uses a new data source, American Mathematics Competitions, to examine
            the gender gap among high school students at very high achievement levels. The data bring
            out several new facts. There is a large gender gap that widens dramatically at percentiles
            above those that can be examined using standard data sources. An analysis of unobserved
            heterogeneity indicates that there is only moderate variation in the gender gap across
            schools. The highest achieving girls in the U.S. are concentrated in a very small set of elite
            schools, suggesting that almost all girls with the ability to reach high math achievement
            levels are not doing so.

            The Gender Gap in Secondary School Mathematics at High
            Achievement Levels:
            Evidence from the American Mathematics Competitions1
            Glenn Ellison

    • JK says:

      So, what’s going on here? Genes? Culture? Methodological issues?

      Sample size. Total N for Asians/Pacific Islanders in the Minnesota sample is 219, of whom 1.37% girls and 1.25% boys scored above the 99th percentile. That probably corresponds to one or two girls and boys scoring that high, which means that no reliable inferences about the gender ratio can be made. However, as shown in Table S2 in Supporting Online Material, the Asian variance ratio is always >1 across different states, i.e., males are more variable (the ratio is lower in Asians than non-Asians, though, which may reflect the fact that more Asians hit test ceilings).

      A recent study concludes that political, economic, and social equality are not consistently correlated with sex differences in academic achievement internationally. By cherry picking data you can show such correlations but these are driven by outliers and do not replicate across survey years. See also this fisking of one of Mertz’s claims.

      • Nita says:

        Sample size. Total N for Asians/Pacific Islanders in the Minnesota sample is 219

        Thanks! I tried to hunt down the original data, but actually reading the summary table would have been a better choice.

      • haishan says:

        More sample size stuff:

        Two recent studies directly address the question of whether greater male variability in mathematics is a ubiquitous phenomenon. Machin and Pekkarinen (19) reported that the M:F VR in mathematics was significantly >1.00 at the P < 0.05 level among 15-year-old students in 34 of 40 countries participating in the 2003 PISA and among 13-year-old students in 33 of 50 countries participating in the 2003 Trends in International Mathematics and Science Study (TIMSS)…a finding inconsistent with the Greater Male Variability Hypothesis.

        So, like, this conclusion depends heavily on statistical power. If we naively assume that there’s a 0.05 probability of incorrectly failing to reject the null hypothesis, then we should see 6/40 or more non-rejections about 1.4% of the time. But in reality, a number of those countries have small sample sizes (Tunisia? Denmark?) and thus low power, and so non-rejections should increase; my gut instinct is that 34/40 significant differences in gender ratios is not close to enough to reject the “Greater Male Variability Hypothesis.” In any case, Hyde and Mertz don’t do the necessary work to prove this; maybe it’s there in the Science paper they reference.

        • JK says:

          Yeah, Hyde and Mertz make no attempt to control for chance fluctuations in the data. A good example of this is the reanalysis of Czech data I linked to above. Based on data from a single study, Mertz claimed there were no gender differences in math performance in Czech Republic, even when five other similar studies showed males to be overrepresented among top performers.

    • Brett says:

      Methodological issues. I worked out the original numbers back from the table in their earlier paper by assuming that she didn’t manage to find any fractional children. The samples (for Asian children) were (N=219):

      Total Asians: 3660, 1835 male, 1820 female
      Asians above the 95th%: 219, 115 male, 104 female (This is apparently what the N referred to)
      Asians above the 99th%: 48, 23 male, 25 female

      You can decide for yourself whether you think that these numbers support the conclusions she drew. You can also speculate on why she didn’t see fit to explicitly include these numbers in her results and I had to back-calculate them.

      • JK says:

        Thanks, I was puzzled by the sample size because the percentages didn’t seem to make sense. While N’s of 23 or 25 give suggestive support for the Asian gender parity hypothesis, more data would be needed to draw any conclusions.

      • Anonymous says:

        Where did you get the 3660 number?

        By the “earlier paper” do you mean the one that Nita cited? or an even earlier one?

        • Brett says:

          I mean the one Nita cited earlier: Table 2 in http://dericbownds.net/uploaded_images/hyde.pdf

          I used a variational method to get the numbers. You can see my spreadsheet at https://docs.google.com/spreadsheets/d/1HlwwMPEzwI6JPybcM4sh95wOz9MrcYL4Kh9YaGSLcN4/edit?usp=sharing

          The short answer is there’s not many sets of integer numbers that will give you the percentages cited in the table: for N=219 above the 95th%, the female and male numbers for the 95th% and 99th% are fixed and there’s only a small amount of wiggle room in the total population (which is 3655, not 3660 as I said earlier – sorry for the error!). For the white population because N is so much larger there’s more potential variation in the numbers so I can’t pin down the counts as precisely – the total population could be as low as ~31K or as large as 43K.

          • Nita says:

            That’s the prettiest thing I’ve seen today 🙂

          • Brett says:

            Thanks! This analysis also got the comment “You must be correct” from the notoriously irascible Greg Cochran, so I think I’m justifiably proud of it.

          • Anonymous says:

            The paper does not claim N=219 for over 95%. It claims N=219 for everyone. Isn’t it more plausible that the paper is slightly wrong on some of the numbers than that it is completely wrong on the meaning of N?

          • Brett says:

            No it doesn’t; all it says is N=219, with the specific population left ambiguous. Check for yourself – the only reference to N=219 is in the table with no additional text. So yes, I find it more plausible that it refers to the population above the 95th% than that they did their math wrong.

          • Anonymous says:

            Saying that N is ambiguous is like saying that F is ambiguous – maybe it stands for male.

          • Nita says:

            They do say:

            Too
            few students scored above the 95th percentile to compute reliable statistics for
            these groups: American Indians, Hispanics, and Black not Hispanic.

            which supports Brett’s interpretation of their “n”.

          • Brett says:

            Well, you’re welcome to your own plausibility heuristics.

            Taking a look at the Missouri Dept. of Education website, I can’t find the exact results Hyde et al. report. But I can find total tested populations for 2010-2014 Grade 11 students, which give total Asian populations of 3,823-4,120 and total White populations of 45,793-50,418, very close to what I back-calculated from the percentages in the Hyde paper.

            http://rc.education.state.mn.us/#

          • Anonymous says:

            Since the paper says that the numbers are from NCLB, it is strongly implied that they are the whole population; so your interpretation is probably correct.

            But that does not justify the paper’s use of N (which is, by the way, not an error in the paper’s favor).

  84. miaoued says:

    Re: Scott’s discussion of the Leslie et al paper, I would like to moot a possible argument in favour of the paper’s conclusion– not so much because I disagree with Scott but because this seems to be a plausible explanation that I have no strong counterargument to.

    We know that especially in “mathematical-logical” areas, men significantly overestimate their ability and women significantly underestimate their ability relative to each other (here). Scott has also discussed the “lottery of fascinations”– the idea that our inborn inclinations predispose us to pursue certain subjects with greater zeal than others, and as a result increase our chance of succeeding in those areas. But our inclinations are also a result of our environment and our beliefs– if we were led to believe that we are less competent than we actually are in a mathematical field, combined with the perception that the mathematical field is itself highly demanding, then we are less likely to invest time or even feel interest in said field (unless you are a masochist). Over time, the disadvantage may well accrue into the poorer results that Scott discussed.

    Responses appreciated.

    • Richard says:

      For an amusing take on gender differences and the whole nature/nurture debate, see “Brainwash”: https://www.youtube.com/watch?v=cVaTc15plVs

      (english subtitles, Norwegian documentary series.)

      if it’s too long, take a look @ 17:30 -19:00

    • Caelum says:

      Assuming this is the study you were linking to (the link was broken for me):

      http://www.ncbi.nlm.nih.gov/pubmed/?term=24303578

      I think “men significantly overestimate their ability and women significantly underestimate their ability relative to each other” is a misrepresentation. Men in the study gave higher self-estimates of a proxy for intelligence, but men also scored higher on measured proxies for intelligence.

      To be clear, the authors claim to have been looking for a Hubris-Humility Effect, which they say in the intro is “men significantly overestimating, and women significantly underestimating, their abilities relative to each other”, but the paper they link to just points out that men have higher self-estimates of a proxy for intelligence than women. Then the authors here say that their hypothesis that men were overconfident and women underconfident was confirmed, because men had higher self-estimates again. But certainly self-estimates alone can’t speak to whether the estimates were “over” or “under” confident.

      EDIT: Not sure how important that all is to your overall question, but I’m also not sure it’s clear from that study alone that gender-ability-perceptions play all that big of a role in math-ability outcome.

    • lmm says:

      From @Caelum’s response it seems this isn’t actually true. But suppose it was. What would we do about it?

      “Affirmative action”-type policies – quotas, or generally just lowering standards for women in mathematical-logical fields – seem to stand an enormous chance of backfiring, if we accept that the differences in results reflect real differences of ability (that are a result of differences in upbringing). Outreach-type events to encourage girls to go into these fields also seem like they would be ineffective.

      We could try to attack it head-on, by giving boys and girls earlier and more accurate feedback about their objective ability levels. Would that help? Would it be politically acceptable? Here in the UK we take national standardized tests at age 7 in English, Mathematics and Science – that would seem like just the ticket. Are girls who score highly in those more likely to become mathematicians in later life? Does any sex gap reveal itself there?

      IIRC Scott’s summary was that girls do slightly better in early school, and then boys overtake them around puberty. That seems to suggest a physical difference rather than a self-image based one.

    • Anthony says:

      The obvious response is that we should measure people’s abilities (and actual accomplishments) as objectively as we can, let them know the results, and make decisions based on those results. Thus both men and women will know how well-equipped they are for math-intensive work (and other types of work, as well), and they will be making more fully-informed decisions when they choose their future studies, work, etc.

      We should also do more research into how important intelligence and other characteristics actually are to success in various endeavors, and work on promoting ways to change those things which are changeable.

      • Caelum says:

        @lmm and @Anthony, I’m not quite sure how what you suggest is substantively different than the current state of affairs. Schools measure aptitude at a lot points during development (at least they did at the public school in the U.S. I went to), and they give children feedback. Is that not everyone’s experience? Even assuming that not all students take the sort of annual district/statewide assessments that I’m talking about, students get grades which are at least some sort of proxy for intelligence/ability in a given subject area.

        I mean to say that all the sorts of proxies for intelligence we currently use (or could) are broadly equally useful for providing feedback to allow students to more accurately estimate their own abilities. I guess I’m not sure what @lmm means by “more accurate” assessments.

        • Anthony says:

          I graduated from a Catholic high school in 1984, but went to public schools in two states for k-8; my older daughter started kindergarten in 2012. My mother taught at a California community college from the late 70s to the early 2000s.

          When I was in elementary school, it was fairly common for children to be administered IQ tests. There were programs for “gifted” (high-IQ) children, and at least in the elementary schools, there was tracking by ability (and doing particularly poorly or well could get you moved down or up a track or more).

          Now that my kids are in elementary school, there’s no evaluation independent of the teacher (unless a parent gets some sort of diagnosis of a learning disability), and there’s definitely no tracking.

          I mean to say that all the sorts of proxies for intelligence we currently use (or could) are broadly equally useful for providing feedback to allow students to more accurately estimate their own abilities.

          Not really. Proxies that depend on effort will produce results that show what the student *has* done, not necessarily what the student is *capable* of. If the point is to motivate smart kids from groups often overlooked (or groups where smart kids are less common, or groups with a culture that doesn’t value schoolwork, or just kids who are lazy), it’s better that they know what others think they *could* do.

          • Anonymous says:

            If the point is to motivate smart kids from groups often overlooked (or groups where smart kids are less common, or groups with a culture that doesn’t value schoolwork, or just kids who are lazy), it’s better that they know what others think they *could* do.

            I just want to applaud this for its rare insight and empathy. (And I do mean rare . . . even parents with more than a passing interest in education don’t seem to give a fig about children who aren’t like theirs.)

          • Anonymous says:

            Also,

            Now that my kids are in elementary school, there’s no evaluation independent of the teacher

            Yes, and think about the personality of the typical elementary school teacher vs. her possibly non-NT nerdy student. It doesn’t bode well for the student.

  85. Red, Grey, and Blue says:

    Hoping I can get some help reconciling my predominately Blue/Grey views with a single conspicuous Red belief: abortion.

    Life-threatening and dangerous pregnancies are perfectly fine; that’s just math. Morning-after pills and very early abortions also seem acceptable, because zygotes and undeveloped embryos seem indistinct from sperm on the personhood-scale. But given that brain development begins as early as 5 weeks of pregnancy, I’m not sure where we decide to draw that line.
    Arguments from “parents who get abortions are parents who can’t afford children, so it’s saving them from a bad life” or the Freakonomics argument that abortions lower crime rates sound creepy and eugenics-y, and not at all in line with Blue/Grey values as I understand them.
    I’m not familiar with prenatal development other than what I can learn from the Wikipedia article, so if there’s a compelling case from biology, I’d love to hear it.
    The strongest argument I know of is from Women’s Rights, where you don’t get to control the life of a young women by forcing her to have a child, and adoption/foster care systems (at least in the States) don’t seem strong enough to completely fill that gap. My problem here stems from the fact that we are taking away rights from the more marginalized group in favor of the comparatively well-off. To be clear: women have certainly been treated unfairly by society up to this point, and measures still have to taken to address that. It just seems like doing so at the expense of a group with literally no representation is dangerous.

    My motivation for this post comes from the fact that on almost every other issue, the Blue/Grey position seems intuitive and intelligent. The most likely explanation I can think of here is that I’m doing something wrong, and I’d like to know what it is.
    Context: I was raised Catholic, but am now atheist. Arguing against abortion from the concept of sin seems stupid.

    • Dinaroozie says:

      One direction you can come at with this is to draw an analogy with someone requiring an organ donation. Suppose I cause a car accident, and though I’m unhurt, someone else is at death’s door and requires a kidney. It seems to me that most people are intuitively uncomfortable with me being compelled by law to give up my kidney, even though the other person will definitely die without it, giving up a kidney is a comparatively non-deadly thing to do (though it has risks), and arguably they’re in this situation because of me. Would you be comfortable with the law compelling me to undergo surgery and give up my kidney in this scenario? If not, what makes that different to you than compelling someone to undergo a risk to their health (pregnancy/birth) so that someone else might live? I’ll note that people might think I’m being a dick by refusing to give up my kidney – they just don’t want the legal system forcing me. I believe there are at least some people who hold a similar view regarding abortion, and I don’t know where that falls in the red/blue/grey continuum.

      Another possible line of argument, though one that likely lots of people find horrible, is the idea that killing people is bad not because ending a life is fundamentally bad, but because of the suffering it causes them and those left behind. Once someone is born, it becomes impossible to kill them without being 100% sure that you’re not hurting someone else, but before they’re born you’re arguably in the clear on that. You can probably tell that I find this argument pretty poor but it’s another possible angle.

      • Menno says:

        If not, what makes that different to you than compelling someone to undergo a risk to their health (pregnancy/birth) so that someone else might live?

        Giving up a kidney is not analogous to pregnancy. Once your kidney is gone, it is permanently gone. There’s no biological mechanism to return you to a pre-donation state.

        While pregnancy has risks and certainly interferes with someone’s daily life for several months, it’s not the case that being pregnant is permanently altering*.

        You also have the difference that the other person in the accident made a decision that put them there. This isn’t to blame them, but their actions even indirectly led to their situation. A fetus has no agency whatsoever, no actions, no decision making capability. And on the other side, while your actions in driving the car led to the situation, it’s not expected or even truly foreseeable. Heterosexual sex leads to pregnancy. That’s the biological function.

        *Yes, pregnancy does cause some permanent changes to a woman’s body and there are non-negligible risks. But again, it’s a normal process unlike removing a kidney.

        • RCF says:

          “Giving up a kidney is not analogous to pregnancy. Once your kidney is gone, it is permanently gone.”

          You are committing the all-too-common error of thinking that “analogous” means “exactly the same in every single aspect”, rather than “having similarities that are pertinent to the current discussion”.

          “You also have the difference that the other person in the accident made a decision that put them there.”

          Not central to the hypothetical. We can imagine that the “car accident” consists of someone driving their car into a house and injuring someone in the house.

      • John Schilling says:

        If you were convicted of a crime, in this case reckless driving, which resulted in an innocent person being left with zero working kidneys, I don’t think I would have much problem with you being ordered to deliver one healthy kidney in restitution.

        Pragmatically, I think the situation comes up rarely enough that it’s not worth the bother of changing the legal code to deal with it, and if someone did propose such a change I’d wonder whether they had something else in mind.

        But infanticide is anything but rare; for reasons ranging from getting a good night’s sleep for the first time in six months, through six-figure financial payoffs and up to the grand eugenics of crime control, killing innocent babies is frequently a tempting expediency. We have two practical Schelling points against widespread infanticide; conception and birth. Both have real problems, but it’s clear that the nuanced versions involving embryonic neural development are a recipe for endless strife.

        Pick your unsatisfying solution.

        • Gbdub says:

          It’s obvious why birth turned into a Schelling point, but it strikes me as a pretty arbitrary one – does a premature birth count? What about induced labor? The difference between a 9th month fetus and an infant seems mainly a matter of location, whereas a pre-conception sperm/egg is fundamentally different from an embryo.

          And as far as compelling women’s bodies, it seems that we’re perfectly willing to compel her to care for the infant after it’s born (unless she formally puts it up for adoption, which is easy immediately after birth but more complicated later) – if that’s truly your objection how do you reconcile the two? Yes, pre-birth the compulsion is literally within the woman’s body, but compelling 18 years of guardianship and care seems pretty arduous as well. Again the distinction seems primarily one of location rather than form.

          • Creutzer says:

            It’s obvious why birth turned into a Schelling point, but it strikes me as a pretty arbitrary one

            That’s kind of the very nature of a Schelling point.

          • Nita says:

            I haven’t seen any political demands for elective abortion of 9-month fetuses. Have you?

            (unless she formally puts it up for adoption, which is easy immediately after birth but more complicated later)

            She can make an actual choice after birth, though. Not the “consenting to drive is consenting to die” kind of choice she “made” by having sex.

            I’m still unhappy with the current system of compulsory parenting, though, because it seems that some parents make their unwanted children extremely, traumatically miserable.

          • Gbdub says:

            @Cruetzer – my understanding is that a Schelling point MAY be arbitrary but doesn’t have to be. We’ve sort of established a third Schelling point of fetal “viability”, but that’s vague and shifting. “Conception” seems like the least arbitrary of the available Schelling points.

            @Nita – you’re right of course regarding post-birth choice in adoption. What I was mainly trying to convey is that we aren’t absolute about bodily autonomy, particularly when there are external consequences. Better examples would be the restrictions on drugs, suicide, and now health insurance. So if we’re being consistent, “my body my choice” can’t quite be the whole argument. At least it’s not enough to totally assuage my squeamishness despite my theoretical support.

            As far as people pushing for late-term abortion, that’s definitely a contentious issue with people on both sides. Part of the Texas bill Wendy Davis famously filibustered was a ban on abortion after the 20th week. And the federal ban on partial birth abortion faced a serious Supreme Court challenge in 2007, being upheld on a 5-4 vote. Not quite 9 month, but there is definitely significant support for post-viability, late-term elective abortion.

            Finally, I’d add that while consent to drive does not imply consent to die in a car wreck, I think we’d have to conclude that a driver has consented to the risk of dying in a car wreck in a way that someone sitting on the couch has not, and it is therefore reasonable to hold the driver in some degree responsible for the consequences of deciding to drive.

            I think I’m steel manning anti-abortion more than I intended to.

          • Nita says:

            @Anonymous

            “Born Alive” is defined as the complete expulsion of an infant at any stage of development that has a heartbeat, pulsation of the umbilical cord, breath, or voluntary muscle movement, no matter if the umbilical cord has been cut or if the expulsion of the infant was natural, induced labor, cesarean section, or induced abortion.

            A human embryo has a heartbeat at the 6th week of gestation. According to “Born-Alive Infants Protection Act”, what exactly should be done after a miscarriage at this stage of development?

        • RCF says:

          It is my opinion that you have arguments that you think are relevant, but rather than presenting them, you are merely alluding to them, and that sort of behavior should be discouraged.

      • Anonymous says:

        I think this scenario is possibly flipping intentionality. Would you respond similarly to the following hypo?

        Suppose you’re going rock climbing with someone. Due to an incidental slip, your partner is hanging from a rope attached to you. The rope is caught around your leg. There is a legitimate chance of serious damage occurring to your leg, up to and including it having to be amputated (which can clearly also result in your death). You’re sufficiently high on the cliff that you know your partner cannot survive the fall.

        Are you morally allowed to cut the rope below you? Should you be legally allowed to? Does it matter if the situation arose because of an intentional choice you made or due to your negligence? Does it matter if the risk is to just your foot or your pinky rather than a larger body part or more vital organ?

        • Gbdub says:

          That still breaks down relative to abortion because 1) the fetus, unlike your climbing partner, had no agency in creating the predicament and 2) assuming a “normal” pregnancy, adoption is an out – sort of like if you’re hanging from the cliff, but you hear the inbound rescue chopper.

          Where it does work pretty well is that my sympathy is somewhat affected by how conscientious you were in selecting your climbing route and managing your safety equipment.

          I’m curious about this because I too find myself generally favoring minimal abortion restrictions on utilitarian grounds, but am squeamish about elective abortions beyond the first trimester or so.

          • Anonymous says:

            1) the fetus, unlike your climbing partner, had no agency in creating the predicament

            I’m not sure this matters. Regardless, feel free to come up with another hypo which embodies this but has intentionality running the right direction.

            2) assuming a “normal” pregnancy, adoption is an out – sort of like if you’re hanging from the cliff, but you hear the inbound rescue chopper.

            This hypo is responding to violinist-style arguments from bodily autonomy. The important feature of the analogy is the danger to your leg, which is analogous to the potential dangers of pregnancy. Adoption would matter in arguments concerning an individual’s obligation to provide continued post-natal care.

            Where it does work pretty well is that my sympathy is somewhat affected by how conscientious you were in selecting your climbing route and managing your safety equipment.

            I think that many people will likewise sneak in some sort of prior responsibility. If this matters, then it’s clearly not a ‘bodily autonomy > life of others’ conclusion. I’m not sure what it is (and I think you can make arguments where prior responsibility doesn’t matter), but we’re not in the region where bodily autonomy trumps all.

      • Anonymous says:

        “. Would you be comfortable with the law compelling me to undergo surgery and give up my kidney in this scenario?”

        False analogy. It’s the baby, not the mother, who undergoes the brunt of the abortion. Would you be comfortable with a law compelling any organ recipient to undergo surgery and give up the organ on the demand of the donor?

      • Matt says:

        I think where this fails is this:

        We know that when we drive we risk maiming ourselves or others. If you get into an accident and maim another driver, you put yourself into that situation, sure, but so did the other driver. A woman choosing to have sex knows she risks a pregnancy, but a fetus did not make any comparable decision to put itself at risk.

        • Anonymous says:

          We know the desires of the other driver (to not be involved in an accident). It’s nonsensical to say that a fetus desires being born, and that’s where the analogy breaks down.

    • mori says:

      A quick google tells me some dogs are as intelligent as infants. If you eat meat, you cause comparable deaths.

      • Richard says:

        I got a long comment about animal welfare below, but the counterargument about eating meat is that the alternative for the animal is not eternal life, but often a rather uglier death.

      • Anonymous says:

        How often do you think people eat dogs?

      • Cauê says:

        I’m sure you’ll get different results if you compare what’s being lost in each case in terms of future life experiences.

      • Deiseach says:

        I don’t eat dog (not knowingly, though after the horsemeat scandal, qui sait?)

      • BrowncoatJeff says:

        That would be comparable iff dogs were known to, in almost every case, grow to be as intelligent as an adult human if we refrain from killing it for 18 years.

        Also, I think from an intuitive, in-group loyalty, and evolutionary point of view there is a bright line difference in our moral obligations to member of our own species and members of any other species.

        • Matt says:

          I don’t see much sense in speaking of morals with species which are utterly incapable of recognizing them.

          • houseboatonstyx says:

            Most mammals do follow the same general set of behaviors as most humans do. ‘Recognition’ of ‘morals’, seems, well, a variety of phlogiston.

      • Shenpen says:

        The what? I think I need to find a name for this, this mistake is made so often. I will call it the Demand Fallacy.

        The Demand Fallacy is assuming that customers are responsible for unethical business practices of vendors, because they generate the demand for it. However, one could say just as well that customers buy those products because the supply exists. It is a two-way relationship, butcher sell meant because customer buy it, but customers buy it because butches sell, because it is offered, available, there. Butchers would not kill animals if ethical consumers would reject it, but also consumers were not able to buy meat if ethical butchers rejecting to do the work.

        The fallacy is assuming that butchers or other vendors are completely at the mercy of financial forces, if customers would want to buy baby meat or their own left hand, they would have to provide that, too, because they desperately need money. This is not actually so. It is not horribly hard for butchers to find some other job. In fact, finding a different source of income sounds easier than giving up your favorite foods. Money is fungible and does not matter much where it comes from, but only steak tastes like steak. (Let’s assume the butchers are a not actually enjoying their job. Shudder.)

        Since there is a mutual causation, the majority of the responsibility falls on those who actually do the killing: the butchers. The customers have a limited responsibility.

        Thus, eating meat does not cause the death of animals. Killing animals to make money off the meat eaters, is what causes the death of animals.

        This is my rebuttal of the Demand Fallacy, or the blame-customers-first fallacy.

    • Richard says:

      The thing is that immediate and easy access to abortion has been tried out in just about all of Europe and it turns out it doesn’t lead to a large increase in the number of abortions, but it leads to a lot safer abortions. From this side of the pond it seems like a no-brainer.

      • Eh? Abortion laws in Europe are generally more restrictive than those in the US, as most Western European countries have mandatory waiting periods (which are rare in the US and have sometimes been found unconstitutional) and limit abortion to the early weeks of pregnancy (unlike the US).

        • Richard says:

          Right, sorry about my rather ill advised use of the word “Europe”. After some digging, I should have said something like “Northern Europe and most of the former eastern bloc, but definitely excluding anything catholic and especially Ireland which appears to be a raging stone-age misogynistic hellhole.” I will let my self out and continue that particular rant over at ozys place.

          Anyway, the actual ethical point I was clumsily trying to make and that still appears to be valid was that there will be abortions whether you like it or not, and until you can change the demand side they might as well be safe abortions. Much like prohibition really.

          The actual figures I wast talking about are valid for Scandinavia, where the number of abortions after legalisation were roughly the same as the numbers of botched knitting needle attempts before legalisation, but it turns out that this happened in the ’60s which by coincidence is when the pill became available so those numbers are probably irrelevant.

          • Harald K says:

            After some digging, I should have said something like “Northern Europe and most of the former eastern bloc, but definitely excluding anything catholic and especially Ireland which appears to be a raging stone-age misogynistic hellhole.”

            No, this would not be enough. You’ve missed MLD’s point: Even perfectly protestant, perfectly northern, non ex-Soviet, non-hellhole and feminist-friendly countries like mine (Norway) do in fact have significantly stricter abortion laws than the US/UK.

          • Deiseach says:

            Ah yes, the backwardness of my country, that we have not yet permitted abortion on demand! Fret not, Richard, your dreams of a secular paradise on earth shall soon be realised, since once more the abortion campaigning is revving up, our present government already passed a bill permitting limited abortion (and yes, the usual suspects have already attacked it as not enough and called for even more liberalisation) and any day now, we will have legislation along the lines of the U.K. (that’s mainly where we crib our legislation from) and then, oh happy happy days to come! When we shall be as delightful a paradise as the U.K. and the U.S.A. and never no more shall unwanted pregnancies be a thing! Every child shall be a wanted child! No more will social workers for cases of abuse, neglect and the like be needed!

            (Of course, I’m long enough in the tooth to remember when all this was promised to us if only we had legalised contraception. Or divorce. Or civil partnerships. Or – well. Any day now, that secular paradise is just around the corner!)

            Yes, it’s horrible being a woman living in a “raging stone-age misogynistic hell-hole”, but somehow I manage to struggle onwards, onwards to the sweet embrace of the grave.

            (Anyone here hear about the big huge riots and burning of cars in the streets and wholesale destruction of property and maimings, injuries and deaths by the uprising of the enraged masses upon our minister for health recently coming out as gay? No? EXACTLY.)

          • Mercy says:

            “Raging stone-age misogynistic hell hole” is pretty cruel, but between the “keeping a clinically dead woman hooked up to life support because she was pregnant” scandal, the “routinely sawing pregnant women’s pelvises apart without consent in order to encourage more births” scandal and the “actually straight up leaving a woman to die rather than aborting her doomed child” scandal, well, it’s not like it came out of thin air.

            It’s not really fair either that India is so heavily associated in people’s minds with gang-rape, or the British with pedophilia; I’m sure that this reputation is out of step with the lived experience of a great many people of britain, most of whom, perhaps even the majority, have never even met anyone whose childhood was ruined by an encounter with a Tory cabinet minister, Asian Minicab driver or beloved light entertainment figure. Nevertheless it’s hard to sympathize with locals whose first complaint is other people’s opinions, even if those are mostly rubbernecking. Some accidents are hard to look away from.

          • houseboatonstyx says:

            (Of course, I’m long enough in the tooth to remember when all this was promised to us if only we had legalised contraception. Or divorce. Or civil partnerships. Or – well. Any day now, that secular paradise is just around the corner!)

            Unfortunately, instead we have seen the end of traditional marriage; now people are marrying their dogs and their trees. Women are bobbing their hair and wearing trousers. The moon landing hoax is beginning to crack; people are beginning to suspect things. Our young men and women are playing Dungeons and Dragons.

            Right here in River City.

            And women have the vote…. And they’re even starting to talk together in a movie.

          • Deiseach says:

            Damn it, I want to respond at length to what Mercy said, but this entire thread is already unwieldy to where it’s getting unusable, and I don’t know if Scott would be up for a separate post on a topic that would definitely invoke the forbidden “gender” if not “race” topics.

            Exceedingly short rejoinder: blame our crappy underfunded overstretched health system, not the availability or otherwise of abortion. You’re no doubt invoking the Savita Halappanavar case; there are grave deficiencies in Irish maternity services which go a lot deeper than simply “if only an abortion had been carried out in time!”

      • Jaskologist says:

        Is your contention that the abortion rate was lower in the US prior to Roe vs Wade?

        Over here, Kermit Gosnell was able to operate for decades, milling multiple people and having untrained people do procedures because pro-choicers made a conscious decision not to inspect his clinic.

        If you’ve heard of Wendy Davis, her sole claim to fame was opposing a bill which primarily simply required abortion clinics to meet the same health and safety standards required of places which do as little as give stitches.

        • Cauê says:

          Let’s try to avoid getting too tribal? This would just be an argument for holding all clinics to the same standards, and all it’s doing here is trying to attach negative affect to abortion.

          • Jaskologist says:

            Tribes are real things, though, and the they influence the actual shape of the debate. In the US, applying the standards of medical cleanliness to abortion clinics is actually controversial, and is actually opposed by Blue tribe. That is relevant when discussing why Blue tribers favor abortion, and addressing the claim that legal abortion leads to clean abortions.

          • Anonymous says:

            *cough* consequentialism?

          • Cauê says:

            “That is relevant when discussing why Blue tribers favor abortion, and addressing the claim that legal abortion leads to clean abortions.”

            I actually agree, but then one should address that when posting the shocking extreme examples.

        • Kevin says:

          If you’ve heard of Wendy Davis, her sole claim to fame was opposing a bill which primarily simply required abortion clinics to meet the same health and safety standards required of places which do as little as give stitches.

          Incorrect. The contents of Texas Senate Bill 5 include:

          Texas Senate Bill 5 is a list of measures that would add and update abortion regulations in Texas. These measures include a ban on abortion at 20 weeks post-fertilization and recognize that the state has a compelling interest to protect fetuses from pain. The bill would mandate that a doctor who performs abortions have admitting privileges at a nearby hospital, and to require that clinics meet the same standards as other surgical health-care facilities in the state. Another provision would require oversight of women taking abortion-inducing drugs such as RU-486. The bill would not apply to abortions necessary to save the mother’s life or to prevent permanent bodily damage from a pregnancy.

          The claim that fetuses feel pain at 20 weeks is not supported by evidence. It is just intended to reduce access to abortions, as is the hospital admitting requirement. Most hospitals in the South do not want to be associated with abortion clinics.

          • Harald K says:

            “The claim that fetuses feel pain at 20 weeks is not supported by evidence.”

            I’m not familiar with the evidence in detail, but there might be some cultural factors affecting how plausible the evidence is judged in questions like these. Since for instance even Sweden draws the line for on-demand abortion at 18 weeks. In Norway it’s 12 weeks, extendable to 18 weeks on appealing to a medical board.

            Never mind pain, in week 20 you’re getting dangerously close to viability. I knew a woman who was prematurely born before what would have been the UK limit of 24 weeks, and she’s a healthy and well 30-year old today. A 20 week limit is far from extreme enough to insist there must be vicarious motives.

          • Irenist says:

            Kevin (and RCF):

            OF COURSE there’s hypocrisy in pro-life support for these laws. The end is to limit abortion, and the health requirements are just a means. But under the present SCOTUS regime, such hypocrisy is usually required. All that conceded, the regulations aren’t *entirely* groundless:

            1. 20 week bans, regardless of one’s views on fetal pain, just seem to be red states testing what they can do after Gonzalez v. Carhart: they’re not *obviously* out of line with the current SCOTUS abortion regime, although of course YMMV.
            Cite: http://www.pewforum.org/2013/01/16/a-history-of-key-abortion-rulings-of-the-us-supreme-court/

            If upheld, such bans would still be far less restrictive than what prevails in Europe, even Scandinavia–as Harald K. mentioned–so they’d hardly put the U.S. red states outside the First World mainstream.
            Cite: http://www.theatlantic.com/international/archive/2013/08/in-liberal-europe-abortion-laws-come-with-their-own-restrictions/278350/

            2. ACOG, an obstetrical organization deeply opposed to SB5, complained that only 0.5% of abortions result in “major complications,” so the bill was unnecessary. Cite: http://www.acog.org/About-ACOG/News-Room/News-Releases/2013/Ob-Gyns-Denounce-Texas-Abortion-Legislation
            However, if you (or your wife, mother, sister, or daughter) end up part of that 1 in 200 women, I imagine you’d want the doc to have admitting privileges, yes? So I think ACOG’s stat actually argues against their anti-SB5 position.

            3. SB 5 (a.k.a. HB2 in the Texas House) defines abortion clinics as Ambulatory Surgical Centers (ASCs). Cite: https://legiscan.com/TX/text/HB2/id/872841

            That would make them subject to the parts of the Tex. Health and Safety Code that already regulate ASCs, already, before the bill, defined as any “facility that primarily provides surgical services to patients who do not require overnight hospitalization or extensive recovery, convalescent time or observation.”
            Cite: 25 Tex. Admin. Code § 135.2(5), online at: http://info.sos.state.tx.us/pls/pub/readtac$ext.TacPage?sl=R&app=9&p_dir=&p_rloc=&p_tloc=&p_ploc=&pg=1&p_tac=&ti=25&pt=1&ch=135&rl=2

            What are some examples of ASCs? Well, according to the Texas ASC Society, their professional group, “the most common procedures [at ASCs are] cataract removal, colonoscopy, and knee arthroscopy.” Cite: http://www.texasascsociety.org/
            So, I don’t know if Jaskologist’s “just stitches” is an exaggeration, but the gist is correct: surely vacuum aspiration and dilation & curettage are at least as much “surgery” as colonoscopy, yes?

            TL;DR: Jaskologist’s claim is correct: the ASC provision was the core of the bill, and it doesn’t take much in the way of “surgery” to end up regulated as an ASC.

          • Kevin says:

            Harald K: See, e.g., Fetal Pain: A Systematic Multidisciplinary Review of the Evidence. The difference between even week 20 and week 21 is huge when it comes to viability: http://en.wikipedia.org/wiki/Fetal_viability#Limit_of_viability

            Irenist: Jaskologist’s claim was that SB5 “primarily simply required abortion clinics to meet the same health and safety standards required of places which do as little as give stitches.” That is an incorrect summary of the contents of the bill, as shown quite plainly in my previous post. No amount of equivocation on your part will change the fact that the bill contained at least 4 distinct provisions, and Jaskologist deliberately chose to focus solely on the least objectionable one. Your edited post still offers no support for the claim that this provision was the “core” of the bill.

            Also, this argument is not persuasive:

            However, if you (or your wife, mother, sister, or daughter) end up part of that 1 in 200 women, I imagine you’d want the doc to have admitting privileges, yes?

            It assumes that abortion clinics receiving admitting privileges instead of shutting down is a possible outcome for this bill. The decision here is not between safe access to abortion or even safer access; it’s between safe access or no/unsafe access. Context matters.

          • Irenist says:

            Kevin,

            Based on media reports, my impression has been that the ASC provision was at least tied with the admitting provision as the main bone of contention. But whether something is the “core” of a bill is subjective, so I’ll withdraw the claim.

            As for the context: sure, most hospitals in Texas don’t want to give abortionists admitting privileges. So maybe the clinic closes, the abortion doesn’t happen, and the 1 in 200 women with major complications don’t end up with major complications. Now, of course you’ll state that far more women out of any 200 will have major complications from giving birth and carrying children than from abortion. And you will be entirely correct. So, sure, context matters: conceded. But the law isn’t prima facie irrational for mandating admitting privileges–the complications do happen.

          • Jaskologist says:

            Tell you what, I am totally willing to alter my view that the “safe” part of “safe, legal, rare” is more than just rhetoric on the Blue side. Just point me to the legislation Blues have pushed or other efforts they have made to ensure that abortion clinics must follow the same health standards as other such medical centers (ASC, urgent care, what have you).

            I genuinely don’t think it’s out there, but I certainly haven’t performed an exhaustive search.

          • Irenist says:

            Jaskologist,

            I think the main pro-choice argument is that, as the National Abortion Federation puts it, “Illegal Abortion is Unsafe Abortion,” i.e., that keeping abortion legal is in itself a huge contribution to safe abortion on their part. I don’t actually disagree: it certainly seems reasonable that legally regulated abortion will be far safer than black market abortion, just as legally regulated alcohol, narcotics and brothels are, AFAIK, safer than those under prohibition regimes (e.g., bourbon is safer than moonshine). My concern about legal abortion is that it’s unsafe (obviously) for the unborn child, not that it improves the safety of the mother killing her child. I’m perfectly comfortable conceding their “safe” argument in the interest of honest rational discourse. Here’s the NAF page I quoted the slogan from, with their supporting stats:
            http://prochoice.org/education-and-advocacy/about-abortion/abortion-facts/

        • RCF says:

          “If you’ve heard of Wendy Davis, her sole claim to fame”

          Not her sole claim to fame.

          “a bill which primarily simply required abortion clinics to meet the same health and safety standards required of places which do as little as give stitches.”

          I have a high confidence that that is not the case, that some random person on the internet asserting it is insufficient cause for me to counteract that high confidence.

          “pro-choicers made a conscious decision not to inspect his clinic.”

          See above, regarding the proposition that this is an honest analysis of the situation.

          ” In the US, applying the standards of medical cleanliness to abortion clinics is actually controversial, and is actually opposed by Blue tribe.”

          And again.

          “That is relevant when discussing why Blue tribers favor abortion”

          Blue tribers support abortion rights, not abortion.

          • Deiseach says:

            RCF, even an association of abortion providers refused to permit Kermit Gosnell to become a member when he applied, his “clinic” was so appalling when they visited it.

            Now, I know feck-all about American newspapers and their biases. Maybe “The Washington Post” is a raging right-wing conservative misogynist homophobic racist classist publication. I have no idea. But they ran a series on the story back when it was breaking:

            When the clinic was first inspected in 1979, it had a medical director on staff who was a certified obstetrician/gynecologist. The certificate for approval after that inspection expired in December 1980, but the next “documented site review was not conducted until August 1989.”

            By then, Gosnell was the only doctor affiliated with the clinic. At that investigation, the state health department “noted several violations of Pennsylvania abortion regulations” but “based on mere promises to improve documentation and filing” granted the clinic approval for another 12 months. There was a similar investigation in 1993, with no result.

            There were no inspections of the clinic over the next 16 years. The health department received multiple complaints about the clinic, including one from a doctor who said that his patients “were becoming infected with sexually transmitted diseases at Gosnell’s clinic when they had abortions there.”

          • Irenist says:

            RCF:

            Blue tribers support abortion rights, not abortion.

            Nope. Not anymore:
            http://rhrealitycheck.org/article/2014/04/03/abortion-blessing-grace-gift-changing-conversation-reproductive-rights-moral-values/

            Here, for openers, are a few ways we might change the conversation:
            ….
            8. Embrace abortion as a sacred gift or blessing.

            10. Honor women who decide to terminate pregnancies just as we honor motherhood.

          • Nita says:

            @Irenist

            Here is what she says under “blessing”:

            An ill-conceived pregnancy is bad. An unintended pregnancy is regrettable. An abortion when needed is a blessing. It is a gift, a grace, a mercy, a cause for gratitude, a new lease on life.

            Here is what she says under “honor”:

            How often do we affirm and honor the wisdom of women who make difficult childbearing choices (abortion, adoption, waiting) so as to best manage their lives and their parenting?

            This is not at all what I expected after reading your quotes.

          • Irenist says:

            Fair enough, Nita. But those headings were the author’s, too. She actually wrote the words, “embrace abortion as a sacred gift.”

            Now, us orthodox Catholics get mocked for believing that “every sperm is sacred,” which (contra Monty Python), we most certainly do not. (A sperm is a mere gamete; life begins at the conception of a zygote.)

            But if I were to say that RH Reality Check (a respected pro-choice advocacy site) published a column opining that “every abortion is sacred,” I’d be denounced as some right wing loon. But look, they did! The days of “safe, legal, and rare” are waning fast in an increasingly radicalized pro-choice movement.

          • Nita says:

            I think you and the author have very different ideas of what “sacred” means. It seems that to her point is “the gratitude we feel about the possibility of escape from the abject despair of unwanted pregnancy is extremely intense and personal, like a spiritual experience”.

            Now, you can object that the word belongs to religious people and she’s using it incorrectly. But she is talking about abortion rights, like RCF said.

          • Anonymous says:

            Here’s another thing the RH Reality Check article says:

            We are, incredibly, faced with . . . a stunningly antagonistic debate about contraceptive technologies that could make as many as 90 percent of unintended pregnancies along with consequent suffering and abortions simply obsolete.

            ^Yes. Yes we are.

    • Wirehead Wannabe says:

      Well, I personally support abortion on the grounds that it’s morally equivalent to not conceiving a child in the first place. I would consider you to ask why you believe murder to be wrong. My answers are, in order from most to least important, “Because their family and friends will grieve, because people don’t like living in fear of being murdered, and because there’s a sunk cost associated with raising someone from a child then ending their life.” None of these problems apply to abortion, and birth is a strong schelling fence to prevent us from a slippery slope of killing toddlers and teenagers.

      • roystgnr says:

        The “sunk cost” answer sounds like a fallacy to me, and although the previous two answers don’t disallow abortion, they also don’t disallow a mass hobo grave in your cellar. (as long as you pick the loneliest hobos and kill them by surprise, which aren’t great mitigating factors to me)

        We’re left with “birth is a strong schelling fence”, which is much more defensible. There’s a continuum between killing a single-celled organism and infanticide, and the moral values we want to map it to are discrete, so there’s got to be a nonsensical discontinuity *somewhere*. The trouble is that once you’re willing to admit nonsensical rules because they happen to also be strong schelling fences, it turns out that “conception” also qualifies, so now everyone’s fighting again.

        • haishan says:

          And then people start proposing increasingly weak Schelling fences in attempts at compromise, which has the effect of turning the debate into a tug-of-war about THIS FENCE SHOULD GO HERE, WOMAN-HATER! NO, HERE, YOU BABY-KILLER!

          • Nita says:

            No, the precarious position of the fence is a good thing because it allows you guys to incorporate science and tech developments in future legal decisions.

            If you build really sturdy fences, eventually you might end up in the future equivalent of modern Saudi Arabia.

          • Brad says:

            >If you build really sturdy fences, eventually you might end up in the future equivalent of modern Saudi Arabia.

            Whoa now; your causality is all wrong. A strong theocracy might *establish* birth as the fence, but how does having the fence up – for any old reason – lead into theocracy?

          • Nita says:

            Well, in the good old days, people used religion to make fences more sturdy – “don’t even think of moving this fence, only an evil infidel would do that!”

            These days, we have internally coherent ideology and philosophical rationalization. And as soon as we persuade everyone that our fence is not arbitrary at all, but obviously in the only acceptable and logical place, the original reasoning will be forgotten. Our fence will cease to be a Schelling fence and become quite Chestertonian. And you know what people say about touching those…

          • drunkenrabbit says:

            @Nita,

            Right, and now when people consider speaking openly about IQ’s relation to race and gender, people shout “don’t even think of moving this fence, only an evil sexist/racist would do that!”

            These days, we have internally coherent ideology and philosophical rationalization.

            Ha. Modern ideology isn’t any more or less coherent than medieval theology, they’re both internally consistent systems with different priors.

          • Nita says:

            @drunkenrabbit

            when people consider speaking openly about IQ’s relation to race and gender, people shout “don’t even think of moving this fence, only an evil sexist/racist would do that!”

            People don’t just consider speaking about it — they actually do speak about it.

            If by “people” you mean public figures (which is an important subset of people, of course), then yes, there is an issue.

            I don’t support shouting people down, although I’m sympathetic to the concern that scientific findings could be used to “justify” unjust policies (see: history).

            Modern ideology isn’t any more or less coherent than medieval theology, they’re both internally consistent systems with different priors.

            …and that’s exactly what I’m saying.

          • Nita says:

            [“do speak about it” was supposed to be a link to Perceptions Of Required Ability Act As A Proxy For Actual Required Ability In Explaining The Gender Gap]

          • drunkenrabbit says:

            …and that’s exactly what I’m saying.

            I see that now, I didn’t catch it before. Note to self, don’t drink and comment.

            People don’t just consider speaking about it — they actually do speak about it.

            I was referring to in-person, face to face. Obviously a lot of subjects get batted around online that are too sensitive for real life. Maybe my social circle is unusual, but most people I know would never poke the subject with a ten-foot pole.

      • Cauê says:

        Yes, you either have to beat the “friendless hobo killed painlessly in his sleep” scenario, or bite that bullet.

        • Andrew says:

          No, it’s already been done.

          “because people don’t like living in fear of being murdered”

          You can’t allow killing off hobos without making everyone live in fear of becoming a hobo, and then being killed. Everyone is a _potential_ hobo (though some more than others).

      • Jaskologist says:

        Schelling points make for terrible moral arguments, triply so when the point itself is highly in dispute, as it is here. How sympathetic are you to past societies which have variously designated foreigners, women, children, Jews, or blacks as non-persons? All of those are Schelling points; why would you expect the future to judge you any less harshly for your own?

        • RCF says:

          “Schelling points make for terrible moral arguments”

          What utter nonsense. Morality is all about Schelling points. “Theft”, for instance, is violation of the Schelling point of “these are the rules for who owns what”.

          “triply so when the point itself is highly in dispute, as it is here.”

          If there is not a consensus about the point, then it’s not a Schelling point. By point out that non-Schelling points are bad, you’re simply supporting the idea that Schelling points are good.

          “How sympathetic are you to past societies which have variously designated foreigners, women, children, Jews, or blacks as non-persons? All of those are Schelling points”

          “Don’t kill anyone” is quite clearly, from an objective standpoint, a better candidate for a Schelling point than “don’t kill any non-Jews”. I really don’t see how you think this supports your thesis.

          • Irenist says:

            “Don’t kill anyone” is quite clearly, from an objective standpoint, a better candidate for a Schelling point than “don’t kill any non-Jews”. I really don’t see how you think this supports your thesis.

            Agreed. So don’t kill anyone. Not even the unborn.

            Now, I assume you mean that birth is a better Schelling point than race. Sure.

            But let’s say you want to run an evil, exploitative slave economy. Skin color is a pretty convenient Schelling point to administer.

            Okay, now let’s say that you want to use people’s bodies as raw materials for biotechnology, without any pesky concerns about consent or human dignity or whatever. Fetal stem cell research, right? Very convenient Schelling point.

            I’ll be over here with “don’t kill anyone.”

          • Andrew says:

            “Don’t kill anyone” clearly doesn’t work if “kill” isn’t interpreted with some qualifiers. Obviously we should kill people who are currently running around with AK47s shooting into crowds when that’s the most effective way to stop them.

            The killing in such cases is, of course, incidental to some other objective (viz. stopping the killing spree).

            In the case of abortion there is also another objective to which killing the fetus is incidental (viz. preventing the would-be mother from becoming so).

            You can certainly argue that the latter objective is unworthy while the former is worthy, but you cannot object on the basis of an absolute opposition to killing unless you also adopt a pacifist stance with respect to various other forms of killing which are more-or-less universally accepted.

          • Irenist says:

            Andrew:

            Everything you say is correct. I was attempting to turn RCF’s motto against RCF’s argument. But for the reasons you mentioned, the motto doesn’t work in any case.

        • Jon Gunnarsson says:

          Can you give any examples of societies that designated the groups you mention as non-persons? Certainly these groups have been legally discriminated or considered inferior in some societies, but that’s not at all the same as being thought a non-person.

          • Irenist says:

            Deleted. See the comment below for the rewritten version.

          • Irenist says:

            @Jon Gunnarsson:
            Well, in the current SCOTUS legal regime around abortion in the U.S., the state has some interest in the rights of the unborn, which increase from the first (where they are minimal) through third trimesters. Thus, the unborn are treated not so much as non-persons as “sub-persons.” This, of course, is essentially the same concept as the Nazi shibboleth of the “untermensch,” or “subhuman.”

            Here’s Himmler on “Der Untermensch”:
            “Although it has features similar to a human, the subhuman is lower on the spiritual and psychological scale than any animal.”

            Now, “lower than any animal” sounds to me like the difference between “subperson” and “nonperson” is kind of just a semantic quibble. So there’s an example: the Nazis.

            BTW: The Nazis applied their subhuman category widely, to Jews, Slavs, gays, Roma, people of color, etc. They were uniquely ghastly. But the idea that some H. sapiens (the unborn, the brain-damaged) are less worthy of care and concern than “any animal” has not left us.

            ETA: Another expression the Nazis used a lot was “useless eaters” (unnütze esser)–the idea being that subhumans not doing slave labor in agriculture, coal mining, chemicals, or armaments weren’t contributing anything to the war effort. And since they weren’t people, the logical thing to do with these “useless eaters” was to kill them or starve them so the useful eaters could have all the food in occupied Europe, what with the Allied blockade-induced food shortages in the Nazi empire. Adam Tooze’s “The Wages of Destruction” book on Nazi economic policy (and its deep implication in the Holocaust) is the go-to reference here.

            Anyhow, it was all very “Freakanomics,” in a way–lots of economic benefits to snuffing out those subhumans.

            ETA2: Apparently RationalWiki has an article about conspiracy theorists who think the New World Order thinks we’re all unnütze esser or something. That’s not at all what I’m talking about. Just about Tooze’s analysis of WWII.

          • Jon Gunnarsson says:

            I don’t think that fits. People who think of foetuses as non-persons (who can therefore be killed) don’t try to exterminate foetuses, or torture foetuses, or publish angry pamphlets portraying foetuses as vile and depraved.

            The Nazis didn’t think of Jews and other “Untermenschen” as non-persons, but rather as persons who are greedy, devious, traitorous, lecherous, etc. and therefore had to be opposed by any means necessary. No one can muster up this kind of hate for a non-person.

          • Jaskologist says:

            “It warn’t the grounding—that didn’t keep us back but a little. We blowed out a cylinder-head.”

            “Good gracious! anybody hurt?”

            “No’m. Killed a n***er.”

            “Well, it’s lucky; because sometimes people do get hurt.”

            The Adventures of Huckleberry Finn

            (Feel free to use the term “subhuman” rather than “non-person.” It’s the concept that interests me, not the specific terminology.)

            (Also, the spam filter censored Huck Finn! I really should have expected that.)

          • Irenist says:

            @Jon Gunnarsson:

            As befits crazy murderous racist villains, the Nazis were ideologically inconsistent. They made a big deal out of the idea that their targets were subhumans, subjecting them to unanesthetized medical experiments, having to defecate in their clothes on forced marches, and other such dehumanizing, bestializing treatment.

            OTOH, the Nazis’ targets of course WERE people, so the Nazis went ahead and passionately hated them the way people hate their human enemies. This reveals an inconsistency in Nazi ideology, but doesn’t change the facts of that ideology.

            Now, you’re right that our society doesn’t get that worked up about unanesthetized cruelty to the unborn (including way past even the most conservative scientific estimates of when fetal pain is certainly possible), just as we don’t passionately hate the livestock we factory farm. In both cases, we torture out of indifference, not hatred. In that regard, our present regimes are perhaps rather more like the antebellum South, or various medieval and ancient societies, where lesser folk weren’t seen as enemies, just as disposable, and occasionally needing condign discipline regardless of whether one felt any passion about the ugly business. (Such passion being ungentlemanly in any case.)

            So sure. The Nazis at least saw their victims as human enough to be worth passionately hating, whereas we don’t even have that much regard for the unborn. You win, I guess?

          • Anonymous says:

            (Also, the spam filter censored Huck Finn! I really should have expected that.)

            Does it also censor Sherlock Holmes, because Watson habitually ejaculated at the breakfast table? And Jane Austen, whose characters often made love in public?

    • Muga Sofer says:

      I’m in pretty much the same position as you; and honestly, I have very strong suspicions that being anti-abortion is the “natural” Blue position. (Which is not the same as the true position, although I think it’s that as well.)

      The reason the Blue tribe is associated with being pro-abortion is coalition politics; pro-abortion campaigners successfully allied themselves with feminists, and pushed the idea that the anti-abortion position was analogous to the anti-gay position (that is, largely motivated by religion and gender roles), which got pro-religion and gender roles people to form an opposing coalition.

      This isn’t all that uncommon; libertarianism is also a clear fit for Blue values, but ended up being a major plank of the Reds. On the other hand, feminism has a history of allying itself with Red values that are eventually expelled (TERFs, that whole anti-porn thing, PIV-critical feminism, etc.)

      On the other hand, I think the … most appealing, if not the strongest … Blue argument on this front is the appeal to science-fiction values. Just like an alien or a robot is only counted as a person if it achieves “sentience” (the intelligence level of an adult human – roughly equivalent to a soul), so we should apply the same standard to our own infants.

      The obvious objection to this is that actually, we value our children a great deal even before they become adults, for obvious evolutionary reasons.

      The entire idea of “sentience” is a pretty clear attempt to simplify away our actual values into something easier to do utilitarian sums with (as I would argue the case of animal rights proves, but if you don’t buy that, then developmentally disabled people.) But it’s still very popular, especially among Blue-tribers.

      Some people have actually bitten this bullet and endorsed baby-eating. In particular, there are cases of abortionists trying “post-natal abortion”, which is fairly horrible. So … hey, that way lies consistency, folks, if you’re interested.

      • Muga Sofer says:

        Oh, and the best way to reduce abortion is to reduce the economic conditions that create it! Solve poverty, and you solve a lot of things.

        • Jaskologist says:

          Do we see lower abortion rates in blue states?

          • Nornagest says:

            Answer appears to be “no”. The highest rates in 2008, according to CDC stats, were in:

            Delaware (40.0)
            New York (37.6)
            New Jersey (31.3)
            Maryland (29.0)

            …and the lowest were in:

            Wyoming (0.9)
            Mississippi (4.6)
            Kentucky (5.1)
            South Dakota (5.6)

            All given annually per 1000 women. Wyoming’s is so low that I suspect we’re looking at bad data of some kind, but I’m willing to trust the others. I don’t think this is a straightforward matter of ideology — the highest-scoring states are strongly blue, but all the lowest-scoring ones but Mississippi are only middling red — but it isn’t economic either; Wyoming is rich per capita and South Dakota is middling, but Kentucky’s one of the poorest states in the Union and Mississippi is the poorest.

            (Let the record show that the strength of these results surprised me.)

          • LTP says:

            We must keep in mind the ease of access of abortion, though. It may be that many people travel from red states where it is hard to get an abortion (due to a mix of tending to be more rural and having the number of clinics being de facto restricted) to blue states where abortions are easy to get.

          • Nornagest says:

            Yeah, on further investigation I’m pretty sure that’s what’s driving the Wyoming numbers: there were no abortion clinics in Wyoming in 2011 (there were three abortion providers), so most women in need of an abortion are presumably traveling out of state and being counted there.

            Similar stuff might be happening in some of the other lowest-scoring states. But I’d only expect this to mangle the statistics so much, and I still see what looks like demographic differences (and still don’t see obvious economic differences) if I throw out the lowest-scoring states.

        • Anonymous says:

          I’m not sure about that. Rich countries have hugely varying rates.

        • Poverty is not the only cause of abortion– for that to be true, women would have to want to bear and raise every child they conceive and can afford.

          The stereotype when abortion was illegal was that better-off women would travel to somewhere that they could get a safe abortion.

          The truth is that not everyone wants children. Among people who want children, not all of them want a lot of children.

      • Nita says:

        “sentience” (the intelligence level of an adult human

        Either your definition of “sentience” or your definition of “adult” is extremely unusual. Pretty much everyone agrees that 5-year-old kids are sentient. I think cats and pigs are sentient.

        It’s still legal to euthanize your cat. I believe it’s unethical to euthanize a cat if someone is willing to take it off your hands and give it a good home. This is also true of most newborn children. But embryos cannot be adopted that way.

        Oh, and the best way to reduce abortion is to reduce the economic conditions that create it! Solve poverty, and you solve a lot of things.

        Well, then perhaps you should advocate for solving poverty, rather than for banning abortion?

        • Muga Sofer says:

          >Either your definition of “sentience” or your definition of “adult” is extremely unusual. Pretty much everyone agrees that 5-year-old kids are sentient. I think cats and pigs are sentient.

          >It’s still legal to euthanize your cat. I believe it’s unethical to euthanize a cat if someone is willing to take it off your hands and give it a good home. This is also true of most newborn children. But embryos cannot be adopted that way.

          You’re right about the 5-year-olds; although when people start setting out their definitions they tend to be excluded. That’s a problem with the definitions, though.

          But I think you’re unusual if you believe cats and pigs are exactly as much “people” as adult humans. (Though I’d argue we have a responsibility to look after cats (e.g. in shelters) until we can find them a home, no? That’s why most people have such strong objections to the fact most shelters euthanize pets. )

          It seems like you’re laying out a different ethical framework, though, rather than defending the “embryos aren’t people” argument? Arguing that actually, many people should be killed if we can’t improve their lives is different to arguing that killing and embryo isn’t really killing. (I think this argument is probably stronger, but less persuasive/popular; I’d be very interested to see the full version.)

          >Well, then perhaps you should advocate for solving poverty, rather than for banning abortion?

          I usually do, yeah.

          • Nita says:

            You’re right about the 5-year-olds; although when people start setting out their definitions they tend to be excluded

            I think you might be confusing sentience and sapience (people do that a lot), unless you think 5-year-olds lack subjective experience.

            But I think you’re unusual if you believe cats and pigs are exactly as much “people” as adult humans.

            I don’t believe that. I think human children surpass cats before they attain full personhood. So, an adult cat is less of a person than a human 2-year-old.

            You’re right, though — I am unusual in that I actually try to stick to “science-fiction values”, because they seem more fair than the alternative (i.e., “those who are more closely related to me should get more rights”).

            That’s why most people have such strong objections to the fact most shelters euthanize pets.

            I don’t think most people have such objections. For instance, look at the most popular reviews of a book arguing for no-kill shelters. And these reviews are by people who really care about animals.

            Arguing that actually, many people should be killed if we can’t improve their lives is different to arguing that killing and embryo isn’t really killing.

            It’s killing for sure, but is it murder? I’m arguing that it’s ethical to kill even a healthy human embryo if it’s unwanted by the person whose body it’s using. Because the only alternative we currently have is forced pregnancy and childbirth, which can be equivalent to torture.

            Now, if we collectively decide that people should be forced to donate organs or blood to victims of their traffic accidents, I’ll have to rethink my position.

          • MugaSofer says:

            >I think you might be confusing sentience and sapience (people do that a lot), unless you think 5-year-olds lack subjective experience.

            I was deliberately confusing them, because I’m using the term in it’s classic science-fiction sense.

            >It’s killing for sure, but is it murder? I’m arguing that it’s ethical to kill even a healthy human embryo if it’s unwanted by the person whose body it’s using. Because the only alternative we currently have is forced pregnancy and childbirth, which can be equivalent to torture.

            >Now, if we collectively decide that people should be forced to donate organs or blood to victims of their traffic accidents, I’ll have to rethink my position.

            Oh.

            I’m not sure what the analogy to euthanizing cats is, then.

          • Nita says:

            A cat, a newborn baby or an embryo depend on you for their survival. But you really, really can’t handle that right now. [In my actual ethical reasoning, I also consider the fact that the embryo is growing inside your body and manipulating your physiology, but we’ll skip it here for the sake of the argument.]

            (A) If you can find someone else willing and able to take care of them, you must transfer the responsibility to that person.
            (B) If that is impossible, you may abandon/kill your dependent.

            All babies in Western countries fall into case A, all human embryos currently are case B. Cats are a mixed bag.

      • Jaskologist says:

        Abortion is a necessary for other Blue positions to be viable.

        Unfettered sexual activity produces babies. In theory contraceptives should prevent this, but in actual practice you still end up with a lot of babies.

        And once you have a baby you can’t party it up anymore. You need to settle down and get to work, or, if you chose a mate poorly, get on the dole (but a child will still cramp your style).

        So, if you want to make sure people don’t get married young, but do have lots of sex, a baby is a problem. Remove the baby and you remove the problem.

        • Cauê says:

          I like this, but only if it’s clear we’re not talking about a conspiracy, but of the affect heuristic and policy debates feeling like they should be one-sided.

          • Jaskologist says:

            I don’t see how it’s conspiratorial. “X is good, and Y is needed for X, so Y is good” is perfectly normal reasoning.

            The conspiracy version is: abortion encourages women to make unwise choices which ultimately result in more unwed mothers, which means more people dependent on the state, which means more guaranteed blue votes. But even that could be reframed as a case of Molochian selection, where those who want more safety net *and* lifestyles which cause people to rely more on the safety net outcompete those who just want the safety net.

          • endoself says:

            How does that actually work as selection? Politicians who cause there to be more people relying on the social safety net benefit all politicans who support the social safety net about equally. You get more support for the social safety net, but the same amount of support for policies that increase the number of people dependent on in, ceteris paribus. Evolution requires differential fitness.

          • Think memetic evolution; you’re right that it does little to help one politician over another with similar views.

        • John Schilling says:

          This. To Blue Tribe, sex is the moral equivalent of a pick-up basketball game, something that all the cool people do because it’s cool and fun, and when you’re done the only consequence is or ought to be that you had a fun time with your cool friends. To Red Tribe, sex is serious business, intrinsically bound with love as part of the trinity – love, marriage, parenthood – that defines the greatest part of most people’s lives. Each sees the other side’s attempt to tie sex to their values as abhorrent.

          I’m a Red-leaning Grey in this respect; even if we conquer biology outright, there’s an emotional component to sex that insists on escaping any “just harmless fun” box you try to force it into. And, yes, for most people marriage and parenthood do trump all the wild oats you could ever hope to sow. But you’re welcome to try either way, so long as you’re willing to live with the consequences.

          Until we conquer biology, some of the consequences may look uncomfortably like dead babies, and Blue Tribe would be well-served by ways of addressing that which can’t readily be reduced to “because sexual freedom!”

          • Salem says:

            This is an extremely perceptive comment. By which I mean I agree 🙂

          • Anonymous says:

            Until we conquer biology, some of the consequences may look uncomfortably like dead babies, and Blue Tribe would be well-served by ways of addressing…

            And, just who do you think has been standing in the way of, as you put it, “conquering biology”? Who has traditionally been against access to contraception?

          • Sparky says:

            Just about 60% of women who get abortions in the US already have one child, and just a little over half of the women in the US who get abortions are over 25 years old. This suggests motivations that might be more inline with traditional “family planning” than living wild and free for a good chunk of those having abortions. I think that even if a more traditional sexual ethos were prevailing in the US, there would still be a place for abortion. It might be a much less tribal issue were this the case, arguably.

            (The above figures are from a study performed by CDC and the Guttmacher Institute back in 2008.)

          • Leo says:

            “Sex is a fun hobby; insisting it must be a big deal won’t make it so” vs “Sex is an eternal contract; insisting it shouldn’t be a big deal won’t free you” seem true of different people. I tend to call them sluts and prudes for short.

            I can’t think of any demographic correlates for that personality trait. Leftists (resp. rightists) tend to think being a slut (resp. prude) is morally correct, but I’m unconvinced it matches their actual psychological needs.

            That said, as Sparky mentioned, this seems only tangentially related to abortion.

          • drunkenrabbit says:

            @Anon,

            There haven’t been any meaningful restrictions on access to contraception in decades. Clearly that wasn’t the conquest we needed.

          • houseboatonstyx says:

            There haven’t been any meaningful restrictions on access to contraception in decades.

            Cost, side-effects, and permanency are, in effect, restricting wider use of current contraceptives. Side-effects and permanency are problems for medical research, and so is some part of the cost.

        • Kevin says:

          Unfettered sexual activity produces babies. In theory contraceptives should prevent this, but in actual practice you still end up with a lot of babies.

          Most unintended pregnancies occur due to a failure to use contraception, or a failure to use it correctly. The pill, condoms, and other short-term contraceptive options are highly susceptible to human error. Long-lasting contraceptives have failure rates from 0.5% (IUDs) to 0.05% (Implanon/Nexplanon). Large-scale adoption of long-lasting contraceptives would dramatically lower the number of unintended pregnancies and correspondingly the number of abortions.

          • houseboatonstyx says:

            Large-scale adoption of long-lasting contraceptives would dramatically lower the number of unintended pregnancies and correspondingly the number of abortions.

            As would (will) development of better contraceptives, and easier access to the existing ones (eg Brazil).

            Development of really convenient and affordable contraceptives has been found controversial, and not tried.

          • Jaskologist says:

            That may well be true, but it’s not the world we live in, nor the one we’ve had for the whole time abortion has been legal. Until such time as the contraceptives you describe really do become widely used, abortion is necessary to make the liberal sexual ethic possible.

          • Deiseach says:

            Like I said, I’ve been around for the “people are going to have sex anyway, that means babies, want to reduce or prevent unwanted pregnancy then permit contraception” argument.

            We got legalised contraception in Ireland. At the moment, I don’t think we can go any more liberal unless we start giving eight year old children in primary school implants.

            Still got a lot of unwanted pregnancies. Oh, you mean contraception is not a magic solution and when it comes down to it, you need to instill a sense of personal responsibility into people to make sure they use it correctly? Stop imposing your mediaeval religious bigotry on us, hater!

          • Kevin says:

            Yes, comprehensive sex education is a necessary part of contraception policy. I don’t think you’ll find many liberals who deny that. But again, a move to long-lasting contraceptives would eliminate most human error. There’s no way to use an implant or IUD incorrectly once you have it.

            I personally would support mandatory birth control implants for everyone at puberty, given the availability of contraceptive technology without significant contraindications (which we don’t quite have yet for women and, unfortunately, definitely don’t have for men). Making it a deliberate choice to be able to get pregnant would help change how society views sex and pregnancy.

        • Anonymous says:

          This is why Playboy has always been a major support of pro-abortion organizations.

        • ADifferentAnonymous says:

          There is much truth in this, but I think you overstate some things. First, I’m not sure the single swinger partier thing is as central to blue values as you make it sound; I don’t think it’s all that rare to be thoroughly blue and consider that lifestyle a bad idea on pragmatic grounds. Second, I don’t know the stats but I’d bet elite blue contraceptive use is actually quite reliable. Lastly, your tone makes it sounds like a pro-life argument, but you can just as easily turn it sounds and say “banning abortion enforces red sexual mores”. I think that’s exactly what feminists do say…

          Also, experiment: is there a surprising amount of pro-lifism among PIV-critical feminists? An abortion ban would be a huge boon to their position, so if people are choosing convenient beliefs you’d expect a strong effect on the margins there.

          • Jaskologist says:

            I think the symmetrical case does hold up; banning abortion enforces red social mores.

            As for the rest, there are numerous Douthat columns on how the blue elite practices a far more conservative sexual ethic than they preach to the masses, who are less able to bear the costs of that lifestyle.

      • Cauê says:

        “Is it sentient?” (or “a human life?” or “a person?”) in this context is working too much like “is it a blegg?”

        http://lesswrong.com/lw/nm/disguised_queries/
        http://lesswrong.com/lw/nn/neural_categories/
        http://lesswrong.com/lw/no/how_an_algorithm_feels_from_inside/
        http://lesswrong.com/lw/oh/righting_a_wrong_question/

        We have this rule about not killing bleggs, but being ok with killing non-bleggs, and sometimes we get stuck arguing whether something is *really* a blegg, instead of considering the purpose of that rule, and what is being gained or lost by each decision in that particular case.

        • Nick T says:

          Tabooing a concept is qualitatively harder when that concept directly appears in your best current statement of your preferences. (Some rules are their own purposes.)

          • Cauê says:

            I expanded on this below.

            The concept is not in my best current statement of my preferences in this case, and I suppose you could say I’m trying to change other people’s statements.

    • As a utilitarian conservative, I agree with abortion on the grounds of the Freakonomics argument. I’m tired of the stark left/right divide on this issue and I do think there is room for middle ground.

      “parents who get abortions are parents who can’t afford children, so it’s saving them from a bad life” or the Freakonomics argument that abortions lower crime rates sound creepy and eugenics-y, and not at all in line with Blue/Grey values as I understand them.

      • Emile says:

        I also find that argument perfectly fine, and don’t consider “eugenics-y” as a point against abortion.

        • Jaskologist says:

          It overlaps with eugenics a bit, but really what you’re discussing is Pre-Crime, where the only punishment is death. If you are comfortable with executing people who are statistically more likely to commit crime, by all means, continue.

          Philip K. Dick would like to have a word with you, though. And maybe also the recent anime Psych-Pass.

          • Emile says:

            Nah, that only works if you count “killing a fetus” as murder; the whole abortion debate is about “where do we draw the line”, you can’t assume an answer to that. And the Freakonomics argument is one (more) reason to draw the line at a place that allows abortion, for pragmatic reasons.

          • Jaskologist says:

            Whether we count abortion as murder is the debate. Freakonomics isn’t side-stepping the debate, it’s just saying “if it reduces the crime numbers, we’re not going to count it as murder,” which can cut a lot of other ways.

            How do the Freakonomic “murder has reduced” numbers look if we count abortions as murders, incidentally?

          • houseboatonstyx says:

            It overlaps with eugenics a bit, but really what you’re discussing is Pre-Crime, where the only punishment is death. If you are comfortable with executing people who are statistically more likely to commit crime, by all means, continue.

            Is that really the argument? Crime is increased by poverty and stress within families, and many families may be tipped into that by an unwanted child.

            Or, more abortions and fewer crimes may both be effects of a common cause, such as leftist policies including more welfare and social services.

          • Emil says:

            [Freakonomics argument] is just saying “if it reduces the crime numbers, we’re not going to count it as murder,”

            No, “it reduces crime numbers” is one reason against counting abortion as murder, but no-one’s claiming it’s a *sufficient* reason.

            I don’t think you’re arguing in good faith here (or even arguing for what you believe as opposed to just poling arguments in things for the sake of being contrarian or for the lulz or something).

      • Anonymous says:

        Alas, it’s wrong. The crime drop they point to can not have been caused by legalizing abortion, because one can commit crimes at any age. To prove that it did, you would have to show crime dropped in the cohorts culled by abortion, rather than the older ones.

        And it’s the other way round. The culled cohorts had a higher crime rate, offset in the total by the decline in the older cohorts.

      • Anonymous says:

        That arguement has some holes. Look up Steve Sailor’s reply.

    • Anonymous says:

      Would the same then not apply in medicine research and social aid? If we were to accept that the unborn are as valuable as the born, and need someone to speak on behalf of them, wouldn’t we have to put a huge amount of resources into miscarriage and stillbirth prevention- research, medical care, excellent living conditions for pregnant women?

      In my experience, blue efforts on behalf of the unborn tend to go into campaigns and work in maternity care and miscarriage care.

    • Deiseach says:

      I’m not going to touch this becuase it’s toxic. Example: recently had to unfollow someone on Tumblr because they reblogged a pro-choice post re: anti-abortion legislation in (I think) Michigan, and the amount of frothing at the mouth about “foetuses” as if human foetuses were some kind of horrific alien tapeworm infestation, including “helping non-sentient foetuses at the expense of born sentient people” had me going “I can’t read this because I’m going to explode with anger, start yelling insults, that’s not going to help, and the attitude on show is so entrenched there’s nothing I can do to change it”.

      So unfollow it was.

      What would you say to, or how would you handle, someone who says your anti-choice attitudes are a lingering effect of the Catholicism which you haven’t adequately shaken off, and you just need to educate yourself more, elevate your consciousness, and unless you possess a uterus shut your big fat mouth on the topic?

      • Nita says:

        Well, the humans in question are fetuses — more developed than embryos, but not infants yet. Tapeworms, on the other hand, do not have a fetal stage of development at all.

        • Deiseach says:

          Yes, but the tone was as if “foetuses” were some horrific invasive alien things not even tangentially human. It was like a less fun version of The Brood.

    • Loki says:

      Abortion is interesting as it’s one of the areas where my thinking is conspicuously un-Utilitarian, possibly, and it may approach the level of a Sacred Value. Basically, I have two hypotheses for why I believe what I believe (I am a largely-blue tribe lefty intersectional feminist):

      #1: While I seem to use utilitarian or at least consequentialist reasoning elsewhere, I actually consider bodily autonomy a Sacred Value. This explains why I feel that I would support legal abortion even if it were proven that a fetus is morally equivalent to a person – a person has an absolute right to decide who gets to be inside their body and may use deadly force to enforce this right.

      #2: I am actually using consequentialist reasoning, but I consider being forced to undergo pregnancy and birth against one’s will a form of torture that is worth more ‘negatons’ (imaginary opposite of utilons) than the murder of an unborn person. This may or may not be a Typical Mind Fallacy thing that stems in part from my own utter horror of the concept of being forced to endure my body warping with the weight of an unwanted living thing growing inside me, making me feel ill and sapping my nutrients until it bursts free in an hours-long painful ordeal, likely ripping one of my favorite body parts and leaving my body permanently scarred and marked by the experience, and also there is a risk it will kill me.

      #3 I am actually using utilitarianism correctly, and I believe the statistically likely value of the life of an unwanted child, if positive, is not large enough to outweigh the negative utility of forced pregnancy and birth.

      • Randy M says:

        So the rightist slur that abortion is sacramental to progressives is not, actually, that far off.

        • Anonymous says:

          The plural of anecdote is not data. That conclusion can not be accurately made from this single instance.

        • RCF says:

          Because of the phrase “Sacred Value”? You’re wildly misinterpreting that.

          • Deiseach says:

            From a 2007 speech by Katherine Ragsdale, Episcopalian clergyperson, and until recently Dean and President of the Episcopal Divinity School:

            “When a woman finds herself pregnant due to violence and chooses an abortion it is the violence that is the tragedy; the abortion is a blessing.

            When a woman finds that the fetus she is carrying has anomalies incompatible with life, that it will not live and that she requires an abortion – often a late-term abortion – to protect her life, her health, or her fertility, it is the shattering of her hopes and dreams for that pregnancy that is the tragedy; the abortion is a blessing.

            When a woman wants a child but can’t afford one because she hasn’t the education necessary for a sustainable job, or access to health care, or day care, or adequate food, it is the abysmal priorities of our nation, the lack of social supports, the absence of justice that are the tragedies; the abortion is a blessing.

            And when a woman becomes pregnant within a loving, supportive, respectful relationship; has every option open to her; decides she does not wish to bear a child; and has access to a safe, affordable abortion – there is not a tragedy in sight – only blessing. The ability to enjoy God’s good gift of sexuality without compromising one’s education, life’s work, or ability to put to use God’s gifts and call is simply blessing.

            These are the two things I want you, please, to remember – abortion is a blessing and our work is not done. Let me hear you say it: abortion is a blessing and our work is not done. Abortion is a blessing and our work is not done. Abortion is a blessing and our work is not done.

            I want to thank all of you who protect this blessing – who do this work every day: the health care providers, doctors, nurses, technicians, receptionists, who put your lives on the line to care for others (you are heroes – in my eyes, you are saints); the escorts and the activists; the lobbyists and the clinic defenders; all of you. You’re engaged in holy work.”

            “Blessing”. “Saints”. “Holy work”. If pro-choice people don’t want us knuckle-draggers to think they’re treating abortion as a sacrament, perhaps they should not use “sacred value” language that so easily confuses our low-IQ tiny brains.

        • Loki says:

          As I think Randy M. is getting at, I’m using ‘Sacred Value’ the way Scott has used it before when discussing morality.

          In this sense a Sacred Value is something you don’t think in rational consequentialist terms about because you consider it (or think you consider it) infinitely valuable, or at any rate more important than anything else. It’s generally a bad thing if you are Utilitarian, because it means you won’t support an action that compromises that value for something else, even in a case where that action, on balance, leads to greater positive utility.

          Also I never said I was the Hive Queen of the Blue Tribe either.

      • being forced to undergo pregnancy and birth against one’s will a form of torture

        In >90% of all cases, you cannot claim that you are undergoing pregnancy “against your will”, in the usual sense of that phrase, as it being “against your will” implies that it’s imposed by an external agent. But pregnancy is a consequence of having sex, and you knew this when you chose to have sex. People experiencing the consequences of their freely chosen actions don’t get to claim that they’re being compelled by hostile forces simply because they didn’t specifically intend those consequences.

        • Muga Sofer says:

          We do, actually.

          If you willingly walk down a dingy back alley where you know there’s a risk of rape – no, hang on, that’s unrealistic. If you drink alcohol knowing that there’s a risk someone might try to rape you while you’re tipsy, that is not considered consent if you do in fact get raped.

          • But the rapist is an external moral agent, not an intrinsic part of your biology. A pregnancy that follows sex is not like a rape that follows drinking, it’s like a hangover that follows drinking. A hangover feels really terrible, but you can’t claim that it wasn’t your fault.

            Unless you decide that the embryo is a free moral agent, and claim that an embryo implanting itself in your womb without your explicit consent is in the same moral category as a rapist. In that case, I’ll allow that your model is consistent, though it is also insane.

          • Nita says:

            an embryo implanting itself in your womb without your explicit consent is in the same moral category as a rapist

            How about a mentally ill rapist who can’t be held responsible for their actions? Is it justified to use lethal force against one, even if you wandered into his or her vicinity on your own free will?

            What if every rape not stopped by lethal force involved hours of pain, have a good chance of leaving your body torn* (like childbirth) and result in months of unpredictable after-effects (similar to pregnancy+postnatal recovery)?

            * all the way to your rectum with 4% probability

          • lmm says:

            @Nita That’s an angle I hadn’t thought of, I do appreciate the insight.( But I’m still going to go with no, you’re not justified in escalating to lethal force even in response to (non-lethal) violence)

        • Nita says:

          The external forces here are your physiology, the embryo’s physiology and the law that would punish you and anyone trying to help you.

        • youzicha says:

          Isn’t the external agent in this example the people who ban abortion?

          For an analogy, if you are a farmer and your harvest fails and you starve, it is perhaps a stretch of language to say that you starved against your will. But if the government forbids you from eating the food you grew and you starve, it sounds much more natural.

        • Dre says:

          If I understand what you’re saying I feel like this standard of “responsibility” isn’t applied anywhere else though. If I drive a car when its icy out, hit a patch of ice, flip the car and get injured, nobody denies that the injury was “against my will”. We take all sorts of risks all the time, to say that everybody has whatever happens to them “coming to them” seems like its giving up morality altogether. (I’m not sure if this will help me rhetocirally, but if you want an external agent you can take something like physics)

          Does it matter how much birth control the people were using? If they can push the probability down to less than a car accident on a trip to the store does that make it not “intended”? (These aren’t rhetocial questions, I’m just not sure I understand your standard of “imposed”)

          • John Schilling says:

            If you have zero functioning kidneys after the car crash, we may sympathize with the fact that you are going to be on dialysis for the rest of your life, but we’ll still frown on your going out and taking someone else’s kidney without their permission.

            If you wind up pregnant after the condom breaks, we may sympathize with the fact that you are going to suffer nine months of unwanted pregnancy and childbirth, but we’ll still frown on your killing a baby to alleviate your own suffering.

            Whether the underlying circumstances are described as “taking responsibility”, “against my will”, or “intended”, are mostly irrelevant. The one question that matters here is: Are you in fact killing a baby?

            If your answer is, “I don’t care, I’m justified anyway because…”, then you have set yourself a very high bar to clear.

          • If I understand what you’re saying I feel like this standard of “responsibility” isn’t applied anywhere else though.

            ?!?!?!

            On the contrary, I feel like this standard of responsibility is applied everywhere, and abortion is the bizarre exception where people refuse to accept it. Personal anecdote: driving after a snowstorm, my car slid off the road and destroyed a road sign. I was not only ticketed for reckless driving, but I was also charged for the cost of replacing the sign, all this despite the fact that I didn’t intend to do any of that. And my punishment was just. As an adult, I’m expected to either have the skill to drive my car safely on icy roads, or to correctly judge that I can’t drive safely under those conditions and then to stay home.

            You’re expected to understand the possible consequences of your action and to exercise judgement and prudence, and you’re expected to pay for the results of your actions even if they weren’t what you intended.

          • Nita says:

            @Mai La Dreapta

            Presumably your punishment was entirely monetary, with no risk of long-term physical consequences?

          • haishan says:

            Presumably your punishment was entirely monetary, with no risk of long-term physical consequences?

            It would probably be an awful idea to do this, but if the state decreed that two strokes of the cane was the punishment for reckless driving, the argument still goes through. Singapore allows caning for “causing [an] explosion likely to endanger life and property,” which is something that you could certainly do unintentionally through recklessness. I personally don’t have a problem with judicial corporal punishment in general or with this Singaporean statute in particular, and I’d be kind of surprised if Mai did.

          • Nita says:

            OK, so let’s assume our respective countries have all turned into Singapore overnight (for the sake of the argument). Still, isn’t it a little unfair that only the woman is “punished” for reckless sex-having?

          • Randy M says:

            Drunk driving?

          • Mary says:

            ” but we’ll still frown on your going out and taking someone else’s kidney without their permission.”

            On the other hand, if you donated a kidney to someone else, under no circumstances whatsoever are you allowed to retrieve it without that person’s consent. Even if you were tricked or forced to donate, you do not get to chop up someone else’s body to reassert your control of your own body.

            To bring the analogy even closer.

          • Nita says:

            Mary, I swear I’d let my kidney go. But my uterus is still inside me, and the recipient of my unwilling “donation” is releasing dangerous chemicals into my bloodstream 🙁

          • LTP says:

            “If you wind up pregnant after the condom breaks, we may sympathize with the fact that you are going to suffer nine months of unwanted pregnancy and childbirth, but we’ll still frown on your killing a baby to alleviate your own suffering.”

            So women who are raped and are impregnated from that rape because they had a few drinks at a bar and got roofied should be forced to carry the baby to term? After all, they *knew* there was a non-zero chance of getting roofied, raped, and impregnated if they went to a bar, so they have to take responsibility.

            It’s absurd. If you double up on a condom and IUD, a woman is probably far less likely to get pregnant from consensual sex than by rape.

          • Anonymous says:

            You’re not having an operation done on your uterus. You’ve having an operation done on the baby.

            Anyway, the death rate of women who give birth or even have miscarriages is distinctly lower than that of women who aren’t pregnant.

          • Nita says:

            @Anonymous

            You’re not having an operation done on your uterus. You’ve having an operation done on the baby.

            If separated from my uterus, the embryo or fetus will die anyway. Would you have no moral objections to abortion if it involved removing the embryo completely intact and gently putting it on a soft blanket?

            the death rate of women who give birth or even have miscarriages is distinctly lower than that of women who aren’t pregnant

            Where is that from, and how is it relevant?

        • Drew says:

          pregnancy is a consequence of having sex, and you knew this when you chose to have sex.

          Similarly, bacterial meningitis is a consequence of going out in public.

          The problem is when you jump from “you accepted a risk of contracting meningitis” (true) to “you accepted a risk of contracting meningitis and an obligation to allow the disease to to run its natural, untreated course

          Those two are very different sorts of risk calculations.

          • Anonymous says:

            …and we’re back to the question of whether a fetus has the same moral standing as a bacterium. This is still going to be critical ground; the bodily autonomy argument can’t get us the entire way.

            Think also of the auto accident case above. You accept the risk of causing someone’s serious, lifelong medical condition as a result of your driving. With this, you accept the risk that you will be financially responsible for a settlement intended to treat this person for that condition. Sure, there’s a theoretical alternative – just kill them. But that’s assuming that it’s a moral option. We can’t assume it if it’s what we’re trying to show.

        • RCF says:

          “In >90% of all cases, you cannot claim that you are undergoing pregnancy “against your will”, in the usual sense of that phrase, as it being “against your will” implies that it’s imposed by an external agent.”

          If the government prevents you from getting an abortion, then the government is an external agent imposing its will.

      • Gbdub says:

        @Loki – if Bodily Autonomy is a Sacred Value, are you equally opposed to laws against use of certain substances, bans on euthanasia, mandatory vaccination, and involuntary commitment for mental illness? I think you have to be if it is truly sacred. I think you’d also have to be against mandatory child support / child care and the health insurance mandate, though these are arguable.

        If you are, I disagree but respect that position as defensible and consistent. In practice, a disturbing number of people speak highly of bodily autonomy as it relates to abortion, but then turn around and support bans on smoking and Big Gulps.

        • Pete says:

          Or, indeed, organ sales.

          The “it’s her body” argument is incredibly poor for a number of reasons. The first is, as mentioned, we already put many restrictions on what people can do with their bodies’.

          Secondly, even the staunchest libertarians will adhere to the rule that the right to swing my arm ends at your face. There is an implicit argument made that the foetus has no moral weight (or at least a moral weight that is significantly less than the moral weight of the woman’s bodily autonomy), which is defensible, but not obvious – certainly not obvious enough to simply assume.

          Thirdly, society as a whole is often being asked to pay for abortions. When this argument is tacked on, which it often is even if only implicitly, it stops being just a case of “your body.” It’s “our money.”

          Fourthly, the impact on society should be taken into account in the moral calculus, including, but not limited to, the effect on the Doctor who performs the operation (again, it’s not only your body, the body of the Doctor is being used as a tool for termination).

          I think, overall, the case for legal (and, in fact, freely available) abortion is fairly convincing, but this particular argument, as usually presented, is a poor one.

          • Nita says:

            the right to swing my arm ends at your face

            And the right to swing your placenta ends at my uterine wall?

        • Loki says:

          Most of those actually? I support voluntary euthanasia, legalisation of most recreational drug use, and I think involuntary commitment is done far too much, but ultimately that that (and mandatory vaccination) it can be justified – largely on the grounds that some people are in no state to consent to anything (just as drugged people cannot consent to sex), which means that me in a psychotic break or as a toddler (I’ve been both) was also not competent to choose to remain out of hospital, or to choose the risk of disease over vaccination. In these special cases, we have to act in the incapable person’s best interests as we’re choosing for them either way.

          I was however speculating on the reasons for my own beliefs rather than arguing for Bodily Autonomy as a Sacred Value. In fact, re: the guy below (I’d totally be cool with the ability to sell your organs if we could be certain it was entirely voluntary btw) none of these are arguments for abortion, they are all hypotheses for why I, as a moral intuition, do support legal abortion.

          The Doctor, obviously, is a consenting and in fact paid participant, so no moral qualms there, and your money already goes to protect freedoms and values you might not agree with (mine pays the guys who make disabled people do paperwork while starving until they kill themselves because then they don’t claim benefits). Less divisively, I consider my right to not give birth if I don’t want to about as important as my right to not get shot if I don’t want to, and ‘your’ money also pays for the cops.

          Since neither of us has the ability to move to a country where our taxes will be used exactly in accordance with our values, we have to just keep working to try and get our countries’ policies to reflect our values as much as possible.

          Oh and re: the moral weight of the fetus I would love it if someone tackled one of my other arguments where I pretty much did say ‘I might be okay with it even if it was killing babies’. 2 and 3 in my post certainly did attach some moral weight to the fetus anyways.

      • houseboatonstyx says:

        Sacred value? Coming at it (or perhaps sneaking up on it) from a different side. Just what a foetus/embryo/zygote ‘is’, is a religious issue. But for most USians, separation of church and state is … pretty sacred.

    • Anonymous says:

      The blue/grey side is generally in favor of reducing the *need* for abortion (through various policies, ranging from access to information about contraception to economic policies that help the poor/middle class). Even though they’re not thought about in this way, they are actually demand-side anti-abortion policies.

      You can also look at what happens in countries that DO prohibit abortion altogether: You have forensic vaginal exams, back-alley abortions, and women thrown in prison. You have women dying of complications from pregnancy (yes, even anti-abortion folks aren’t for that, but it turns out to be very hard to legislate around).

      • Menno says:

        Even though they’re not thought about in this way, they are actually demand-side anti-abortion policies.

        In that case, everyone is in favor of reducing abortion. No side thinks that their policies are detrimental to the poor and middle class. It’s a little misleading to only categorize one group as working on the demand side.

        • Nita says:

          In that case, everyone is in favor of reducing abortion.

          Uh… of course? An unwanted pregnancy is a negative experience no matter how soon and in what way it ends. What the “pro-abortion” side would like best is affordable, available and effective contraception, not more abortions.

        • Anonymous says:

          Empirically, everyone *is* in favor of reducing abortion. Just look at the reaction to the recent declines in abortion rates – both Planned Parenthood and anti-abortion activists want to claim responsibility!

          And of course one one thinks their policies are detrimental to the poor or working class. They just managed to think this in different ways.

    • Tarrou says:

      Ethically and legally speaking, there is a question that needs answering which then solves our abortion issue, and that is when life begins.

      The catholics have an answer, which I don’t necessarily agree with, but they have one. I have never heard a specific answer from the pro-choice side that made any sense at all. And this is bloody crucial. You MUST be able to distinguish “human” from “non-human”, at one single nanosecond in time, otherwise we’re just playing with gray moral risk areas.

      There are plenty of options, but no consensus on even how to go about deciding. You could go with heartbeat, brainwave activity etc. Some people say “birth”, but that’s imprecise. Some abortion procedures involve partial “birth”. Is it when the child crowns? When the cord is cut? Is there a right to abort if it was intended, but the child is accidentally delivered alive? These are all questions that need answers, but I fear we have only politics.

      Personally, I support abortion as a temporary expedient until birth control is perfected. If the pill were 100% effective, I could see supporting a total ban on abortion except in cases of rape. I hope that technology will eventually render this dispute moot, but for now, I take the moral risk.

      • Nita says:

        There’s some tension between making birth control more effective and believing that legal personhood* begins at conception.

        Currently, emergency contraception pills do not prevent implantation. I would gladly trade that “feature” for higher effectiveness.

        Also, your argument about birth not being instant works equally well for conception/fertilization – it’s not an instant event either.

        * “life” goes on continuously, all the way back to chemical evolution

        • Tarrou says:

          Absolutely, I’m not plumping for the catholic definition of the beginning of life.

          But since a fetus can survive outside the womb long before birth, there’s a gray area. We can certainly say that the life cannot possibly begin prior to fertilization, and cannot possibly be denied once it is completely detached from the mother, breathing, kicking and screaming. So the reality lies somewhere in that roughly nine-month term. At some point, one life becomes two, and the absolute right of a woman to decide what happens to her body (which I support) ends, and the duty of us all to recognize and support the rights of both lives begins.

          It is at that point, ill defined as it may be, that abortion is no longer a moral option. I do think pro-lifers have framed the issue correctly. It is primarily a problem of when a fetus becomes a person.

        • Deiseach says:

          My problem with the word-juggling around “sure, we finally accept that it’s life, even that it’s human life, but it’s not a human person“, is that this makes personhood not an innate and inalienable quality (you know, like those inalienable rights to life, liberty and the pursuit of happiness? that all persons, being created equal, possess?) but a status that may be bestowed and removed by law.

          I think we’re going down that route with euthanasia; arguments for mercy-killing on the grounds that Granny isn’t really herself anymore now she has dementia are skating close to saying “Granny isn’t a person anymore”.

          Personhood will not be something you possess as of right; it will be something you achieve and can lose, e.g. the Catholic theological notion of “the age of reason” in its secular form of “sentience” (as I’m seeing argued on here: are 2 year olds sentient? how about 5 year olds?) will become the deciding factor, and once you lose that (or are judged to no longer have the capacity for it), then you’re not a person and “killing no murder”.

          And then we widen the scope, and some persons are not persons because they never had the capacity for true sentience (come on down, Peter Singer?) and so to dispose (humanely of course! no unnecessary suffering!) of them is not murder and possibly not even really killing. It is the reduction of suffering and the increase of utility and hedonism.

          But if some humans can be life, can be humans, but are not persons, why limit it to grounds of age or severe defect?

          We’ve had examples in our history of defining certain groups as life, as humans, but not persons: rather they were chattel, or living tools. Why should we expect ourselves to escape? You and I are persons right now – as long as we continue in possession of our faculties. Let us lose them by physical or mental illness or damage, and we are no longer persons, and have no claim to the rights which formerly we enjoyed, or to the protection of the law; rather, we rely on the decision of ‘legal’ human persons as to whether our continued existence is a benefit or a burden.

          If we start nibbling away at the roots, who knows when the tree will fall?

          • Nita says:

            Thanks for the thoughtful reply, Deiseach.

            I understand what you’re concerned about. But the “safe” approach you’re defending — simply consider all human organisms of any shape legally and morally equal — feels just as wrong and dangerous to me.

            If I could save either you or a liquid nitrogen tank containing a frozen human embryo, I wouldn’t flip a coin. I would choose you.

            With modern technology, we can keep human bodies alive even after whole-brain death (and I mean death, not coma). Are they still people? Then doctors won’t be able to save some of the lives they’re now saving with organ transplantations.

            Should every miscarriage be investigated as thoroughly as a child’s sudden death? Were doctors like Barnett Slepian mass murderers who deserved to die? I don’t believe that.

            No matter where we draw the line, there will be some loss, some suffering. But I think we can both make the least terrible choices and prevent sliding down a slippery slope if we anchor our idea of personhood to something observable, like brain function and behaviour.

          • Irenist says:

            Nita,

            I’m with Deiseach, but I just wanted to say that yours was a thoughtful post, too.

          • Irenist says:

            Nita,

            If I could save either you or a liquid nitrogen tank containing a frozen human embryo, I wouldn’t flip a coin. I would choose you.

            I think it’s possible both to say that we’d save a person before a tank of embryos, and to say that the embryos are people.

            Let’s say there’s a fire and I can only save you or a tank of embryos. I’ll save you: you and yours would be far more harmed by your death than the embryos would be by theirs. You have a personal life narrative you want to continue, your loved ones love you, etc.

            Now, let’s say there’s a fire and I can only save you or a roomful of five hospice patients. I’m still going to save you, because those hospice patients were going to die soon. But that doesn’t mean that I don’t think of the hospice patients as persons!

            Likewise, although I’d be comfortable saving you rather than the five hospice patients, it would be immoral for me to just walk into a hospice and shoot five patients.

            Further, although it would be right for me to save you instead of a tank of frozen embryos, it would, IMHO, be immoral for me (if I were female) to abort an embryo because I didn’t want any children.

            A lot of bioethical debate turns on whether the unborn or the terminally ill ought to be killed, and on whether they’re people. The “would you save 100 embryos or one adult” scenario is often used to suggest that us pro-lifers don’t *really* think embryos are people.

            I’m attempting to say that’s not right. Another example. My daughter is a person. But I don’t think she should have the right to vote, because she’s only a toddler.

            Similarly, I think the unborn are persons. But that doesn’t mean that they are persons who should have ALL the same care and concern directed toward them as born people. Just more care and concern than the present abortion-on-demand regime.

            Essentially, I look at being anti-abortion as being very broadly analogous to being a carnivore who objects to inhumane factory farms and won’t buy their meat. Such a carnivore isn’t saying nonhuman animals should have the same rights, just that “not being tortured” is a right they merit.

            Now, the carnivore still thinks it’s okay to kill nonhuman animals, and doesn’t (one would hope) think nonhuman animals are persons. So the analogy is very imperfect. But it’s meant to highlight that care and concern can be bounded.

            The unborn are people. But it’s okay for it to be legal to abort them to save the mother’s life. Hospice patients are people, but I’m not saving them instead of you. My daughter is a person, but she doesn’t get a vote. Etc. Personhood needn’t entail identical treatment.

      • Cauê says:

        “Ethically and legally speaking, there is a question that needs answering which then solves our abortion issue, and that is when life begins.”

        Wait, why?

        If you kill me, what’s being lost are all the life experiences I would have otherwise in the future. If you kill a fetus, what’s being lost is the exact same thing. Why does it matter where we set the boundaries of the mental categories in our maps?

        (there are good objections to what I said above and I consider this a complicated problem, but “when life begins” looks almost entirely irrelevant to me)

        • Tarrou says:

          In what way? One life becomes two. This is a pregnancy.

          A woman has the right to decide her own medical procedures, but doesn’t have the right to beat a nine-year-old to death with a spade. So deciding the point in time when one turns into the other is the crux of the whole matter.

          • onyomi says:

            Yes, I think a lot of confusion arises in the abortion debate because of tendency to view the fetus as somehow part of the woman’s body, implying that eliminating it would be equivalent to electing to cut off one’s own pinky finger, which, while weird if you’re not a Japanese gangster, most people would not see as morally problematic.

            But, of course, the foetus has its own DNA, its own developing organs, etc. and everybody intuitively thinks of it as ANOTHER person growing inside a bigger person, not a big person growing an extra head, torso, etc. inside of themselves.

            As far as viewing the fetus as some kind of “invader,” I did find this article very interesting:

            http://www.nytimes.com/2006/03/14/health/14preg.html?pagewanted=all&_r=0

            Not that I’d say that this fact alone justifies abortion, but it does put a new perspective on things: tl;dr, the fetus IS, in fact, a foreign thing in the woman’s body which fights for resources and alters her chemistry, sometimes detrimentally, for its own benefit.

          • Cauê says:

            As I said above: there’s a rule saying we can’t kill bleggs, but we can kill non-bleggs. So the way we tend to approach the problem is asking whether it’s a blegg. And each one runs their blegg-sorting algorithms and get different answers, because we’re talking about something that’s round and blue but not furry.

            Then the ones who get “yes, blegg” treat the “don’t kill bleggs” rule as 100% applicable, and the ones who get “no, not blegg” treat it as 100% not applicable – several people on this post have stated the moral weight is the same as an animal’s, or not even that.

            We basically agree about all the facts of the matter, fighting only about “is it actually really a blegg though”.

            So, I got a problem with this.

            What we should be doing is taking a look at the rule, understanding what it’s supposed to do, and answer whether it’s applicable in that case. I don’t care if we call it a blegg, but does it contain vanadium?

            Why shouldn’t we kill bleggs? What does this change in the world that we’re trying to prevent? What is lost? What is being gained and lost with each decision in this case?

            Whether we put the fetus in the category blegg or not-blegg is a fact about our maps. If we care about the territory, that’s what we should be thinking about.

            The way I personally look at it, what is valuable about human life are our subjective experiences. If I were killed, what I (or rather future me) would lose would be the experiences that won’t happen but otherwise would. If the acting of killing a fetus would result in its future subjective experiences not happening that otherwise would, then this is a fact about the territory, whether we paint it as a blegg or a not-blegg in our maps (and it doesn’t change whether it’s “currently a blegg” or “not yet a blegg”).

            I don’t know if I managed to be clear, but I’m not sure in which direction to move. I’ll wait for some feedback.

          • Nita says:

            @Cauê

            what is valuable about human life are our subjective experiences

            So, is there any difference between murder, abortion, contraception and abstinence under this approach?

          • Cauê says:

            “So, is there any difference between murder, abortion, contraception and abstinence under this approach?”

            Differences between murder and abortion are external to the one who’s dying – the sorrow of those who stay alive, effects on society, expectations, etc. From the point of view of the one who dies, I see no difference (if I die now, I will lose my future, it can’t remove the past, so the fact that I have lived a few years can’t mean *more* is lost than would have been if I had died in the womb).

            As for contraception and abstinence, I see a big difference when I weight it by probability. It’s not possible anymore to say that “this would happen with high probability if not for this action”. But I do feel confused when I think about this.

          • Jiro says:

            Caue: What do you do when you say “Okay, I won’t argue about whether it’s a blegg. I’m actually interested in whether it contains vanadium”… and then someone else comes along and tells you that ‘contains vanadium’ is almost as arbitrary a concept as ‘is a blegg’? It seems to me that questions like “is it sentient” are *already* an attempt to narrow ideas down to what people are really concerned with.

          • Cauê says:

            I think the question is insufficiently unpacked. We *can* still go further and ask “so what, why does that matter?”.

            For instance, we may ask if a comatose patient is currently sentient (although we don’t wonder whether current low brain activity means they’re not currently a person), but on deciding whether to kill them that’s not the important question. We ask how likely they are to wake up.

            That we don’t use the same standard on talking about fetuses, and instead wonder whether they are currently sentient (or human/alive/a person), is something to be explained.

            Perhaps the comatose patient is more blegg-like, or had started in the blegg bin to begin with, so we don’t focus on that and think in a more consequentialist way.

        • lmm says:

          I think the charitable interpretation of the grandparent is that we need to find the point in between deciding not to conceive (ok in descriptive ethics) and killing your child shortly after birth (not ok in descriptive ethics) at which these actions go from ok to not ok, despite all having the same consequences.

      • Peter says:

        I’m not sure I follow.

        It seems like we only need distinction when your moral system distinguishes “human” from “non-human”

        If we suppose that there might be an alien race with similar intelligence to humans and think about how we would react to them in terms of morality, we can see that this distinction is weird.

      • RCF says:

        “Ethically and legally speaking, there is a question that needs answering which then solves our abortion issue, and that is when life begins.”

        Life began in abiogenesis. The issue isn’t when “life” begins, and using the term “life”, when clearly something else is meant, shows a refusal to be precise that is highly indicative of a contempt for honest discourse.

        “I have never heard a specific answer from the pro-choice side that made any sense at all.”

        You ask a question that doesn’t make sense, and you don’t get an answer that makes sense? Surprise, surprise.

        “You MUST be able to distinguish “human” from “non-human””

        And now you’re switching from “life” to “human”. And there’s a difference between “non-human” and “not a human”.

        “at one single nanosecond in time, otherwise we’re just playing with gray moral risk areas.”

        Wow. Dichotomies don’t get much more false than that. We can have moral precision without having nanosecond precision.

        “Some people say “birth”, but that’s imprecise. Some abortion procedures involve partial “birth”. Is it when the child crowns? When the cord is cut?”

        You’re making nitpicking arguments that aren’t really controlling.

    • Anonymous says:

      What is the difference between the eugenicsy “it’s saving them from a bad life” and “adoption/foster care systems (at least in the States) don’t seem strong enough to completely fill that gap”? You seem to accept the second and reject the first, even though they seem identical to me.

    • Wrong Species says:

      Somewhere in the second trimester seems like a good compromise. Wikipedia tell me that many neurobiologists believe that the fetus feels pain somewhere around 24-26 weeks so that could be a good schelling fence.(I don't really know how to cite on this website http://en.wikipedia.org/wiki/Prenatal_perception#Prenatal_pain)

      However, I think the best thing that could happen is that the pro life and pro choice people keep arguing about the issue forever. If the pro-life side wins out then abortion could be restricted completely. If the pro-choice side wins, then they might decide that there isn't really that much of a difference between a nine month fetus and a newborn. Like you, I'm not exactly sure where the line should be drawn but the status quo doesn't seem too bad.

      • Anthony says:

        As I understand it, fetal viability at 23 weeks is effectively zero (less than 1%), while at 24 weeks, it’s somewhere around 30%. And while the second number has gone up a bunch in the past decades due to recent medical advances trying to save premature babies, the first number hasn’t, even though people are trying to figure out how to change it, too.

        So there’s something of a Schelling fence somewhere around 24 weeks, though it’s more like a line of fenceposts with nothing between them compared to conception and birth.

        (I also understand that the techniques used for abortions change at about the same level of fetal development, which makes for another possible Schelling fence by banning particular abortion methods. But my understanding of this is less complete, and I may be wrong about it.)

      • Anonymous says:

        Yes, the start of the third trimester has always seemed like the most sensible bright line to me. It coincides with the point of viability and the higher brain development necessary to feel pain (source). The fact that basic brain development begins at 5 weeks doesn’t make a 5-week-old fetus into a person. Plenty of animals have brains of some sort, and they’re not persons.

    • Lane says:

      Blue Tribe mid-twenties woman here, in favor of legalized abortion but not 100% confident in my position. My usual reasoning:

      — Empirically, people are going to seek abortions no matter what; keeping them legal means they can be kept safe and regulated.
      — Pretty much everyone seems to agree that abortion is okay to save the life of the mother. However, I suspect that in practice, banning all but life-saving abortions would mean that doctors would sometimes err on the side of not performing them, even when they were necessary. If abortion were regulated to this extent, I think it would be impossible to avoid the deaths of some women. (Compare to this account of parental notification laws in action — http://shorttext.com/6133b6bf *)
      — Making abortion illegal might risk a slippery slope to regulating/banning contraception again (one shouldn’t logically follow from the other at all, but conservatives seem intent on getting rid of both, so).
      — And, honestly, if abortion were made illegal, my life and the lives of most of my friends would be in more danger, our careers could be delayed or permanently derailed, we could be tied for two decades to partners who turn out to be unsuitable… our lives would just be much worse, and I’m not willing to put a country full of young women in that position for entities that aren’t sentient and can only be seen with a microscope.** Maybe this is selfish, but seriously, when I hear pro-life arguments, all I can hear is, “I’d rather sacrifice your life, and the lives of people you love, than compromise on principles that don’t affect me in the slightest.”

      I do think that abortion is probably immoral. I have an IUD because it seems to be the most foolproof form of contraception, and if I were to accidentally get pregnant I would strongly consider keeping it, even though it would mess up my life a lot right now. But I don’t think I have the right to make this decision for other people.

      *Reposted from the now-deleted fugitivus.net. Most relevant quote:

      Girls who can’t tell their parents about their abortions? After you pass a parental notification law, they still can’t tell their parents. Girls who can tell their parents? After you pass a parental notification law, they still tell their parents, unless they fall into an ill-defined legal loophole – then they tell their parents but still have to come get a bypass. A parental notification law accomplishes two things: 1) it takes the girls who can’t tell their parents and penalizes them for not being able to tell their parents and, 2) it takes a portion of the girls who can tell their parents and makes them go through the process anyway.

      **This is only true for the first few weeks, yes yes.

      • Tarrou says:

        This isn’t to discount your experiences, only to show that lived experience differs.

        Most of the abortions people I know have gotten are because the woman stopped taking her birth control, or somehow conspired to get pregnant for some other purpose.

        I served in the Army for some time, and it was extremely common for all the women in a unit to mysteriously get knocked up two months before a deployment, get to stay home, and have an abortion a week or two after the unit left. I’ve also seen a dozen or so abortions because the woman was trying to get a guy to marry them, and he wouldn’t.

        I perfectly sympathize with those whose upbringings are so strict they don’t have the information or the resources to plan their own sexuality. But how many people does that describe? All those women you are worried about catching pregnant, are they all in a cultish compound? Do they have access to a Walgreens? Because the generic pill is $12 a month, and condoms run about that for a sexually active male in a relationship (I may be projecting here). It’s not bloody hard to avoid. If a grown woman can’t avoid getting pregnant (excepting rape etc.) I see no reason why she would be capable of making any other decisions about her body.

        All these worries seem to me to be more based on movies about the fifties than modern reality.

      • Gbdub says:

        “Conservatives seem intent on getting rid of both [abortions and contraceptives]”

        Sorry to go off on a tangent, but this is a (sometimes intentional) misrepresentation of the conservative position that’s really messing up the debate (publicly, not so much here).

        Conservatives in general support the availability of contraception at a high rate. Even Catholics, who officially oppose all birth control, are much more wishy-washy where the rubber meets the road (pun intended).

        There are pretty much only two areas where “conservatives” as a whole, reject the blue tribe contraceptive position:
        1) conservatives oppose abortion and consider some contraceptive forms to be abortifacients. Blue tribes disagree with their classification.
        2) conservatives oppose forcing private organizations/corporations to pay for birth control.
        Neither of these positions strikes me as support for “getting rid of” contraception, unfortunately that’s the spin these positions get amongst the Blues, and it damages the quality of debate.

        Oh, there’s a third one: conservatives tend to support abstinence-only sex ed. But again, this is less driven by desire to eliminate contraception than a horrified reaction to the thought of their kids having sex. I think it’s an irrational position, but describing it as “denying access to birth control” is still uncharitable.

        • Anonymous says:

          Gbdub, the counterargument would be that those positions do, in actuality, result in reduced access to contraception and more unwanted pregnancies. When critics of the conservative position say that it constitutes “denying access to contraception,” they are talking about effects, not intent. I don’t agree that we should politely ignore this.

          • John Schilling says:

            Counterargument to what, exactly? Lane, Tarrou, and Gbdub seem to be debating the claim, “Conservatives seem intent on getting rid of [contraceptives]”. Lane’s words, arguing for that proposition. So at least some of the critics absolutely are talking about intent, not effect.

            And they are also explicitly talking about denying access or “getting rid of” contraceptives, not marginally reducing access by expecting the user to pay the costs. Because if those costs really are $12/month, which I believe is plausible in most cases, that’s an extremely marginal reduction in access, yet the language being used is that of outright prohibition.

            If you’re just going to argue that preferred policies of Red Tribe would have the unintended consequence of marginally reducing access to birth control, you’re not so much countering anyone’s argument as ceding about 90% of the position actually under debate. To me, this looks like classic motte-and-bailey at work, and I do expect Blue Tribe will keep trying to reclaim the bailey of “Red Tribe secretly wants to ban contraceptives outright; we need to crush Red Tribe utterly or all women will be reduced to nothing but breeding stock”.

          • Anonymous says:

            Counterargument to taking into consideration only their intent, for the sake of being charitable. (I mean, I don’t see jaskologist being particularly charitable wrt liberal positions. Do you have nothing to say about that?)

          • Gbdub says:

            Saying that anything that reduces the use of x is equivalent to getting rid of x is not merely uncharitable, it’s absurd.
            I would hope that we could agree that the following positions are all meaningfully distinguishable:
            1) all contraceptives are sinful and should be banned.
            2) I can tolerate contraceptives, but certain contraceptives are too much like abortion and should be banned.
            3) contraceptives should be legal, but I don’t want my tax dollars funding them
            4) contraceptives should be legal, and it’s okay to spend tax dollars providing them to those truly in need, but private organizations shouldn’t be forced to buy them for everybody.

            Really only the first one could be called “getting rid of contraceptives” – applying that label to 2 is a bit uncharitable but not horribly so, 3 and 4 have nothing to do with getting rid of contraceptives at all. Unless you want to argue that not supporting something as a positive right is the same as getting rid of it.

            In practice, I honestly see basically zero conservatives openly supporting position 1, so few that any chance of it gaining actual political traction is effectively null. As for 2, there definitely are a few conservatives supporting it, but still a definite minority. 3 and 4, yeah now you’re talking in terms that will get you somewhere with a lot of Reds. Basically, while I think a lot of Reds would prefer a world with less out-of-wedlock boning, the number willing to take political action against the Pill is a lot lower than typical Blue rhetoric would suggest.

            So if you want to say, “conservative policies would result in lower use of contraceptives, and that has negative societal consequences that we should care about”, fine, that’s a good debate worth having. But saying they favor “getting rid of contraceptives” is, in my mind not merely uncharitable but so warping of the actual conservative position as to actively prevent reasonable discussion on the issue, which is a shame, because I think the discussion of whether the state has responsibility and authority to mandate “free” contraceptives for everyone, vs. that responsibility falling in the people actually having sex, is an important one without a simple answer.

        • drunkenrabbit says:

          Regarding sex ed, it’s not entirely irrational, they feel that the school giving instructions on how to have safe sex is tantamount to endorsing premarital sex. In their eyes, its like responding to a school heroin epidemic by handing out clean needles and showing videos on safe injection. Abstinence-only sex ed attempts to treat the problem (highschoolers having sex) rather than the side effects (STDs and pregnancy).

        • RCF says:

          “2) conservatives oppose forcing private organizations/corporations to pay for birth control.”

          That’s the conservative framing. The liberal framing is that employment consists of employers giving compensation to employees, and employees then being free to do with that compensation as they wish, but conservatives want to set a certain class of compensation apart, and prohibit its use in the procurement of contraceptives.

          “Neither of these positions strikes me as support for “getting rid of” contraception”

          Letting the government decide what is “acceptable” contraceptive is certainly is far down the road towards “getting rid of contraception”. If liberals said “We don’t want to get rid of Christianity, we just think some denominations are evil, and we want those banned”, do you think conservatives would say “Oh, okay. As long as you don’t want to get rid of Christianity”? Also, birth control pills do not cause abortions, and I think that letting Catholics pretend that this is a matter of opinion is harmful to the debate.

          “In practice, I honestly see basically zero conservatives openly supporting position 1”

          Rick Santorum says that birth control is “not okay”, and thinks thinks that states should be allowed to ban it. Operation Rescue founder Randall Terry said that “we” (presumably, referring to Operation Rescue) want to make pill, IUDs, morning after pills, and the patch illegal.

          “so few that any chance of it gaining actual political traction is effectively null.”

          There was a time in this country when contraception was illegal in many states, and it wasn’t a change in popular opinion that changed that; it took a Supreme Court opinion to change it.

          “As for 2, there definitely are a few conservatives supporting it, but still a definite minority.”

          The Texas GOP platform opposes the morning after pill.

    • lmm says:

      If you’re thinking for yourself at all you should expect to have *some* beliefs that differ from your political affiliation (I refuse to use this colour tribe terminology). If anything I would say having all your beliefs align with your peers would be more worrying; it would suggest you were just following the groupthink.

      (And, fwiw, I think you’re probably right – and probably not as alone as you think. I’m a left-liberal but anti-abortion, and there’s another reply claiming a similar position.)

    • Anonymous says:

      Red-blue mosaic here.

      The issue is relatively simple to me. I don’t see newly born infants as ‘persons’ in the moral sense. My intuituion places their moral status somewhere similar to mice or rabbits. Inflicting pain or death on a mouse should only be done with significant justification, but the mouse’s life weighs less than say a cat or a pig’s, and much less than a human’s. Given the choice between keeping your pet mouse alive and undergoing a risky, expensive, and excruciating experience, I would not consider euthanizing the mouse to be immoral. Thus, I don’t consider abortion to be immoral.

      By this line of reasoning, why isn’t infanticide moral? Well, for starters, the continuation of the infant’s life is no longer conditioned on someone else experiencing childbirth. The big issue to me is uncertainty. At some point in the first few years of life, an infant obtains the mental qualities that make it a ‘person’. I don’t know exactly where this point is, but it seems certain to me that it’s after birth, so I choose birth as a conservative Schelling point to adopt.

      • houseboatonstyx says:

        By this line of reasoning, why isn’t infanticide moral? Well, for starters, the continuation of the infant’s life is no longer conditioned on someone else experiencing childbirth.

        +

        Thanks for a clarifying view, which points toward consequences.

    • Princess_Stargirl says:

      I am personally ok with infanticide for the first few days of life. So obviously I am fine with abortion. One can imagine a spectrum of infanticide norms depending on long after birth infanticide remains allowed. I think abortion is best thought of as being a mild point on this spectrum.

      • Wrong Species says:

        Why the first few days, and not after? If we ever get to that point, I’m not sure what conceivable argument you could use against infantcide in general.

        • Princess Stargirl says:

          Oh idk the right place to draw the moral line. The first few days is a conservative estimate. At birth is reasonable schelling point.

          In terms of morals I do not really think boolean morality is the best model. I think the “badness” of killing a baby monotonically increases as the baby ages. When the baby is only a few days old I think its probably only a little bad to kill the baby*. So killing the baby is a reasonable choice in many situations. As the child ages it becomes more and more bad to kill them. At some point it becomes VERY bad to kill them, but the “badness” increases smoothly with time imo.

          *Though its probably bad enough that you shouldn’t commit even first few day infanticide (or late term abortion) without a pretty good reason.

      • Matthew says:

        Interesting. I’m for abortion on demand, on the grounds that

        a)early-term embryos don’t have moral weight
        b)late-term abortions of convenience are a myth; I’m willing to take the word of women who seek them that they have compelling reasons for doing so.

        However, I’m not okay with infanticide, not because of some magical change at the birth canal, but because birth removes the bodily autonomy argument from the calculus.

        I find the argument “You must bear that foetus to term even if you don’t want it, because someone else will adopt it” repulsive, but I also find the argument “You should be able to terminate that newborn that is no longer inside your body, even though someone else would adopt it” repulsive. Once the kid’s born, the people who want to take care of it have a stronger moral claim than the person who wants to kill it, and more DNA in common doesn’t change that.

        • Princess Stargirl says:

          I agree if you can easily give the child away you should do this instead of killing the child. So in our current society I do not think infanticide is ok in most cases.

    • Lesser Bull says:

      My guess is that your beliefs are more consistent than your compatriots think.

      As I’ve become more Red, I find myself becoming less pro-life than I used to be.

    • Dale says:

      I recently wrote a long post on the Effective Altruist perspective on abortion, considering the ways that standard rationalist / effective altruist beliefs either support or undermine standard arguments about abortions. It was pretty well received on LW and the EA forum.

      It’s too long to quote here, but in very brief summary of a couple of the arguments, though I go into many more in the article:

      Moral Uncertainty implies assigning some credence to fetuses being morally valuable, and even if low this can dominate QALY calculations.
      Replacability implies aborting a baby and replacing it with another later in time is ok.
      Animal Rights implies larger moral circles, which increases the probability they include unborn.
      Xrisk means caring about future people who haven’t even been conceived yet.

      https://effectivereaction.wordpress.com/2014/12/31/blind-spots-compartmentalizing/

      • Wrong Species says:

        I just read your piece and it was very good. My two cents: I think the fact that the fetus is a potential person should be given more weight, although I don’t believe it works by itself. One more animal isn’t going to change the world, another person might.

    • blacktrance says:

      I think the better question to ask is this: why should fetuses have rights? Suppose abortion really is harmful for fetuses – that by itself doesn’t imply that it shouldn’t be legal. While there’s an advantage to be gained by cooperating with adults by recognizing their rights (i.e. restricting ourselves in what we may do to them), which is why we should do it, there’s no similar advantage for restricting ourselves in our dealings with fetuses, which is why it’s instrumentally rational to not give them any rights.

      This also works as an argument against animal rights.

      • Wrong Species says:

        It also works as an argument against protecting small children. Where do you draw the line?

        • blacktrance says:

          I don’t know where the best place to draw the line is, but I support legalizing infanticide.

          • Wrong Species says:

            Are you so willing to support infanticide that you allow 4 year olds to die if that ends up being the logical extreme?

          • blacktrance says:

            I’m not sure what you mean by “the logical extreme”. But as I said, I don’t know where I’d draw the line.

          • onyomi says:

            To blacktrance and anyone else who supports infanticide… uh, why?? Assuming the newborn isn’t somehow so horribly disabled as to preclude a healthy life of any sort, this seems obviously immoral to me.

            I mean, have you seen a newborn? It seems obviously to me to be a little person, and that killing it would be murdering a person. I’m aware that some ancient cultures were okay with infanticide, but then, some ancient cultures were also okay with a father killing an eight-year old for disobeying him.

          • Nita says:

            @onyomi

            Well, I’m not a true member of the babykiller coalition, but…

            I mean, have you seen a newborn?

            They look something like this, I guess? And all they do is sleep, scream and nurse (except the ones who haven’t figured out how to suck yet).

            If anyone deserves human rights because of how they look, it’s these guys.

          • onyomi says:

            Well I certainly would favor banning hunting of primates. I’m morally inclined toward not eating any mammals, though my love of bacon is thus far too strong.

            But I think the “potential for rich experience” factor must carry some weight. The newborn may not have a rich inner life yet, but it is still very obviously a baby human. A baby human may be stupider than your dog, yet we place more value on the life of the baby human because we know it has a very high likelihood of developing into something much more self-aware than the dog.

            Taken to the extreme, this would seem to preclude all abortion, since any fertilized egg has the potential to develop into a deep thinker some day, though I think in the very early stages there is a lot less inevitability, and therefore less respect demanded. Some fertilized eggs never manage to attach to the placenta, even without taking contraceptive measures to prevent such attachment; yet I don’t think we mourn the loss of a possible family member every time that happens, even if we know it does (losing a baby a bit later in the pregnancy, however, seems like it could be pretty darn traumatic, however).

            Of course, how much “potential for future inner life” should factor into our calculations is a separate, debatable question, but I’d say it’s already pretty much accepted that it counts for something, at least. Otherwise, killing a dolphin or chimp would be as bad as murdering a 5-7 year old human, which I don’t think accords with most people’s intuitions, though human intuition is probably inherently biased in favor of humans, admittedly.

          • Anonymous says:

            @Nita: who says they don’t already?

            http://www.scientificamerican.com/article/argentina-grants-an-orangutan-human-like-rights/

            (Note: I have no specific allegiance to the news site I linked, I just post the first English result I found for the story).

          • blacktrance says:

            onyomi:

            I’m not a moral intuitionist. It may seem immoral and repulsive at first, but that doesn’t mean it’s wrong. It is instrumentally rational to keep the option of infanticide on the table, even if one doesn’t intend to use it – there’s no advantage for us to be gained if we restrict ourselves in our dealings with babies.

            As for the newborn being a person – I’m not particularly enthusiastic about the concept of “person” (in the moral sense). It’s useful because it labels a cluster in thingspace, but it also introduces a false dichotomy between “persons” and “things” – the former is really a subset of the latter.

          • onyomi says:

            Blacktrance,

            Are there any options you’d take off the table?

          • blacktrance says:

            In general, no, unless I’d get something better by giving up one of my options – and that’s not the case with abortion, infanticide, or animal rights.

          • onyomi says:

            I think it’s misleading to say “I’m not against infanticide” if what you mean is “if I could go back in time and kill baby Hitler, I’d do it.”

          • blacktrance says:

            That’d certainly be misleading, but that’s not my position. If someone wanted to kill their own baby, I’d be okay with it for the same reason I’m okay with people getting abortions.

            (However, there is the caveat that in current society, infanticide is much less normal than abortion, so if I knew someone was going to engage in it, I’d be concerned that they’d have an evil character. But that would still be a “morally neutral thing likely to be done by an evil person”, and not actually a bad act in itself.)

          • thirqual says:

            Something I noticed is not often well known in discussion about infanticide: the practice is incredibly common in nomad societies, and was widespread in sedentary societies too.

            (note: the rest of this is meant as independently of infanticide for sex selection as practiced in India for example)

            Exposure (ancient Greece) or ritual sacrifices (Carthage) was often preferred (to stay in ancient Europe). Those practices survived until the XXth century in some countries (as in, not really fought efficiently by law or condemned strongly by society even when the law was clear).

    • Scott Alexander says:

      “But given that brain development begins as early as 5 weeks of pregnancy, I’m not sure where we decide to draw that line.”

      I mean, I think we decide to draw the line at birth. I’m kind of being facetious, but I don’t think it’s a bad place. Before birth the fetus seems to have less mental activity than a cow in pretty much every way, and we’re okay with killing cows. Of course, that continues for a long time after birth, but birth is a sufficiently bright line that it seems to address line-drawing concerns.

      • Cauê says:

        We’re not usually ok with killing people who currently but not permanently have less mental activity than cows, though.

    • Sarah says:

      For me, the most salient fact about abortion is that a sixth of pregnancies end in abortion.

      Abortion, like meat-eating, may be wrong, but even if it’s morally equivalent to murder, it’s absolutely impossibly to treat it as legally and socially equivalent to murder. You can’t lock up all the meat-eaters, and you can’t lock up all the women who have abortions. And you can’t really use the social emotions of disgust and fear which we usually reserve for murderers, against *that* large a percentage of the population.

      There are a number of moral issues that take this form. Maybe we are all obligated to give more to charity than we do; but it’s just not *practical* to treat all non-EAs as if they’re murderers.

      Slavery was a similar issue: it was, of course, wrong, but there were just too many slaveowners for the US to treat it as a *crime* in the sense of a deviant aberration which the community punishes. You had to fight a war to end slavery, and arguably we have not eradicated all its traces.

      Any time you try to argue that a *common* behavior is extremely immoral, you are in the position of calling *ordinary* people villains. You’re going to get pushback — “hey, lots of people I know do that thing! it’s normal! are you saying we’re bad people?” Now, it’s totally possible for a common behavior to be immoral. But it’s really hard to treat a common behavior like a *crime.* (You can make a common behavior illegal — e.g. drug use — but people won’t respect the law, and they won’t react with horror to a drug user the way they react to a thief or a murderer.)

      I happen to think that some behaviors modern society thinks of as “normal” — for instance, the fact that so few people will help friends in need — are historically abnormal and ethically unacceptable. But even though I believe that, I have to put a weird sort of “personal reality goggles” on to see someone who flakes on their friends as despicable rather than “normal.”

      And my brain just *can’t* see meat-eating or abortion as making you a “bad person”, even though I’ve heard the arguments that they’re wrong. They seem like normal behavior to me; maybe I’m just grading society on a curve, but my brain simply won’t parse these kinds of violence as criminal/aberrant rather than licit. The trend of civilization is away from violence — slavery, torture, corporal punishment, marital rape, etc. used to be normal and now they’re (mostly) not. So I guess I would predict that any given kind of violence will eventually become abhorrent to our descendants. But it doesn’t seem that abortion or meat-eating are abhorrent to *our* society *now*, and so they don’t read as abhorrent to me.

      • Cauê says:

        Yes, this is strong. I find myself in the weird position of not being able to honestly distinguish abortion from murder, and yet sympathizing so little with the fetuses and so much with the women that I don’t get emotionally worked up about it.

        (but still somehow get worked up about bad arguments surrounding it)

        • Anthony says:

          but still somehow get worked up about bad arguments surrounding it

          Good God, yes. I’m moderately pro-choice – I’d put the line around 24 weeks, and maybe some restrictions on earlier abortions (if it’s really just another medical procedure, we should treat it like one…). But there’s nothing like an emotional pro-choice argument to make me feel like a pro-lifer.

      • Kiya says:

        One-sixth seemed surprisingly high to me, so I went and found a source that seems to agree; maybe more like one-fifth.

        Apparently half of all pregnancies in the US are unintended. That’s a lot more than I would have guessed.

        • Anthony says:

          There’s “unintended” and “unwanted”. A friend of mine’s wife got pregnant when they were not planning to do so – they’d been planning to wait a few more years. But they decided to go through with it, and have a kid now. That’s an “unintended” pregnancy by pretty much any reasonable definition, but it’s not the sort of thing people are appealing to when they talk about “unintended pregnancies” in arguments about abortion.

      • Irenist says:

        @Sarah:

        Great points. I struggle with this as a pro-lifer, too. I see abortion as murder (not just killing), yet I don’t abhor the mothers as “murderers”; I just think they’re making a bad choice. But how can there be murder without a murderer?

    • Andrew says:

      “Arguments from “parents who get abortions are parents who can’t afford children, so it’s saving them from a bad life” or the Freakonomics argument that abortions lower crime rates sound creepy and eugenics-y, and not at all in line with Blue/Grey values as I understand them.”

      I think you should reconsider that evaluation.

      The “red” tribe describes its anti-abortion stance as “pro-life” and I think that the moniker is an accurate descriptive: their basis to opposition is genuinely founded on an ideological commitment to value (human) life for its own sake.

      On the other hand, the “blue/grey” tribes are vastly less likely to see this value in life for its own sake. They will see life as valuable, but for some reason outside of its mere livingness.

      This is evidenced by the fact that Democrats support legalizing euthanasia, whereas Republicans do not.

      I have an explanation for this, which is not very charitable to the Republicans. I will state it, but it is not my point. My main point is that I think you are mistaken about what the “tribes” actually think.

      To state my (not very charitable) analysis of the reasons: I think that the pro-lifers on some level do not know why human life is valuable. Instead, they accept it as received wisdom. As such they are not equipped to reason about the exceptional cases, where the reasons for valuing human life do not apply and so the particular life is not valuable.

      But again, my point here is not to say that. My point is you should re-consider your idea about what “blue/grey” values are.

      • Anonymous says:

        I object to the conflation of greys with blues.

        • Andrew says:

          It’s useless to say so if you don’t say why.

          • onyomi says:

            Because there are many “greys” who don’t associate their views with those of the “blues,” or who associate as strongly with “reds” as “blues.” I guess most of them are probably right-wingish anarchists/libertarians like me.

      • Irenist says:

        Andrew:

        Overall, your comment above is very insightful.

        I’d like to challenge your explanation. Now, you said it’s not your main point, so if you don’t feel like responding, I certainly won’t be offended or anything. But in case you do:

        I think that the pro-lifers on some level do not know why human life is valuable. Instead, they accept it as received wisdom. As such they are not equipped to reason about the exceptional cases, where the reasons for valuing human life do not apply and so the particular life is not valuable.

        Well, there are going to be people in any movement who are just following received wisdom. And if we’re primarily talking about the G.O.P. generally rather than pro-life activists and leaders in particular, than sure.

        However, I think you’ll find that, e.g., Thomist theologians’ accounts of human flourishing rooted in a synthesis of Aristotelian virtue ethics and Christian piety and charity are at least thought-through, contrary to your characterization. You’re of course free to disagree with them, but I think it’s fair to say that theologians and bioethicists on the pro-life side aren’t just parroting received wisdom: they’re doing real thinking on these issues, even if not thinking you find ultimately convincing. The theocons and biocons at, e.g., First Things or The New Atlantis might be wrong, but they are a thoughtful lot.

        That said, why IS human life valuable, and which human lives aren’t? Is it a combination of personhood, lack of suffering, and capability for you? Or something else? E.g., I assume you’d say that fetal lives aren’t valuable (as lacking personhood) and the lives of pain-stricken terminally ill people seeking euthanasia aren’t valuable to the persons themselves, and so forth. Is that about right, or am I way off?

      • Jaskologist says:

        Interesting that you would consider this view uncharitable to the Reds. They would state the same facts, but with the opposite connotation.

        The term for what you are describing is “Imago Dei.” All humans, being in the image of God, are automatically endowed by their creator with certain unalienable rights. This is received wisdom.

        Those who do not receive this wisdom are less likely to value life for its own sake, and also very likely to erode away at those fundamental human rights. This is evidenced by the fact that Democrats support legalizing euthanasia, whereas Republicans do not.

        The historical argument comes into play at this point. Imago Dei was in fact instrumental in coming up with the concept of humans rights at all. There are plenty of thinkers (CS Lewis and Chesterton, among others) who will tell you that without it there is no basis for human rights at all, and they will point you to the many times people have sought another basis and ended up deeming many human “lives not worth living” as evidence.

      • John Schilling says:

        To state my (not very charitable) analysis of the reasons: I think that the pro-lifers on some level do not know why human life is valuable. Instead, they accept it as received wisdom

        A similar analysis would be that pro-choicers on some level do not know why human life is valuable. Instead, they reject it as worthless until proven otherwise.

        As for me, I would characterize exactly one of these analyses as “uncharitable”. And I know which group of people I would rather have as neighbors, or rulers.

  86. nico says:

    So I really enjoyed this SSC post about Singer on Marx.

    My question: can anyone recommend (or produce) a similar piece of writing about Edmund Burke? Main main questions are:
    1) Of the obvious things that Burke said, which only become obvious after Burke said them?
    2) I understand that Burke was “wrong” about a few things (class structure comes to mind), but I can’t tell if those were things he put his money on, or just an artifact of translating things 200+ years into the future.

  87. rsaarelm says:

    The last few posts have had pingback notices coming in from blogspam sites, might want to block those somehow.

  88. Jiro says:

    Obviously the statement about Multiheaded repaying the money to effective charities is intended to forestall the argument “wouldn’t it be, by your own utilitarian reasoning in other posts, better to send the money to effective charities rather than to Multiheaded?”

    I’m not convinced it really does forestall that, though. That same reasoning would suggest that if we lend the money to some better cause (perhaps microloans to poor third worlders), wait to get paid back, and then immediately donate the amount paid back to effective charities, that would still be better than lending the money to Multiheaded, and having the amount paid back go to effective charities.

    (It would also suggest that nobody should just donate to effective charities; they should always lend the money, get paid back, and then donate it, thus getting two uses out of the money.)

    Perhaps Scott should just bite the bullet and admit that nobody actually has moral values that are equivalent to utilitarianism, and say that we should lend money to Multiheaded because Multiheaded is someone we know, if not well, at least more than we know a random third worlder. Of course, that would lead to people spending money on themselves and their families instead of on effective charity, and even preferring their own countrymen over foreigners, and that’s terrible.

    • Elissa says:

      See, when we were talking about Ialdabaoth, I included a tangent about how, no, this isn’t the same as effective charity, but here are some other reasons you might want to spend your money this way, and by the way purchasing fuzzies is totally allowed. Which made the appeal kind of verbose, but now I feel reassured that it was necessary. (Did you complain about this when Scott recommended buying books or earplugs?)

      • Anonymous says:

        Jiro did not bring up effective altruism out of the blue, but because Scott implied that this is effective altruism. Which he did not about earplugs and books. This tells you nothing about the hypothetical in which you provide no disclaimer on Ialdabaoth.

        • Elissa says:

          Jiro did not bring up effective altruism out of the blue, but because Scott implied that this is effective altruism.

          How do you figure? I don’t see that implied anywhere. He just didn’t explicitly disclaim it like I did.

          • Anonymous says:

            Multi intends to pay as much as possible forward eventually with donations to effective charities.

          • Elissa says:

            …that does not imply that giving money to Multi is effective altruism. It didn’t imply that when Ialdabaoth said the same thing, either. It’s still kind of nice, right?

            (srsly ‘tainted altruism’ is such a stupid bias)

          • Anonymous says:

            OK, if Ialdabaoth said the same thing, then the only difference is the disclaimer. My last sentence was wrong.

            I stand by my first two sentences, but I regret writing them because they add nothing to Jiro. I should have reread him and realized that.

    • AR+ says:

      I don’t believe in EA so giving money to my in-group doesn’t cause me any problems whatsoever.

      Kinda confused, though. I thought, “oh, hey multiheaded needs money. I suppose I can help out,” and then realized I had no idea where that impulse came from. I’m certainly adjacent to multiheaded in a social sense, but I haven’t talked w/ her very much at all. Best I can figure is that it’s the effect of reading so much of her back and forth as an active participant, not even w/ her but just in the conversation happening around us, that it feels kind of like how it might if everyone in the thread was hanging out in person. In that case, it wouldn’t seem weird at all that there would be some sense of community towards someone you’ve almost never even talked to one-on-one.

      But it does seem weird because we weren’t there together in person, and indeed she may not have read any of my stuff, or if she did she hated all of it and I had no way to gauge her reaction (except those times where she makes her reaction… explicit), and it’s not like she could tell when I was reading her posts or tumblr, so rather than it being like I was hanging-out and listening, it’s more like either ease dropping or being a member of an audience. Feels vaguely stalker-ish to even think of her as “somebody I know.”

      And then of course there’s also the problem of whether I should be actively funding communist immigration to the West, but in the end I don’t think that was a real factor in the decision.

    • Pseudonymous Platypus says:

      Perhaps Scott should just bite the bullet and admit that nobody actually has moral values that are equivalent to utilitarianism

      Or perhaps we’re not all perfect moral beings who are consistently able to live up to our moral ideals despite our feelings, biological urges, etc etc etc.

      • Alexander Stanislaw says:

        Do you think that giving money to Multiheaded is a moral failing ?

        • Pseudonymous Platypus says:

          It’s not a moral failing in the sense of being actively bad, but if you look at it from a strictly utilitarian perspective, then it is morally suboptimal, because you could (probably) generate more utility by donating that same money elsewhere.

          I’m not discouraging people from donating to Multiheaded, though. Quite the contrary; I did so myself. I was merely pointing out that I don’t think Jiro’s attempt to use this as an argument against utilitarianism works.

          • RCF says:

            It’s not an argument against utilitarianism, it’s an argument against the proposition that rational reflection on one’s moral values culminates in utilitarianism.

          • Pseudonymous Platypus says:

            I still don’t think this case works to make that argument. For instance, I can rationally reflect on my moral values and decide that I shouldn’t illegally download TV shows, but I might still do it anyway, because how else am I supposed to get Top Gear when I live in the States? Uh… that’s just a hypothetical example, of course…

            Anyway, returning to the point, I can rationally reflect on my moral values and decide that utilitarianism is the system to which they best conform (or vice versa). But that doesn’t mean I’ll be a perfect utilitarian. Am I missing something?

        • Elissa says:

          It’s a moral failing in the same way that buying books Scott recommends (instead of borrowing them from the library, say, and donating the money to AMF) is a moral failing. Only less so, because there’s a pretty good chance that Multi will ultimately pay your donation forward.

    • Anonymous says:

      See also “why our kind can’t cooperate”.

      >It would also suggest that nobody should just donate to effective charities; they should always lend the money, get paid back, and then donate it, thus getting two uses out of the money.

      Moneylending *is* a thing people do to increase value, and when you lend without interest you are basically saying “Your well-being is “interest” enough for me”, except for the whole thing where you still get mad if they don’t pay you back because they can’t.

      So, if you think the “good” done to Multiheaded is worth sacrificing what you’d gain on interest, it’s still a good idea. Not *necessarily* better than microloans, mind you, but still.

      Also, philanthrolocalism. There is an argument that any charity done to a person you somehow know is superior to charity done to strangers from a risk/reward perspective, because you have reduced risk in the form of hyper-detailed understanding of what that money is doing. I don’t know if I buy it, but it’s an argument that exists.

      • llamathatducks says:

        The argument in your last paragraph, slightly modified, is a big part of why I insist on donating to local charities even though money spent locally is less “effective” than money spent abroad. I’m still donating to strangers, but these are strangers in my culture whose needs I can come reasonably close to understanding and whose perspective on the work done by the charity in question I can pretty easily find out, so I feel rather more confident than with faraway charities that the help is helping in a way that people want to be helped.

        (Another reason is that I selfishly want my city to be as good a place as it can possibly be, which includes helping people survive and thrive in this city.)

        • If you are interested in doing the most good, you don’t necessarily have to understand the needs of the people you are helping; what you need to have is evidence that your money will accomplish more if donated to the charity in question than it will if donated elsewhere. And it is unclear why, once you factor in other sources of evidence, such as randomized controlled trials, having a better personal understanding of locals should on balance give you stronger reason to help this group of people. I’d also say that we should discount the evidential force of arguments that support conclusions that we want to believe on independent grounds. Acknowledging that you have selfish reasons for helping locals in your city should make you distrust, to some degree, that these are precisely the people you should also be helping from an altruistic perspective.

      • Anonymous says:

        CS Lewis made the virtue ethics argument that helping those far away while neglecting those you actually see and know made your benevolence mostly imaginary.

    • SFG says:

      You people are trying too hard. You don’t need logical excuses to give an unfortunate transgendered Russian money, just do it because it makes you feel good. 🙂

  89. Ian James says:

    I’ve been meaning to post in one of these open threads for a while. I ordered some MealSquares in early December. I returned from a trip on January 2nd to find–after a few weeks of anticipation–they had finally arrived in the mail! I tore open the package, set my first square on a plate, and proceeded to have a bite of what might be the single most atrocious piece of “food” I’ve ever tasted. Now, I’m a Soylent early adopter, so I wasn’t expecting this thing to taste “good” by any ordinary definition. Still, I was completely unprepared for the rubbery texture (as if they had thrown a bit of astroturf in the microwave to soften it up a little) and pungent prune juice-like flavor. I kept eating until I finished the square, hoping with each bite that I would somehow “adjust” to the taste. Not only did the adjustment fail to occur, a horrible aftertaste lingered in my mouth until I finally washed it out with some ice cream.

    As I’ve said, I approached this from the perspective of a Soylent early adopter trying to figure out whether multiple nutritionally complete foods could have complementary places in my diet. In particular, I’m not crazy about carrying Soylent around with me in a water bottle–it goes bad after several (eight? ten?) hours out of the fridge, and even before that, it’s fairly unpleasant when warm. The solution turned out to be, not MealSquares, but Soylent brownies. I bake them once or twice a week and carry some around at all times. Eating two or three of those (along with a cup of tea) is just as good as a glass of Soylent or a conventional meal.

    That said, I think MealSquares are a fine idea in principle, and I hope the company can iterate their product into something nutritious and palatable. But unless the batch I got was a complete fluke… I think it’s too early to be advertising, guys.

    P.S. If anyone else is curious about the brownies, I think Lee Cauble’s recipe is an excellent starting point, but in my version I reduced both the sugar and the cocoa powder from 1/2 cup to 3/8 cup (6 tbsp). This makes them less of a dessert and more of an everyday food, which reflects how I use them. Also I found that adding 2 tbsp coconut oil gave them a “fudgy” texture. You’re welcome, I guess? There are long odds that anyone else cares about this… then again, if anyone did it would probably be SSC commenters, right?

    • Glen Raphael says:

      I found mealsquares vaguely tolerable after forcing myself to eat several of them. But just barely. The first one was the worst – truly disgusting – and the next two weren’t much better.

      Mealsquares are MUCH better at room temperature than fresh from the refrigerator. There are two reasons for this: (1) the coconut oil gets waxy when cold, (2) the chocolate chips lose most of their flavor when cold. However, at room temperature the coconut oil makes it *even more sticky* which is an annoyance – you have to eat it with a fork or pick it up wrapped in a paper towel or plastic bag to avoid sticky fingers.

      • AR+ says:

        I’ve found the new fluffier version to be best when heated up.

      • Vaniver says:

        Mealsquares are MUCH better at room temperature than fresh from the refrigerator.

        I will offer a counter-recommendation: try them straight from the fridge, at room temperature, and heated up in the microwave. You will probably have strong preferences for one of those, and it is likely to disagree with others’. (I strongly prefer them straight from the fridge.)

    • AR+ says:

      Hmm, maybe try another one from a different pack? I remember several months back that one of them had these bad tasting gooey parts that I attributed to early-development bugs.

      I find it odd that different people have had such wildly different reactions, from “these are good,” to “it tastes like… food,” to ” single most atrocious piece of ‘food’ I’ve ever tasted.” I thought maybe I was just weird but I gave my mother one and she liked it to, so I guess it’s just a really polarizing food.

      Though… how long was your trip? Remember that these things currently have a shelf-life closer to fresh fruit than commercially available energy bars, especially if unrefrigerated. If it arrived in December and sat in your mail box for a couple of weeks, they might simply have spoiled.

    • Elissa says:

      I did find that, after eating them every day for a week or so, my perception of the taste had very noticeably shifted from “I really don’t like this,” to “I really don’t mind this.” Which is still not great, obviously, but ultimately the extreme density and dryness were more offputting to me than the taste.

    • drunkenrabbit says:

      What was the point of meal squares supposed to be in the first place?

      • AR+ says:

        As I understand it, nutritionally complete whole food w/o the trouble that this would normally require.

      • John Maxwell IV says:

        You could imagine many attributes a food item could be rated on: prep time, nutritional value, taste, cost, etc. MealSquares is an attempt to do very well on the “prep time” and “nutritional value” dimensions while scoring OK to good on taste and cost. But beyond that, we’d like to offer people a sort of “default food” they can eat for the most mundane 1/3 of their meals, removing some of the mental overhead associated with figuring out what they’re going to eat.

        (We’re trying to communicate these benefits on our homepage, but it’s possible we could do this better… if anyone wants to offer feedback, either here or via our contact form, I would love that.)

        • Shenpen says:

          I have an idea for you. Since a lot of people could do well to lose some weight, they should be eating less than their daily calorie requirements, and it would be better if this default meals would provide as few as possible calories, so that they can allow more calories to sneak into their normal meals.

          So I recommend making a calorie reduced version.

          But think it over because it may turn out to be a fluke. There is a chance that most of your customers are rather thin – because your whole business idea seems to be that it is for people who don’t really like eating in general, don’t take a lot of pleasure out of it, but prefer to be healthy. That is pretty much the opposite of the typcial psychological profile of obese people.

      • Eli says:

        Your question presupposes that there actually is a point.

    • John Maxwell IV says:

      Hi Ian, MealSquares cofounder here. We had a bad batch around a month ago and I’m wondering if you might have been affected. Would you be interested in receiving a free sample of our latest iteration to help us determine what’s going on? If so, you can reply to me via our contact form using whatever email address you got your order confirmation email at (and also send us whatever your preferred mailing address is if it has changed).

      (Apologies if you were indeed affected by a bad batch. We try to make it clear on our website that we are a beta product… quality control is one of the many dimensions that we’re currently trying to improve on as a company. If anyone else thinks they might have been affected by a bad batch, please contact us using the email address you used for your order.)

      • Vaniver says:

        I was curious if that was the case- I had a pack that was so bad a month or two ago that I almost canceled my subscription because of it, and then every pack since has turned out to be fine.

      • vV_Vv says:

        What did this ‘bad batch’ imply other than a bad taste, I may ask? Were the ingredients in the wrong proportions? Were they spoiled? Did you get some sort of contamination?

        I’m asking because you advertise your product as something people are supposed to consume as staple food, therefore if there are quality problems there could be significant health issues.

        • John Maxwell IV says:

          I’m asking because you advertise your product as something people are supposed to consume as staple food, therefore if there are quality problems there could be significant health issues.

          Understood; that’s a reasonable concern. In this case, the bad batch was caused by incorrect ingredient ratios (we’re not sure on the details).

      • Ian James says:

        OK, I’m impressed that you guys are trying to make it right. I’m contacting you now.

    • zz says:

      By “early adopter”, do you mean that you bought soylent in the Kickstarter phase, or were you DIYing it before Rob had started making noises about commercializing it? I ask because if you’re in the latter category, I have a lot of trouble thinking you switched to the commercial product, which has implications in whether the soylent brownies will work for me, since I believe my DIYed mix is both cheaper and better-sourced than the commercial one.

    • Brandon Berg says:

      Aren’t some of the nutrients in Soylent (e.g. vitamin C) destroyed by heating?

    • Princess_Stargirl says:

      I have ordered several rounds of mealsquares and none of them ever had a “pungent prune taste.”

  90. People who follow UK politics, what are your predictions for the general election?

    I’m pretty much following Ian Dale’s predictions because he seems to have the best methodology. http://iaindale.com/posts/2014/12/29/general-election-predictions-the-complete-list

    So overall Conservative 275-285
    Labour 290-300
    LibDem 22-27
    UKIP 1-5
    SNP 15-20
    Plaid 2-4
    DUP 7-10

    Thoughts?

    • kieran M says:

      I accidentally turned electionforecast.co.uk into a link for the whole reply, so my previous message got eaten, but that’s what I’m going with: its a Bayesian hierarchical model after all, so you’ve got to love it.

      If you look at any competing models for predicting the next election you’ll see a lot of differences, particularly in which party will be the largest in the next election. There are several reasons for this

      1-We lack local polling, meaning assumptions have to be made about how national polls translate locally
      2-Parties such as the SNP, UKIP and the Greens are getting unprecedentedly high numbers, and how these translate at the polls will make a huge difference

      As far as I can tell, we can be pretty sure (barring some shake up in the next few months) that

      1-There will be no absolute majority
      2-The following parties will gain seats: Labour, SNP, UKIP, maybe the Greens
      3-The following parties will lose seats: Conservatives, Lib Dems.

      Other than that, it’s really very up in the air as to where Labour and the Conservatives will be come election day.

    • Zorgon says:

      I think Dale and several other pundits are being conservative (small ‘c’) regarding the scale of the Lib Dem crash. It’s natural to want to revert it a bit to the mean, but barring the hardcore faithful, the Lib Dems are dead in the water. I fully expect to see them lose multiple deposits.

      (I may be a little gleefully anticipating this whooping, I admit.)

      I have absolutely no idea if the Greens’ gains are going to translate into seats. If they’re at all clever about focusing on potential wins, they could easily manage 2-3 at least, which is a serious step towards getting the UK media to break their painfully obvious embargo on them. But I don’t see good signs in that regard, they seem to be pushing for a full nationwide campaign; in some places that isn’t a bad idea (like my own constituency where pulling into second will leave them in a good position to take the seat next time) but in the Home Counties and the North it’s a complete and total waste of time, effort and money.

      Labour are resurgent. It’s quite impressive how quickly the machine’s picked up, and it’s even more impressive the extent to which they’re going to avoid letting their weird-ass robot leader in front of the cameras. It reminds me of the Tories trying to avoid acknowledging John Major was their leader back in the 90s.

      All in all it’s way too chaotic to call. I think there are going to be some huge upsets, but I have no idea what exactly. I just wish there was a center-left party remaining in British politics that didn’t have the Greens’ baggage stapled to it.

      • Deiseach says:

        With the British “first past the post” system, I think the votes will translate into a whopping loss for the LibDems which will be spread among smaller parties like the Greens, but not in sufficient blocs to elect more than few surprise seat-takers. I think it’ll be Labour and the Tories fighting it out for seats, with Greens, SNP, UKIP, Monster Raving Loony Party candidates coming in third, fourth or fifth – doing much better than in the last election, and mopping up the lost LibDem and swing Tory and Labour votes, but not enough to really be huge.

        If, say, the Tories offer to give the Greens concessions in return for letting them form a minority government in order to avoid a hung parliament, I think the Greens should avoid that like the plague (take warning from what befell the Green Party in my own country when they went into power-sharing!)

    • Anonymous says:

      I think it’s really hard to say, because it depends on so many factors. I would think:

      1. The votes for UKIP, Greens, etc, will not hold up. People will vote for mainstream parties. UKIP will do very well as a protest vote in safe working-class Labour seats, but I’d say they are more likely to lose than gain MPs.
      2. The Lib Dems’ death has been much exaggerated. Dale is right that they’ll get 25 seats or so. Yes, they’ll lose some deposits too.
      3. The SNP will be triumphant in Scotland, which may be the most important long-term development.
      4. The election will be local. Labour will fight back in the distant London suburbs (eg Enfield), but lose in the nearer suburbs (eg Ealing).
      5. I would not be the least surprised to see the Conservatives remain as the largest party.
      6. No party will gain a majority, and there is a very decent change we’ll have a second election within 18 months, fixed-term parliament law notwithstanding.

      • Adam Casey says:

        I agree re Lib Dems, the constituency polling seems to show rather sharp differences in incumbancy for them. They’ll cling on to a higher proportion of their seats than of their deposits.

    • Adam Casey says:

      He seems impressively meticulous. But I worry that in a setup like that a small systematic error in calling marginals could turn into a large overall distortion.

      Obvious case in point: The betting markets seem to think he’s rather pessimistic about SNP and UKIP’s chances. http://www.sportingindex.com/spread-betting/politics/british/mm4.uk.meeting.4888412/uk-general-election-seats-markets

      Personally I agree with Dale on UKIP and disagree on SNP. So I backed the SNP side of the ledger when it was at 22, and sold UKIP on 9.5. But the Ashcroft constituency polls are going to be crucial in going from “I can’t believe they can overturn that big a majority” to “it seems like n% of Lab voters will switch in constituencies like this”.

      As for the overall: I put a lot of weight on the final outcome being “seriously hung” to borrow a wonderful phrase, with LabLibSNP being the only viable government. And not a small amount of weight on this being true in spite of Con getting most votes and most seats.

      • Anonymous says:

        Given how badly the Conservative coalition went for them, the Lib Dems might not be interested in another coalition, even with a party they share more values with. The SNP and UKIP don’t seem interested in coalitions; it’s not to their benefit to get wrapped up in Westminster politicking when much of their identity comes from being against the prevailing political establishment. The most they might do is negotiate a deal on a referendum in exchange for confidence and supply votes. So I think it will be a minority government of whoever gets the most seats out of Labour or the Conservatives.

        • Salem says:

          I could see a minority Conservative government with a supply agreement with the DUP (as they did with the UUP in 96-7) surviving for a full term, but that requires the Conservatives being very close to a majority themselves. Otherwise, I think no coalition means a second poll.

          That’s why I think Labour are playing a high-risk game trying to ‘de-capitate’ Clegg.

        • Adam Casey says:

          If the Lib Dems don’t want a coalition what would be the point of them standing? Sure, this coalition hammered their vote, but they don’t care about votes. They care about implementing policy. To do that you need ministers.

          • Salem says:

            You might equally argue that if the Lib Dems truly cared about implementing policy, they’d join the Labour or Conservative Party and work from within.

            Any sensible explanation for why the Lib Dems exist as a party and act the way they do has to take into account the idea that (1) they are playing a long game (2) they enjoy taking a stand and (3) they are not very interested in being held responsible. All of these make it very possible that they might not want to form a coalition after the coming election, at least on terms that would be acceptable to a major party. After all, their predecessors didn’t form a coalition in 1974 or 1977, when they had ample opportunity to do so.

    • Deiseach says:

      I think it’ll be much like the elections in Ireland; God alone knows who we should vote for since they’re all useless but the sitting government may squeak back in.

      I imagine the Conservatives will hang on, maybe with the help of a few independents or fringe parties. I can’t see Labour managing a comeback (who can bear to contemplate Milliband as Prime Minister?) The Liberal Democrats will suffer the usual fate of the junior party in an unpopular coalition government and be (relatively) destroyed at the polls. May be some ‘protest votes’ cast for UKIP. Outside of England, it’s hard to forecast the effect the failed independence referendum will have in Scotland; I suspect the SNP will suffer, but will Labour pick up their voters or where will they go?

      • Muga Sofer says:

        >I think it’ll be much like the elections in Ireland; God alone knows who we should vote for since they’re all useless

        Someone should probably do something about that.

    • haishan says:

      This year’s election seems unusually ripe for tactical voting, with the two major parties plus UKIP and Lib Dem all commanding >10% in polls of many constituencies. As such, I’d urge more skepticism than usual of statistical models. That said, you should retain your usual amount of skepticism of pundits. My meta-prediction: betting markets will outperform any single model or pundit. (I realize this isn’t exactly going out on a limb, but still.)

      • Adam Casey says:

        Bonus factor making tactical voting even more important: Single constituency polling for the first time in GE history.

    • kaninchen says:

      A majority government seems highly unlikely. Based on the data at http://electionsetc.com/, I give greatest credence to a situation in which Labour and the Tories have roughly equal numbers of seats and any ruling coalition requires at least three parties for a majority. The most natural coalition would be Labour + SNP + Lib Dems, but even that won’t be very stable. I would not be surprised if we have another election within 12 months in such a scenario.

      If there is a second election, I think the Tories will benefit – one would expect political chaos to help the party of order and stability.

    • BD Sixsmith says:

      People who follow UK politics, what are your predictions for the general election?

      Despair.

  91. AR+ says:

    A fan has recut the entire Hobbit trilogy into a single 4 hour film. I haven’t seen the original trilogy, but I’ve read the book, and I’m going to watch this version. I’ve long felt like movies are too long, as a general rule. Like, the Nightmare Before Christmas was 90 minutes. A lot of classic Disney movies are 90 minutes. That is a good length for a movie to have.

    For anyone who tried them months back and didn’t like their texture or how extremely dense they were, MealSquares underwent a major change in baking process a month ago that made them much fluffier. I found them tolerable before but now I actually enjoy eating them.

    • I’m increasingly convinced that television series are a better storytelling method than films. At least for book adaptations (contrast GoT and LOTR)

      • drunkenrabbit says:

        Definitely. I can’t think of a single book I’d rather see adapted into a movie rather than a decent miniseries. Especially Dune, the movie attempts at that have been terrible.

        Also, Lonesome Dove is both a miniseries, and the best western I’d ever seen. Some John Ford classics aside.

        • stillnotking says:

          I wanted to hate the Lynch movie, but the truth is that it had me from the moment Virginia Madsen said “A beginning… is a very delicate time.”

          It may be overblown, messy, and incomprehensible to anyone who hasn’t read the novel five times, but goddammit, it has something. The thing that every David Lynch project has, and which there is no word for. Twinpeakiness?

          • John Schilling says:

            I want to see the version that has the look and feel of the Lynch version, including the casting, with the script and plot of the Sci Fi channel miniseries. We can keep a few key bits of Lynchian dialogue, if you like.

          • Erl says:

            I believe that the accepted DFWism is “Lynchian(ism)”

          • Sarah says:

            I like the Lynch movie, but it drives me nuts that everyone is white.

            This isn’t a “representation” thing, it’s a “fidelity to the book” thing. The Fremen are clearly coded as Arab, they use Arabic words, the *default* would be to cast people from roughly that part of the world. (I would buy East Asian, because of “Zensunni.” I would also buy making everyone a mix of races because it’s a far-future society. But everyone being white just feels *artificial*.)

          • pwyll-alt says:

            David Lynch’s adaptation has all kinds of problems, including terrible dialogue and intermittent lack of fidelity to the book’s plot, but nonetheless I think it does an amazing job of building an immersive world. The set design and costume design are breathtaking, and it’s near the top of the list of films I’ve seen in terms of the sense of atmosphere it conveys.

        • Anonymous says:

          Have you seen the documentary about the first attempt to make a Dune movie?

      • huntz0r says:

        There is a fanedit version of the original LOTR films by Kerr which I cannot recommend highly enough. The editor pulled together all the raw material from the theatrical and extended editions, deleted scenes, etc.; then cut out everything that wasn’t in the books, and reassembled the film into six parts that follow exactly the narrative order and structure of the books.

        As a result it tells the story much better than either the theatrical or extended cuts, yet clocks in at a very marathon-able 7 hours and 39 minutes. And it truly is a joy to watch — the incredible world that Jackson et al built for the film really shines once it is relieved of all the films’ frustrating deviations, expansions, and compromises.

        Much of the problem with these films is just an unfortunate limitation of the format. You can have a “table-setting” episode of a TV series and come back hard the next week, but that’s a hard thing to get away with when people have to wait a year for the next film. So you can either be faithful to the larger story arc and end up with a film that’s vaguely unsatisfying on its own (see: Mockingjay Part 1), or you have to screw with the pacing and shoehorn in extra plot threads with climaxes and denouements that don’t exist in the source material.

        I might watch this edit, but the ultimate edit will be made sometime after all three Hobbit films are released as extended editions, and all the material is there to be sorted and sifted and reassembled in the best possible way. I expect it should only need to be 4-4.5 hours to tell the whole story as Tolkien wrote it, and I have no doubt it will be amazing.

        • Anonymous says:

          Much of the problem with these films is just an unfortunate limitation of the format. You can have a “table-setting” episode of a TV series and come back hard the next week, but that’s a hard thing to get away with when people have to wait a year for the next film.

          So, wait, is this really an inherent limitation of the format? What stops them from making both films ahead of time and releasing them in rapid succession?

      • cassander says:

        a movie is a short story, at most a novella. If you have something long and complicated (i.e. more than a couple main characters and themes) there just isn’t space in a movie to include all of it.

    • drunkenrabbit says:

      Along those lines, has anyone seen “The Phantom Edit”? Is it worth a watch?

      • youzicha says:

        I haven’t seen that, but I did watch Star Wars: Turn to the Dark Side, which I guess is an even more extreme version of the same idea. I thought it was certainly more enjoyable than the original movies, but still not really worth the time it took to watch it.

      • zz says:

        I cannot recommend Star Wars: Uncut strongly enough.

      • huntz0r says:

        The Phantom Edit is alright, but there have been many and several better fanedits since. The Q2 edits of the prequel films are all quite good, and make those films watchable and even enjoyable.

        Another one worth getting is Star Wars Episode IV: Revisited by Adywan — which is, essentially, the THX Edition DVD that Lucas never gave us. That description doesn’t quite do justice to the editor, who put a ton of care and attention into it, and it really shows.

        • Anonymous says:

          Can you discuss this in further detail? I watched The Phantom Edit a few years ago and found it just… unobjectionable; I’ve not heard about any of the others.

        • Luke Somers says:

          Adywan’s revisits of aNH and tESB are good, and really clean things up. I wonder what he’d do with RotJ. Maybe the capital ship battle would look like they were actually fighting each other.

    • Izaak Weiss says:

      I would honestly wait until someone cuts the whole thing down to 2 hours. I’ll probably make my own cut at some point.

      I’ve seen all the films and you could easily cut it down further.

  92. Alejandro says:

    “@Sylvester Now that I’ve helped you catch the bird, will you help me catch the mouse?” Tom tweeted.

    “@Tom Sorry, I can’t; he’s standing just outside the border of my hunting district” Sylvester jerrymandered.

  93. anon says:

    Something I’ve been wondering about for a while

    How likely is a homeless person in the United States to find permanent housing again (IE what percentage escape homelessness)? My perception has always been that once you’re down and out you’re more than likely to be homeless for good, but I haven’t been able to find any numbers to confirm or refute that.

    • Jadagul says:

      I think that depends on where you put the threshold for “become homeless”? Like, I’ve read that if you look at all the people who’ve ever spent the night in a homeless shelter, the most common number of total nights for a person to spent in a shelter over their lifetime is “one.” The second-most-common number is “two.”

      Like most problems, the populations divide into “a large group of people who need help dealing with temporary problems” and “a smaller group of people with incredibly crippling collections of problems.”

      • Anonymous says:

        I would think that many of those one-nighters actually have a home but are too drunk to find it. (Extrapolating from personal experience here, but sounds plausible nevertheless.)

        • Deiseach says:

          Speaking from experience in a social housing job (all eight months of it), temporary homelessness can happen for several reasons: generally it’s young people getting kicked out of the family home or relationships breaking up and one partner or the other has nowhere to go. Drink and drug addictions play a part here as well, but we refer people to emergency accommodation (hostels and sheltered housing associations) and try to work out something more medium to long-term (you are not going to get social housing merely because you’re homeless, unfortunately).

          Longer-term homelessness is (a) people are being evicted from rental properties or the rents are going up beyond their ability to pay, and they can’t find replacement accommodation within their means (b) people lost their jobs, can’t pay the mortgage, the bank is repossessing their house and ditto on the accommodation front (c) people with severe substance abuse and/or mental health problems.

          Quite a substantial proportion of the people in category (c) are unable to handle independent living; even if accommodation is found for them or they get social housing, they can’t cope with struggling to live by a budget, pay rent regularly, etc. They end up back on the streets or doing the rounds of ‘couch surfing’, hostels and emergency/temporary accommodation. This can even end up, as in a case from a few months ago that was all over the Irish news media, in people dying on the streets of exposure or the end results of their hard lives. Even with help and support, they fall through the cracks.

      • roystgnr says:

        Aside from the “crippling vs temporary” divide, there’s also a “crippling vs inconveniencing” divide. The HHS definition of homelessness includes “no home, living on the street”, “no home, living in a motel”, and “no home, living with our aunt”, despite some significant practical differences between those categories. Other agencies’ definitions try to exclude all or part of that third, “doubled up” group, whose size can range from millions to tens of millions of people depending on the precise definition.

        • Mary says:

          Author Michael Flynn had some mirth at the expense of those definitions on discovering that he was homeless until he was six. (At the time, he thought his family was living at his grandparents’.)

    • Vaniver says:

      A while back, I came across the rule of thumb that ten homeless people must be offered long-term housing in order to reduce the long-term population of the homeless by one. (There are a bunch of factors here, which sibling comments explain better) That implies to me that we would expect people to be homeless for roughly one tenth of the relevant timeframe.

      It also seems like homelessness, like so many other things, is probably modeled by a power law- less than 20% of the homeless are homeless for more than 80% of the total human-nights not spent in a home.

      So people who are so seriously mentally ill or destitute that they can’t maintain a home are likely to not suddenly be able to maintain a home, but people who are in between homes are very unlikely to become seriously mentally ill or destitute because of that.

    • lmm says:

      Not the US, but one interesting part of Stuart: A Life Backwards was that, having been homeless, it wasn’t such a big deal to him. So even though he had a (government-provided) house, he would sometimes e.g. sleep rough after a night visiting a friend if he’d missed the last bus.

      But it sounded like he was one of the worse cases, with a large stack of issues – and even then, he’d be housed – through local government, or with a girlfriend, or some other acquaintance – for months at a time, and then it would break down.

      Not sure how much any of it will apply to the US, but it was a fascinating book.

    • Anonymous says:

      New York City has collected statistcs on permanently homeless familie iclooked it up a few years ago and don’t have the link . Th number was startling: fifty. Yes, fifty permanently homeless in New York City! Most families use shelters during a crisis when they are too disorganized temporarily to use all the social welfare and hosing benefits they normally use.

      • Nornagest says:

        I’d like to know their definition of “permanent” before drawing any strong conclusions from this.

    • dipitty do says:

      A friend got out of homelessness by coming into an inheritance.

      Short of that, I think he’d have died on the streets. He almost did. Being homeless and schizophrenic is pretty rough.

      People who are homeless due to being between jobs or having been evicted, etc., probably have a decent chance of getting back on their feet. But people who’ve been homeless for a year or more probably have very bad chances.

      Oh, and if you’re curious about my friend’s story, he wrote a book about it: http://www.barnesandnoble.com/w/friends-and-foes-a-memoir-jim-williams/1120837854?ean=9781312626560
      It’s not literature, but it tells the tale.

  94. Let assume for the moment that the factors that make for differential maths performance are decided somewhere after birth and before the final years of high school. (The non-genetic explanation for the results)

    What would those be, and what interventions could be made to change them?

    • Levi Aul says:

      There’s always hormone therapy. Presumably, targeting the sexually-dimorphic portions of people’s brains with hormones from the opposite sex could puff those portions up to full size. Drastic, obviously, but since many neurons have sex-linked hormone receptors built in, there’s really no way around it. (Well, you could play with aromatase levels instead…)

      • Deiseach says:

        Drastic, but if people are really serious about Increase Intelligence Now, and the measure of high intelligence is “how well do you perform on tests of advanced mathematical concepts?” then yes.

        The Hyde and Mertz paper makes me wonder if the reason, as Scott mentioned in the other post, that gender differences show up drastically around 17 is not so much to do with puberty and the effect of sex hormones etc. but rather that as children get older and advance through school, the curriculum gets tougher.

        Therefore, around the age of 17, you’re going to hit complex problem solving, and if males have a wider spread of ability (more likely to be at either end of the bell curve), then the percentage of boys who have greater mathematical ability will now exhibit their superiority by beng able to tackle the more complex problems.

        That is, the pubertal effect which Scott expected to show up around 13-14 is delayed not because of physiology but because of the curriculum. The girls hit the limit of what hard work can do without natual ability when the mathematics taught gets sufficiently complex in a certain grade, and the small percentage of ‘genius’ boys begin to pull ahead?

        • Nita says:

          Look, everyone!

          In the previous thread, Deiseach expressed concern that people would round the nuanced arguments about higher variance down to “girls are dumber”.

          In this thread, Deiseach writes:

          The girls hit the limit [..] and the small percentage of ‘genius’ boys begin to pull ahead

          …implying that acing a high-school level maths test requires a level of ‘genius’ beyond any girl’s limit.

          This makes me update in favour of Deiseach’s original concern.

          • Anonymous says:

            Go troll somewhere else please.

          • Operationalizing Anonymous says:

            Nita, I don’t think Deiseach is the anti-feminist droid you’re looking for.

          • Nita says:

            @Operationalizing Anonymous

            I’m saying: if even Deiseach herself can’t avoid rounding it off in casual conversation, then what can we expect of everyone else?

          • Operationalizing Anonymous says:

            @Nita: I think she’s trying out an explanation to see whether it works. I don’t get the impression she thinks women are dumb. (Also, FWIW, she may be one of the people who agree with you on, like, 85% of stuff. Target judiciously.)

          • Nita says:

            @Operationalizing Anonymous

            Well, obviously* I agree with what she meant in that comment in particular, and appreciate her as a commenter in general.

            * or — actually not obviously, apparently? in any case, thanks for the feedback, kind anon 🙂

          • Deiseach says:

            Do I contradict myself?
            Very well then I contradict myself,
            (I am large, I contain multitudes.)

            And if you took a view of me from the dorsal side, you’d agree to the largeness.

            All right, let’s go round the mulberry bush one more gyration. Are there gender differences when it comes to both extremes of the bell curve for mathematical and other intelligences? It would appear so; the scientists say thusly and who am I to contradict them?

            Therefore, does this mean that a percentage of all the boys studying maths or underwater basket weaving will be both stupider and smarter than the cohort of girls? That appears to be a reasonable conclusion.

            Us being humans, are we reasonable? No, we’re damn well not. Taking a factual statement that some boys, and a greater number of them, will be better or more gifted or have greater innate ability for mathematics than girls will be seized on by everyone with an axe to grind or a point to prove.

            There will be those who say “You see? There is no such thing as institutional sexism! Women are just not biologically able to do the work at the highest level, and therefore we do not need to change anything about the present set-up of society and education, in saecula saeculorum!

            There will be others who decry the results as all part of a Vast Patriarchal Conspiracy and the corruption of science aimed at keeping women down, and denying them their rightful place in equal numbers at the top of the tree.

            I was NOT saying that a particular level of genius was beyond ANY girl’s level; I WAS saying that at the very highest level of classes, there will – because of the vagaries of the bell curve – be more boys able to tackle the topics than girls. That is all that I was saying.

            Now I’m going to go and spend my time more fruitfully, like banging my head against the wall or writing more quasi-idolatrous litanies of adoration for fictional characters as portrayed in popular televisual media.

          • Tom West says:

            I think Nita is spot on. I remember about a month after the Summers’ controversy a slightly burned out male high school computer science teacher telling me (professional-to-professional) that even the president of Harvard said women weren’t up to the hard sciences.

            He wasn’t a terrible teacher, but his misinterpretation of Summers’ comments was all he needed to allow himself to stop making the effort to encourage girls into CS.

            I have no doubt his story (which pretty much matched the Internet’s interpretation once you left the science-related blogs) was repeated many thousands of times over.

            “Women can’t do science” is a nice simple paradigm that matches a huge number of people’s natural understanding, and anything that can be misinterpreted into confirmation, will be.

            When an authority like Summers (president. Harvard!) “confirms” it, there’s very little that is going to dislodge this obvious “fact”.

            Certainly those who misinterpret are hugely more numerous and thus far more significant to the lives of men and women than the insignificant few who will spend the effort to understand the details of what is actually being said.

          • lmm says:

            All words, all explanations are simplifications; “the earth is round” is false, but not as false as “the earth is flat”. “Women can’t do science” is false – but is it more or less false than “women and men are equally good at science”, or “sex differences in scientific ability are unimportant”?

          • houseboatonstyx says:

            On behalf of the Women’s Lib Guild of 1970….

            In 2008 I read up on the controversy about Summers’s luncheon of 2005. I say, a pox on the woman/women at that luncheon who put out the “women cannot” version and attributed it to Summers, and the others of their local tribe who took up the cry. Which then rippled on and on till it reached burnt-out math teachers, etc.

            I suspect some of the motive was to get Summers out of Harvard — without realizing how much damage their misquote would do to feminism and women in general in the outside world.

          • Tom West says:

            Imm: but is it more or less false than “women and men are equally good at science”

            A hell of a lot less false, and a huge amount less damaging for society as a whole.

            houseboatonstyx: Yes, they certainly didn’t help. But Summers was a public individual. He doesn’t get to say things (in public) that *will* be misinterpreted without paying a price. Same goes for politicians and CEOs. That’s why they’re paid well.

          • haishan says:

            But Summers was a public individual. He doesn’t get to say things (in public) that *will* be misinterpreted without paying a price.

            Summers’ comments were public only for very unusual values of “public”:

            The conference, on women and minorities in the science and engineering workforce, was a private, invitation-only event, with about 50 attendees.

            Basically, if Nancy Hopkins hadn’t been triggered by Larry Summers’ horrible misogyny, his remarks would have been heard by a few dozen people. If public figures can’t say things that will be misinterpreted in front of a few dozen people, we’re more doomed than I thought.

          • RCF says:

            “But Summers was a public individual. He doesn’t get to say things (in public) that *will* be misinterpreted without paying a price. Same goes for politicians and CEOs. That’s why they’re paid well.”

            So, any time someone lies about what a public figure said, it’s the public figure’s fault, for being so careless as to say something that someone would lie about?

            Also, it’s lmn, not Imn.

          • cypher says:

            “Women can’t do science” is false – but is it more or less false than “women and men are equally good at science”, or “sex differences in scientific ability are unimportant”?

            Both are significantly less accurate than “we should judge women in science on their own ability as individuals, since we’re already pulling from a self-selected pool, not randomly pulling women from the general population.”

          • lmm says:

            @cypher That’s dodging the question. Of course under ideal circumstances we would always get to know everyone in great detail before ever trying to infer anything about them, even if they were, I don’t know, wearing a load of swastikas or something. But in practice there are only so many hours in the day, only so many conversational opportunities, and six billion people to choose between; the only way to live is to make a certain level of snap judgement based on the limited information available. Given that, for the cases where I have to decide quickly, I’d like my snap judgements to be as accurate as they possibly can be, taking into account any known correlations. Wouldn’t you?

        • Tom West says:

          haishan: The conference, on women and minorities in the science and engineering workforce, was a private, invitation-only event, with about 50 attendees.

          I understood it was a limited audience. I hadn’t quite realized how limited. Still a bad move (obviously), but less egregious than I had realized.

          RCF: So, any time someone lies about what a public figure said, it’s the public figure’s fault, for being so careless as to say something that someone would lie about?

          There are certain things that are pretty darn easy to twist. If your words are likely to feed a flame, then you don’t get to say those words.

          Why are public people so careful about what they say about Islam after a terrorist incident? Because they understand that there words *will* be twisted to justify reprisals against innocents if there words contain even a hint of ambiguity.

          Also, it’s lmn, not Imn.

          And thanks for the correction – lmm it is.

          • Deiseach says:

            If there is any way anyone at all can possibly put the wrong construal on someone you say, it will happen. This goes double for public figures. Just look at any media coverage of remarks made by the current pope, who is very prone to off-the-cuff, unscripted remarks.

            His latest one, if you believe what you read in the papers, is that he told Catholics not to be breeding like rabbits. Cue outrage on both right and left for being disrespectful, etc.!

            I haven’t gone searching for the actual text of his remarks, but I rather imagine that there was a tiny bit of context that we need to know here!

          • Anonymous says:

            If anything, private (but not too private) statements are worse, because the person twisting your words has an easier time controlling the narrative. A truly public statement is up for everyone to hear/see/read, so they can form their own opinions regarding them.

          • Anonymous says:

            Your theory predicts that when Summers published his transcript that the lies would stop.

          • Anonymous says:

            There are alternative, stronger explanations for governments figures refusing to be factual about Islam. Ideology and the scope and popularity of murder of infidels. They say it’s due to concern for reprisals, but why do you believe them? There have been virtually no individual reprisals either here or in Israel, while up the failure to address the ideology of murderous Islam could be argued endangers many more lives, by hamstringing our homeland security. For example, FBI training manuals are not allowed to use the word Islamic, therefore cannot tariff agents re jihadi front groups, agents, methods, recruitment strategies.

          • Matthew says:

            There have been virtually no individual reprisals either here or in Israel,

            Um, what?

            Most recent available report is 2012:

            http://www.fbi.gov/about-us/cjis/ucr/hate-crime/2012/topic-pages/victims/victims_final

      • haishan says:

        Is it possible that getting rid of the plastics that sap and impurify our precious bodily fluids could help?

      • Anonymous says:

        Has synaptic pruning been looked at wrt this? Does it occur later in men and if so could that have an effect?

  95. Secretariat says:

    Count me as interested in a 3/1 SSC meetup.

  96. Pingback: Open Thread #7: War Plans | Thing of Things

  97. Anonymous says:

    Are you going to make another post here or on your tumblr if there is a Berkeley SSC meetup or should I add something else to my feed?

    • Charlie says:

      Yes.

      Okay, that’s not a very polite answer, but I do think that not only would society be better in some non-obvious ways, but also that we’d take it for granted.

      • Anonymous says:

        I already edited out my question. It was below my high standards for hypothetical questions.

        • Desertopa says:

          It makes the train of conversation rather confusing to do this if people have already replied to you though.

    • Desertopa says:

      I think the only way for every book to be a great and substantive piece of literature is for there to be very few books. Great literature seems defined more in terms of impact than quality in common usage.

      If every book were of a standard of quality necessary to be a great and substantive piece of literature, fewer people would probably enjoy reading, because there are already enough really good books for people who enjoy good books to read them very often, but there would no longer be enough bad books to satisfy people who really want a trashy romance or thriller or something.

      • Levi Aul says:

        I think, when you talk about substance/impact, you’re more thinking of the trappings of the genre of narrative people vaguely refer to as “litfic”, not really any qualities pertaining to literary merit per se.

        Great literature can happen in any genre. You can have action or mystery or romance novels that are seminal works with high literary merit, containing skill applied in constructing deep characters; clever and original prose; a narrative structured and paced to impart thematic insights at the right moments; etc. You can have all these things, while also having fun.

        However, it seems that, at least in the Western and possibly Russian literary traditions, the sort of depressive personality required to have deep insights about the human condition and put them to words is anti-correlated with the desire to make people enjoy reading what you write.

        I think some of these authors might learn a thing or to from reverse-adapting the plot of a video game or movie into a novel. That way, the “fun” factor comes pre-installed, and it’s just the literary merit that needs to be worked in.

        • Anonymous says:

          I don’t think video game and movie narratives for the most part adapt very well to novels, or vice versa. Different narratives are better suited to different media.

          Of course, what stories are suited to certain media and what people actually use those media for is often quite different. For instance, I think that video games as a medium have considerable advantage over movies as a vehicle for romance narratives (I could go into this at greater length, but it’s well past 4:00 am where I’m posting this at the moment.) But there’s such a dearth of video games with well written romances that it’s easy to find lists of top video game romances which contain entries where the “romances” are entirely fanon, whereas for decades there have been countless skilled writers competing to create successful romance movies.

          • Nornagest says:

            I think that video games as a medium have considerable advantage over movies as a vehicle for romance narratives

            I’d be interested to hear more about this.

          • Anonymous says:

            Not the same anon, but there is the fact that agency gets people super invested, and I guess narrative branching can enrich some or someother aspect of the romance genre?

          • Mary says:

            “Different narratives are better suited to different media.”

            How true. Perhaps most easily seen by reading Order of the Stick and Rusty and Co..

            Both take place in avowed D&D universes — in Rusty a character consults the Monster Manual — but being webcomics, they get away with things that no DM would ever survive doing. Not only do both parties have central characters, there are scenes with only NPCs — and even prolonged stretches of nothing but NPCs.

          • Desertopa says:

            Okay, to clarify from before (previous anon was me,) I think video games have a major advantage over movies as a vehicle for romances for the following reasons:

            First off, length. As a medium intended to be consumed entire in a single sitting, I think movies are badly handicapped in terms of conveying deep and believable romances. Given the time constraints of the format, it’s hard to portray the development of a credible romance from the beginning, because so many of the ways that people in real life demonstrate rapport and long term compatibility are difficult to convey quickly. The upshot is that most romance movies portray characters behaving irresponsibly and making absurd overcommitments while in a state of limerence, because otherwise it’s hard to progress to any kind of resolution within the time constraints. So in this regard video games are advantaged not because they’re uniquely suited for romance, but because movies are particularly unsuited for romance, and the huge proliferation of romance movies owes more to the fact that romances are one of the most universally popular plots, and movies one of the most lucrative forms of mass media, than to any kind of suitability of the medium.

            Second, video games’ potential for open-endedness means that players can choose between multiple romantic options and select the ones they feel the most affinity for. An article I read a while back argued that video games were inherently a worse medium for romance than movies (with which I disagreed categorically on every point,) and one of its main points of contention was that in a video game, you’re presented with a selection of options, and if you don’t like any of them, the romance will inevitably fall through for you. This is of course true, but it ignores the fact that in a movie, there are no options for the audience, and if they’re not sold on the only romantic relationship presented by the movie, the narrative will also fall through. Multiple options provide the player with something of a safety net; it’s more likely that at least one option will be to the player’s liking.

            Third, the interactivity of the medium can help induce the players to identify with the characters. Particularly if the player’s own choices are providing fuel for the romantic narrative, it’s likely to make them feel more invested in the romance than if they were simply passive observers.

            Of course the big caveat to all this is that it’s given the assumption of an equal quality writing between the different media. Video games might have more potential as a romantic medium in theory, and yet the video game company most known for its attempts to capitalize on these elements is Bioware, whose major writing staff to this day consists of many of the same people whose attempts at romance I was already cringing through when I was thirteen and playing Baldur’s Gate 2. The actual standards of romance writing throughout the video game industry as a whole are really incredibly low, to the point that it’s no surprise if people get the impression that the medium isn’t even capable of conveying compelling romances.

          • anon a gnon says:

            Interesting, Desertopa seems to have independently rederived the entire genre of dating sims / eroge.

            For those uninitiated, dating sims is a class of video games where you play someone (usually a male highschool student) that goes to school and has to make several choices throughout the day on a person you wish to talk to (to get closer to them) or an activity you wish to participate in (to better yourself in some way). These do tend to be very long, have multiple distinctive choices and also have failure conditions to make sure you are committed. This maps well onto your three points.

            The writing tends to be very plain and populist

            I personally do not have much experience in this field, other than having played amagami[0], but it is a genre specifically fitting your requirements. It seems to have died a quiet death in the 2000s, but, again, not an expert.

            The other branch, which I am much more familiar with, eroge (a portmanteau of erotic and gemu). The standard format is very similar (be high schooler, talk to people etc. etc.), but the emphasis is more on the text and the number of choices is much smaller. However, it seems like most of the writing talent is does not typically do romance. Looking at the top 10 entries on erogamescape right now[1] and discounting the one recently released game (which is most likely riding the launch hype). We have 3 sci-fi stories, 2 are literary/drama, 2 are romance, 2 are gameplay oriented conquest RPGs, 1 fantasy adventure. But I suppose erogeis closer as a genre to interactive fiction in general rather than the dating sim subset.

            I’m a fan of visual novels (the english neologism for eroge) but the only way to access them is either through fantranslations (often poor) or waiting for some existing small company to localize it (Manga gamer, Sekai project or JAST).

            I am happy to answer any further questions you may have.

            [0] Translation: Love bite. More information on the system can be found here: https://vndb.org/v1281 (warning, the site as a whole is nsfw)
            [1] http://erogamescape.dyndns.org/~ap2/ero/toukei_kaiseki/toukei.php?kihon_mode=title&mode1=tokuten

        • Creutzer says:

          However, it seems that, at least in the Western and possibly Russian literary traditions, the sort of depressive personality required to have deep insights about the human condition and put them to words is anti-correlated with the desire to make people enjoy reading what you write.

          I don’t know, Dostoevskij strikes me as a rather singular case…

        • Mary says:

          “However, it seems that, at least in the Western and possibly Russian literary traditions, the sort of depressive personality required to have deep insights about the human condition and put them to words is anti-correlated with the desire to make people enjoy reading what you write.”

          “Why is it that all men who have become outstanding 1
          in philosophy, statesmanship, poetry or the arts are
          melancholic?” Aristotle

        • Anonymous says:

          I’m actually quite familiar with eroge, although I haven’t played any that are mechanically “dating sims,” I’ve had experience with a fair number of visual novels. But while there are a few that I’d describe as having better romantic narratives than you’ll find in mainstream non-eroge video games, on the whole, the standards of writing are really low (given that the games are lower investment and lower return compared to mainstream video games, and can sell largely on sex appeal, they have a tendency to attract an even lower tier of writing talent,) and the general tropes of the genre are pretty terrible in terms of constructing credible romance.

          In a representative eroge romance, the protagonist and love interest spend time together, possibly in constant friction with each other, possibly thrown together by trying circumstances, possibly by common interest, but usually with only weak rapport between the characters. Interest is conveyed by awkwardness and flashes of sexual tension, or occasionally by sexual harassment. Then, eventually, one character will confess their feelings for the other, the other will declare mutuality, and the two will consequently agree that they are in a relationship, and will have sex immediately. This is usually the substantial endpoint of the relationship’s progression, although some further conflict is occasionally introduced and resolved after this point.

          There are some visual novels with legitimately interesting and compelling narratives, but for a medium which theoretically focuses so much of its resources on romance, it generally does a very poor job at that. I occasionally wonder if the games which focus on women as a target audience (or “otome games”) handle romance better, but even speaking as a guy who enjoyed female-targeted works like Pony Club books as a kid, I’ve often been left with the distinct impression that works produced for the mass market of Japanese women are actually written for the consumption of some kind of aliens.

          • Nita says:

            I can’t really wrap my mind around a single-player romance game. In real-life romance, the most salient thing to me is how two complex, unpredictable individuals somehow manage to develop mutual interest, affection and trust.

            In my experience, written stories can simulate that to some extent, but games? Aren’t NPCs too predictable?

          • Desertopa says:

            An NPC can be as complex and difficult to predict as any novel character, if you invest that level of writing into them.

            In eroge at least, characters tend to be extremely predictable because players get pissed off if the choices required to “get” a character are too obtuse, and choice generally has little bearing on the outcome of the relationship beyond the “get/don’t get” dichotomy, or possibly “good end/bad end.”

          • jaimeastorga2000 says:

            In my experience, written stories can simulate that to some extent, but games? Aren’t NPCs too predictable?

            Visual novels are mostly text-based. Think of a visual novel as a Choose Your Own Adventure book, except that each scene is illustrated with an anime screenshot and there is an original soundtrack playing in the background while you are reading.

          • anon a gnon says:

            I’m actually kinda confused at your experience saying that video game writing is generally superior, because I tend to find most of the English translated VNs of better quality than typical video game writing, and most of the companies with big name artists that can drive sales have not had any translations (sans Navel’s Shuffle, I suppose anything by Key could qualify?). If you’re referring to prose quality, there are something like three translators out of all of the ones that I know who are halfway competent and understand how to put sentences together, so it could be that.

            I’m not going to deny that there is a long tail of incredibly bad writers, it’s just that I can’t really think of many translated entries that tries to focus on high school romance and is pretty terrible. wrt to writing (Nekopara?)

          • Anonymous says:

            I’m just gonna leave this here: http://ahatestory.com/

        • Eli says:

          However, it seems that, at least in the Western and possibly Russian literary traditions, the sort of depressive personality required to have deep insights about the human condition and put them to words is anti-correlated with the desire to make people enjoy reading what you write.

          Frankly, if someone thinks you need a depressive personality to have deep insights about the human condition at all, I think that someone is either a depressive trying to prop up their own goddamn ego, or just a particularly stupid snob.

      • Mary says:

        For every book to be a great and substantive piece of literature necessarily requires that the author produce no inferior books on the route to producing great and substantive pieces.

        Most writers of immortal works of literature aren’t up to that.